Está en la página 1de 998

ATI COMPREHENSIVE 2.

1. Following abdominal surgery, a client's abdominal wound edges are separating, and the wound is draining a large amount of serous drainage. Thenurse should place the client: Incorrect: This position is incorrect because it can increase tension on the suture line, and cause further wound separation and tearing (dehiscence).Incorrect: This position is incorrect because it can increase tension on thesuture line, and cause further wound separation and tearing (dehiscence).Correct: The semi-Fowler's position decreases tension on the wound, and it may prevent further separation and tearing of the wound(dehiscence).Incorrect: This position is incorrect because it can increase tension on the suture line, and cause further

wound separation and tearing (dehiscence).

flat on the back with legs straight. in high-Fowler's position with legs straight. in semi-Fowler's position with the knees slightly bent. on the left side with knees bent .

2. The PN is preparing to administer an enteral feeding to a client. To prevent gastric cramping and discomfort due to the feeding, the nurse should: Correct: Cold formula can cause gastric discomfort. With enteral feedings, particularly via gastrostomy tube,

the formula reaches the stomach quickly, with little or no opportunity to be warmed, as oral feedings would as they pass through the mouth and esophagus.Incorrect: Tube placement is confirmed prior to beginning each feeding. This action does not prevent gastric discomfort. However, checking tube placement does help prevent the infusion of the formula into the lungs.Incorrect: To prevent gastric discomfort, the concentration of the tube feeding formula needs to be advanced gradually. Full-strength formula may cause gastric discomfort, especially when the first few feedings areadministered.Incorrect: The head of the bed should be elevated at least 30 during the feeding and for at least 30 minutes after feeding. This is done to reduce the risk of aspiration, however, not to prevent cramping and discomfort. allow time for the formula to reach room temperature prior to administration. determine tube placement once every 24 hours. prepare to administer full-strength rather than diluted formula. elevate the head of the bed during and after feedings.

3. The nurse is caring for a child with cystic fibrosis (CF). Which intervention willhelp to prevent respiratory complications? Incorrect: Clients with CF should not receive cough suppressant syrups. These children need to cough frequently to clear lung secretions.Correct: Nebulization with mist or aerosol therapy followed by chest physiotherapyhelps to keep secretions free-flowing. The pulmonary effects of CF are progressive, and bronchial secretions must bekept moist.Incorrect: The child should change positions frequently to promote drainage from the lungs, and promote aeration of the lungs.Incorrect: Children with CF can safely receive the pertussis vaccine. These children need protection from pertussis because this infection causes severe respiratory complications. Encourage the use of cough suppressant syrup.

Give frequent nebulization treatments. Limit changing the child's position to conserve the child's need for oxygen. Withhold the vaccine for pertussis.

4. The nurse is caring for a client following insertion of a pacemaker. The client isplaced on continuous ECG monitoring because it will: Incorrect: This is incorrect because pacemaker voltage settings are adjusted manually at the time of insertion.Incorrect: A chest x-ray is used to check the placement of pacer wires after a pacemaker insertion.Correct: The heart rate may change following pacemaker insertion because the pacemaker fails to maintain the pre-set heartrate. This problem can be detected immediately with continuous ECG monitoring.Incorrect: Fluoroscopy is used to determine dislodgement of pacer leads after a pacemaker insertion. Dislodgement can be prevented with bedrest andminimal arm and shoulder activity. allow the primary care provider to adjust voltage settings. check placement of the pacer wires. detect a dramatic change in heart rate.

determine dislodgement of pacer leads.

5. The nurse is administering eye drops to a client. To prevent injury, the nurse should: Incorrect: The nurse should ask the client to "look up" before instilling the eye drops. This action reduces stimulation of the corneal reflex and injury to the eye, should the client jerk away.Incorrect: Eye drops should never bedropped directly onto the cornea as this action may injure the cornea. The nurse should deposit the medication onto the lower conjunctiva.Correct: As a safety precaution, the nurse administering eye drops should rest his hand on the client's forehead. In case the client moves, the nurse's hand will move at the same time, lowering the risk that thedropper will hit the client's eye.Incorrect: When administering eye drops, it is essential to have an adequate amount oflight. However, the nurse should not shine a bright light directly into the client's eye. ask the client to "look down" before instilling the eye drops. drop the eye drops directly onto the client's cornea. rest his hand on the client's forehead. shine a bright light into the client's eye.

6. Which statement is true regarding the behavior of clients who are in pain? Incorrect: Many clients avoid conversation and social contacts when they are experiencing pain. Clients with chronic pain may become withdrawn and isolated.Incorrect: Clients' reactions to pain are often influenced by theircultural and ethnic background. The nurse needs to consider each client's cultural background when assessing a

client's pain.Incorrect: Clients often place their hands over the painful area as a self-protective or guardingmechanism to prevent further pain.Correct: Many clients fail to report or discuss their pain or discomfort with nursesand other caretakers. Thus, the PN needs to assess clients for pain on a routine basis. Clients experiencing pain may engage in social activities for distraction. Clients from different cultures react to pain in the same way. Clients in pain usually avoid touching the painful area. Clients who are in pain may not report their pain to the nurse or other caretakers.

7. A client is being discharged from same-day surgery following cataract extraction from the right eye. The nurse will instruct the client to: Correct: Lifting requires straining, which increases pressure in the eye and may disrupt suture lines.Incorrect: The client should not bend forward or lower the head. This action increases pressure in the eye and could disrupt suture lines.Incorrect: Mild pain is normal. However, moderate to severe pain should be reported to the surgeon.Incorrect: The client should sleep on the unaffected (left side) to reduce pressure in the eye. Increased pressure could disrupt the suture lines. avoid lifting anything heavier than five pounds until cleared by the surgeon. bend from the waist to pick up objects on the floor. call the surgeon immediately if he has any discomfort. sleep on his back or on his right side.

8. Substance abuse is diagnosed when the person's involvement with drugs oralcohol:

Incorrect: Substance abuse is likely to cause or contribute to family conflict. However, family conflict is not a diagnostic criterion for substance abuse.Incorrect: Substance abuse usually leads to physical health problems overtime. However, physical illness is not a diagnostic criterion for substance abuse.Correct: A client has a problem with substance abuse when that person begins to develop interpersonal difficulties, and is not able to perform their roleadequately at work or at school.Incorrect: A person abusing substances may come to the attention of the law. However, the development of legal difficulties are not a diagnostic criterion for substance abuse. causes family conflicts. causes physical illness. interferes with the person's ability to function. results in legal problems.

9. A newly employed nurse discovers that some medication doses are incorrect. Coworkers admit that changes in medication orders have not been processedcorrectly, but they advise the nurse to administer the medication anyway. The nurse should:

Incorrect: This action is an unsafe practice, and it violates the five rights of medication administration.Incorrect: Reporting the incident to the State Board of Nursing is not the first step in resolving theissue.Correct: The nurse needs to inform the nurse manager about the unsafe medication practice and incorrect medication dosages. The nurse manager can then determine how to resolve this intradepartmental issue.Incorrect: Resigning will not resolve the issue and it will allow the unsafe practice to continue. give the dose that is available. report the incident with documentation to the State Board of Nursing. report this unsafe practice to the nurse manager. resign due to unsafe practices.

10. The nurse is teaching a client with tuberculosis about ways to reduce spreadof the disease to others. Teaching is effective when the client states: Incorrect: Tuberculosis is spread by droplet nuclei, and not through contact with the skin of an infected person.Incorrect: Tuberculosis is spread by droplet nuclei, and not through blood contact.Incorrect: Clients with tuberculosis need to cover their mouths when they laugh to reduce spread of the disease.Correct: Tuberculosis is spread by droplet nuclei. Thus, covering the mouth when coughing decreases the release of droplet nuclei into the air, and the spread of the disease to other people. "I can transmit tuberculosis to others by touching them." "I can transmit tuberculosis to others through contact with my blood." "I don't need to cover my mouth when I laugh." "I'll cover my mouth with a tissue when I cough."

11. The nurse performs a physical assessment on a newborn baby. Which finding, if noted, is abnormal and needs to be reported? Incorrect: Apnea lasting 5-15 seconds is periodic apnea, and is normal for the newborn. No intervention is required as long as there is no change in the infant's heart rate.Incorrect: A blue color in the fingers and toes is called acrocyanosis, and it is a normal finding in the newborn in the first couple of days after birth.Incorrect: Gagging orchoking is common in the hours following birth because the infant was in a fluid-filled environment for the gestation. There should be a bulb syringe in the infant's crib at all times to suction the mouth, pharynx, and nose, and clear theairway as needed.Correct: Normal respirations range for the newborn is 30-60 respirations per minute. Rapidbreathing is a sign of respiratory distress, which may indicate sepsis or other complications and should be reportedimmediately. Apnea lasting 5 to 15 seconds Blue color in the fingers and toes Gagging or choking Respirations of 80 per minute

12. The nurse is providing discharge teaching to a client who is at high risk forinfection. The client asks if there is any way to prevent getting a cold. Whichresponse from the nurse is correct? Incorrect: The common cold is caused by a virus. Antibiotics are not prescribed for the common cold.Incorrect: This is not practical advice, and it does not necessarily eliminate exposure.Incorrect: A mask may decrease risk of infection, but it is not the most effective measure.Correct: Handwashing is the most effective preventive measure for a cold. "Ask your doctor for an antibiotic that you can take."

"Stay indoors during the height of cold and flu season." "Wear a mask when going outdoors during cold and flu season." "Wash your hands frequently during cold and flu season."

13. The nurse is interviewing the wife of a client who is in the middle stage ofAlzheimer's. The wife tells the nurse that she wants to keep her husband at homeas long as possible. Which response by the nurse is most helpful? Incorrect: This statement is not supportive of the wife. The wife may be quite capable of providing care, especially with instruction and other assistance.Incorrect: Some clients with Alzheimer's disease become aggressive, but not all. If aggression occurs, the client can be treated with medication or behavioral interventions.Correct: Withsupport and education, a family caregiver may be able to provide an appropriate home environment for the client with

Alzheimer's disease. This reply accepts the wife's wishes and offers practical help.Incorrect: This comment is not supportive of the wife. It assumes that the nurse knows what is best in this situation. Realistically, only the wife can make this important decision.

It is not realistic for a person your age to care for a client diagnosed with Alzheimer's disease." "Keeping your husband at home could be dangerous. Alzheimer's clients may become violent." "Let's talk about the help you will need so you can continue to care for your husband at home." "To maintain your own health, you need to consider nursing home placement for your husband."

14. The nurse is caring for a client with Buck's traction. In which situation mustthe nurse intervene in order to maintain effectiveness of the traction? Incorrect: This is a correct application of the traction boot. The Velcro straps keep the boot in place to promote effective traction pull.Correct: Effective traction requires the client to be in good body alignment. This positionis out of alignment and requires intervention. Improper body position may decrease the effectiveness of the pull from the traction.Incorrect: This is the correct position for the footplate because it promotes effective traction. If the footplate is placed against the footboard of the bed, effective traction may not be achieved.Incorrect: This is a correct setup for the traction. If weights are resting on the bed or the floor, the force or line of pull of the traction is disrupted.

The boot is secured by three Velcro straps around the leg.

The client is lying with hips midline and shoulders to the left of midline. The traction footplate is resting 6 inches from the footboard. Traction weights are hanging freely at the foot of the bed.

15. What is the purpose of the Apgar score after birth? Incorrect: Apgar scores do not correlate with the gestational age, and are not used for this purpose.Incorrect: Although babies that score low on the Apgar may be transferred to the ICU, this is not the purpose of the Apgar.Correct: The one-minute and five-minute Apgar is a ten-point scoring tool used to assess the need for resuscitation systematically in the newborn. This score reflects the baby's physical condition at birth.Incorrect: Apgar scores are not used to evaluate the baby for long-term health needs or health problems.

To estimate gestational age To determine if the baby needs to be transferred to the intensive care unit (ICU). To determine if the newborn needs resuscitation. To predict if the baby will have long-term problems.

16. A client with severe emphysema has a moon face and a buffalo hump related to long-term steroid treatment. The client states, "I look so horrible." Whichresponse by the nurse is therapeutic? Incorrect: This is a judgmental statement, it has no bearing on disease etiology in this case, and it does not

address the client's anxiety about a distressing change in appearance.Incorrect: This is not a therapeutic response. The client's symptoms are related to long-term steroid use and not overeating. Also, the nurse is not addressing the client's anxiety about a distressing change in appearance.Correct: This is a therapeutic response. This response indicates that the nurse is receptive to listening to the client's feelings about distressing body imagechanges.Incorrect: This is not a therapeutic response. The nurse is not addressing the client's anxiety about a distressing change in appearance.

"If you hadn't smoked all those years, this would not have happened." "I will refer you to the dietician for a weight-loss diet." "This must be very upsetting for you. Would you like to talk about it?" "You must try to get used to your appearance as you need to take steroids."

17. A client develops wheals as a result of an allergic reaction to a medication. When documenting the reaction, the nurse correctly describes a wheal as: Incorrect: This is a description of a macule, not a wheal. Freckles or flat moles are examples of macules.Incorrect: This is a description of a papule, not a wheal. Warts or elevated nevi are examples of papules.Incorrect: This is a description of scales, not wheals. Dandruff or dry skin are examples of scales.Correct: This is the correct description of a wheal. Urticaria (or hives) is characterized by the development of wheals.

a flat, nonpalpable, brown lesion with an irregular boarder. a slightly elevated, palpable mass with a clearly defined border. fine, silvery-white, irregularly-shaped flakes that adhere to the skin.

an itchy, elevated, reddened mass with an irregular border and shape.

18. A 7-year-old child arrives in the emergency department with multiple injuries and bruises that are in different stages of healing. The parents report that thechild received injuries on the legs and torso from falling off a bike. What is thelaw in this situation? Incorrect: It may be helpful for a counselor to interview the child and the parents, and provide counseling. However, counseling is not a requirement in this situation.Incorrect: Any health care provider who observes suspicious injuries on a child must report abuse. Reporting abuse is not the sole responsibility of primary careproviders.Incorrect: Bike safety is important, but this child's injuries are more suspicious of abuse than a fall from abike.Correct: By law, suspicious injuries such as bruises on various parts of a child's body must be reported to the authorities. Child Protective Services is the agency that oversees the protection of children from abuse. A counselor must talk with the child and the parents. A primary care provider must notify authorities of suspected abuse. The child must be taught bike safety. The child's injuries must be reported to Child Protective Services.

19. The nurse is caring for a client following a transurethral resection of theprostate (TURP). Which action reduces the risk of postoperative bleeding? Correct: CBI with normal saline helps to prevent bladder spasms, which may cause postoperative bleeding.Incorrect: The client should not remain in a supine position. The client needs to turn, cough, and deep breathe to prevent respiratory complications.Incorrect: Urine output should be measured every 2 hours.Incorrect: Theclient needs to increase fluid intake to a minimum of 2000-2500 mL/day to keep the urine clear.

Adjusting the continuous bladder irrigation (CBI) fluid to maintain a colorless or light pinkdrainage return. Maintaining the client in a supine position for the first 24 hours after surgery. Measuring urine output every 4-6 hours Restricting fluid intake to less than 1800 mL/day

20. During the preoperative phase, nursing management for cardiac surgery isprimarily focused on: Correct: Client teaching during the preoperative period helps to reduce the client's anxiety. The client is encouraged to ask questions and discuss concerns. Family members should also be included in the teachingsessions.Incorrect: Diagnostic studies establish the baseline data that play a key role in determining the need forsurgery. These studies are not the primary nursing focus during the preoperative phase.Incorrect: Providing the client with information about blood transfusion, autotransfusion, and autologous blood donation is important. However, teaching about blood transfusions is not the primary nursing focus during the preoperative phase.Incorrect: Physicalassessment is most important during the postoperative phase of cardiac surgery. It is not the primary nursing focus during the preoperative phase. client teaching. diagnostic studies. blood transfusion. physical assessment.

21. The role of the nurse who works in a rehabilitation unit is to: Correct: Nurses are responsible for coordinating client care with other departments.Incorrect: The nurse manager is responsible for the annual budget.Incorrect: The nurse manager is responsible for establishing the unit's goals with input from the staff.Incorrect: The nurse manager/supervisor is responsible for adequate staffing. coordinate client care with physical and occupational therapy. develop an annual budget for the unit. establish goals for the unit. reassign personnel to other units to ensure adequate staffing.

22. The nurse is preparing an 11-year-old boy for removal of a skeletal traction pin and application of a hip spica cast. The boy says he wants his parents withhim during the procedure. The most appropriate nursing action is to:

Correct: The child will be supported and comforted by the presence of his parents. However, the procedure may be too anxiety-producing for the parents to witness.Incorrect: The child should be familiar with the staff in the procedure room, but parents should also accompany the child to give support if possible.Incorrect: The presence of a nurse may be comforting, but children usually receive the greatest sense of support from their parents.Incorrect: Usually parents are allowed and encouraged to accompany their child during this type of procedure. allow the parents to be with the child. introduce the child to the staff in the procedure room. explain to the child that the nurse will be with him for support. explain to the child why the parents cannot be present.

23. The best method for verifying the identity of a client prior to theadministration of a medication is to: Incorrect: Identification tags at the end of a client's bed are not reliable. The client may have become confused and moved into another bed, or the tag may belong to the previous client.Correct: Checking an intactidentification bracelet is the best method for verifying a client's identity prior to administering medications.Incorrect: This is an inappropriate method for identifying the client. The names above the bed and outside the room may be thatof a former client.Incorrect: Asking clients to verify their identity is not a safe method for identification and may result in medication errors. Clients may be confused, anxious, or disoriented, and thus provide unreliable information aboutthemselves. check the identification tag located at the bottom of the client's bed. check the medication administration form against the client's identification bracelet.

compare the client name above the bed with the name plate outside the room. state the client's name and ask the client if he or she is that person.

24. An immobilized client requires passive range of motion (ROM) for each joint. Which technique is correct when performing these exercises? Incorrect: It is safe to flex the joint until slight resistance is felt, but not beyond. Exercising a joint until full resistance is felt could damage the affected joint.Incorrect: Exercising a joint to the point of pain increases the risk for trauma to the joint. The nurse should note the point at which the client experiences pain and stop the exercise before this point is reached.Incorrect: Hyperextending the joint may increase the risk of injury to the joint and is thereforecontraindicated.Correct: Support of distal joints during passive ROM reduces the risk of injury and damage to affectedjoints. Passive ROM exercises should be performed slowly and carefully to ensure support of the client's joints. Continue to flex the joint until full resistance is felt. Exercise the joint to the point of pain. Hyperextend healthy joints to increase flexibility. Support the client's joints that are distal to the joint exercised.

25. A nurse is providing home care for a 7-year-old client with muscular dystrophy. The client has been receiving home schooling. A care conference is planned to discuss if the client can attend regular school classes. What is thenurse's role in the conference? Incorrect: This is not the role of the nurse during the conference. The home school teacher can provide

valuable information in this area.Incorrect: This is not the role of the nurse during this conference. The classroom teacher is the person who will assess the client and determine the classes the client can attend, based on theinformation provided by the nurse as to the child's capabilities and limitations.Incorrect: This is not the role of the nurse during this conference. The social worker is the person who assists the client and family with financial needs.Correct: The nurse can provide valuable information about the child's capabilities and limitations. This information will assist the other members of the conference to plan activities that will enhance the child's potential. Describing the client's usual method for learning Determining which regular school classes the client can attend Discussing the client's need for financial assistance for school supplies and activities Providing information about the child's capabilities and limitations

26. Which finding is an early symptom of poor oxygenation in the client who hasexperienced an acute myocardial infarction (MI)?

Incorrect: Cyanosis is a clinical sign that develops in the later stages of poor oxygenation. Hypoxia, or poor oxygenation, must be identified at an early stage.Incorrect: The heart rate tends to increase in the client with poor oxygenation. The heart rate increases to compensate for inadequate oxygenation and to maintain cardiac output.Correct: Shortness of breath is an early indication of respiratory distress following an acute MI. Unlesspromptly identified and treated, poor oxygenation can result in cardiogenic shock.Incorrect: Venous stasis is not a manifestation of poor oxygenation. Pooling of blood results from inadequate circulation in the lower extremities. People who are obese or have congestive heart failure are at risk for venous stasis. Cyanosis Slow heart rate Shortness of breath Venous stasis

27. The nurse is caring for a client with hepatitis A. The client asks how he acquired this type of hepatitis. Which response by the nurse is accurate? "Thetype of hepatitis you have can be transmitted through: Correct: Hepatitis A has a fecal-oral transmission route.Incorrect: Hepatitis B can be transmitted through exchange of body fluids.Incorrect: Hepatitis B and D can be transmitted through sexual contact.Incorrect: Hepatitis Cis transmitted through percutaneous exposure to blood and plasma. The highest incidence occurs in IV drug users and individuals with hemophilia. contaminated food." body fluids." sexual contact."

sharing IV drug needles."

28. The nursing assistant has reported a fever of 102.2 F (39 C) in a client withmeningitis. Which action by the nurse is appropriate? Incorrect: Analgesics do not necessarily have antipyretic properties, so these drugs may not lower body temperature.Incorrect: Rapid or excessive lowering of body temperature can cause shivering and cause bodytemperature to rise even more.Correct: An antipyretic acts to lower body temperature. It is important to reduce the client's temperature because fever increases the risk of seizures and increased intracranial pressure.Incorrect: Because of the risk of seizures, the client should not get into the shower. In addition, cool water can chill the client, resulting in shivering and a rebound increase in body temperature. Administer prescribed analgesics. Apply ice packs to the client's axillae and groin. Check to see if there is an order for an antipyretic. Have the client take a shower in cool water.

29. The nurse is talking about the goals of therapy with a small group of clientson the inpatient unit. The nurse notices that one client, who is usually talkative, isnot participating in the discussion. The nurse identifies this client behavior as: Incorrect: Antisocial behavior violates the rights of others. Although this client is not participating in the group discussion, the behavior is not antisocial.Incorrect: Histrionic behavior is very dramatic and brings attention to theclient. This client's behavior avoids attention to the self.Incorrect: Obsessive-compulsive behaviors include checking, ordering, counting, and undoing. The client's behavior is not obsessive-compulsive.Correct: Passive behavior is

identified by a lack of active participation and involvement in discussions and activities that involve the individual. Thisclient is not actively participating in therapeutic group discussion. antisocial. histrionic. obsessive-compulsive. passive.

30. The nurse is assessing a client with diabetes mellitus who is at risk fordeveloping hypoglycemia. Which symptom occurs with hypoglycemia? Incorrect: Deep, rapid respirations are a symptom of hyperglycemia, not hypoglycemia. Kussmaul's respirations are the body's attempt to "blow off" the extra acid produced with diabetic ketoacidosis.Correct: Diaphoresis is a symptom of hypoglycemia. The sympathetic nervous system is activated when there is a decrease in

the amount of glucose available to the cells. Activation of the sympathetic nervous system produces profuse sweating.Incorrect: Excessive thirst is related to hyperglycemia, not hypoglycemia. When the blood contains higheramounts of glucose, the body attempts to dilute the blood with intracellular and extracellular fluid. As a result, the tissues lose water, causing thirst.Incorrect: Frequent urination is related to hyperglycemia, not hypoglycemia. When the blood contains higher amounts of glucose, the body attempts to dilute the blood with intracellular and extracellularfluid. This extra fluid increases urine output.

Deep, rapid respirations Diaphoresis Excessive thirst Frequent urination

31. Which is the critical step in assuring correct identification of a newborn? Incorrect: Checking identification bracelets is an important later step in the process of infant identification. However, it is not the first critical step in this procedure.Incorrect: Many hospitals do provide identification bracelets tofamily members. This step provides some measure of security, but it is not the critical step in the process of infant identification.Incorrect: Footprinting of newborns is helpful in identification, but it is not the critical security measure forcorrectly identifying newborns.Correct: The infant needs to be correctly identified with identification numbers at birth. This security measure is critical and must take place before the infant leaves the birth setting. Identification bands areapplied to the infant's wrist and ankle. The numbers are documented in the medical record. The mother and father of the baby also wear the identification bands during the entire hospitalization of the baby.

Comparing identification bracelets of mother and baby before leaving the baby in themother's room Providing identification bracelets for all family members Obtaining identification footprints at birth and checking them at discharge Identifying the newborn before mother and newborn are separated after delivery

32. The nurse is caring for a client with acute respiratory failure. To reduce the client's fear and anxiety, the nurse should: Correct: The nurse needs to explain procedures calmly, using simple terminology and a soothing manner. Simple explanations will keep the client informed, while decreasing the fear of the unknown.Incorrect: The staff should not limit visits from family members. Visitations from the family may help to decrease the client's fear and anxiety.Incorrect: While a quiet environment may promote rest, it is not the best intervention to reduce fear and anxiety. Verbal support and reassurance with concrete actions to intervene are more likely to decrease the client's anxiety than leaving him alone in a quiet room.Incorrect: While the nurse may refer some questions to the primary care provider, it is not the best intervention for reducing fear and anxiety.

explain all procedures using simplified terms. limit visits from family members. maintain a quiet environment at all times. refer the client's questions to the primary care provider.

33. The nurse is planning care for a preterm infant and the parents. To promote the infant-parent bond, the nurse should: Incorrect: One nurse or case manager should act as the liaison with the baby's parents. Preterm infants are hospitalized for long periods. Receiving information from many different health care workers can confuse parents.Correct: Parents should feel free to visit their preterm infants. The nurse should also encourage the parents to touch and hold their infant.Incorrect: The baby's siblings and grandparents should visit the infant. The support of other family members can strengthen the parents' ability to cope.Incorrect: The nurse should explain the equipment toparents. However, parents should not be responsible for using or changing settings on the baby's equipment. encourage all health providers involved in the infant's care to keep the parents informed. encourage the parents to visit the infant frequently. limit visitors to only the mother and father. teach the parents to use the equipment involved in care of the baby.

34. The nurse is assessing a client who is receiving oxygen therapy. The client may be developing oxygen toxicity. A sign of oxygen toxicity is: Incorrect: A pulse rate of 90 is within the normal resting pulse rates for adults, and it is not a sign of oxygen toxicity.Correct: Bradypnea, or abnormally slow breathing, is a sign of oxygen toxicity. The nurse should immediately

report this finding so that the oxygen flow can be adjusted to meet the body's oxygen demand.Incorrect: A respiratory rate of 12 to 18 breaths per minute is normal, and is not a sign of oxygen toxicity.Incorrect: Warm, moist, pink skin is asign of adequate oxygenation, not oxygen toxicity.

a pulse rate of 90 beats per minute. a respiratory rate of 6 breaths per minute. a respiratory rate of 12 to 18 breaths per minute. warm, moist, pink skin.

35. Which question should the nurse ask in order to assess a client for risk ofsuicide? Incorrect: This is a judgmental question that introduces the issue of whether it is sinful to attempt or commit suicide. It forces the nurse's value system on the client, and it may stop the client from disclosing suicidalthoughts.Incorrect: Suicidal behavior among family members puts the client at higher risk of suicide. However, this question will not give the nurse information about this particular client's thoughts.Correct: Direct questioning is themost effective and efficient way to determine if a person is suicidal. The nurse needs to assume a caring, nonjudgmental attitude toward the client.Incorrect: This is a leading question that tells the client what the nurse wants to hear. This judgmental approach prompts the response, "Oh, no, I haven't thought about hurting myself." "Do you realize that suicide is a sinful act?" "Has anyone in your family ever committed suicide?" "Have you tried to hurt yourself, or are you thinking of hurting yourself?" "You haven't thought about hurting yourself, have you?"

36. A pregnant client calls the telephone triage nurse and states, "I think I may bein preterm labor." Which question should the nurse ask first? Incorrect: While assessing the client's knowledge base concerning labor is important, it is not the first question the nurse should ask.Correct: A history of preterm labor is the most important risk factor for subsequent preterm labor and delivery.Incorrect: While this is an important question, it is not the first question the nurse should ask. The client may be experiencing Braxton-Hicks contractions, which are normal during pregnancy.Incorrect: While the EDD is an important consideration in the assessment of preterm labor, it is not the first question the nurse should ask. "Have you discussed the signs of impending labor with your Certified Nurse Midwife(CNM)?" "Have you experienced preterm labor before?" "How often are your contractions?" "When is your estimated date of delivery (EDD)?

37. The nurse is participating in a health fair. If noted, which client should the nurse refer for an osteoporosis evaluation? Incorrect: African-American women are less susceptible to osteoporosis because they have a greater bone mass than white women do.Correct: Decreased mobility and immobility increase the risk of osteoporosis becausebone is resorbed faster than it is formed. Also, loss of bone mass occurs with aging.Incorrect: Women who have a

small, thin frame are more susceptible to osteoporosis than are women with a large frame.Incorrect: Men are lesssusceptible to osteoporosis because they have a greater bone mass than women do. Also, walking decreases the risk of osteoporosis because it increases muscle activity and bone formation. A 60-year-old African-American woman. A 65-year-old woman who is confined to a wheelchair A 65-year-old woman with a large frame. A 70-year-old man who walks daily.

38. A factory worker reports to the employee health clinic after chemicals splashed into his left eye. The nurse's immediate action will be to: Incorrect: The nurse should not apply a dressing because it may actually keep the chemical in contact with the eye.Incorrect: Assessing the client's health history is important, but it is not the first action the nurse should take inthis emergency.Correct: The most important emergency action is to dilute the chemical and rinse it from the eye. Performing this intervention immediately will help to minimize injury to the eye. The nurse can use normal saline or plain water without checking for client allergies.Incorrect: Drops will not be sufficient to flush the chemical from theeye. This is not the appropriate emergency treatment.

apply a dry dressing. assess the client's health history, including allergies. flush the eye with normal saline or water. instill normal saline drops in the affected eye.

39. The nurse is discharging a 15-year-old girl following hospitalization for aseizure disorder. After receiving instructions for home care, which statement by the client indicates a need for further education? Incorrect: Alcohol ingestion, stress, and fatigue can increase the risk for seizures. Clients who are aware of precipitating factors may feel more in control.Incorrect: All of the anticonvulsant medications can cause abnormal development of the fetus. The client should postpone childbearing until she no longer takes this medication.Incorrect: Exercise appears to reduce the frequency of seizures. Teens are encouraged to participate in their regular activities.Correct: State regulations differ, but most states allow teenagers to apply for a driver's license after they have been seizure-free for a year. "Alcohol, stress, and fatigue can make me more prone to seizures." "I shouldn't get pregnant while I'm taking an anticonvulsant medication." "I'll need to keep up my exercise routine." "I'll never be able to drive."

40. A nurse is participating in a community program designed to educate the public about depression. To increase depression awareness, the program will

stress that: Correct: Clients with depression are usually treated with antidepressant drug therapy. Individuals with severe depression may require electroconvulsive therapy (ECT).Incorrect: There is no one cause of depression. However, stressors in one's social environment can add to the life burden with which a person must cope.Incorrect: People arenot able to use willpower to end a depression. Depression is a biopsychosocial illness that usually requirestreatment.Incorrect: All age groups are vulnerable to depression. However, elderly adults are more vulnerable to depression due to the increased number of life losses they experience. depression is a treatable disease. depression is caused by sociological stressors. people can use will power to resolve a depression. middle-aged persons are more vulnerable to depression than other age groups are.

41. The nurse is preparing a client for a fecal occult blood test. Which statementprovides the most accurate information about this test? Incorrect: Dietary preparation is generally required for this test. The client needs to avoid red meat, poultry, and raw vegetables before and during the test to prevent a false positive reaction.Incorrect: One positive test is notdiagnostic for blood in the stool. Three tests must be positive in order to confirm the presence of blood in the stool.Incorrect: The client should avoid vitamin C and aspirin during the test period because these preparations may result in a false positive result.Correct: The fecal occult blood test is a useful diagnostic tool for detecting blood in feces that may be due to colon cancer. Dietary preparation is not necessary for this test. One positive test indicates gastrointestinal bleeding.

The client can continue to take vitamins and aspirin during the test period. This test detects small amounts of blood in the feces.

42. A client is making her first visit to the prenatal clinic. The nurse is completing a pregnancy profile. This client has the most common pelvic shape, which is: Incorrect: About twenty percent of women have the android pelvic shape. An android pelvis is typical of males, and it is not favorable for labor and birth.Incorrect: About twenty-five percent of women have the anthropoid pelvic shape, and it is not favorable for successful labor and birth.Correct: Fifty percent of women have the gynecoidpelvic shape. A gynecoid pelvis is considered adequate for successful labor and birth.Incorrect: Only five percent of women have the platypelloid pelvic shape. This is a flat shape, and it is not adequate for successful labor and birth.

android.

anthropoid. gynecoid.

platypelloid.

43. The nurse is providing palliative care to a client with advanced terminalovarian cancer. Which nursing measure is most essential? Correct: When providing palliative care for the client anticipating death, the nurse should monitor the client closely for any sign or symptom of distress. The nurse then administers pain medications as needed until the pain is controlled.Incorrect: Most clients who are dying are more comfortable if they do not eat or drink.Incorrect: Clients whoare dying may not benefit from artificial hydration via an intravenous infusion, and this procedure may add to theirdiscomfort.Incorrect: Clients nearing the end of life are not necessarily depressed. However, the client and family maybenefit from counseling and emotional support. Administer additional pain medication as ordered to alleviate breakthrough pain. Encourage the client to eat by offering preferred foods. Maintain a patent intravenous line to provide adequate hydration. Obtain a referral for counseling to help the client with depression.

44. The nurse is caring for a client who observes kosher dietary laws. Whichstatement best describes this cultural food pattern? Incorrect: Members of the Hindu religion and others may advocate a vegetarian diet. However, dietary rules and restrictions vary according to the doctrines of individual sects.Correct: Kosher dietary laws of the Jewish religiondo not allow the mixing of milk products and meat at a meal. Milk products must be served first, followed, later, by the meat.Incorrect: Some Roman Catholics abstain from eating meat on Ash Wednesdays or on Fridays during Lent. Some older Catholics may abstain from eating meat on all Fridays of the year.Incorrect: Members of the Islam faith

are taught to abstain from pork and alcohol. Traditionally, meats that are allowed should be killed and blessed according to cultural rules. A vegetarian diet is the preferred diet. Dairy products and meat are not eaten together. Meat is not eaten on Ash Wednesday or on Fridays during Lent. Pork and alcoholic beverages are not allowed.

45. The nurse is caring for a client with an indwelling Foley catheter. To help prevent a urinary tract infection (UTI), the nurse should: Correct: The drainage bag should be kept below the level of the client's bladder to promote drainage and prevent a UTI.Incorrect: Specimens should be obtained from the designated port using sterile technique. To prevent UTI, it is essential to maintain a closed drainage system.Incorrect: To promote drainage and prevent UTI, the drainage bag should be kept below the level of the bladder. Also, the tubing should be coiled on the bed.Incorrect: To prevent UTI, the client should ambulate with the drainage bag below the level of the bladder. The closed drainage should not be interrupted. keep the catheter bag below the level of the bladder for proper drainage. obtain specimens by separating the catheter from the drainage tube. place the drainage bag and tubing on top of the bed by the client's feet. separate the drainage bag from the catheter when the client is ambulating.

46. What are common clinical symptoms associated with digitalis toxicity?

Incorrect: Dizziness and light-headedness are not associated with digitalis toxicity.Incorrect: Fatigue and ataxia are common problems associated with abnormalities of the nervous system. They are not associated with digitalis toxicity.Correct: Manifestations of digitalis toxicity in the visual system include green or yellow-colored vision, visual halos, double vision, and blurred vision.Incorrect: Neither tinnitus (ringing in the ears) nor ear pain isassociated with digitalis toxicity. Dizziness and syncope Fatigue and ataxia Yellow vision and halos Tinnitus and ear pain

47. The nurse is talking with an expectant couple about nonpharmacologicmethods of pain relief. Which statement by the couple indicates the need forfurther teaching?

Correct: Non pharmacologic methods of childbirth pain management are widely used today, even though many types of anesthesia are available.Incorrect: Breathing strategies, relaxation techniques, and imagery should bepracticed prior to labor. Practice increases the woman's ability to skillfully use these techniques during the actual labor.Incorrect: There are a variety of pain-relieving techniques available to mothers in labor. The pain in labor may becaused by physiological, social, cultural, environmental, and psychological factors.Incorrect: The supportive presenceof a significant other is known to have a positive effect on the length of labor, and it can reduce the pain of labor. "Nonpharmacologic techniques are rarely used today for pain relief." "Nonpharmacologic methods of pain relief are most effective if practiced before labor." "There are a variety of pain-relief techniques, because no one factor causes pain in labor." "Women have shorter labors and less pain when a support person is present."

48. The nurse is caring for a client who has a new ostomy. Which action will the nurse take when performing ostomy care? Incorrect: Moisturizing soap should not be used because it may leave a residue on the skin that will interfere with pouch adhesion.Correct: Using a measuring device such as a stoma gauge helps ensure that the ostomy pouchfits properly.Incorrect: The supine position is a more comfortable position for ostomy care.Incorrect: The seal should be removed slowly and carefully to reduce trauma and irritation to the skin. Cleanse the skin around the stoma with moisturizing soap and water. Measure the stoma site using a measuring device. Position the client on the side to perform ostomy care. Remove the seal from around the stoma quickly and in one motion.

49. A toddler is scheduled for surgery. To help the toddler cope with hospitalization, the nurse should: Correct: A well-loved toy or comfort object will help the toddler cope with feelings of separation from home during hospitalization.Incorrect: Toys from the play area may distract the child, but it will not help the toddler to cope with fears.Incorrect: A tour the day before admission is appropriate for the older child. It is best to prepare a toddler right before a procedure because they have little concept of time.Incorrect: Storybooks may help to prepare a toddlerfor hospitalization. However, toddlers do not learn from factual explanations as would school-age children. ask the parents to bring a favorite toy or blanket from home. give the child toys from the playroom for distraction. give the parents and child a tour of the unit the day before admission. have the parents read a book to the child that explains hospitalization.

50. The nurse is caring for a client with congestive heart failure (CHF). Whichaction should the nurse take first to relieve the client's dyspnea? Incorrect: This nursing action collects data for assessment, but it does not relieve the dyspnea nor does it promote comfort.Correct: Fowler's position helps the client breathe by decreasing venous return to the heart, andincreasing thoracic capacity. This nursing action will relieve the client's dyspnea and promote comfort.Incorrect: This nursing action may help to relieve the client's anxiety, but it is not the first action the nurse should take to relieve dyspnea and promote comfort.Incorrect: This nursing action will reduce peripheral edema in the legs, but it will not relieve dyspnea.

Auscultate the client's breath and heart sounds. Adjust the client's bed to Fowler's position. Encourage the client to take deep breaths. Support the client's legs with pillows.

51. The nurse recognizes that pointing out strengths to a client will promote: Incorrect: Self-concept refers to one's perception or view of oneself. In some cases, a clearer selfconcept may include perception of negative qualities, and this may lower the client's selfesteem.Incorrect: Narcissism is totalself-involvement. People with narcissistic personalities have an unhealthy form of self-esteem. They have an inflated self-concept and behave as though only their needs are important.Incorrect: Self-awareness may not promote selfesteem. As people become more aware of their feelings, they may also become more anxious or depressed.Correct: Self-esteem is a person's judgment of his or her own worth. The nurse can strengthen a client's self-esteem by pointing out the client's strengths and positive qualities.

a clearer self-concept. narcissism.

self-awareness self-esteem

52. An angry client approaches the nurse and makes loud demands. Which is thebest response from the nurse? Incorrect: This response humiliates the client by belittling his behavior and comparing it to that of children.Incorrect: This response is evasive and designed to ignore the client's request. An angry client may also perceive this response as hostile.Correct: This response encourages the client to understand the effects of his behavior and to adhere to appropriate social standards.Incorrect: This response uses anger and indignation to address the client's anger, and it may promote more angry remarks by the client. "Act like an adult, not a child." "I will get to you as soon as I can." "I will take care of your requests when you lower your voice." "Who do you think you are to yell at me?"

53. To decrease the risk of respiratory acidosis in the postoperative client, the nurse should: Incorrect: Although it is important to monitor oxygenation in the postoperative client, oximetry will not decrease the risk of respiratory acidosis.Incorrect: Assessing chest sounds in the postoperative client is important,

but this intervention will not decrease the risk of respiratory acidosis.Incorrect: The level of oxygen in the blood has no relationship to respiratory acidosis. Providing oxygen will not decrease the risk.Correct: The nurse needs to ensure that a client's pain is controlled following surgery. Pain at the incision site can prevent a postoperative client from breathing deeply, which can increase the risk of respiratory acidosis. assess oxygen concentration in the blood using a pulse oximeter. evaluate the client's chest sounds. provide oxygen via nasal cannula. provide pain medication as necessary.

54. The nurse is placing a nasogastric tube in a client who has a decreased levelof consciousness. Which nursing action will decrease the risk of injury to theclient? Incorrect: The nasogastric tube should not be iced. Icing may cause the nasogastric tube to stiffen, which increases the potential for trauma to the mucous membranes.Incorrect: The length of the tube should be determined by measuring from the tip of the client's earlobe to the nose, and then from the nose to below the xiphoidprocess.Incorrect: Clients with a decreased level of consciousness should not receive anything by mouth duringinsertion of a nasogastric tube. The client may choke or may aspirate the fluid.Correct: The semi-Fowler's position reduces the risk of incorrect placement of the tube in the trachea or bronchi, and the aspiration of vomitus. The client should remain in the semi-Fowler's position during feeding and for 30 minutes after feeding. Cool the plastic nasogastric tube in ice for ease of insertion. Determine the length of the tube by measuring from the client's nose to the sternum. Instruct the client to swallow small amounts of water to ease passage of the tube. Position the client in a semi-Fowler's position.

55. The nurse is assessing a client with chronic obstructive pulmonary disease(COPD). Which clinical manifestation indicates that the client is developing rightsided heart failure? Incorrect: The client with right-sided heart failure may have an elevated blood pressure due to fluid volume excess.Correct: An early sign of right-sided heart failure is dependent edema in both lower extremities. This manifestation is the result of increased venous pressure and poor venous return due to diseased blood vessels in thelungs.Incorrect: The client with right-sided heart failure may have distended neck veins due to fluid volume excess.Incorrect: The client's weight increases with right-sided heart failure because of fluid retention. Blood pressure within the client's baseline Dependent edema in both lower extremities Flat neck veins Weight unchanged or slightly lower

56. The nurse observes that a 7-hour-old-baby has mucus in the back of hismouth. Which intervention is a priority? Incorrect: Documenting and monitoring findings is important, but the baby needs to be suctioned first.Incorrect: Suctioning the nose first may cause the baby to aspirate oral mucus. The mouth should be suctioned first.Correct: Gentle suctioning removes secretions. Suctioning the mouth and then the nose prevents possibleaspirations of oral secretions.Incorrect: Mucous production is normal in the period of transition to extrauterine life. Itdoes not warrant reporting to other health care providers. Document the finding and continue to monitor the baby. Gently suction the baby's nose, then the mouth, with a bulb syringe. Suction the baby's mouth and then the nose with a bulb syringe. Report the finding to the nurse practitioner or health care provider.

57. The nurse is instructing a client who has just been diagnosed with mildhypertension. Which lifestyle modification can the nurse suggest to help theclient control hypertension? Incorrect: Clients should exercise for 30 minutes every day at a moderate to moderate-intense activity level. Moderate-intense activities include brisk walking, jogging, and swimming.Correct: Excessive alcohol consumption isstrongly associated with hypertension. Hypertensive clients who drink alcohol should limit their alcohol intake to 1ounce per day, or the amount of alcohol in 2 ounces of 100-proof whiskey, 8 ounces of wine, or 24 ounces of beer.Incorrect: Reducing the fat content of the diet does not substantially lower blood pressure. However, fat reduction may slow the progress of atherosclerosis, and reduce overall cardiovascular risk.Incorrect: To control hypertension, clients should not add salt during preparation of foods, and they should avoid foods high in sodium. Exercise intensely for 30 minutes every day. Limit alcohol intake to 1 ounce per day.

Substantially reduce fat content in the daily diet. Use salt in food preparation only.

58. Due to client reports of inadequate pain management, the staff has selectedpain management as an area for improvement. Which is the best method forobtaining data about the quality of pain control? Incorrect: A decrease in the administration of pain medications does not indicate that clients have improved pain control.Incorrect: Although a good practice, this method does not provide data concerning the effectiveness of pain management.Incorrect: This information would be difficult to obtain. There is usually no record of the time when a client requests pain medication.Correct: This method measures the effectiveness of the pain medication. Also, data concerning pain management is easy to retrieve. Check client charts for a decrease in the administration of pain medications. Check client records for each client's pain level at the time of medication administration. Check that clients receive pain medications within 30 minutes of the request.

Inquire about each client's level of pain within one hour of the client receiving an oralanalgesic.

59. The nurse is caring for a client who is in labor. It is safe to administer meperidine (Demerol) to the client: Correct: It is important to administer narcotic analgesics during the correct phase of labor because narcotics have a depressant effect on fetal respirations.Incorrect: Meperidine is usually administered for pain management

after the latent phase of the first stage of labor because of the depressant effect of the narcotic analgesic on the infant's breathing.Incorrect: Meperidine should not be administered for pain management during the second stage oflabor because of the depressant effect of the narcotic analgesic on the infant's breathing.Incorrect: Meperidine is not administered during the transition phase of labor because it has a depressant effect on fetal respirations. during the active phase of the first stage of labor. before the latent phase of the first stage of labor. during the second stage of labor. during the transition phase of labor.

60. The nurse is caring for a client with acute renal failure who is at risk forhyperkalemia. Which problem is a clinical manifestation of hyperkalemia? Correct: Potentially life-threatening cardiac arrhythmias are clinical manifestations of hyperkalemia. Other symptoms of hyperkalemia are irregular pulse rate, vomiting, reduced urinary output, and muscle weakness.Incorrect: Leg cramps are clinical manifestations of hypokalemia.Incorrect: Constipation is a clinical manifestation ofhypokalemia.Incorrect: Anorexia is a clinical manifestation of hypokalemia. Cardiac arrhythmias Leg cramps Constipation Anorexia

61. A nurse is asked to evaluate a fellow employee. The questionnaire contains items about quality of client care, cleanliness of the environment, andcompleteness of documentation. This type of evaluation is called: Incorrect: Annual review is the summation of the individual's work, which may include peer review.Incorrect: Continuous quality improvement is a system for creating organization-wide participation in planning and implementinga continuous improvement plan.Correct: Peer review is a process during which a group of practicing nurses evaluatethe quality of another nurse's performance.Incorrect: Shared governance is a method of placing professionalaccountability directly in the hands of each participant. annual review. continuous quality improvement. peer review. shared governance.

62. The nurse is teaching a class to women in the first trimester of pregnancy.

The overall goal of prenatal education is to provide: Incorrect: While learning about breathing techniques is important, the overall goal of prenatal education encompasses much more. The nurse offers information about a healthy lifestyle and nutrition to prevent illness and promote optimum health for the mother and for the developing fetus.Incorrect: While a tour of the maternity unit mayallay anxiety in expectant parents, the goal of prenatal education encompasses much more.Incorrect: While learningconditioning exercises is important, the overall goal of prenatal education encompasses much more. Informationabout healthy lifestyle and support to the pregnant woman and family are important.Correct: Prenatal educationfocuses on providing information, skills, and social support to family members. Clients who receive information earlyduring a pregnancy tend to experience fewer risk factors and lead healthier lifestyles. a method for relaxed breathing during labor. a tour of the hospital maternity unit and explanation of equipment. information about conditioning exercises. information and support for family members.

63. The nurse can most accurately monitor a client for fluid retention by: Incorrect: While measuring fluid intake may be important, it is not an accurate way to monitor if the client is retaining fluid.Incorrect: While measuring urine output is important, it is not an accurate way to monitor if the client isretaining fluid.Incorrect: The intake of foods high in sodium can cause fluid retention. However, this action is not an accurate way to monitor if the client is retaining fluid.Correct: An important sign of fluid retention is weight gain. Seventeen ounces of fluid are retained by the body for every pound the client gains. measuring fluid intake. measuring urine output. recording intake of foods high in sodium.

weighing the client daily.

64. A client who has irritable bowel syndrome is in a bowel retraining program. As part of this program, the client will be instructed to: Incorrect: These foods should be avoided because they are gas-forming, and may cause discomfort.Incorrect: Water intake should not be decreased. Water is needed to balance the stools and prevent constipation.Incorrect: Anticholinergics are used for their antispasmodic effect and should be taken before

meals.Correct: Bran and bulk-forming laxatives help to normalize the stools. Bran pulls fluid into what wouldotherwise be a hard, dry stool and adds bulk to what would otherwise be a watery stool.

add foods such as beans, cabbage, and cauliflower to the diet. decrease water intake to 2 cups a day to avoid diarrhea. take anticholinergic medications 2 hours after meals. take bran or bulk-forming laxatives.

65. The nurse is instructing a client with iron-deficiency anemia about atherapeutic diet. To promote erythropoiesis, this client should increase his intake of: Incorrect: Although boiled eggs have nutritional value as a source of protein, they are not involved in erythropoiesis.Incorrect: Although canned soups can suffice for a meal, they are high in sodium and they are not involved in erythropoiesis.Correct: Red meats, especially liver, are rich in iron. Red meats promote erythropoiesis, which is the production of red blood cells.Incorrect: Although whole milk has nutritional value as a source of protein, it is not involved directly in erythropoiesis. boiled eggs. canned soups. red meats. whole milk.

66. After a gastrectomy, clients usually have a nasogastric (NG) tube in place. Which nursing measure will help to prevent or unplug a clogged NG tube? Incorrect: The nurse should not instill water into the tube unless it is specifically ordered by the primary care provider.Correct: The nurse needs to check regularly that the NG tube is draining properly and is free of kinks.Incorrect: The nurse should not irrigate the tube unless ordered by the primary care provider. Irrigating the tube may disrupt the suture line.Incorrect: Repositioning the tube should be done by the primary care provider to avoiddisrupting the suture line. Administering a bolus of water through the tube every 2 hours to keep it clear Checking that the NG tube is patent Irrigating the tube with saline every shift Repositioning the tube if it is not draining properly

67. The nurse suspects that a preschool-age child may have iron-deficiencyanemia. Which clinical manifestation is produced by iron-deficiency anemia? Incorrect: A high hemoglobin level is a symptom of polycythemia, and not iron-deficiency anemia.Incorrect: Joint pain is associated with sickle cell anemia, and not iron-deficiency anemia.Correct: A lack of dietary iron preventsproper hemoglobin production, which results in pale skin and mucous membranes. Iron-deficiency anemia alsoproduces fatigue and poor muscle tone.Incorrect: Petechiae are a symptom of aplastic anemia, and not of irondeficiency anemia. A high hemoglobin level Pain in joints Pale mucous membranes

Petechiae

68. Which question will elicit the most helpful data when assessing a client'sorientation to reality? Incorrect: This question assesses whether the client is experiencing auditory hallucinations. The question does not determine the client's orientation to reality.Incorrect: This question assesses the client's ability to think abstractly. The question does not determine the client's orientation to reality.Incorrect: This question assesses the client's insight into his or her mental health problems and treatment. The question does not determine the client's orientation to reality.Correct: This question asks the client to identify his or her current location, which is one aspect ofreality orientation. A disoriented client may not be cognizant of location, time, or persons. "Do you hear voices when no one is around?" "How are an apple and an orange alike?" "What does your medication do to help you?" "Where are you right now?"

69. The diagnosis of a sexually transmitted disease (STD) should be reportedaccording to the county health department policy. The purpose of reporting STDs is to: Incorrect: The client may be interviewed by a public health nurse, but this is not the purpose of reporting STDs.Incorrect: The health department will notify sexual partners that they may be infected with an STD. However, the health department cannot disclose the client's identity as this would be a serious breach of confidentiality.Correct: Timely reporting of STD cases to the health department facilitates the rapid notification of sex partners and the detection and prevention of further outbreaks.Incorrect: While the client would benefit from learning about the use ofcondoms, this is not the purpose of reporting STDs. ensure that the client is interviewed by a public health nurse. notify sexual partners and identify the individual who has infected them. prevent further transmission of the disease within the community. provide instruction to the client on the use of condoms during sexual intercourse.

70. Which is the preferred intramuscular injection site for adults and for childrenolder than seven months? Incorrect: The deltoid muscle can be used safely only when injecting small amounts of medication or when other sites are inaccessible. The radial and ulnar nerves and brachial artery lie within the upper arm structure and canbe injured by intramuscular injections in this area.Incorrect: This is not a preferred site for intramuscular injections because of the close proximity of the sciatic nerve, which can be injured by needles or medication.Incorrect: The vastus lateralis is the preferred site for infants under seven months of age. This site should not be used for extremely

thin clients.Correct: The ventrogluteal injection site presents the least amount of risk for nerve damage, tissue necrosis, or pain when compared to other intramuscular sites. Deltoid site Gluteus medius site Vastus lateralis site Ventrogluteal site

71. The nurse is assisting with a screening program for cervical cancer andunderstands that the earliest and most common symptom of this type of canceris: Incorrect: Abdominal or pelvic pain is a late symptom of cervical cancer.Incorrect: Feeling pressure on the bladder or bowel is a late symptom of cervical cancer.Correct: Vaginal bleeding is the first symptom of cervical cancer. In the early stage of cervical cancer, the client may experience spotting after douching or sexual intercourse.Incorrect: Weight loss and generalized wasting occur during the later stages of cancer. abdominal or pelvic pain. feelings of pressure on the bladder or bowel. vaginal bleeding. weight loss.

72. The nurse is teaching a client with chronic bronchitis how to use a metereddose inhaler. After inhaling an aerosolized bronchodilator, the client should hold her breath:

Incorrect: This is not the correct technique when using a metered-dose inhaler. A bronchodilator is used to increase the diameter of the air passage and should normally not elicit a cough.Incorrect: This is not the correct technique when using a metered-dose inhaler. Delivery of aerosolized medication occurs within 3-5 seconds after inhalation.Incorrect: This is not the correct technique when using a metered-dose inhaler. Rapid exhalation leads to increased elimination of aerosolized medication.Correct: When using a metered-dose inhaler, the client should exhaleslowly through pursed lips after holding the breath for a few seconds. The client needs to use the correct technique for maximum benefits from the medication. for a few seconds and exhale slowly through the nose until a cough is elicited. as long as possible and exhale slowly through the mouth. for a few seconds and exhale rapidly through pursed lips.

for a few seconds and exhale slowly through pursed lips.

73. A client is making rude and suggestive comments to other clients on the inpatient unit. To control the client's disruptive behavior, which action should the nurse perform first? Incorrect: Asking other clients to ignore the disruptive behavior will not help the client to deescalate rude behavior. Also, other clients may find it difficult to ignore this person's upsetting remarks.Correct: Separating the clientfrom others will help the client regain control of his disruptive behavior. This is the first action the PN should take in this situation.Incorrect: Enforcing consequences for disruptive behavior may be effective and it can help to prevent future episodes of rude behavior. However, this is not the first action the PN should take.Incorrect: Ordering the client to stop the disruptive behavior will not be effective. The client who is being disruptive usually does not have control over the behavior and cannot stop the behavior at will.

Ask the other clients to ignore the behavior. Assist the client to his room. Take away the client's privileges. Tell the client to stop the disruptive behavior.

74. A nurse is working as a home care nurse on a 12-hour shift. The client requires 24-hour skilled nursing care. When the nurse assigned for the next shiftdoes not arrive after 30 minutes, the nurse should:

Incorrect: The agency needs to be informed that the nurse for the next shift has not arrived. However, leaving prior to the arrival of a replacement is client abandonment.Incorrect: The family may be notified of the problem, but the client requires full-time skilled nursing care. Leaving before another nurse arrives constitutes clientabandonment.Correct: This action informs the agency of the problem so that another nurse can be assigned. The nurse is correctly following the chain of command.Incorrect: Since the shifts are for 12 hours, staying for another shiftmeans the nurse would be on duty for 24 hours, which is not safe. inform the home care agency before going home. notify the family before going home. request the home care agency to provide another nurse. stay with the client for the next shift.

75. When a client's intracranial pressure is increasing, which change usuallyoccurs first? Correct: Increasing intracranial pressure impairs perfusion of brain tissue, which results in decreasing alertness or confusion. If the pressure is not normalized, the client may progress to coma.Incorrect: Pupils dilate withincreased intracranial pressure, but this is a late sign.Incorrect: Increased systolic pressure does occur with increasedintracranial pressure. However, this is a late sign.Incorrect: One-sided weakness may occur, especially with a tumor that increases pressure in one side of the brain. However, this is a late sign of increased intracranial pressure. Decreasing level of consciousness Dilated and fixed pupils Increased systolic blood pressure Weakness of one side of the body

76. A client with a cerebral vascular accident (CVA), also called brain attack, has receptive aphasia. Which communication strategy should the nurse use tocommunicate with this client? Incorrect: Receptive aphasia affects the ability to comprehend both written and spoken language. This language deficit is caused by a disruption in the neuronal pathway, whereby the client is unable to process the message.Incorrect: The client with receptive aphasia does not comprehend words, and therefore cannot use them forspoken or written communication.Correct: Receptive aphasia is caused by damage to Wernicke's area of the brain. The client with this problem cannot comprehend spoken or written language. Aphasias vary in degree and type of deficit.Incorrect: Receptive aphasia affects the ability to comprehend both written and spoken language. Therefore, this is not a helpful communication strategy. With expressive aphasia, a client can understand verbal expression, but is unable to articulate thoughts and ideas in speech or writing.

Allow extra time for verbal responses. Encourage the client to write messages rather than trying to speak.

Encourage nonverbal communication, including gestures and pictures. Rely on written rather than verbal communication.

77. Peptic ulcers have been linked to several causative factors. Which factor isconsidered the primary cause of peptic ulcer disease? Incorrect: Caffeine ingestion is not a primary risk factor for peptic ulcers. However, caffeine is a gastric irritant.Incorrect: Chronic use of aspirin is not a primary risk factor for peptic ulcers. However, aspirin is a gastric irritant.Incorrect: Cigarette smoking is not the primary risk factor for peptic ulcers. However, cigarette smoking may increase the harmful effects of Helicobacter pylori.Correct: This bacterium is responsible for 80 percent of gastric ulcers and 90 percent of duodenal ulcers. Caffeine ingestion Chronic use of aspirin Cigarette smoking Infection with gram-negative Helicobacter pylori

78. The nurse is caring for an elderly client who has been living alone and isdehydrated due to an inadequate intake of fluid. Which sign indicatesextracellular fluid volume deficit (ECFVD)? Incorrect: Blood pressure decreases in clients with ECFVD due to lack of circulating fluid volume.Correct: Heart rate increases in clients with ECFVD. The body tries to compensate for a lack of fluid volume by increasing the rate of circulation.Incorrect: Urinary output decreases in clients with ECFVD due to a lack of circulating fluid volumeand decreased renal perfusion.Incorrect: Neck vein distention is sign of fluid overload and not a sign of fluid volumedeficit.

Increased blood pressure Increased heart rate Increased urinary output Neck vein distention

79. The nurse is instructing a client with constipation. To prevent constipation, the client should: Correct: An adequate daily fluid intake helps to prevent the formation of hard, dry stools.Incorrect: It is not necessary to have a bowel movement every day as long as the stool is soft and moist.Incorrect: Daily use of mineraloil is contraindicated because it may inhibit absorption of fat-soluble vitamins.Incorrect: Suppositories and enemas are harmful on a daily basis because they may cause habitual dependence.

drink two to three liters of fluid every day unless contraindicated. have a bowel movement every day. take 30-60 mL of mineral oil each morning. use suppositories or enemas daily.

80. A 5-year-old child has been admitted to the hospital in sickle cell crisis. Which intervention is essential? Incorrect: Vitamin B12 is given to combat pernicious anemia, not sickle cell anemia.Incorrect: Transfusions of packed cells may be administered if the child's hemoglobin is below 12g/dL. However, whole blood transfusions

are not administered during sickle cell crisis.Incorrect: Anticoagulant therapy is not useful in treating sickle cell anemia.Correct: Adequate hydration, pain relief, and oxygenation to prevent further sickling are primary interventions for clients in sickle cell crisis. Administer vitamin B12 to combat the anemia. Give transfusions of whole blood. Initiate prescribed anticoagulant therapy. Provide adequate hydration.

81. A previously healthy 80-year-old client with no underlying heart or pulmonary disease is hospitalized with lobar pneumonia. Because of the client's age, thenurse should use caution when implementing which order? Incorrect: Bed rest decreases cardiovascular and respiratory demands and conserves energy, which will benefit the client. However, prolonged bed rest can lead to various health complications, including reduced lung

expansion, slower bowel motility, decreased stamina, pressure injury to skin, diminished appetite, and alteration in mood and mentation.Incorrect: Increasing the client's fluid intake will help to liquefy secretions and make them easierto expectorate. This order poses no risk to the client as long as there is no history of congestive heart failure (CHF). Elderly persons tend to have decreased thirst and may be marginally hydrated, especially when ill.Incorrect: Oxygentherapy increases the availability of oxygen to meet cellular demands. Lowflow oxygen therapy poses no risk to theclient. Caution should be used when high-flow oxygen is used for the client with chronic obstructive pulmonarydisease.Correct: The effort it takes for the client to cough may increase shortness of breath and further compromise oxygenation. Thus, the nurse should assist the client with turning, coughing, and deep breathing, while carefully observing the client for dyspnea.

Bed rest Moderate increase in fluid intake Oxygen 2L/nasal prong Turn, cough, and deep breathe

82. The nurse is caring for a client with diabetes mellitus who has a blood sugarof 430 mg/dl. Which drug should the nurse prepare to administer? Incorrect: Although glipizide is a blood-sugar lowering agent, its onset of action is 1 to 1.5 hours. This client's blood sugar is elevated way above the normal value of 70-100 mg/dL, and must be reduced quickly.Incorrect: Glucagon should not be administered because it increases blood glucose. This client's blood sugar is elevated way

above the normal value of 70-100 mg/dL.Incorrect: Although NPH insulin is a blood-sugar lowering agent, its onset of action is 1 to 2 hours. This client's blood sugar is elevated way above the normal value of 70-100 mg/dL and must be reduced quickly.Correct: The client's blood sugar is elevated way above the normal value of 70100 mg/dL. Regular insulin has an onset of action of 30-60 minutes, which should decrease the client's blood sugar. Glipizide (Glucotrol) Glucagon NPH insulin Regular insulin

83. The mother of a 10-year-old child diagnosed with type I diabetes asks thenurse, "Will my child need to give herself injections? Why can't I do the injectionsfor her?" The most appropriate response is: Correct: Children can learn to administer their own insulin once they are 8 years old. Children need to develop skills in monitoring their own blood glucose levels and self-administering insulin.Incorrect: A parent or supportperson needs to be skilled in blood glucose monitoring and insulin administration. There may be times when the child is ill and needs the parent to administer the insulin.Incorrect: Children who are older than 8 can be taught self-caretechniques, including how to inject insulin.Incorrect: Typically, children with type I diabetes require the administration of regular and intermediate-acting insulin. "It is important to encourage your child to take an active role in her own care." "You do not need to learn this technique because your child is old enough to inject herself." "Your child is too young to learn to give herself injections." "Your child will be taking tablets and no insulin."

84. The nurse is working with community services that provide care to thehomeless. An important short-term goal for homeless clients with mental illness is to: Incorrect: Homeless clients often have schizophrenia and other serious mental problems that are characterized by suspicion and paranoia. Therefore, clients who are mentally ill and homeless may be unwilling to meet with a group of people.Correct: Homeless clients with mental illness need to receive health care services in theirown living environment. Health care services should be provided in the streets, homeless shelters, and otherenvironments that are easily accessible for the homeless.Incorrect: Clients with serious mental illness will not benefitfrom insight-oriented psychotherapy. Mental illness is the most common type of disorder found in homeless populations.Incorrect: Homeless clients with mental illness are unlikely to attend or participate in regularly scheduledtherapy sessions. Attending scheduled meetings is an unreasonable expectation for these clients. involve clients in group therapies with other homeless clients. provide health care services in the client's own environment. provide insight-oriented psychotherapy.

schedule weekly individual therapy sessions for clients.

85. A client is recovering from abdominal surgery. The intravenous infusions and antiembolic devices have been discontinued. The client still requires frequentdressing changes due to excessive drainage. Which nursing intervention is costeffective? Incorrect: Ordering supplies for one or two days is appropriate, but not for one week. The excess supplies will be discarded if not used, which is not cost-effective.Incorrect: An ostomy drainage bag is an expensive item and requires a physician's order.Incorrect: All linen in each client's room has to be laundered before it can be used for another client. Laundry is one of the high costs of each unit. Consequently, this is not a costeffective action.Correct: Equipment is charged to the client until it is processed by the supply center. Order one week's supplies for dressing changes and place in the client's room. Place an ostomy drainage bag over the incision. Place extra linen in the client's room so that it will not have to be retrieved before changing the bed. Remove the intravenous infusion pump and antiembolic device, and place them in adesignated area to return to the supply center.

86. A client with systemic lupus erythematosus (SLE) asks the nurse why she is always tired. Which explanation is correct regarding the changes that occur withSLE? Incorrect: A low white blood cell count increases the risk of infection. It is not necessarily the cause of

fatigue.Incorrect: A low platelet count increases the risk of bleeding and bruising. It is not necessarily the cause of fatigue.Correct: A low RBC causes anemia, which creates a decrease in energy.Incorrect: Pain can be emotionally and physically draining. However, it is not necessarily the cause of fatigue. "A low white blood cell count commonly occurs with this disease." "Decreased platelet counts occur in most clients with SLE." "The disease is often accompanied by a low red blood cell count." "The pain associated with your inflamed joints can be very exhausting."

87. The nurse can safely administer oral medications to a client who has: Incorrect: Oral medications should not be administered to clients who are on gastric suction. However, liquid oral medications can be given through a nasogastric tube if suction is not attached.Correct: Clients with intactswallow, cough, and gag reflexes generally have no difficulty taking oral medications.Incorrect: Oral medications should not be administered to clients who are vomiting. However, clients can safely take oral medications during intermittent periods when they are not nauseated or vomiting.Incorrect: Medications may need to be administeredparenterally to clients with sluggish bowel sounds. Alterations in gastrointestinal function, such as reduced peristaltic activity, interfere with oral drug absorption, distribution, and excretion. gastric suction. intact swallow, cough, and gag reflexes. intermittent nausea and vomiting. sluggish bowel sounds.

88. The nurse is caring for a client who has undergone surgical repair of anabdominal aortic aneurysm. While observing for postoperative complications, thenurse suspects a paralytic ileus. A clinical manifestation of paralytic ileus is: Correct: Absent bowel sounds upon auscultation of the abdomen is one indication of a paralytic ileus. Other clinical signs include excessive gas accumulation, enlarged abdomen, and generalized tympany.Incorrect: A bulge or nodule detected in the abdomen upon palpation indicates a hernia, and not a paralytic ileus.Incorrect: Black, tarrystools containing digested blood is an indication of cancer or bleeding in the upper gastrointestinal tract from ulcers orvarices.Incorrect: A humming or swishing sound heard through a stethoscope over the abdomen is called a bruit. Presence of a bruit helps to confirm a diagnosis of aneurysm, not paralytic ileus. absent bowel sounds upon auscultation. an abdominal bulge or nodule. black, tarry stools containing digested blood. a humming or swishing sound upon auscultation of the abdomen.

89. The nurse is teaching a client who has symptoms of mastitis about care ofthe breasts at home. The client should be taught: Correct: It is safe for a client with mastitis to continue breastfeeding. Many lactation consultants advocate continued breastfeeding, because it aids in resolution of the mastitis.Incorrect: It is safe for the woman to takeanalgesics for pain due to mastitis.Incorrect: Women who are on antibiotic therapy for mastitis may continue to breastfeed without imposing harm to the baby.Incorrect: Many women with mastitis use a firm breast support as a comfort measure. to avoid interrupting breastfeeding. to avoid taking analgesics for the mastitis. to stop breastfeeding if antibiotics are prescribed. not to wear a bra until the mastitis resolves.

90. The nurse is instructing a client with cholelithiasis about the importance ofeating a low-fat diet. A food product that is labeled as low-fat contains no more than: Incorrect: A food with this label is not a low-fat food. Clients on a low-fat diet need to check food labels carefully for fat content.Incorrect: Foods with this label are fat-free foods. Clients on a low-fat diet need to check foodlabels carefully for fat content.Incorrect: A food with this label is not a low-fat food. Clients on a low-fat diet need to check food labels carefully for fat content.Correct: Clients on a low fat diet need to check food labels carefully for fatcontent. Labels may also describe a food low in fat as a "low source of fat." Serving sizes on food labels arestandardized to make it easier to compare the nutritional value of similar products. 5 grams of fat per serving. 0.5 grams per serving.

10 grams of fat per serving. 3 grams of fat per serving.

91. The nurse is caring for an elderly client with urinary incontinence. To promotecontinence, the nurse should: Incorrect: Although incontinence briefs may keep the client's clothing and bedding dry, they do not promote continence.Incorrect: Learning Kegel exercises is not the best intervention for this elderly client who is incontinent. While Kegel exercises do eventually strengthen the pelvic floor muscles, it takes time and effort for them to be effective.Correct: Providing the client with an opportunity to void every 2-3 hours promotes regular emptying of the bladder and prevents overdistention.Incorrect: Restricting fluids after the evening meal helps to decrease nocturnalincontinence, but would have no effect on incontinence during the day. have the client wear incontinence briefs. instruct the client in Kegel exercises. provide the client with an opportunity to void every 2-3 hours. restrict the amount of fluid consumed after the evening meal.

92. The nurse is teaching a procedure to a client from a different culture who hasrecently arrived in the United States. To assess the client's understanding of the procedure, the nurse will: Correct: The nurse should ask for a return demonstration to make certain the client understands how to perform the procedure. Clients from other cultures may nod and smile even though they may not understand theprocedure or what the nurse has said.Incorrect: Focusing on the task suggests that the nurse is more interested inthe task than in the client.Incorrect: This action does not ensure that the client knows how to perform the procedure.

Also, the client may have difficulty reading English.Incorrect: Raising the voice does not make words moreunderstandable, and a raised voice may suggest hostility. ask for a return demonstration. focus on the task to be performed. give the client reading materials illustrating the procedure to take home. speak louder so the client will understand what is said.

93. A 52-year-old client experiencing hot flashes and night sweats expresses interest in taking hormone replacement therapy (HRT). Which statement by thenurse provides accurate information about HRT? Incorrect: Hormone replacement therapy (HRT) does not lower the risk of heart disease among postmenopausal women.Incorrect: Only 35-40 percent of postmenopausal women in the United States useHRT.Incorrect: Short-term use of hormones may be associated with risks such as blood clots (especially in women who smoke) and cholecystitis.Correct: Although the findings of the Women's Health Initiative study have demonstrated the risks of HRT outweigh the benefits for the majority of postmenopausal women, recent evidence refutes long-held beliefs supporting the use of HRT. The decision must be individualized in collaboration with the health care provider.

HRT decreases a woman's risk for heart disease. HRT is used by 75-80 percent of postmenopausal women. Short-term use of HRT is not associated with risks. There is controversial evidence regarding risks and benefits of HRT.

94. The Certified Nurse Midwife (CNM) delivers a healthy term newborn. After the cord is clamped, the CNM hands the crying baby to the nursing student forimmediate newborn care. What should the student do first? Incorrect: Identification of the newborn is essential, but it is not a critical need for the newly born, wet baby.Correct: The baby should be dried quickly to prevent evaporation heat loss and chilling. Cold stress can cause

respiratory distress. However, if the baby is not crying, the most important first step is to provide for a patent airway and provide oxygen if the baby is cyanotic.Incorrect: The baby is crying and has a patent airway. Excessive suctioning can stimulate the vagal nerve and cause apnea. Overzealous suctioning should be avoided unless there isan obstruction in the airway or copious mucus in the oral cavity.Incorrect: Weighing the baby is part of the initial physical assessment, but is not the priority for a newly born, wet baby. Obtain footprints and apply the ID bands to mother and baby. Quickly dry the baby. Suction the baby using a bulb syringe to the side of the mouth. Weigh the baby and measure for length.

95. The nurse is providing information about cancer prevention at a health fair. Which statement is correct about reducing the most significant risk factor forlung cancer? Correct: Cigarette smoking is the most significant risk factor for lung cancer, accounting for about 85 percent of lung cancer cases.Incorrect: Environmental pollutants may contribute to lung cancer risk, but it is not possible toeliminate this from daily life.Incorrect: A mask may lesson the chance of a person who is immunosuppressed, such asthose receiving chemotherapy, of acquiring a secondary infection related to airborne agents. However, wearing a mask is not known to decrease the risk of developing lung cancer.Incorrect: Chest x-rays are not designed to screen for lung cancer. Mammograms are an effective radiologic screening test used for the detection of breast cancer for women over 35-40 years of age and those with an increased risk for the disease. Avoid cigarette smoking. Avoid exposure to environmental pollutants.

Wear a mask when working outside. Schedule an annual chest x-ray.

96. The nursing staff is working with a client to reinforce certain behaviors. An indicator that behavior modification strategies are successful is when desiredbehaviors: Incorrect: Reinforcing certain behaviors does not cause those behaviors to continue at the same rate. How frequently a client performs a behavior depends on whether the client receives positive or negativereinforcement.Incorrect: Reinforcing behaviors does not decrease the rate of their occurrence. How frequently a clientperforms a behavior depends on whether the client receives positive or negative reinforcement.Correct: Reinforcingor rewarding certain behaviors causes them to increase in frequency or occur more often. The client who is rewarded for certain behaviors is likely to repeat those behaviors.Incorrect: Ignoring behaviors may cause the behaviors to

stop. How frequently a client performs a behavior depends on whether the client receives positive or negativereinforcement.

continue at the same frequency. decrease. increase. stop occurring.

97. The nurse is working with a client through the phases of a helping nurseclient relationship. Which therapeutic communication technique should be usedduring the working phase? The nurse should: Correct: Helping clients express their feelings is an important component of the working phase of a helping relationship. During the working phase, nurse and client mutually plan the client's care and then activate the careplan.Incorrect: Evaluating goal achievement with the client occurs during the termination phase of the helpingrelationship. During the termination phase, the nurse helps the client re-establish independent living or transfer toother health care services.Incorrect: Assessing the client's health status begins during the orientation phase of the helping relationship. Assessment of the client's physical and mental health should be ongoing throughout the nurseclient relationship.Incorrect: Setting the tone for the nurse-client relationship occurs during the orientation phase ofthe helping relationship. During the orientation phase, the nurse should let clients know that their values arerespected and their opinions are valued. encourage and help the client express feelings about her own health and health goals. evaluate goal achievement with the client. perform an initial assessment of the client's health status. set the tone for the relationship by demonstrating a caring attitude.

98. The nurse is assessing a new client in the neurology clinic. Which nursingobservation describes absence seizures? Correct: This behavior is typical of absence seizures, formerly called "petit mal seizures."Incorrect: The aura and complete loss of consciousness is typical of a tonic-clonic seizure (formerly called "grand mal"), not absenceseizures.Incorrect: Repetitive motor behavior is typical of a complex partial seizure, not absence seizures.Incorrect: Progressive neuromuscular activity is typical of a focal motor seizure (a type of partial seizure), not absence seizures. The client briefly stares into space and does not respond to her name. The client describes feeling funny and then waking up on the floor sometime later. The client repeatedly smacks her lips for several minutes. The client's right eyelid begins to flicker followed by muscle twitches of the right face, arm, and hand.

99. The nurse is meeting a client for the first time. The client, who is angry, reports that his request for pain medication has been ignored by the earlier shift. To improve client satisfaction, the nurse will: Correct: Immediately addressing the client's need for pain medication will decrease the conflict and improve client satisfaction.Incorrect: The nurse should check the record prior to administering pain medication. However, this action will not resolve the client's immediate need for medication.Incorrect: Defending the actions of the previous shiftwill not resolve the client's need for pain medication and may increase client dissatisfaction.Incorrect: Reporting theincident to the nurse manager will not resolve the client's immediate need for pain medication.

ask the client if he still requires pain medication. check the record to see if any pain medication was administered. inform the client that the previous shift was understaffed. report the incident to the nurse manager.

100. The nurse is interviewing clients as part of a skin cancer screening program. Which statement made by a client may indicate the development of skin cancer? Incorrect: This is not a sign of skin cancer. Prolonged exposure to the sun causes increased damage to the epidermal cells. Blistering may develop as a result of severe sunburn.Incorrect: This is not a sign of skin cancer. Flat scaly areas on body parts usually indicate actinic keratosis. These lesions generally develop in older clients on areas that are exposed to the sun.Incorrect: This is not a sign of skin cancer. Desquamation or peeling is a result of

erythema following sunburn.Correct: A mole that changes color to brown or black is a danger signal that can indicate skin cancer.

"I have blisters on my legs that developed after I spent a day at the beach." "I have this flat, scaly area on my neck." "The skin on my nose is drying and peeling off." "This mole on my arm is turning a dark brown color."

101. The nurse is assessing an outpatient client for possible dysfunction of thethyroid gland. Which is an early symptom of hypothyroidism? Incorrect: Always feeling warm is a symptom of hyperthyroidism, not hypothyroidism.Correct: Fatigue is an early symptom of hypothyroidism. Fatigue develops because the thyroid gland is not producing enough thyroidhormone to maintain the metabolic functions of the body.Incorrect: Hand tremors are a symptom of hyperthyroidism, not hypothyroidism.Incorrect: Insomnia is a symptom of hyperthyroidism, not hypothyroidism.

A constant feeling of warmth Extreme fatigue Hand tremors Insomnia

102. The nurse is often required to handle biohazardous materials. Whichstatement about double bagging is accurate? Correct: To correctly double bag contaminated materials, one person needs to be inside of the isolation room and one person needs to be outside of the room. To prevent contamination, the assistant outside of the room holds the double bag with the top edge of the bag folded over gloved hands.Incorrect: Double bagging does not takeplace inside of the isolation room. Contaminated materials should be bagged, sealed, and labeled as biohazardousinside of the room, and then handed to an assistant outside of the room for double bagging.Incorrect: The nurse needs to wear gown, gloves, and sometimes a mask before coming in contact with contaminated materials. The assistant outside of the room should also wear protective clothing.Incorrect: Double bagging of contaminatedmaterials requires two people. One person works inside of the isolation room and one person works outside of the room. Double bagging is performed by an assistant who stands outside of the isolation room. Double bagging is performed inside of the isolation room. Double bagging may be performed without gowning, gloving, or masking as long ascontact with the client does not occur. Double bagging of contaminated materials can be performed by one person.

103. A child is undergoing skin testing for allergies. Why does the child need to remain in the waiting room for 30 minutes after receiving the injection? Correct: If a child is going to have an anaphylactic reaction to an injected allergen, it generally occurs within 30 minutes after the injection. The child will need to have immediate care if a reaction occurs.Incorrect: Skin testing

procedures vary from clinic to clinic. This is not the reason a child needs to wait for 30 minutes following injection of an allergen.Incorrect: Epinephrine is injected if the child has an anaphylactic reaction to the allergen. Epinephrine isnot injected on a routine basis.Incorrect: The time it takes to interpret skin testing varies. This is not the reason a childneeds to wait for 30 minutes following injection of an allergen. A reaction to an allergen usually occurs within 30 minutes. A serum of the child's positive allergens must be prepared and sent home. Epinephrine must be injected 30 minutes after the test to counteract the effects of the test. It takes 30 minutes for skin testing to be interpreted.

104. A client with angina pectoris is taking nitroglycerin (NTG) for pain when an anginal attack occurs. Which symptom indicates that an adverse effect tonitroglycerin is occurring? Correct: A pounding headache is the most common adverse effect produced by nitroglycerin tablets. The client should notify the primary care provider if the headache lasts for more than 15 to 20 minutes followingadministration of nitroglycerin.Incorrect: Dyspnea is not an adverse effect of nitroglycerin.Incorrect: Dizziness may occur after administration of nitroglycerin, but it is not the most common adverse effect. Clients should sit down or lie

down after taking this medication.Incorrect: Pain radiating to the jaw is a symptom of angina. This symptom is relieved, not caused, by nitroglycerin.

A pounding or pulsating headache Dyspnea Dizziness Pain radiating to the jaw

105. The nurse is caring for a client in the third trimester of pregnancy. Which danger sign should the nurse immediately report to the nurse midwife orphysician? Incorrect: Frequent urination is a common discomfort that women experience during the third trimester. This may be related to the pressure of the fetus on the bladder. Another reason for more frequent urination is the relative increase in fluid volume during pregnancy.Incorrect: While leg cramps can be an annoying problem during the third trimester, they are not dangerous. The cause of leg cramps may be low calcium levels. The nurse should educate theclient about dietary changes to increase calcium intake.Correct: A severe headache accompanied by visual disturbances is a warning sign that the client is developing pregnancy-induced hypertension and impendingeclampsia.Incorrect: Vaginal discharge is common during the third trimester and is considered a minor discomfort. If the discharge is greenish in color or malodorous, then the health care provider may evaluate the pregnant woman forinfection. Frequent urination Leg cramps

Severe headache with visual disturbances White vaginal discharge

106. The nurse is teaching a client with rheumatoid arthritis how to maintainmaximum function. Which instruction by the nurse is accurate? Correct: Therapeutic joint and muscle exercises increase joint mobility and range of motion, decrease pain, and increase muscle strength.Incorrect: Prescribed pain medications for rheumatoid arthritis provide the pain reliefthat clients need to function better. Salicylates and nonsteroidal, antiinflammatory drugs do not produce drowsiness.Incorrect: Consistency of daily exercise is important to maintain mobility and function. If pain increaseswith exercise, the client should stop and report the pain to the primary care provider.Incorrect: Assistive devices promote safe mobility and should be used as needed. "Continue to perform the exercises the physical therapist taught you." "Minimize the use of pain medications because they will make you drowsy." "Perform your daily range-of-motion exercises only on your 'good' days." "To develop your strength, minimize the use of assistive devices."

107. A client who is in labor may have a cephalopelvic disproportion (CPD). Which assessment provides information about the risks for the mother and forthe baby? Incorrect: The biophysical profile is a useful assessment during the antepartum period. It is not used to assess CPD.Incorrect: The Nitrazine test tape is a pH sensitive tape that will turn blue upon contact with amniotic

fluid. This test is not used to assess CPD.Incorrect: Non-stress tests are used to assess fetal wellbeing during the antepartum period. They are not used to assess CPD.Correct: An ultrasound examination provides information about pelvic size, shape, and the position of the fetus within the pelvis that is used to evaluate risk due to CPD. Biophysical profile Nitrazine test Repeated non-stress tests Ultrasound examination

108. While providing home care to a disabled child, the nurse is informed by acoworker that the mother entertained a man the previous night. The nurseshould: Incorrect: The nursing staff should not discuss a family member's private life. It is important to respect the rights of clients and families, and not breach client confidentiality.Correct: Team members involved in the client' s careshould be given only relevant information about the client and family status. The nursing staff should not discuss a

family member's private life.Incorrect: This is not the first step to take in discouraging the coworker from disclosing client information.Incorrect: The most effective method to discourage gossip about clients and their families is to not participate. If the coworker continues to gossip about the family, the nurse may need to discuss the issue with the manager.

discuss the mother's relationship because it influences the client's care. inform the coworker that this is information that the nurse does not require to care for the client. inform the State Board of Nursing about the coworker's breach of client confidentiality. report the coworker to the home care manager for violation of confidentiality.

109. The nurse is helping a client with thrombophlebitis apply antiembolic hose. Antiembolic hose are beneficial because they: Incorrect: The application of antiembolic hose is beneficial because it decreases venous hypertension.Correct: Antiembolic hose maintain external compression on the extremity with the blood clot, thereby minimizing venous status and edema. Clients with thrombophlebitis should also elevate the extremity wheneverpossible.Incorrect: Wearing antiembolic hose helps to reduce peripheral edema, but it does not prevent edema. Diuretic therapy is the treatment of choice for peripheral edema.Incorrect: Antiembolic hose promote healing of venous leg ulcers, especially when used in combination with hydrocolloid dressings. If the ulcer fails to respond totreatment, then the ulcer is debrided, and tissue from a donor site is used for skin grafting.

elevate venous hypertension. minimize venous stasis. prevent peripheral edema. promote debridement of venous leg ulcers.

110. The mother of a 4-month-old infant asks the nurse, "What is the best way towean my child from breastfeeding?" Which response by the nurse is mostappropriate? Incorrect: The baby should not receive whole milk until a year old.Incorrect: Once breastfeeding is established, a gradual change to another feeding method is the best way to wean.Incorrect: Breastfeeding is usuallythe best choice for most babies. However, the nurse should not impose a personal opinion on a client.Correct: Gradual discontinuation of breast feedings prevents breast engorgement and pain for the mother. The infant will also be more satisfied if the mother substitutes one breast feeding at a time with formula. "A cup or bottle of whole milk can be substituted for one feeding." "At this age you can just switch over to the bottle without any trouble." "Breastfeeding is best for the baby. You really should continue." "Breastfeeding should be discontinued gradually."

111. The nurse is caring for a client who has postoperative pain and has an orderfor morphine sulfate. The nurse should not administer this pain medication if theclient has: Incorrect: An increased blood pressure may be a response to pain. Morphine may be safely administered to clients with an elevated blood pressure.Incorrect: Increased heart rate is a response to pain. Morphine may be safelyadministered to clients who are experiencing an increased heart rate.Correct: A respiratory rate of 10 is below normal

for the adult client. Morphine may further depress respirations and is contraindicated unless the respiratory rate is ator above normal.Incorrect: Morphine may be safely administered to clients with an elevated body temperature. a blood pressure reading of 148/92. a heart rate of 102. a respiratory rate of 10. oral temperature of 100.4 F (38 C).

112. A client sits quietly through the unit's group meeting. Which statement by the nurse is most likely to encourage the client to interact in the group? Incorrect: It is important to clarify the staff's expectations of client behavior during group. However, in this situation, the client may perceive the PN's comment as a rebuke or chastisement.Incorrect: This comment has an accusatory aspect to it, and it does not promote a therapeutic environment for the client or the group.Incorrect: This comment is confrontational. Clients may not know why they are failing to interact. For example, a client may feelthreatened by the size of the group, and therefore not participate.Correct: The nurse is encouraging the client to

speak by commenting on the client's silence. The nurse is also asking the client to contribute thoughts and ideas to the discussion.

"Every group member is expected to participate in these sessions." "The purpose of group is for clients to share and you have not said a word." "Why aren't you talking and participating in the group?" "You have been very quiet today. What thoughts would you like to share with us?"

113. The nurse is administering antibiotic therapy to a client with an abdominalwound infection. Which finding indicates that the antibiotic therapy is effective? Incorrect: An increased pulse rate can be a sign of infection. The antibiotic therapy is not yet effective in controlling the wound infection.Incorrect: Tenderness and pain are signs of infection. The antibiotic therapy is not yet effective in controlling the wound infection.Correct: Drainage is an indication of infection. Thus, the antibiotic therapy is probably effective if there is no drainage in the wound area.Incorrect: Redness is a sign of infection. The antibiotic therapy is not yet effective in controlling the wound infection. The client's pulse rate is 112 beats per minute. The client reports tenderness around the wound area. No drainage is noted on the incision dressing. The edges of the incision are reddened.

114. The nurse is caring for a client who is receiving peritoneal dialysis. Thenurse needs to assess the client for which complication? Incorrect: Air embolism is a complication of hemodialysis, not peritoneal dialysis.Incorrect: Exsanguination is a complication of hemodialysis, not peritoneal dialysis.Incorrect: Clients on dialysis of any kind are more likely toexperience hypotension as a complication of therapy.Correct: The client with a peritoneal catheter is at risk for peritonitis. This complication may result from improper care of the catheter prior to, during, or after the procedure. Air embolism Exsanguination Hypertension Peritonitis

115. A client is in the terminal stages of an illness. To provide the grieving family with emotional support, the nurse should: Incorrect: Initially, some family members may not want to assist in the client's care. However, the nurse can encourage family members to help once they are emotionally ready.Correct: Grieving family members may react to a loved one's terminal illness with fear or immobilization. Providing information about the hospital and the client's carehelps to empower family members, and may give them a sense of control.Incorrect: The grieving family should be allowed unlimited visitation during the terminal phase of the client's illness. Limiting visitation may only increaseanxiety and grief for the client and family.Incorrect: While grieving, family members should talk with the primary care provider for information and planning for medical care, but it is the nurse that supports the family emotionally. The

nurse's presence and assistance can help the family cope with grief. immediately involve family members in the client's care. inform the family about hospital routine and the client's basic care. restrict family visitation in order to reduce the client's fatigue. suggest that family members contact the primary care provider.

116. Following a thoracentesis, the nurse should position the client: Incorrect: This is not the correct way to position a client following a thoracentesis.Incorrect: This is not the correct way to position a client following a thoracentesis.Incorrect: This is not the correct way to position a client following a thoracentesis. In order to promote optimum lung expansion of the affected lung, the nurse should position the client with the unaffected side down.Correct: The nurse should position the client on the unaffected side for at least one hour following a thoracentesis. This position will promote expansion of the lung on the affected side, and help to prevent pneumothorax. in a prone position for one hour. in semi-Fowler's position for one hour. on the affected side for one hour. on the unaffected side for one hour.

117. The nurse is teaching tracheostomy care to a client who is beingdischarged. Which step of tracheostomy care should the client be taught to dofirst? Incorrect: This is not the first step in tracheostomy care. The soiled ties are changed at the end of the procedure. Clearing the airway to promote effective ventilation and air-gas exchange takes priority over changing a dressing.Incorrect: This is not the first step in tracheostomy care. Clearing the airway to promote effective ventilation and air-gas exchange takes priority over changing a dressing.Incorrect: This is not the first step in tracheostomy care. Clearing the airway to promote effective ventilation and air-gas exchange takes priority over changing a dressing.Correct: The first step during tracheostomy care is to suction the tracheostomy. Suctioning removes secretions that may trigger coughing during the procedure. Change the soiled ties. Cleanse the inner cannula with a hydrogen peroxide and sterile water solution. Remove the old dressing and cleanse the stoma. Suction the tracheostomy.

118. The nurse is caring for a client in a long-term care setting. Which precautionminimizes the client's risk for falls? Incorrect: Oriented clients may require only one side rail to minimize the risk for falls. The side rail should be placed in the up position to support the client who is getting in or out of bed.Correct: Properly fitted, non-skid shoesreduce the risk of the client slipping and falling when ambulating. Elderly clients and those with neurosensorydisabilities are at the greatest risk of falling.Incorrect: The client needs to move slowly when transferring from bed to chair. Getting up too rapidly may cause dizziness and falls due to postural hypotension.Incorrect: A handrail 1/2 inch

from the wall is difficult for clients to grasp. A 2-inch distance from the wall allows clients to grasp the handrail firmly for support. Four side rails up on the client's bed at all times Properly fitted, non-skid shoes Rapid transfer of the client from bed to chair Smooth, slip-resistant handrails installed at least 1/2 inch from the wall

119. The nurse is assessing a single working mother with multiple sclerosis (MS). The woman states she is still working but feels tired all the time. To assess the client's fatigue, which question should the nurse ask next? Incorrect: Exercise is important, but fatigue is an ongoing problem for clients diagnosed with MS. This is not the best initial question to evaluate the client's fatigue.Incorrect: Sleep is important, but fatigue is an ongoing problemfor clients with MS. This is not the best initial question to evaluate the client's fatigue.Incorrect: Nutrition is important, but fatigue is an ongoing problem for clients diagnosed with MS. This is not the best initial question to evaluate the client's fatigue.Correct: This question is helpful because it explores how the client uses her available energy. The nurse can then help the client look at her priorities and make decisions about what is important. For example, preparing evening meals is important but keeping a spotless house may be unnecessary. "Are you exercising regularly?" "Are you getting at least 10 hours of sleep every night?" "Tell me about your usual diet." "What activities take priority when you are at home?"

120. Which is the correct technique for administering chest compressions whenperforming cardiopulmonary resuscitation (CPR) on an adult? Incorrect: This is incorrect because the sternum must be depressed 1.5 to 2 inches for the normalsized adult during CPR.Incorrect: This is incorrect because the landmark notch should be located with the middle and indexfingers.Incorrect: This is incorrect because both hands should be parallel to each other with fingers extended and off of the chest.Correct: For the proper compression technique, the elbows are locked into position, the arms arestraightened, and the shoulders are positioned directly over the hands. This technique ensures that the thrust for each chest compression is straight down on the sternum. Depress the sternum 1 to 1.5 inches straight down. Identify the sternal notch and place hands in the center of the chest. Place hands together, lock fingers, and rest them on the chest.

Thrust downward with locked elbows and straight arms.

121. The nurse is assessing a client who has developed hyponatremia due toprolonged vomiting. Which sign is a manifestation of hyponatremia? Incorrect: Cardiac dysrhythmias are not a manifestation of hyponatremia. Dysrhythmias develop with hypokalemia.Incorrect: Diarrhea is not a manifestation of hyponatremia.Correct: A client with hyponatremia has a lower-than-normal serum sodium level. When the plasma sodium concentration falls, water enters the brain cells as aresult of osmosis. Swelling of the brain cells causes mental confusion and lethargy.Incorrect: Thirst is a manifestation of hypernatremia, not hyponatremia.

Cardiac dysrhythmias Diarrhea Mental confusion Thirst

122. Several new employees are being oriented. Which task is appropriate for thenurse when orienting new nursing staff to the institution? Incorrect: The nurse manager is the best source of this information.Incorrect: The nurse manager is the best person to provide this information. The human resources department is also a good source of information about organizational structure.Incorrect: Assessing the nurse's ability to correctly calculate pharmacology problems is the

role of the education department.Correct: The nurse's role includes documenting the care provided to clients.

Describing the nursing theory used by the institution Describing the organizational structure of the institution Teaching the calculation of drug dosages Teaching the correct method for documenting client care

123. Which statement is true regarding changes in bowel elimination patternsrelated to age? Incorrect: Adolescents experience rapid growth of the large intestine and increased secretion of hydrochloric acid. Therefore, adolescents are at increased risk for altered elimination such as constipation or diarrhea.Incorrect: The ability to control defecation does not occur until the child is 2 to 3 years of age.Correct: Infants have smallerstomachs and more rapid peristalsis. Therefore, infants require more frequent feedings and have a larger number of bowel movements compared to adults.Incorrect: Older adults have decreased muscle tone in the perineal floor andweakness of the anal sphincter. These physiological changes cause older adults to have greater difficulty in controlling defecation than younger adults do.

Adolescents experience very few, if any, bowel elimination changes. Bowel control is usually achieved by 18 months of age. Infants require more frequent feedings and have more bowel movements. Older adults have better muscle tone and fewer bowel problems than younger adults do.

124. The nurse is providing discharge teaching to a client with HIV. Which statement made by the client indicates that the client understands how to preventcomplications? Incorrect: The client with HIV is at risk for oral opportunistic infections. The client should use a soft-bristle toothbrush or soft tooth sponges to decrease oral mucous membrane trauma.Incorrect: The client with HIV requires ahigh-protein, high-calorie, and low-microbial diet. Fresh fruits, vegetables, and undercooked meat may containorganisms that cause infection in the immunosuppressed individual.Correct: The client with HIV is at high risk for infections. Avoiding large crowds during flu season decreases the risk of exposure.Incorrect: Medications used to treat HIV have multiple side effects, which include lack of energy.

"Dental hygiene is important and I should use a hard-bristle toothbrush." "Now that I am on medication for HIV, I can eat fresh fruits, fresh vegetables, and, my favorite, steak cooked rare. "I should avoid crowded places during the flu season." "The only side effect that I can expect from my HIV medication is feeling tired."

125. The nurse is positioning a client with pulmonary edema. Which position willpromote improved respiratory function?

Correct: High Fowler's position promotes greater lung expansion, improves respiratory function, and relieves the client's anxiety.Incorrect: This position will compromise lung expansion, promote a feeling of air hunger, and increase the client's anxiety.Incorrect: This position will compromise lung expansion, promote a feeling of air hunger, and increase the client's anxiety.Incorrect: This position will compromise lung expansion, promote a feeling of air hunger, and increase the client's anxiety. High Fowler's Prone Side-lying on the left side Supine

126. The nurse is caring for a client with osteoarthritis in the lower extremities. Inorder to promote mobility, the nurse should encourage the client to: Incorrect: Ice is not used to decrease swelling in osteoarthritis. Heat should be applied to increase blood flow and decrease pain in the joints.Incorrect: Increased protein can lead to weight gain, which will aggravate the painand inflammation associated with osteoarthritis. Weight reduction helps to decrease the mechanical stress on the joints.Incorrect: Joint rest is recommended to decrease pain in inflamed joints. Active range of motion can cause joint overuse, which contributes to pain and inflammation.Correct: Quadriceps-setting, isometric exercises are standardtherapy for strengthening the muscles needed for ambulation and thus improve mobility. However, recent research cautions that increased quadriceps strength can actually worsen knee osteoarthritis in clients who have malaligned orlax knees apply ice packs to swollen joints twice a day.

increase dietary protein. perform active range-of-motion exercises for inflamed joints. perform quadriceps-setting, isometric exercises.

127. A client has a nursing diagnosis of impaired verbal communication related to removal of the larynx. To facilitate communication in the postoperative period, the nurse should: Incorrect: While it is helpful to consult with the speech therapist for long-term methods of communication, this is not the most important nursing intervention in the immediate postoperative period.Incorrect: Participation in asupport group may assist the client in developing a network of people. However, this intervention is not the most helpful in the immediate postoperative period.Correct: The nurse should provide the client with a work board, pad andpencil, Magic Slate, or computer. These alternative communication devices will help the client express needs and concerns.Incorrect: The nurse should speak to the client in a normal tone of voice. consult with the speech therapist. refer the client to a laryngectomy support group. provide the client with a means for writing. speak slowly and in a louder voice when instructing the client.

128. Which statement is true about AIDS/HIV prevention? Incorrect: Adolescents are particularly vulnerable to AIDS/HIV. The growing rate of sexual activity and sexuality transmitted disease among adolescents makes them particularly vulnerable to AIDS/HIV.Incorrect: AIDS

medication may reduce the risk of a mother passing the virus to a fetus, but it does not reduce transmission betweenadults.Correct: HIV progresses more rapidly in infants and children than in adults. Some adults with HIV may have anincubation period of 10 years.Incorrect: HIV can be transmitted by any exchange of body fluids, includingheterosexual sex. Adolescents are not as vulnerable to getting AIDS/HIV as adults are. AIDS medication reduces the risk of HIV transmission between adults. HIV has a long incubation period of 10 years in some adults. HIV is not transmitted by heterosexual sex.

129. A client who has just been admitted to the hospital for chemotherapy has very poor vision. However, the family states that the client functions independently at home. Which intervention is most useful for preventing fallsduring hospitalization?

Correct: Clients with poor vision can navigate in a setting as long as it remains consistent. Keeping chairs, the overbed table, and equipment in the same place throughout the hospitalization will help the client learn toambulate safely in the room.Incorrect: This instruction prevents the client from functioning independently and imposesunnecessary restrictions.Incorrect: Clients are more likely to be injured climbing over full side rails. Half rails remindclients that they are in an unfamiliar setting and may remind them to call for help when getting out of bed.Incorrect: Leaving the lights on continuously may contribute to sleep deprivation. It would be better to have a nightlight or leavethe bathroom light on in case the client needs to go to the bathroom at night. Advise the staff not to move furniture or equipment in the client's room. Instruct the client not to get out of bed without assistance. Keep full side rails up at all times. Leave the room lights on at all times.

130. The nurse is caring for a client with dementia. Which nursing action willencourage the client to become more independent? Incorrect: This question is too open-ended for a person with dementia to answer. The question asks the client to process information, and provide a response that may be too difficult for a person with impairedthinking.Correct: This nursing intervention promotes independence by encouraging the client to participate in selfcare. The nurse's words give specific directions that help to prompt the client's action.Incorrect: Clients with dementia may be unable to feed themselves because it is too complex a task. The nurse may need to feed this client to preventthe development of nutritional deficiencies.Incorrect: The comment is placing too many demands on a client with dementia. The client may likely perceive this statement as an overwhelming request. Asking the client, "What do you want for breakfast?"

Offering the client a washcloth and stating, "Please wash your hands with this washcloth." Placing a tray of food before the client and stating, "See if you can feed yourself." Saying to the client, "You tell me what you want to do today."

131. Which action will the nurse take when caring for a client who has asynthetic, long-leg, weight-bearing cast? Incorrect: Warm, moist heat will increase swelling and pain, and should not be applied to the fractured area.Correct: Elevating the leg provides support to the extremity, and helps promote venous return to the heart, whichdecreases swelling.Incorrect: Joints that are not immobilized should be exercised. The client may require passive range-of-motion exercises to maintain function.Incorrect: Clients with weight-bearing casts can ambulate and do notrequire bedrest. Synthetic casts dry quickly, and clients can walk on synthetic casts immediately after application. Apply warm, moist heat to the fractured area. Elevate the leg on pillows to the level of the heart. Encourage the client to keep legs and toes still. Maintain the client on bedrest.

132. The team nurse is called to the soccer field for an injured player. The 13yearold boy has severe pain in his knee with localized edema, and the kneeappears deformed. What should the nurse do? Incorrect: This may be appropriate for later treatment, but it is not the appropriate emergency intervention.Incorrect: Applying ice to an injured knee is helpful, but having the child sit on the sidelines with this injuryis not appropriate.Incorrect: This may be appropriate for later treatment, but it is not the appropriate emergency intervention.Correct: The child's knee should be x-rayed to determine if the ligaments around the knee are torn, the

bone is fractured, or the kneecap is displaced. The nurse should also immobilize the child's knee and apply ice. Apply an ACE bandage and give the child crutches to use. Apply ice to the knee and have the child rest on the sidelines. Give the child an anti-inflammatory medication. Take the child for an x-ray examination.

133. Which measure is most likely to help prevent neurosensory hearing loss? Incorrect: Periodic assessments can detect hearing loss, but they do not prevent it.Correct: Longterm exposure to very loud noise is a common cause of neurosensory hearing loss.Incorrect: Cottontipped applicatorsshould not be used for cleaning the external canal because they can force debris further into the ear. They can also traumatize the lining of the canal and contribute to infection/injury.Incorrect: Impacted cerumen can obstruct the external canal causing conductive hearing loss, not neurosensory hearing loss.

Encourage clients to have an annual hearing evaluation. Encourage rock musicians to utilize hearing protection. Instruct children to clean the external canal daily with cotton-tipped applicators. Routinely inspect the external auditory canal for impacted cerumen in older clients.

134. A client on bedrest has a reddened area on the coccyx. Which action by the nurse will help prevent further skin breakdown? Incorrect: Drying agents such as powder or cornstarch should not be applied, because they can cause further skin damage. Mild soap and water should be used to cleanse the skin.Incorrect: Raising the head of the bedincreases the shearing forces on the sacral area, and increases the risk of pressure ulcers. The client should not be placed in a semi-reclining position.Incorrect: Massage of the reddened area is contraindicated. Massaging can furtherdamage the traumatized skin and tissues.Correct: Frequent position changes redistribute pressure on the client'sskin. This intervention increases blood flow to the tissues, and helps prevent further redness and skin breakdown.

Applying powder to the area after bathing Elevating the head of the bed 45 degrees Massaging the reddened area three times a day Turning and repositioning the client every 2 hours

135. A newborn baby has rectal temperature of 97.1 degrees F (36 degrees C). What should the nurse do in response to this finding? Incorrect: Monitoring is necessary, but monitoring will not help to raise the temperature of a baby in cold

stress.Correct: A radiant warmer will reduce heat loss by convection, and will help to return the baby's temperature to within the normal range.Incorrect: The initial bath should be postponed until the baby has a temperature in the normalrange. Evaporative heat losses and cold stress can cause complications, such as respiratory distress, and should beavoided.Incorrect: Wrapping the baby in blankets and adding a cap will not substantially help to raise the temperatureof the chilled newborn to the normal range.

Continue to monitor the baby. Place the baby under a radiant warmer set at 98.6 degrees F. Quickly bathe and dry the baby to prevent evaporation. Wrap the baby in double blankets and a knit cap.

136. A few days after giving birth, a mother states that she is feeling sad, irritable, and very tired. The nurse concludes that the mother may be experiencing: Incorrect: Clinical depression develops in clients who are not new mothers and who have never been pregnant. However, a history of psychiatric illness is a risk factor for postpartum psychosis.Correct: Postpartum blues are characterized by irritability, sadness, and fatigue that quickly resolve. Clients may develop these symptoms duringthe first few days after delivery. Fifty to 80 percent of all postpartum clients experience postpartum blues.Incorrect: Postpartum depression is characterized by persistent symptoms of sadness and inadequacy that occur 2-6 weeksafter birth. Ten to 20 percent of all new mothers experience postpartum depression.Incorrect: Postpartum psychosis is characterized by delusions, cognitive changes, and extreme emotional lability. The disorder may progress to mania

in women who do not receive adequate treatment. The disorder is rare, and develops in only 1 to 2 percent of new mothers.

clinical depression. postpartum blues. postpartum depression. postpartum psychosis.

137. Nurses use standard or universal precautions for clients in all clinicalsettings. Which activity is a component of standard precautions? Incorrect: Single-use items are desirable when implementing standard precautions. However, disposable items are not always practical because of the expense. Non-disposable items in the client's room should be properlycleaned and reprocessed according to facility guidelines.Incorrect: A private room is preferable, but it is not required for the implementation of standard precautions. However, a client with an infectious disease that may contaminate theenvironment will require a private room.Incorrect: Nurses need to wear masks and eye protection or face shieldswhen client care activities may generate splashes or sprays of blood or body fluids.Correct: Clients in all settings area potential source of infection, regardless of their diagnosis. Therefore, all care providers need to wear gloves to prevent contact with a client's blood or body secretions.

Disposable equipment should be used in the client's room. Hospitalized clients should be assigned to private rooms to maximize standard precautions. Nurses should always wear masks and eye protection when providing personal care to clients. Nurses should wear gloves when touching a client's blood, body fluids, secretions, excretions, and non-intact skin.

138. The PN is caring for an elderly client with Alzheimer disease who is confused, and who wanders into the rooms of other clients. Which client needhas the highest priority? The need for: Incorrect: All clients who are in pain need intervention. However, the relief of pain is not the most important need for elderly clients who are confused.Incorrect: Reality orientation is an important need for elderly clients who areconfused. However, it is not the priority need.Incorrect: It is inappropriate and illegal to restrain an elderly person who is confused. Restraints may cause the client to become more confused and agitated.Correct: Safety is the priorityneed for elderly clients who are confused. These clients also need reality orientation, redirection, comfort, and pain relief. pain relief. reality orientation. restraint. safety .

139. The nurse is discharging a client following total hip replacement. Whichstatement indicates that the client understands home care instructions?

Incorrect: Rocking chairs should be avoided after hip replacement. Rising from a rocking chair can cause a greater than 90-degree flexion of the hip. This flexion can cause movement, stress, and injury to the hip.Incorrect: After discharge, clients should continue to place a pillow between their knees when lying down. The pillow prevents adduction of the hip, a greater than 90-degree flexion of the hip, and injury to the hip.Incorrect: Assistive devices suchas walkers are usually needed for only a short period of time. Clients are generally able to resume normal activities ofdaily living in 3 months.Correct: A raised toilet seat prevents flexion of the hip greater than 90 degrees. If a client risesfrom a low seat, hip flexion increases. This amount of movement and stress can injure the hip. "I can't wait to get home to sit in my rocking chair." "I will be glad to get rid of that pillow between my knees when I lie down." "I'll have to use this walker for the rest of my life." "My wife had a raised toilet seat installed in the bathroom."

140. The nurse is caring for a postoperative client who is at risk for atelectasis. Asign of atelectasis is: Correct: When atelectasis develops, the lungs do not expand fully with deep inspirations. Thus, breath sounds are not detectable at the bases of the lungs.Incorrect: Respiratory rate and pattern are increased inatelectasis, because it is more difficult for the client to meet the need for oxygenation.Incorrect: Chest expansion isdecreased in atelectasis, because there is decreased chest movement on the affected side.Incorrect: Atelectasis mayresult from the production of thick, tenacious secretions. This client is most likely to have a productive cough. decreased or absent breath sounds at the bases of the lungs. decreased respiratory rate and pattern. increased chest expansion. non-productive cough.

141. While transcribing the physician's orders, the nurse discovers an error thatcould potentially harm the client. How should the nurse respond to this situation? Incorrect: The nursing supervisor may offer an opinion concerning the physician's order. However, only the physician who wrote the order can provide final clarification of the order.Incorrect: The physician's nurse cannot legally verify or correct the physician's order without consulting with the physician. It is the physician's responsibility toclarify the order.Incorrect: Nurses may be held liable for harm that comes to clients due to implementation of incorrectorders. A nurse should never carry out a physician's orders if she suspects an error.Correct: The nurse needs to

speak directly to the physician who wrote the order. The physician can then decide if an error has been made and rewrite the order if necessary.

Ask the nursing supervisor to review and clarify the order. Call the nurse in the physician's office and ask for clarification of the order. Carry out the order since nurses are not liable for damages that may occur. The nurse should contact the physician who wrote the order to confirm and/or clarify the order.

142. Which statement is true regarding the application of topical medications tothe skin? Incorrect: A gauze dressing may be applied over areas being treated with topical medications. Dressings help to prevent soiling of the clothes and wiping away of the medication.Incorrect: Liniments are applied by rubbingthem gently but firmly onto the skin.Incorrect: Rubbing is contraindicated because it may cause irritation of the skin. Lotions are spread lightly and evenly onto the skin surface.Correct: Prior to applying topical medications, skinencrustation should be removed by carefully cleansing the area to be medicated. Skin encrustations can interferewith contact of the medication with the skin. Dressings should not be applied over area being treated with topical medications. Liniments are applied by dusting them lightly onto the skin. Lotions and creams are applied by firmly rubbing them into the skin. Skin encrustations are removed before applying topical medications.

143. Which client is at greatest risk for pulmonary embolism? A client: Correct: Immobility promotes venous stasis, which increases the risk of clot formation.Incorrect: NSAIDs

increase the risk of bleeding, and are not known to increase clot formation.Incorrect: Aspirin decreases platelet aggregation and thus, decreases the risk of clot formation.Incorrect: A low body weight does not increase the risk for clot formation. who is immobile. receiving NSAID therapy. receiving aspirin therapy. with low body weight.

144. Intraocular pressure is measured during a routine eye examination to detectglaucoma: Incorrect: Glaucoma is a painless disease that causes blindness if not treated early. Many clients experience no symptoms until the problem is discovered during a routine examination.Incorrect: Glaucoma is a chronic conditioncaused by increased intraocular pressure. While this disease is progressive, it can be controlled with medication andsurgery.Correct: Because glaucoma does not produce symptoms, clients are unaware of the increased intraocular pressure that gradually causes permanent damage to the eye. Routine screening may detect increased intraocularpressure. The client can start treatment before vision loss occurs.Incorrect: Increased intraocular pressure due to glaucoma is not related to high blood pressure. before it becomes painful. during the early stages when it can be cured. early, before vision loss begins. in clients with mild hypertension.

145. The nurse is performing a closed, intermittent bladder irrigation for a client.

When implementing this procedure, the nurse should: Correct: Easy access to needed supplies and equipment prevents possible contamination of supplies and equipment when performing the procedure.Incorrect: The solution should be injected into the catheter port with a sterile needle. This nursing action maintains the integrity of the closed system and prevents contamination of the system.Incorrect: Sterile normal saline is the solution of choice for bladder irrigation because it is similar to plasma, and decreases client discomfort.Incorrect: Rapid administration of irrigation solution is contraindicated because itincreases the risk for bladder spasms. Irrigation solution should be injected slowly into the bladder. arrange supplies and equipment at the bedside or on the overbed table. disconnect the catheter from the drainage tubing when ready to instill the solution. draw sterile water into the syringe in order to have the solution ready for instillation. Inject the sterile solution rapidly into the urinary bladder.

146. The PN who is working on a pediatric unit is qualified to be the primarynurse for which client? Correct: PNs may be the primary nurse for clients who are in stable condition.Incorrect: Caring for a client receiving blood-brain barrier disruption therapy is beyond the scope of PN practice.Incorrect: Caring for a client whois in an unstable condition is beyond the scope of PN practice.Incorrect: The administration of chemotherapy is beyond the scope of PN practice. A 3-year-old client who is immobilized following surgery for hip dysplasia A 7-year-old client receiving blood-brain barrier disruption therapy for an astrocytoma A 10-year-old client requiring neurological checks every 30 minutes for a closed head injury A 14-year-old client receiving chemotherapy for leukemia

147. The nurse is planning nursing care for a client following amputation of a leg. Which intervention is the initial step for helping the client accept alterations inbody image? Correct: This is the initial step in helping the client adjust to an altered body image. Establishing a trusting relationship allows the client to express fears and concerns related to the amputation and an altered bodyimage.Incorrect: While passive range of motion exercises improve mobility, they will not initially promote acceptance of body image.Incorrect: This is not the initial step in caring for a client with an altered body image. However, an amputation support group may help the client later in the treatment program.Incorrect: Teaching self-care activities is not the initial step in helping the client adjust to an altered body image. However, self-care is an important later step inthe treatment program. Establish a trusting relationship with the client. Perform passive range of motion on the amputated limb. Refer the client to an amputation support group.

Teach the client self-care activities.

148. When caring for the client receiving chemotherapy, the nurse understands that the most serious side effect of antineoplastic drugs is: Correct: Bone marrow suppression is a life-threatening complication that places the client at high risk for bleeding and infection.Incorrect: Nausea and vomiting are common side effects of chemotherapy, but they are not themost serious.Incorrect: Phlebitis may develop when chemotherapy is administered through peripheral veins. However, advances in delivery have minimized this side effect.Incorrect: Stomatitis may occur as a side effect of chemotherapy, but it is not a life-threatening condition. bone marrow suppression. nausea and vomiting. phlebitis at the infusion site. stomatitis.

149. The nurse determines that a client with depression is improving when she: Incorrect: Giving away personal possessions may indicate that the client is planning suicide, and it may therefore be a dangerous warning signal.Incorrect: The lack of pleasure in activities one formerly enjoyed (anhedonia) is a sign that the client's depression is not improving.Correct: Increased social interaction is a sign thatthe client's depression is lifting.Incorrect: Early morning awakening (EMA) is a sign that the client has depression thatis not improving. begins to give away her possessions.

reports lack of pleasure in activities previously enjoyed. spends more time interacting with others. states that she awakens at 3 AM most mornings.

150. A client newly diagnosed with Addison's disease is preparing for discharge from the hospital. As part of the discharge instructions, the nurse will teach the client how to: Incorrect: Blood-glucose monitoring is performed for patients with diabetes mellitus, not Addison's disease.Incorrect: A high-carbohydrate, high-protein diet with adequate sodium is recommended for clients with Addison's disease.Incorrect: Glucagon is a medication used for the emergency treatment of hypoglycemia. Client'swith Addison's disease require intravenous hydrocortisone for emergency treatment.Correct: A client with Addison's

disease (adrenal insufficiency) will need to take corticosteroids for life. The adrenal gland does not produce enough cortisol to sustain the body's metabolic functions. Providing medication instruction is the most important aspect ofdischarge teaching for clients with Addison's disease.

perform blood-glucose monitoring. prepare a low-fat, low-protein diet. self-administer emergency glucagon. take corticosteroid medications.

151. The nurse is planning care for a 16-year-old client diagnosed with Hodgkin'sdisease. Which nursing diagnosis is the nurse most likely to find on the client'scare plan? Incorrect: Hodgkin's disease does not impair gas exchange.Incorrect: Hodgkin's disease does not cause fluid volume problems.Incorrect: Hodgkin's disease produces painless enlarged lymph nodes.Correct: The course of treatment for Hodgkin's disease is long and difficult. Some adolescents with this disease live from day to day wondering if symptoms will reoccur.

Gas exchange, impaired Fluid volume deficit Potential for acute pain Risk for powerlessness

152. The nurse is assessing a 16-year-old girl diagnosed with anorexia nervosa. Which symptom will the nurse expect to observe? Correct: Clients with anorexia nervosa may have arrested or delayed sexual development due to extreme malnutrition.Incorrect: Clients with anorexia nervosa may experience hypothermia, not hyperthermia.Incorrect: Mostpatients with anorexia nervosa have hypotension, not hypertension.Incorrect: Clients with bulimia nervosa may havea normal body weight, while clients with anorexia nervosa have a very low body weight.

Amenorrhea Hyperthermia Hypertension Normal to slim weight range

153. Which activity can reduce a man's risk for cancer of the prostate? Correct: The American Cancer Society recommends annual screening for prostate cancer beginning at age

50. Clients should begin screening at age 40 if they have other risk factors for cancer. The most effective screening procedures are digital rectal examination and the prostate-specific antigen (PSA) test.Incorrect: Cigarette smoking isnot a risk factor for prostate cancer. Major risk factors related to prostate cancer include family history, history of sexually transmitted diseases, a high-fat diet, and a sedentary lifestyle.Incorrect: Vasectomy will not decrease the riskfor prostate cancer.Incorrect: TSE will not reveal the development of prostate cancer, but it can reveal early testicular cancer.

Annual screening beginning at age 50 Entering a smoking cessation program Having a vasectomy before age 35 Performing a monthly testicular self-examination (TSE)

154. When counseling a client with coronary artery disease who is on a lowsodium diet, the nurse will encourage the client to eat which foods? Correct: Fresh fruits are low in sodium and are nutritious. The USDA's Food Guide Pyramid recommends two to four servings of fresh fruit daily.Incorrect: Muffins and processed bran cereal are high in sodium, and are not included on a low-sodium diet. Clients on low-sodium diets should use low-sodium baking powder for baking muffinsand other items.Incorrect: Instant potatoes are high in sodium and are not included on a low-sodium diet. The client can eat boiled potatoes, which are low in sodium.Incorrect: Tomato juice and V-8 juice are high in sodium and are not included on a low-sodium diet. Other juices, such as apple, orange, grapefruit, and grape, are recommended in small

amounts.

Fresh fruit Bran muffins Instant potatoes

Tomato juice

155. When caring for hospitalized clients and observing standard (universal) precautions, the nurse does not need to wear clean gloves when: Correct: Gloves are not required when caring for a client with intact skin. Gloves are required when touching body fluids, excretions, secretions, mucous membranes, and non-intact skin.Incorrect: Sterile gloves are required inthis situation to reduce the transmission of infection-causing bacteria.Incorrect: Gloves are required during exposureto body fluids for the protection of the nurse from infectious diseases.Incorrect: Sterile gloves are required forinsertion of an indwelling urinary catheter device. Clean technique prevents the ascent of contaminant bacteria intothe urinary tract. bathing a client with intact skin. changing a dressing with drainage. changing linens of an incontinent client. draining a urinary catheter bag.

156. The nurse is performing a postpartum assessment. The client has a positiveHoman's sign, which may indicate: Incorrect: Involution is the return of the uterus to the nonpregnant state following birth. This process is not indicated by a positive Homan's sign.Incorrect: A positive Homan's sign does not indicate nerve damage. It is a clinical finding that may indicate thrombophlebitis. The nurse needs to report this finding immediately.Incorrect: Apositive Homan's sign may indicate inflammation, but not infection.Correct: A positive Homan's sign is characterized by pain in the calf or behind the knee when the client's foot is dorsiflexed. It is an early sign of femoralthrombophlebitis and the nurse needs to report the finding immediately. involution. popliteal nerve damage.

postpartum infection. thrombophlebitis.

157. A mother with gestational diabetes has just delivered a newborn baby. To prevent newborn hypoglycemia, the nurse should anticipate having to: Incorrect: Infants born to diabetic mothers should not require intensive care if they receive regular feedings, and are closely monitored for hypoglycemia.Correct: Feeding the infant at regular intervals maintains a stable glucoselevel. Jitteriness and lethargic behavior indicate hypoglycemia in the newborn.Incorrect: Infants born to diabeticmothers develop hypoglycemia, not hyperglycemia. The use of insulin is not indicated.Incorrect: Infants born to diabetic mothers usually do not receive a high-calorie formula during the first 24 hours. Early enteral feeding withstandard 20 calories/ounce formula or breast milk is usually effective in maintaining a normal blood glucose level. arrange to transfer the infant to the Intensive Care Nursery. monitor the infant's blood glucose levels and provide enteral feedings. monitor the infant's blood glucose levels, and provide sliding-scale coverage with insulin. provide the infant with 27 calories/ounce formula for the first 24 hours of life.

158. Two clients diagnosed with myasthenia gravis have been admitted. One is inmyasthenic crisis; the other is in cholinergic crisis. Which initial assessment iscritical for both clients? Correct: Both myasthenic crisis and cholinergic crisis result in profound muscle weakness, thus both clients are at risk for hypoventilation. As a result, both may require mechanical ventilation. Therefore, it is critical to assess respiratory status immediately.Incorrect: It is important to assess level of consciousness, but it is not the most critical

initial assessment.Incorrect: It is important to assess pupil activity, but it is not the most critical initial assessment.Incorrect: Urine output and characteristics are not affected by complications of myasthenia gravis. Breathing and oxygenation Level of consciousness Pupil response to light Urinary specific gravity

159. A client reports that he is hearing voices that are condemning him. Which is the best response from the nurse?

Incorrect: This response demonstrates doubt or questioning, and hints that the nurse does not believe or acknowledge the client's experience.Incorrect: Telling the client that the voices are not real fails to acknowledge the client's experience. This response may provoke an argument, because the voices are real to the client.Incorrect: While it is important to acknowledge the client's experience, it is not therapeutic to question whether the clientconverses with the voices.Correct: This response clarifies reality for the client and still acknowledges the client's feelings. "Are you positive you hear voices? Maybe you are hearing noises from the TV." "Certainly you realize that these voices are not real." "Did you listen to the voices and talk with them?" "I don't hear any voices, but I think it would be frightening to hear voices condemning me."

160. Which task may be delegated by the nurse to unlicensed assistivepersonnel? Incorrect: This assignment is beyond the scope of practice for unlicensed assistive personnel.Correct: A client who is two days post-surgery will usually be active and involved in self-care. Unlicensed assistive personnelcan safely bathe this client.Incorrect: This assignment is beyond the scope of practice for unlicensed assistive personnel.Incorrect: This assignment is beyond the scope of practice for unlicensed assistive personnel. Assessing a client during the immediate postoperative period. Assisting a client who is two days postoperative with bathing. Discussing discharge planning needs with a family. Regulating the infusion rate of continuous bladder irrigation.

161. The nurse is assessing an adult client's vital signs. Which assessmentfinding requires further evaluation? Incorrect: Usually there is a variation between apical heart rate and radial pulse, and thus this is a normal finding. An excessive pulse deficit (difference between heart rate and radial pulse) may indicate an alteration in cardiac output.Incorrect: An elevated pulse rate is an expected finding in an adult client experiencing acute pain. Acute pain and anxiety increase sympathetic stimulation, which increases heart rate.Correct: A weak, thready radial pulse that is difficult to palpate is an abnormal finding in an adult. This finding may indicate disease or the development of an emergency such as hemorrhagic shock.Incorrect: The normal respiratory rate for an adult is between 12-20 breaths per minute. Normally, respirations are slower during sleep. Thus, a respiratory rate of 14 breaths during sleep is a normal finding. An apical heart rate of 78 beats per minute and a radial pulse of 77 beats per minute A radial pulse of 102 in a client who reports acute pain A weak, thready radial pulse that is difficult to palpate A respiratory rate of 14 breaths per minute during sleep

162. When teaching a paraplegic client to use a wheelchair, the nurse stresses the need to shift body weight every 15 minutes because: Incorrect: Muscle spasms may occur in paraplegia, but they are not caused by a failure to change

positions.Incorrect: While repositioning does help maintain arm strength, it is not the main reason for repositioningwhile in a wheelchair.Incorrect: Muscle mass in the legs can be maintained only by active exercise, which is not possible with paraplegia.Correct: Changing positions improves circulation to the buttocks and sacrum, which preventsskin breakdown. remaining in one position causes muscle spasms. repositioning strengthens the arms, which makes it easier to transfer. repositioning will maintain muscle mass in the legs. shifting body weight helps prevent skin breakdown.

163. A nurse is assessing an African-American client who is depressed. Which factor is a major obstacle to culturally-sensitive assessment and care? Correct: Stereotyping is an unsubstantiated assumption that all people of a certain race, gender, ethnicity, or socioeconomic background are alike. It is important for the nurse to overcome stereotyping when caring forclients.Incorrect: Although it is helpful for nurses to recognize how their own cultural beliefs differ from those of their clients from other cultures, it is not a major barrier to delivering culturally sensitive care.Incorrect: A nurse who accepts cultural differences can provide culturally sensitive care to clients from different cultures. It is not necessary to belong to the same cultural group.Incorrect: Nurses cannot be expected to understand the values of every culture

they might encounter in delivering care. It is certainly helpful to understand a particular client's cultural values, but it isstill possible to be culturally sensitive without a detailed knowledge of each culture.

The tendency to stereotype clients A lack of interest in one's own cultural heritage A personal ethnicity different from that of the client A lack of understanding of the client's cultural values

164. Proper technique for giving oral hygiene to a client on bedrest includes: Incorrect: Foam swabs are useful for removing debris from the mouth, but do not effectively remove plaque from the teeth.Incorrect: Dentures are cleaned more effectively when removed. If the client feels uncomfortable aboutremoval, the nurse should offer the client privacy during removal, and then return to take the dentures to the sink for cleaning.Incorrect: Although hydrogen peroxide provides foaming action, it can irritate oral mucosa and alter the normal flora of the mouth, predisposing the client to infection. Normal saline solution is a better choice.Correct: Calledthe sulcular technique, this method of brushing the teeth not only removes plaque, but also cleans effectively under the margins of the gums.

using foam swabs to remove plaque from the teeth, as they are less abrasive to the gums than toothbrushes are. brushing dentures while they are in place in the mouth, as some clients feel selfconscious about removing their dentures.

using hydrogen peroxide to cleanse the oral mucosa of comatose clients, as its foaming action will help mobilize adherent debris in dried secretions. cleaning the teeth by angling the toothbrush 45 degrees to the teeth and gums, as this will help clean under the gingival margins.

165. Preschool children who are developing normally usually engage in whichtype of play? Correct: Children in the preschool years play together in a similar activity. Their play is not organized, and there are no rules or responsibilities.Incorrect: School-age children engage in cooperative play. In cooperative play, children play within an organized structure.Incorrect: Observational play is typical of infants. The baby intently watches others play, but does not actively engage in play activities.Incorrect: Parallel play is typical of the toddler years. This play style has two children playing side by side, but seldom interacting with each other.

Associative play Cooperative play Observation Parallel play

166. The nurse is caring for a client who is receiving an intravenous infusion of 5percent dextrose in water. After multivitamins are added to the infusion, the clientreports severe itching. The client is most likely experiencing:

Correct: An allergic reaction may develop following the administration of any intravenous medication, including vitamins. The nurse should immediately notify the primary care provider because severe itching is a clinical sign of allergy.Incorrect: The client's symptom of itching is not consistent with fluid-volume overload. Signs of fluid overload include shortness of breath and abnormal breath sounds.Incorrect: The client's symptom of itching is not consistent with intravenous infiltration. Intravenous infiltration occurs when a needle or catheter penetrates throughthe vein and fluid leaks into the surrounding tissues, producing swelling.Incorrect: The client's symptom of itching is not consistent with phlebitis. Signs of phlebitis include a swollen, warm, reddened intravenous site that is tender tothe touch.

an allergic reaction. fluid volume overload. intravenous infiltration. phlebitis.

167. A client is concerned about sexual relationships following a mastectomy. Which nursing diagnosis will the nurse most likely find on the client's care plan? Incorrect: This client may develop a body image disturbance due to the loss of a breast. This nursing diagnosis does not specifically address the woman's concerns about her sexuality following a mastectomy.Correct: A

woman who expresses concern about her sexuality following mastectomy is at risk for altered sexuality patterns. The client may report difficulties, changes, or limitations in sexual activities or behavior, as well as feelings of loneliness and loss.Incorrect: Skin integrity may be impaired following a mastectomy, and it may impede sexual relationships. This nursing diagnosis does not specifically address the woman's concerns about her sexuality followingmastectomy.Incorrect: Low self-esteem and negative feelings may occur following a mastectomy. This nursing diagnosis does not address the client's concerns about her sexuality.

Body image disturbance Risk for altered sexuality patterns Risk for impaired skin integrity Situational low self-esteem

168. The nurse is obtaining a health history from a client with asthma. Whichinformation is most helpful to know when planning interventions to prevent futureasthma attacks? Incorrect: While it is important to determine the client's understanding of the disease state and anticipated interventions, it is more important that the nurse initially obtain a thorough health history. For healthier living, the nurse and client together may plan ways to modify the home environment and avoid precipitating asthmaticevents.Incorrect: Although the definitive diagnostic workup usually includes pulmonary function tests, initially the nurse's primary role is to obtain a thorough health history for the client suspected to have asthma. This includes information pertaining to the home environment, precipitating events, frequency, quality, and duration of asthmatic

episodes.Incorrect: A positive history for asthma can provide a clue for identification of this health condition in personswith clinical symptoms. While family history is important, initially the nurse's role in obtaining a thorough health historymust include information about the client's home environment as well as discussion about precipitating events, frequency, quality and duration of asthmatic episodes.Correct: It is important to identify environmental triggers for asthma attacks such as cigarette smoke, cat hair, dust, and pollen. This information will help the client make changesin the home and work environment, and thus decrease the risk of an asthma attack. The client's knowledge about asthma The results of pulmonary function tests Family history of asthma Known triggers for asthma attacks

169. The nurse is interviewing parents who have just brought their child to theemergency department. The nurse should suspect child abuse when: Incorrect: The absence of the parent does not indicate that the child has been abused.Incorrect: The child's responses may be caused by pain and fear of the emergency department setting. The child's responses should be assessed, but they may not indicate abuse.Correct: Health care providers should suspect abuse when children haveinjuries that are not explained by their parents or caretakers.Incorrect: Parents may be upset by their child's injuriesand respond inappropriately to questions about the injuries. Their inappropriate responses in the emergency department do not necessarily indicate abuse. the parents leave during the examination. the child appears to be frightened. the child's injury is not consistent with the history of the injury. the parents behave inappropriately.

170. A nurse floated to a telemetry unit is assigned a client who is receiving an investigational medication. The charge nurse instructs the nurse to titrate theintravenous infusion to maintain a systolic blood pressure of 90 mm Hg. Thenurse should: Incorrect: This action is beyond the PN's scope of practice.Correct: Administration of investigational medication is beyond the PN's scope of practice.Incorrect: This is not sound practice, because the medication may have effects the nurse may not recognize.Incorrect: This action is patient abandonment and grounds for termination. ask another nurse to provide a simple method to calculate the medication dosage during titration. ask to be assigned to another client who is not receiving an investigational medication. provide care for the client, but request that the charge nurse titrate the medication. refuse to care for the client and go home.

171. A confused client with a head injury and multiple fractures repeatedly tries to get out of bed. A nursing assistant suggests applying a vest restraint. How should the nurse respond? Incorrect: Although nurses must recognize the client's rights, this client is confused and cannot make rational decisions. The staff will need to use safety measures other than restraints to protect the client.Correct: The confused client with a head injury is likely to resist restraints, which can increase intracranial pressure.Incorrect: Restraints willbe harmful in this case because client resistance can raise intracranial pressure. When restraints are used, clientsmust be monitored closely to prevent injury.Incorrect: Any type of restraint is likely to be resisted by the client, which could increase intracranial pressure. Therefore, restraints are not generally recommended for these clients. "If the client insists on getting up, that is his decision." "Restraints are not generally recommended for clients with head injuries." "Restraints can be applied as long as the client is monitored closely." "Wrist restraints are more appropriate when clients have head injuries."

172. The nurse is assisting with an influenza immunization clinic. A client asks about the need for taking the vaccine. Which is the correct response from thenurse? Incorrect: The flu vaccine is recommended for all adults, especially those with a chronic disease.Correct: Prevention through immunization is the best measure to avoid infection with the influenza virus.Incorrect: The flu vaccine is recommended for all adults, especially those over the age of 65.Incorrect: The flu vaccine is recommendedfor all adults, especially health care workers. "If you do not have a chronic disease, the vaccine is not necessary." "The influenza vaccination is the best way to avoid the flu."

"Vaccination is not recommended for individuals over 65." "Vaccination is not recommended for health care workers."

173. The nurse is monitoring the blood glucose level of a hospitalized client. Theclient's blood glucose finger stick result reads 60 mg/dL. Which action should thenurse take? Incorrect: The nurse should not administer regular insulin because insulin will lower the client's blood sugar further.Incorrect: The nurse should not wait to contact the primary care provider because the client's blood sugar is dangerously low.Correct: The client's blood sugar is dangerously low. The nurse needs to give the client a concentrated sweet such as orange juice immediately, after verifying that the client is conscious and is unlikely to aspirate.Incorrect: The nurse should not wait to contact the supervisor because the client's blood sugar is dangerously low. Administer regular insulin according to the sliding scale. Call the primary care provider and await further direction. Give the client a rapid-acting glucose. Notify the nursing supervisor.

174. The nurse is instructing a client who is being discharged following a vaginalhysterectomy. To reduce the risk for long-term complications, the nurse should provide the client with information about: Correct: The client needs to strengthen these muscles to prevent leaking of urine or difficulty with sexual intercourse.Incorrect: Care of dressing and drains is not a part of the care following a vaginal hysterectomy.Incorrect:

Sequential compression devices are used during the immediate postoperative period and are not a concern atdischarge.Incorrect: The incentive spirometer is used during the immediate postoperative period, and it is not a concern at discharge. exercises to strengthen the abdominal and pelvic floor muscles. the care of dressings and drains. use of sequential compression devices. use of the incentive spirometer.

175. The nurse is caring for a client who has bipolar disorder and is receivinglithium carbonate (Eskalith, Lithane). Lithium is classified as:

Correct: Lithium is a mood-stabilizing medication that is prescribed to decrease mood swings in clients with bipolar disorder. Severe mood swings may range from depression to mania.Incorrect: Antianxiety medications are given to reduce stress and tension in clients who are anxious or have panic attacks.Incorrect: Antidepressant medications are used to elevate a depressed mood. They are prescribed for clients with clinical depression.Incorrect: Antipsychotic medications are given to decrease hallucinations in clients with psychosis such as schizophrenia.

a mood-stabilizing medication. an antianxiety medication. an antidepressant medication. an antipsychotic medication.

176. The nurse is caring for a disoriented 75-year-old client who is a Jehovah'sWitness and needs a blood transfusion. The client's chart contains advance directives that prohibit administration of a blood transfusion. The nurse canexpect the hospital staff to: Incorrect: It is not necessary to contact social services concerning advance directives or a living will.Incorrect: Advance directives should not be disregarded unless it is a matter of life and death, and there is doubt about the client's commitment to the tenets of this faith.Correct: If the client is unable to communicate his wishes, then it is appropriate to follow his advance directives and not administer blood. Advance directives are legally binding documents.Incorrect: It is not legal to honor the family's request to nullify the advance directives.

contact social services to confirm advance directives. disregard advance directives and administer the blood. follow the client's advance directives and his wishes. honor the family's request to nullify the advanced directives.

177. A client is scheduled for an intravenous pyelogram to evaluate for the presence of renal calculi. This procedure is contraindicated in the client who has: Correct: The contrast media in this procedure contains iodine. Clients who have an allergy to shellfish are assumed to be allergic to iodine.Incorrect: The hypersensitivity (allergy) to latex is not a contraindication for this procedure.Incorrect: A previous history of renal calculi is not a contraindication for this procedure.Incorrect: Although accessing an intravenous site may cause increased discomfort for the client, it is not a contraindication.

an allergy to shellfish. a hypersensitivity to latex. a previous history of renal calculi. small veins that are difficult to access.

178. To measure central venous pressure (CVP), the nurse must make certain that the zero mark on the manometer is positioned at the level of the client's:

Incorrect: Positioning the zero mark of the manometer at the level of the client's left atrium will produce an inaccurate reading.Incorrect: Positioning the zero mark of the manometer at the level of the client's left ventricle willproduce an inaccurate reading.Correct: Positioning the zero mark of the manometer at the level of the client's rightatrium will ensure an accurate reading. CVP monitoring is used to assess intravascular fluid status.Incorrect: Positioning the zero mark of the manometer at the level of the client's right ventricle will produce an inaccuratereading.

left atrium. left ventricle. right atrium. right ventricle.

179. A client who is diagnosed with Parkinson's disease reports, "Sometimes I can't get my legs to work when I start to walk. It's like they are frozen." Whichsuggestion would be helpful to the client? Incorrect: This symptom is common with Parkinson's disease, but there are some strategies that can help the client cope with the problem.Incorrect: This symptom is not an adverse effect of levodopa. It is a symptom of Parkinson's disease that may be managed with some simple strategies.Correct: This strategy is often helpful when the client cannot initiate voluntary movement.Incorrect: The client may eventually need a walker or wheelchair for

mobility, but other interventions should be tried first. Continued activity helps the client remain ambulatory as long as possible.

"This is common with Parkinson's disease, and there is nothing you can do about it." "This problem may be caused by taking too much levodopa." "Try walking in place and looking in the direction you want to go until you are able to step forward." "You should consider using a wheelchair instead of trying to walk."

180. A client has diabetes mellitus. Which step is correct when measuring blood glucose levels from the client's finger stick blood sample? Incorrect: The site should be prepared for puncture by cleansing with an alcohol swab, or by washing the site with soap and water.Incorrect: Lowering the client's arm increases blood flow to the site.Incorrect: Most monitoring devices require only a small sample of blood. Excessive squeezing may increase pain andbruising.Correct: The side of the finger is the preferred site for puncture when obtaining a blood glucose sample. The side of the finger is less responsive to pain than the pad of the finger is. Cleanse the puncture site with an antibiotic swab. Hold the client's arm above his head for 30 seconds. Squeeze the client's fingertip to obtain a large sample of blood. Use the side of the client's finger for puncture.

181. During discharge of a newborn baby girl, the nurse instructs the parentsabout the best way to prevent sudden infant death syndrome (SIDS). Whichstatement by the nurse is accurate?

Incorrect: Breast-fed babies do have a lower SIDS rate than formula-fed babies. However, this statement does not instruct the parents in the best method for preventing SIDS.Incorrect: There is a strong correlation between exposure to smoke and the occurrence of SIDS in infants. However, this statement does not instruct the parents in the best method for preventing SIDS.Correct: The incidence of infant death from SIDS has decreased since parents have been taught to position infants on the back instead of the stomach.Incorrect: Infants of closely spacedpregnancies do have a higher occurrence of SIDS. However, this statement does not instruct the parents in how to prevent SIDS. "Breastfeeding lowers the risk of SIDS." "Do not allow anyone to smoke in your house or around the baby." "You should place your baby on her back to sleep." "Your baby is more at risk for SIDS since you also have a 1-year-old."

182. The nurse is preparing a client to undergo paracentesis for chronic ascites. Which statement by the client indicates that he understands the procedure? Incorrect: Clients with ascites are usually maintained on a potassium-sparing diuretic and a sodiumrestricted diet to reduce further ascites.Correct: The client understands that his breathing will improve after the procedure. The removal of abdominal fluid will reduce pressure on his diaphragm.Incorrect: Clients with chromic ascites may need to have the procedure repeated because the abdominal fluid usually returns.Incorrect: Abdominal paracentesis may be performed as an outpatient procedure. "I can stop taking my diuretic medicine after I have this procedure done."

"I will be able to breathe better after this procedure is over." "I will be glad to get this procedure over with, so that I won't have to have it done again." "I will have to be admitted to the hospital to get this procedure done."

183. The nurse is monitoring the fetal heart rate while caring for a mother inlabor. Which tracing on the electronic fetal monitor indicates a non-reassuring pattern? Correct: Loss of initial accelerations indicates that the fetus is being stressed by labor.Incorrect: Shallow early decelerations commonly result from pressure on the fetal head, and they are seen in most vaginal deliverytracings.Incorrect: Short-term variability is the beat-to-beat variability seen with internal monitoring, and it is a sign offetal well-being.Incorrect: Variable decelerations are a benign rhythm pattern, and it indicates fetal well-being. To some degree, this pattern indicates good fetal reserves and ability to respond to uterine contractions.

Loss of initial accelerations Shallow early decelerations

Short-term variability Variable decelerations

184. The nurse is planning a diet for a woman with gestational diabetes. The mealplan should include: Incorrect: The client should eat three regular balanced meals a day to prevent hypoglycemia, and an evening snack to prevent morning ketonuria.Incorrect: Simple carbohydrates should be limited in the diabetic diet. They are readily absorbed by the body and can cause hyperglycemia.Correct: Complex carbohydrates, especiallysoluble dietary fibers, lower carbohydrate absorption and improve insulin sensitivity.Incorrect: Concentrated sugars should be eliminated from the diabetic diet because they cause hyperglycemia. two meals a day and no snacks. simple carbohydrates. complex carbohydrates. concentrated sweets.

185. The nurse is caring for a client with a pneumothorax who has chest tubes inplace. Which nursing action will help prevent recurrence of the pneumothorax? Incorrect: Coiling tubing loosely may keep tubes from kinking. However, it is not an effective measure to decrease the recurrence of pneumothorax.Incorrect: This position promotes comfort for the client. However, it is not an effective measure to decrease the recurrence of a pneumothorax.Correct: An air tight seal around the chest tube is essential to keep air from leaking back into the pleural space.Incorrect: Sanguineous drainage would be expected

with a hemothorax, not a pneumothorax. Coil the extra chest tubing loosely. Keep the head of the bed elevated at a 45-degree angle. Maintain a tight seal around the chest tube with a dressing. Monitor the chest tubes for sanguineous drainage.

186. The nurse observes shortness of breath in a client who is receiving totalparenteral nutrition (TPN). The nurse should immediately: Incorrect: Although it is important to monitor glucose levels in clients receiving TPN, this is not the nurse's first action in this situation.Incorrect: Documenting findings is necessary, but this is not the nurse's first action in this situation.Correct: A complication of TPN is congestive heart failure (CHF), which may result from fluid overload. Shortness of breath may be an early sign of CHF.Incorrect: Weighing clients with TPN is important to monitor weight loss or gain. However, this is not the nurse's first action in this situation. check the client's blood sugar by finger stick. document the finding on the clinical record. report the finding to the RN. weight the client.

187. The nurse is preparing a client to go home and is teaching the client aboutself-care. Which technique will help to increase the client's ability to learn? Correct: Clients who have visual or hearing disabilities will find it difficult to learn. The PN should make

certain that these clients are wearing glasses or hearing aids before initiating instruction.Incorrect: Although a cool, comfortable room may be important for the learning environment, it does not necessarily improve a person's ability tolearn. However, the nurse should ensure that the client feels comfortable and free from pain before starting instruction.Incorrect: The nurse should ideally teach the client one topic per learning session. Presenting 3 or more topics per session may confuse the client and delay learning.Incorrect: The nurse should keep technical medical terms to a minimum when teaching, unless the client is a health professional. The nurse needs to define terms for theclient and present examples that clarify instruction. Make certain that the client is wearing glasses or using a hearing aid if indicated. Place the client in a cool, comfortable room prior to instruction. Present no more than 3 topics at a teaching session. Use the correct terminology when explaining procedures.

188. A nurse is meeting with the family of a client who has a serious mental illness. Families should be included in discussions about the client because:

Incorrect: Family members need reassurance that clients with serious mental illness can live independent and fulfilling lives if properly treated.Incorrect: Family members who are included in the planning and care of a lovedone with mental illness are less likely to deny that the client needs psychiatric treatment.Correct: Family involvementis critically important to the success of treatment for a client with mental illness. Meeting with the family also givesfamily members an opportunity to express the anger, fears, helplessness, or concerns they may feel.Incorrect: Families need to understand that mental illness is treatable. Important therapies for mental illness include medication, psychotherapy, group therapy, and social or vocational interventions. clients with serious mental illness may require 24-hour care by the family. families tend to deny the seriousness of the client's mental illness. family involvement in the client's care is a vital part of the total treatment program. some clients may have mental illnesses that are too serious to treat.

189. The nurse is orienting a client to an inpatient unit. Which is the best way toprovide a new client with structure and consistency? Incorrect: Criticizing the client's behaviors may cause the client to feel angry and defensive. This is not an appropriate nursing action.Incorrect: Discussing penalties could make a new client feel anxious and vulnerable. This nursing action does not promote structure, consistency, or a therapeutic environment.Correct: Clients feel more secure in a new environment when they are provided with the rules and regulations they will need to follow. Clients also need to understand the staff's expectations for their behavior.Incorrect: Isolating a new client may cause the individual to feel anxious and alone. When possible, it is best to help a new client socialize with other individuals onthe unit. Criticize negative behaviors.

Discuss penalties for disruptive behavior. Explain the rules and regulations on the unit. Isolate the client from other clients.

190. Which statement is true concerning routine childhood immunizations? Correct: Studies indicate that many parents believe that the immune system can be weakened by too many immunizations, and that children receive more vaccines than are necessary.Incorrect: The average toddler has 10 to 12 colds a year. Children with slight cold symptoms should receive immunizations at health maintenance visits to maintain the immunization schedule.Incorrect: Children who are immunosuppressed should not receive the live virusvaccines.Incorrect: There is no need to start a routine immunization series over when a child misses a scheduled appointment. Children may not receive immunizations because their parents have misconceptionsabout immunity. Children should not receive immunizations if they have a stuffy nose, but no fever. Children who are immunosuppressed should receive live virus vaccines. Children who miss scheduled times for immunizations must start the series over again.

191. A client with chronic renal failure has a dialysis access site in his left forearm. The nurse is teaching the client about home care of the access site. Which statement by the nurse is correct? Incorrect: Blood pressure should not be taken in the arm with the access site, as it may impair circulation and increase the risk of clotting.Incorrect: Jewelry may be restrictive, impair circulation to the access site, andincrease the risk of clotting.Incorrect: The client is at risk for infection. Redness

and swelling at the access site shouldbe reported immediately to the primary care provider.Correct: Blood specimens should be obtained from this client's right arm. A tourniquet should not be applied to the left arm because it would decrease blood flow through the access site and increase the risk of clotting. Blood pressure may be taken in either arm. Jewelry can be safely worn on the arm with the access site. Redness and swelling at the access site should be reported within 24 hours. The right arm should be used to obtain blood specimens.

192. The nurse is caring for a client with a highly irregular heart rhythm. Theelectrocardiogram (ECG) confirms that the client has atrial fibrillation. This clientis at risk for:

Incorrect: Atrial fibrillation is not associated with an increased risk for angina pectoris.Correct: The risk of stroke increases five-fold with atrial fibrillation. The atria are not contracting effectively, which can result in the formation of a blood clot or thrombus. The thrombus can then break loose and travel to the brain, causing stroke.Incorrect: Atrial fibrillation is not associated with an increased risk for heart block.Incorrect: Ventricular tachycardia is associated with a myocardial infarction, not with atrial fibrillation. angina pectoris. a stroke. heart block. myocardial infarction.

193. The nurse receives a client request for pain medication at the same timeanother client is returning from surgery. The nurse should: Incorrect: A client returning from surgery requires immediate assessment by a nurse.Correct: A client returning from surgery requires immediate assessment by a nurse.Incorrect: A client returning from surgery requires immediate assessment by a nurse.Incorrect: Unlicensed assistive personnel are not permitted by state law toadminister pain medication. administer the pain medication first because it will only take 5 minutes. ask another nurse to administer the pain medication so the client returning from surgerycan be assessed immediately. ask the unlicensed assistive personnel to put the postoperative client in bed while thenurse administers the pain medication. obtain the pain medication and ask the unlicensed assistive personnel to administer itwhile the nurse checks the postoperative client.

194. The nurse is instructing a client who is taking diuretics to reduce edema, and who has developed hypokalemia as a result. An appropriate food choice for this client is: Correct: Bananas provide a good dietary source of potassium. Other potassium-rich foods include: oranges, peaches, raisins, dates, dried apricots, strawberries, cantaloupe, almonds, and potatoes.Incorrect: Chicken is not agood dietary source of potassium.Incorrect: Canned soups are not a good dietary source of potassium, and they arehigh in sodium, which causes fluid retention.Incorrect: Lunch meats are not a good dietary source of potassium, andthey are high in sodium, which causes fluid retention. bananas. broiled chicken. canned soups. lunch meats.

195. A client is admitted to the acute care facility for probable acute appendicitis. The nurse caring for the client should: Incorrect: Laxatives and enemas are contraindicated because they may trigger or complicate a rupture of the appendix.Incorrect: Use of a heating pad is contraindicated because the warmth may increase inflammation andthe risk of a rupture of the appendix.Correct: The client is kept NPO in case the client does have an acute appendicitis and requires surgery. Restriction of food and fluids prior to surgery prevents vomiting and the aspiration of vomitus into the lungs during anesthesia.Incorrect: Semi-Fowler's position is the most comfortable position for aclient with appendicitis.

administer a laxative or enema. place a heating pad on the client's abdomen for comfort. place the client on NPO status. position the client in a flat, prone position for comfort.

196. The nurse is caring for a client who has undergone a modified radicalmastectomy and refuses to look at the incision. Which nursing diagnosisaccurately addresses the client's problem? Incorrect: The client may find it difficult to admit the loss of a breast, but this is not the appropriate nursing diagnosis in this situation.Correct: This client may experience a disturbed body image related to the loss of a sexuallysignificant body part. Many women develop concerns about their appearance and femininity following mastectomy

and find it difficult to look at their incision.Incorrect: The client may grieve over the loss of a breast, but this is not the most appropriate nursing diagnosis in this situation.Incorrect: The client should not be labeled as noncompliant because she is not ready to look at her incision. This is not an appropriate nursing diagnosis.

Denial Disturbed body image Dysfunctional grieving Noncompliance

197. A primigravida in her 1st trimester of pregnancy states, "My husband is acting oddly about my pregnancy. He says he's happy, but he rarely talks about the baby." Which response by the nurse provides the best explanation of the father's behavior? Correct: Even with planned pregnancies, fathers may experience ambivalent feelings. Becoming a new father is a major role change.Incorrect: While this statement is essentially correct, it makes light of the mother'sconcerns and it is not the best statement.Incorrect: This statement does not explain the father's viewpoint, and may provoke anxiety. The father's absence is not an indication that he does not care about his family.Incorrect: This statement is the nurse's opinion, and it may cause the client to become more anxious. Also, there is no proven link between ambivalence and difficulties with bonding. "Fathers are often ambivalent during the early months of pregnancy. Is there anything

else you can tell me about his feelings?" "I wouldn't worry about it. Almost all new dads are like that, and they eventually come around." "I noticed that the baby's father isn't with you today. I can see why you're worried." "The baby's father may have difficulty bonding with the baby."

198. The nurse is caring for a 3-month-old infant with a congenital heart defect. Which intervention will help to prevent impairment of growth? Incorrect: High-calorie formula should be used to provide adequate calories, not thickened formula. Thickened formula is given to treat gastroesophageal reflux.Incorrect: Larger feedings will tax the child's body andpromote overexertion. It is best to provide the infant with smaller feedings at more frequent intervals.Correct: Spacingactivities reduces the workload on the infant's heart, and conserves energy. Consequently, more calories will be available to the infant for growth and development.Incorrect: Supplemental feedings are beneficial and provide theinfant with more energy for growth and development. Add cereal to feedings to thicken the formula and add calories. Give larger feedings to allow more caloric intake. Organize activities and feedings to allow the infant to conserve energy. Restrict supplemental bottle feedings.

199. Which nursing action promotes comfort for the client with second-degreeburns of the hand and arm? Incorrect: This is not the correct time to administer pain medication to a client who is undergoing a burn dressing change.Correct: Giving the pain medication 30 minutes prior to the dressing change allows time for themedication to take effect. Burn dressing changes are often painful procedures.

Because of the medication, the client'scomfort will be increased during the procedure.Incorrect: Topical antibiotics are used to decrease infection in the burnarea, not to decrease pain.Incorrect: Family members may be asked to leave during dressing changes to decreasethe risk of infection. Administering the pain medication immediately following the dressing change Administering the prescribed pain medication 30 minutes before dressing changes Applying a topical antibiotic cream to the burn area before cleansing Encouraging family members to stay with the client during dressing changes

200. When caring for the client with chronic bronchitis, which nursing diagnosishas the highest priority? Incorrect: Alveolar destruction is associated with emphysema, not chronic bronchitis.Incorrect: Altered nutrition may be a problem for this client, but it is not the nursing diagnosis with the highest priority.Incorrect: Activity intolerance may cause discomfort for the client, but it is not the nursing diagnosis with the highest priority.Correct:

Ineffective airway clearance has a high priority for clients with chronic bronchitis. Increased mucous production and impaired ciliary action results in ineffective airway clearance and reduced air-gas exchange.

Impaired gas exchange related to alveolar destruction Altered nutrition, less than body requirements, related to dyspnea Activity intolerance related to dyspnea on exertion Ineffective airway clearance related to increased production of thick mucus.

201. The nurse is caring for a client who has a firm uterus following delivery. Which condition is the most likely the cause for excessive lochia or bleeding? Incorrect: A boggy uterus is a uterus that is soft, not firm. A soft uterus allows the vessels at the placental implantation site to bleed.Incorrect: A distended bladder may cause lochia to be less than expected during recovery. The uterus cannot contract fully when the bladder is distended.Incorrect: Subinvolution results from infection, whichcauses the uterus to enlarge. Subinvolution is more likely to occur following the recovery period.Correct: When aclient has a firm uterus, excessive bleeding may be due to a laceration that was not repaired after delivery. Lacerations may be perineal, vaginal, or cervical.

A boggy uterus A distended bladder Subinvolution of the uterus

Vaginal or cervical lacerations

ATI. MENTAL HEALTH

1. An elderly client admitted to an acute care facility is agitated, quarrelsome, angry, and assaultive with peers. Risperidone (Risperdal) is prescribed for the client. Whichbehavior should the nurse use to evaluate the effectiveness of the medication? Incorrect: Drowsiness is a side effect rather than a therapeutic effect of risperidone.Incorrect: Akathisia is motor restlessness and is an adverse reaction of antipsychotics. This is a component of the extrapyramidal side effects of these medications.Correct: Risperidone is an antipsychotic medication that has a more rapid onset than haloperidol (Haldol) and has fewer extrapyrmidal symptoms. Anger and assaultive behavior are the target symptoms that the medication is intended to diminish. Relaxation is a therapeutic effect along with an interest in surroundings, improvement in self-care, and the increased ability to concentrate.Incorrect: It is unlikely that the client will becomeeuphoric and hyperactive with the use of risperidone.

Drowsiness Akathisia Relaxation Euphoria

2. A client experiencing a dissociative fugue engages in unexplained, intentional

wandering. Which other characteristic occurs with this dissociative disorder? Incorrect: Memory loss associated with a fugue is transient. The individual has no recollection of the activities related to the fugue.Incorrect: The person who experiences a fugue often maintains the ability to perform ordinary tasks, however, has no memory of past identity or wandering activities.Correct: A dissociative fugue is the state of purposeful wandering or travel, often with an assumed identity. The inability to recall personal information is typical. A fugue is similar to dissociative amnesia; both are triggered by a traumatic event or severely disturbing situation.Incorrect: The language use of the person experiencing a dissociative fugue is clear, coherent and appropriate. This individual engages in purposeful wandering in response to a traumatic event. He/she appears lucid, in spite of the common adoption of a new identity and lack of recollection for past events.

Permanent memory loss Failure to perform ordinary tasks Inability to recall personal identity Incoherent language use

3. A client tells the nurse on the night shift that she cannot sleep because a man is in her room. The client points to a coat hanging in the closet and says, "There he is!" Whichthought process disturbance is the client experiencing? Correct: An illusion is a misinterpretation or distortion of an object present in the environment. In this scenario, the coat represents an illusion.Incorrect: A hallucination is a false sensory impression that does not exist in

reality. Most hallucinations are visual or auditory.Incorrect: Confabulation is a fabrication of events or situations to fill in the gaps in memory, usually in a plausible way.Incorrect: A delusion is an intensely-held, false belief that cannot be corrected by logical reasoning. Delusions of paranoia are defined by excessive suspiciousness; grandiosity ischaracterized by exaggerated self worth; reference is evidenced by statements of unrealistic associations; andpersecution relates feelings of unjust action, thought, behavior or judgement.

Illusion Hallucination Confabulation Delusion

4. A client comes to the emergency department immediately after rape and assault. She exhibits symptoms of anxiety, fearfulness, and uncontrollable crying. Which intervention should the nurse implement immediately? Correct: Rape is an act of violence that can leave deep emotional scars related to anxiety, fear, distrust, physical injury, and sexual dysfunction. The primal need for need for safety is a critical issue. The nurse interacting with the victim can provide reassurance of safety from the attacker and tangible strategies to achieve protection fromharm. It is common for victims of rape to feel responsible for the sexual assault. The nurse's role is vital in providing support to the client who experienced the physical, emotional and spiritual violation of the self.Incorrect: A psychiatric referral is indicated but is not the initial intervention.Incorrect: Sexually transmitted diseases and pregnancy areinappropriate topics to address in the initial phase after a rape. The discussion of these matters will increase the victim's anxiety. It is more appropriate for the nurse to initially provide emotional support and reassurance.Incorrect: Conducting a physical assessment for injury and calling the police is appropriate, but is not the initial nursingintervention. The physical assessment may be perceived as invasive by the client if she is not emotionally prepared and ready for this to occur.

Reassurance of physical safety and emotional support are indicated prior to physical assessment and police report. Reassure the client that she is safe and that she is not to blame for the sexual assault. Assess the client for signs of post-traumic stress disorder and request a referralfor psychiatric treatment.

Administer medication to prevent sexually transmitted diseases and providepregnancy counseling. Perform a complete physical assessment to determine the extent and nature ofinjury and notify the police.

5. A client diagnosed with antisocial personality disorder exhibits which behavior? Incorrect: The infliction of harm to oneself is an essential feature of borderline personality disorders. Another salient feature of the disorder is the pattern of unstable interpersonal relationships, mood lability, impulsive behavior and the disruption of self-image.Incorrect: Suspiciousness is a primary symptom of a paranoid personality disorder. Mistrust of the fidelity of a spouse or sexual partner is common. Misperception of the intentions and regard of othersis a characteristic of this personality type. They are often preoccupied with thoughts of presumed malevolence by well-meaning persons.Correct: The salient feature of the individual with an antisocial personality disorder is the disregard for the feelings and rights of others. The individual with antisocial personality disorder is extremely manipulative, has no social conscience, and freely uses others for personal gain. A predominant pattern of criminality and substance abuse is typical.Incorrect: The salient feature of histrionic personality disorder is the extreme emotional nature of behavior and drive for attention. The client with a histrionic personality disorder demonstrates traits of vanity, self-indulgence and is prone to emotional outbursts.

Self-destructive activities Suspiciousness and hostility Manipulation of other people for personal gain Emotional lability, extroversion and dramatic actions

6. In order to provide a safe environment for the client diagnosed with delirium, whichnursing intervention is most appropriate? Correct: Delirium represents a group of cognitive disorders that occur as a result of physiologic, infectious, metabolic or toxic disease affecting brain metabolism and function. The disorder characterized by disorientation, impaired cognitive function, memory impairment and confusion. The speech is incoherent. The delirious person needs a quiet environment, supervision, and orientation to surroundings with reduction of stimuli andhazards.Incorrect: Frequent stimulation should be avoided for persons with delirium. Vital signs and neurologic assessments should be performed frequently. The nurse can obtain a history and assess symptoms as well as assist in obtaining laboratory and other diagnostic data for the confirmation of the diagnosis.Incorrect: Complete darkness isnot appropriate and will increase disorientation and confusion. To reduce combativeness, the use of restraints should be avoided.Incorrect: Visitation of family and friends may be appropriate, but stimulation should be minimized. The use of sedatives may alter the client's level of consciousness, worsening the condition.

Reduce stimuli and provide supervision and reality orientation. Provide hourly stimulation and perform frequent neurological and vital sign assessments.

Keep client's room in total darkness and apply restraints, as needed. Restrict visitation of family and friends and administer sedation, if necessary.

7. A client is agitated and refuses to take the prescribed oral medication. The nurse tells the client that if he will not take the medication orally, it will be administered by injection. Which act has the nurse committed? Correct: Assault is the threat of force or harm. The nurse has threatened the client to administer medication in a more painful route than warranted as a punishment for the undesired behavior.Incorrect: Although this situation may be interpreted as malpractice since the nurse acted inappropriately in a professional role, assault is more clearlyexhibited.Incorrect: Battery describes an intentional act of commission of harm or maltreatment. If the nurse actuallyadministered the medications to the client by injection when the oral route was indicated and preferred by the client, then battery describes the nurse's action.Incorrect: Negligence is the act of omission resulting in harm.

Assault Malpractice Battery Negligence

8. The nurse and client with anxiety disorder are in the orientation phase of a therapeuticrelationship. Which is the most appropriate behavioral outcome resulting from this phaseof the relationship?

Incorrect: Although the client may have some interest in discovering the root of his emotional problems, relief from anxiety is the most important outcome of therapy.Incorrect: Reduction or elimination of anxiety is the priority for care.Incorrect: The client may want to be perceived as mentally healthy, but the priority for therapy is a reduction in anxiety.Correct: The priority for care of the client with anxiety is a reduction in the level of anxiety with the ability to recognize and effectively cope with anxiety in future.

Learn the cause of the client's emotional problems and maladaptive behavior. Prevent emotional problems through lifestyle and behavioral changes. Have the family and friends perceive the client as mentally healthy. Experience relief from anxiety and learn new coping strategies.

9. A preadolescent girl has greater physical growth, including breast development, in comparison to peers. Recently, she has become the subject of teasing by the neighborhood boys regarding her physical shape. The remarks cause a great deal ofanxiety and emotional hurt. Which type of crisis is the preadolescent experiencing? Incorrect: A family crisis typically involves the loss or addition of a family member. A change in family dynamics or altered morale among the members may mark a family crisis.Correct: A developmental (maturational) crisis may occur at a time of normal maturational transition or age-related change. The individual who experiences a

dysfunctional coping pattern and is overwhelmed by the adaptation that must occur may experience a developmentalcrisis. In this example, the pubescent period with the physical, emotional, social and psychologic changes mayrepresent a source of ineffective coping and crisis.Incorrect: A situational crisis may occur in response to a specific, acute and overwhelming event or unpredictable situation. A developmental crisis is generally triggered by a predictable stage of human development, such as pubescence.Incorrect: Adventitious crises result from natural or national disaster, or crime and violence. Large-scale affairs affecting the greater community or individuals within are atype of situational crisis. The scenario described above does not represent a problem that impacts the greater community or likely appear in the news media.

Family Developmental Situational Adventitious

10. A client in a long-term-care facility with Alzheimer's disease is frustrated because shecannot remember the time to attend the group "sing-a-long." Which nursing interventionis most appropriate?

Correct: The client will function best with time to prepare for the activity. The nurse who accompanies the client to the group activity provides reassurance and comfort. The care for the client with dementia is emotionally, physically and even spiritually draining. The role of the nurse is to engage in caring actions that allow the dignity to the human being with a cognitive disorder.Incorrect: The client with dementia may be unable to read the schedule or comprehend the time.Incorrect: The client with a cognitive disorder may have difficulty with spatial relations, includingreading a clock.Incorrect: Announcing the activity may be appropriate, but does not help the client get to the proper location on time.

Remind the client of the time prior to the activity and offer to take her to the location. Give the client a printed schedule and highlight the activities of interest to her. Place a clock in the client's room and instruct personnel to remind her of the time. Request the activity director to announce the activity over the intercom system.

11. A client diagnosed with severe depression is scheduled to receive the first of a series of electroconvulsive (ECT) treatments. The client verbalizes apprehension about thetreatment. Which nursing response is most therapeutic?

Incorrect: Although electroconvulsant therapy (ECT) will improve depression, this information, given as part of the procedural preparation, is not necessarily reassuring to the client. Typically, the client will want to know more concrete details regarding the ECT and the anticipated response to the procedure.Incorrect: Although the client will be relieved to learn that there is no pain with ECT, the greatest reassurance is the presence of a nurse during the procedure.Correct: Reassurance with concrete details about the procedure will help the client with apprehensionbetter than false reassurance with no specific information about ECT. It is likely to be comforting to the client that the nurse will be present during the procedure.Incorrect: Although transient amnesia occurs , the emphasis on this occurrence is likely to heighten anxiety.

"Electroconvulsive therapy will eventually improve your depression." "You will not have any pain or discomfort; supplementary oxygen will be provided to you." "The nurse will remain with you until you are awake and fully oriented." "You will have temporary memory loss during and after the procedure."

12. Which nursing interventions are most effective in assisting a client with stressmanagement?

Correct: The nurse can assist the client in developing strategies to enhance strengths and coping skills,

such as the use of guided imagery or deep breathing and relaxation exercises. The nurse may teach the client with stress and anxiety to use rehearsal techniques, recreational activities or play therapy for young children.Incorrect: It isnot the nurse's role to develop a support system for the client.Incorrect: Long-term psychotherapy may not beindicated. Stress management and crisis intervention are often managed with short-term therapy.Incorrect: Commonly used defensive mechanisms which generally signal ineffective coping may include: repression, suppression, projection, introjection, reaction formation, denial, displacement, and regression.

Provide strategies to enhance coping skills for stress reduction. Develop a support system for the client to prevent stress. Arrange long-term psychotherapy to learn techniques for crisis prevention. Use defense mechanisms for coping with stress and anxiety.

13. The client with schizophrenia, undifferentiated type, exhibits psychotic behavior. Which signs and symptoms can the nurse expect the client to demonstrate?

Incorrect: Depression and psychomotor retardation may occur, but are not typical of this psychotic condition.Incorrect: Intense affect and self-mutilation are characteristic of borderline personality disorder.Correct: Delusions and bizarre behavior are typical in undifferentiated schizophrenia.Incorrect: Profound hyperactivity and

elation are characteristic of the manic phase of bipolar disorder, and are not seen in schizophrenia.

Depression and psychomotor retardation Intense affect and frequent self-mutilation Delusions and bizarre behavior Profound hyperactivity and elation

14. A client with depression and suicidal ideology is admitted to an inpatient unit of a psychiatric hospital. The client is reluctant to express her feelings and speaks of pastlosses. Which is the priority nursing diagnosis?

Incorrect: The client has not mentioned self-recrimination or guilt.Incorrect: This diagnosis may be appropriate but more information needs to be gathered before making this diagnosis.Correct: The client has verbalized past losses and is not coping effectively as evidenced by suicidal feelings.Incorrect: Powerlessness may be applicable, but is not the priority nursing diagnosis, and cannot be applied until more is known.

Situational low self-esteem related to depressed state and self-recrimination Dysfunctional grieving related to fixation about past losses Ineffective individual coping related to feelings about past losses and suicidal thoughts. Powerlessness related to anxiety and fear of continued major losses

15. A client with obsessive-compulsive disorder reports to the nurse that he has perpetual tardiness for work due to a need to repetitively perform a daily routine eachmorning before leaving the house. Which nursing intervention is most therapeutic?

Incorrect: It is not advisable for the client to wait until arrival at the employment site to perform the daily routine. Postponing the ritual to a time or setting with no consistent opportunity to perform or complete the routine islikely to provoke additional anxiety.Correct: The client with obsessive thoughts may need to perform compulsive behavior in an effort to reduce anxiety. Until better control of the compulsive disorder is achieved, a technique for anxiety reduction may be to allow additional time to perform the ritual prior to departure for work.Incorrect: Anticipation of the ritual is likely to increase the level of anxiety and compulsive behavior to follow.Incorrect: Performing the repetitive ritual within the employment setting is likely to provoke more anxiety and possiblycompromise work performance.

Advise the client to wait until he gets to work to repeat his routine. Direct the client to awaken earlier each morning in order to allow time to repeat the routine. Recommend rehearsal of the anticipated ritual before going to sleep at night. Suggest that the client ask his boss to permit adequate time each morning to perform the routine.

16. A client diagnosed with paranoid schizophrenia refuses to take the medication

because he is convinced that it contains poison. Which nursing intervention is mostappropriate initially?

Incorrect: Calling the primary care provider and responding to the paranoid thoughts will reinforce them.Incorrect: Assault by injection of the medications is threatening to the client and illegal.Correct: Use of a casual approach is not likely to exacerbate paranoid thoughts. The nurse will need to watch the medications being taken to avoid hoarding.Incorrect: Asking the client for an explanation of his behavior may increase suspiciousness and reinforce the desired behavior.

Ask the client if he wants the primary care provider to be called. Tell the client that if he refuses the medication by mouth, it will be given by injection. Give the medications and water to the client in a matter-of-fact manner. Ask the client for an explanation of why he thinks the medications are poisoned.

17. A client receives lithium carbonate (Lithobid). If given concurrently, which medication may cause a dangerous interaction with lithium?

Incorrect: There is no known interaction between B12 and lithium.Incorrect: Chloral hydrate is a sedative. There is no known interaction between chloral hydrate and lithium.Incorrect: Allegra is an antihistamine/decongestant which may be taken with lithiumCorrect: Lasix, a diuretic, could be potentially dangerous to the client because of thedepletion of fluids. This fluid volume deficit can increase the concentration of lithium in the blood causing toxicity.

Cyanocobalamin (Vitamin B12) Chloral hydrate (Noctec) Fexofenadine (Allegra) Furosemide (Lasix)

18. A client is admitted to the substance abuse treatment unit of the psychiatric hospital for detoxification related to heroin abuse. Which medication is used most often for the detoxification of heroin?

Correct: Methadone is a synthetically developed compound used for heroin detoxification. The addictive properties, including the euphoric effects and withdrawal symptoms, are reduced. Heroin addicts receiving controlled doses of oral methadone are likely to suffer fewer of the social and medical problems related to intravenous drug addiction.Incorrect: Codeine is an opiate and is not indicated for heroin detoxification.Incorrect: Ritalin is used forattention deficit disorder, not substance abuse disorders.Incorrect: Antabuse is used to deter alcohol consumption,

not heroin withdrawal.

Methadone (Dolophine) Tylenol with codeine (Tylenol # 3) Methylphenidate hydrochloride (Ritalin) Disulfiram (Antabuse)

19. A client with anorexia nervosa states to the nurse, "I never feel good enough for my family. I am always ruining everything." Which nursing diagnosis is most appropriate?

Incorrect: Risk for ineffective family coping may be appropriate but there is no evidence of family disorganization. Family pressure to succeed is a common contributing factor in the disorder. The psychologic characteristics of the anorexic are: perfectionist, poor self-image, reliant on other's perception for personal validation, eager to please and succeed, and lacking the desire for individual attention.Correct: Anorexia nervosa is an eating disorder characterized by the refusal to maintain a normal body weight. The anorexic controls weight by restricting calories or engaging in the binge-purge cycle of eating. Multiple factors may contribute to the body image disturbanceincluding issues of self-esteem, family relationship problems, and perfectionism.Incorrect: Social isolation is anoutcome of anorexia-bulimia; the priority diagnosis is related to the etiologic factors of body image distortion and poorself-image. The individual with a binge-purge pattern of eating suffers social isolation due to the nature of the bulimic

ritual.Incorrect: Anxiety may be a contributing factor to anorexia, but the primary reason for anorexia is low selfesteem and a distorted body image.

Risk for ineffective family coping related to family disorganization Body image disturbance related to low self-esteem Impaired social interaction related to dysfunctional family interaction Anxiety related to unmet needs and goals

20. A client diagnosed with anxiety disorder begins taking alprazolam (Xanax). The nurse provides medication teaching. Which instruction is most appropriate?

Incorrect: There is no contraindication for ingestion of aged cheese. The client receiving MAO inhibitors is instructed to avoid this product.Correct: Alcoholic beverages should not be ingested with Xanax due to the additive depressant effects. Xanax is an antianxiety drug and has CNS depressant properties.Incorrect: There is no contraindication for intake of chocolate.Incorrect: Processed meats do not interact with Xanax.

Avoid aged cheese. Avoid alcoholic beverages. Decrease chocolate intake. Decrease intake of processed meats.

21. Which drug has the greatest potential for extrapyramidal side effects?

Incorrect: Bupropion (Wellbutrin) is an antidepressant. Extrapyramidal side effects occur in clients taking antipsychotic medications.Incorrect: Setraline (Zoloft) is an antidepressant. Extrapyramidal side effects occur in clients taking antipsychotic medications.Incorrect: Alprazolam (Xanax) is an antianxiety medication. Extrapyramidal ofside effects do not occur in clients taking this classification of medications.Correct: Extrapyramidal side effects including: dystonia, akathisia, pseudoparkinsonism and tardive dyskinesia typically occur in clients takingantipsychotics. Clozapine (Clozaril) is an antipsychotic drug.

Bupropion (Wellbutrin) Setraline (Zoloft) Alprazolam (Xanax) Clozapine (Clozaril)

22. A client expresses anger toward the nurse who delays taking the group on a therapeutic walk. The nurse states to the client, "We are very busy. Two nurses are absent from work today. We are attempting to help you the very best we can under the circumstances. I will address your concerns later." Which does the nurse's response demonstrate?

Incorrect: A paternalistic behavior is condescending to the client. The nurse with a paternalistic attitude may

placate the client's concerns or needs and act in a superior mannerIncorrect: False reassurance is not exhibited inthis example of a communication technique.Incorrect: The nurse is using a defensive approach to communication rather than judgmental one.Correct: This non-therapeutic communication technique may cause resentment by the client and block communication.

Paternalistic behavior False reassurance A judgmental attitude Defensive behavior

23. A client diagnosed with schizophrenia tells the nurse in an angry tone, "I hate all of them." Which nursing response is most appropriate?

Correct: This response reflects active listening techniques, which promote therapeutic and open communication.Incorrect: This response reflects a judgmental attitude by the nurse and is not therapeutic.Incorrect: This response does not focus on the client or the therapeutic relationship.Incorrect: Offering time and self is therapeutic but in this case the client's feelings are neither acknowledged nor explored.

"You hate all of them? Tell me about them." "It is never good to hate people; you hurt yourself." "I have feelings of hatred at times too." "I think that I need to stay with you while you are angry."

24. A client is brought to the emergency department in an unconscious state with no observable injuries. Paramedics at the site note empty bottles of lorazepam (Ativan) andvodka. In the acute care setting, the nurse performs vital sign assessment every fifteenminutes due to the risk for which physiologic complication?

Correct: The client is at a high risk for respiratory depression due to the central nervous system (CNS) depressant effects of toxic levels of alcohol and benzodiazepines (lorazepam), particularly when taken in combinationIncorrect: Hypertensive crisis does not occur in clients with CNS depression.Incorrect: The most serious side effect of tricyclic antidepressants is cardiac toxicity. Benzodiazepines are anxiolytic that acts as a tranquilizer. The primary risk to the individual who ingests toxic amounts of benzodiazepines in combination with alcohol isrespiratory depression.Incorrect: The client should be closely monitored for respiratory depression, not hyperthermia. Toxic levels of alcohol and benzodiaepines, particularly when taken in combination, have CNS depressive effects, and may result in a diminished respiratory drive.

Respiratory arrest Hypertensive crisis EKG changes Rapid temperature elevation

25. The charge nurse documents the following assessment findings regarding acoworker who reports to work: dilated pupils, difficulty concentrating, paranoid thinking, violent temper outburst, and grandiosity. Based on the assessment findings, the nurse may conclude that the coworker has symptoms related to usage of which substance?

Correct: Cocaine is a central nervous system stimulant. Symptoms of cocaine usage include: tachycardia, dilated pupils, elevated blood pressure, insomnia, anorexia, impaired judgment, poor concentration, temper outbursts, paranoia, elation, and grandiosity. Depression usually follows the high and subsequently reinforces addictive behavior.Incorrect: Symptoms of amphetamine usage include: tachycardia, dilated pupils, nausea and vomiting, twitching, impaired social interaction and hyperactivity.Incorrect: Heroin use results in constricted pupils, decreased respirations, hypotension and slurred speech.Incorrect: Marijuana use includes talkativeness, increased appetite, redeyes, and inappropriate laughter.

Cocaine Amphetamine Heroin Marijuana

26. A client obtains therapy at a substance abuse rehabilitation facility for non-medicinal

use of lysergic acid diethylamide (LSD). During the rehabilitation phase, the nursemonitors the client closely for which condition?

Correct: Flashbacks frequently occur when the client is withdrawing from hallucinogens and may occur at any time for the first few months or even years after cessation of use of the substance.Incorrect: Euphoric statesoccur during the ingestion of marijuana, not LSD.Incorrect: Delusions are not likely to occur when the client is withouthallucinogen use.Incorrect: Panic reactions do not usually occur when the client is free of hallucinogens.

Flashbacks Euphoric states Delusions Panic reactions

27. A client comes to the outpatient psychiatric clinic with the following complaints: slurred speech with abnormal tongue movements, muscle spasms in the neck, andcontinual pacing with a shuffling gait. As the client's history is collected, which should the nurse suspect?

Correct: The signs and symptoms are related to extrapyramidal effects of the use of antipsychotic medications, particularly the phenothiazines. There are three reversible side effects including acute dystonia,

akathisia and pseudoparkinsonism. Tardive dyskinesia is the most serious side effect and is not reversible.Incorrect: The client clearly has side effects related to antipsychotic medications. Dementia and delirium are two examples of cognitive impairment disorders. The client is not demonstrating behavior seen in these conditions.Incorrect: Muscle rigidity coupled with slurred speech and shuffling gait indicate extrapyramidal side effects of antipsychotic medication.Incorrect: Alcohol toxicity may cause slurred speech and an unsteady gait, but does not explain the pacing or muscle spasms.

Extrapyramidal side effects related to antipsychotic medication Evidence of a cognitive impairment syndrome Signs and symptoms related to substance abuse Alcohol toxicity with documented serum levels

28. After dinner a female client goes to the lounge area of an acute care mental healthfacility with brightly colored, rouged cheeks, excessive amount of accessories in her hairand on her wrists, and three pairs of false eyelashes. She is wearing a sheer nightgownand high heels. Which nursing action is most appropriate for the care of this client?

Correct: Redirecting a client's behavior in a non-judgmental and non-threatening manner is the most appropriate nursing action.Incorrect: The inappropriate dress of the client may be distracting to others involved in a

unit activity. Her appearance may be counter-productive to the therapeutic environment of others in the group.Incorrect: Reminding the client to dress appropriately is condescending and nontherapeutic. Her poor choicefor apparel and outward appearance is a reflection of intrapersonal issues. The client has not forgotten to dress in a socially acceptable manner; instead, she lacks the discernment and proper judgment to do so.Incorrect: Insisting thatthe client wear facility clothing is non-therapeutic and inappropriate. The nurse can work with the client to make appropriate personal clothing and external appearance choices. The benefit of allowing the client to remain in her own clothing is related to issues of self-esteem, compliance, respect for the individual, and selfdetermination.

Redirect the client to her room and assist her with putting on appropriate, personal apparel. Allow the client to wear what she likes and get her involved in a unit activity. Remind the client that she has to wear appropriate clothing while in the lounge area. Provide the client with facility clothing because of her inability to makeappropriate apparel choices.

29. Which classification of drugs is used most often during the acute detoxificationperiod of alcohol withdrawal?

Correct: Antianxiety medications, especially the benzodiazepines, are most commonly prescribed for alcohol

withdrawal.Incorrect: Antidepressants are not usually prescribed during the first 24-48 hours of acute detoxificationIncorrect: Anticonvulsants are used for seizure disorders, not during detoxification.Incorrect: Antimanic drugs are used for bipolar disease, not detoxification and alcohol withdrawal.

Antianxiety Antidepressants Anticonvulsants Antimanic

30. The nurse assesses the mental health status of a client newly admitted to an acutecare facility. Which client statement is most indicative of a positive mental health state?

Incorrect: The perception of self-efficacy is an indicator of positive mental health status. However, overcompensation in one area of competence may occur as a result of a deficiency in another area.Correct: The client with long-term goals for self-improvement and independent living indicates healthy mental function.Incorrect: The lack of insomnia or sleep disturbance does not necessarily indicate a positive mental health status. For example, a person with depression may sleep excessively.Incorrect: Quality friendships and effective social relationships may

indicate that the client has limited social skills or some underlying problem with interpersonal relationships.

"I help my friends with their computer problems and academic needs." "I will graduate from college next semester and already have job offers" "I never have sleep problems like other members of my family." "I have one close friend that I see and talk to every day."

31. Which nursing intervention is initially most appropriate for the care of the homelessperson with mental illness?

Correct: The nurse engaging in active listening is most likely to gain the homeless client's trust and pave the way to an open relationship. The nurse has the opportunity to gather significant psychosocial assessment data through the history.Incorrect: For the nurse to establish a support system and obtain temporary housing for the client is neither a realistic nor appropriate intervention. Problem identification proceeds planning of interventionstrategies.Incorrect: A treatment plan may facilitate the client's insight into his problems, however, the primary intervention is to practice active listening to the client. Verbalization of feelings is therapeutic. One of the primary rolesof the nurse in this setting is to obtain assessment data that can serve as the basis for the therapeutic care of thehomeless person.Incorrect: Exploration of the factors that lead to homelessness is not appropriate in the initialpsychosocial assessment. These issues may be explored after rapport is established.

Listen attentively to the client's conversation for insight into multiple problems. Establish a support system and obtain temporary housing for the client. Formulate a treatment plan to develop the client's insight into his problems. Explore with the client the factors that lead to homelessness.

32. After witnessing a drowning, a client develops paralysis of both legs. Extensiveneurological tests reveal no evidence of a physical cause for the paralysis. Based on theclient's clinical presentation, which can the nurse conclude about the client?

Incorrect: Clients with hypochondriasis are preoccupied with somatic function and have frequent complaints of physical illness. These clients often doctor shop because they believe the complaints are not taken seriously. Thecondition usually interferes with social and work roles to some degree. Hypochondriasis persists in spite of the lack ofmedical evidence supporting the physiologic problem.Correct: Conversion disorder is the condition of loss of bodyfunction, such as paralysis or blindness, that is not related to a physiologic disorder. This condition results from a catastrophic event leading to serious deficits in motor or sensory function. A conversion experience may arise out of apsychological conflict, which is not in the conscious awareness of the individual. The symptoms are not intentionally produced. The individual may have an indifferent attitude or affect regarding the nature of the symptoms, referred to as la belle indifference, which is out of proportion with the severity of the symptoms.Incorrect: Catatonia is seen in the client diagnosed with schizophrenia. There are two types of catatonic schizophrenia. One form is characterized bystupor, and immobility. The other is characterized by excessive, impulsive, destructive behavior.Incorrect: The client does not display typical symptoms of severe anxiety or panic.

Hypochondriasis Conversion disorder Catatonic schizophrenia Anxiety disorder

33. While watching television in the lounge of a mental health facility, the client states quickly and abruptly to the nurse, "The sun is shining. My son loves purple. Somewhereover the rainbow. Oh, where could my little dog be." Which alteration in thought process is the client experiencing?

Incorrect: The consecutive use of these statements is an example of flight of ideas, not abstract thinking which is the ability to conceptualize ideas or finding meaning in proverbs. Flight of ideas is the successive adjoining ofnon-related thoughts and multiple topics. Correct: The consecutive use of these statements is an example of flight of ideas, which is a continuous flow of speech whereby the client jumps from one topic to another. Incorrect: Word saladis a mixture of words and phrases that have no meaning. Incorrect: A neologism is a word invented by the individual that has meaning only to that person.

Abstract thinking Flight of ideas Word salad Neologism

34. An individual who is a regional sales manager seeks mental health care due tooverwhelming feelings of fear and anxiety when speaking in front of a group. The nursegathers assessment data about the physical manifestations that occur at the time theindividual engages in public speaking. Reported symptoms include: trembling, blushing, sweating, nausea, vertigo, and lightheadedness that interfere with the ability toadequately communicate the intended information. The nurse developing a plan fortherapeutic management of the symptoms understands that which condition is mostlikely?

Correct: A social phobia is an intense and enduring fear of social or performance situations that severely affects the daily life of the individual. Anxiety attacks are common with exposure to situations whereby the individual fears embarrassment, judgment, or perception as crazy or stupid. The fear of public speaking is a classic social phobia. Incorrect: A specific phobia is a severe and persistent fear of a clearly identifiable source, object, situation, or experience. Common categories for specific phobias include: animals (dogs), reptiles (snakes), nature (lightening), bodily experience (blood, injections, vomiting), and situations (heights, airplane travel, elevators). Individuals usually recognize that their fears are in disproportion to phobic source. Incorrect: School phobia is a form of a separationanxiety disorder whereby the individual experiences excessive anxiety, often to the point of panic, with exposure to a school setting. The response of separation from the parent is excessive and inappropriate to the situation. The duration of the disturbance is four weeks or greater; onset is prior to age eighteen. The phobia causes significant

distress and disturbances in the social and academic functioning in school. Incorrect: Agoraphobia, the most severe and pervasive form of phobic disorder, is the marked fear of being in places or situations that escape may be difficult or embarrassing. Some examples of agoraphobic fears are: being on a bridge, traveling on a bus or train, or beingalone in a crowd.

Social phobia Specific phobia School phobia Agoraphobia

35. A client who is a victim of domestic abuse and spousal battering tells the nurse that she feels powerless to leave her marriage because she is financially dependent on herspouse. Which intervention is most appropriate?

Incorrect: The cycle of domestic abuse is common. However, information regarding abuse occurring in previous generations is not helpful for the physical protection or emotional coping with a violent infliction of harm. Incorrect: Although understanding of the spousal anger may aid in the client's coping with the abuse, the primary concern is safety and security. The nurse's role in working with the client suffering harm from a spouse is not to

validate the abuser's situation with possible explanations for anger. Correct: The nurse has a vital role in the physical and psychosocial assessment of the client who is a victim of spousal abuse. The determination of physical safety andsecurity is the most important intervention for the client in a situation of potential harm. A plan for action may be formulated after baseline information about the situation is obtained. Incorrect: Although it may be therapeutic for the client to identify her personal strengths and positive areas of her life, the most critical concern is safety and security. Maslow's hierarchy of needs identifies physical safety, security and survival needs at the basic level of human functioning. The nurse plays a vital role in obtaining physical and psychosocial assessment data relating to the victim's extent and nature of abuse. After baseline information is about the victim's situation is revealed, a plan for action may be formulated.

Explain to the client that her spouse probably came from an abusive family. Explore with the client the reasons for spousal anger to aid in a betterunderstanding. Determine the extent of injury and the client's perception of financial dependency. Assist the client to identify personal strengths and positive areas of her life.

36. Chlordiazepoxide hydrochloride (Librium) is frequently ordered for alcoholwithdrawal. The administration of Librium is contraindicated with which medical condition?

Incorrect: Shingles is not a contraindication for the use of Librium. This infectious disease is related to herpes zoster. There is no relationship between Librium use and the rate of complications associated with herpes zoster. Correct: Hepatic dysfunction is likely to occur in the client with alcohol dependency. The side effect of Librium is hepatic impairment; therefore, the use of the medication is contraindicated if hepatic dysfunction is detected. Incorrect: Librium is not contraindicated for the client with glaucoma. Incorrect: There is no association with Librium use and Cushing syndrome.

Shingles Hepatic dysfunction Glaucoma Cushing's syndrome

37. A client with an antisocial personality disorder exhibits frequent episodes of hostility with angry, negative remarks directed towards the nursing staff. Which is the most therapeutic nursing intervention?

Correct: Clients with antisocial behavior are unable to form attachment to others and lack judgment. They

show no guilt or remorse for their actions and demand instant gratification. Many appear to enjoy conflicts. Settinglimits for inappropriate behavior is necessary. Incorrect: Ignoring the client is not therapeutic. Since clients with antisocial behavior are very demanding; ignoring the client will result in conflict. Incorrect: Additional antipsychotic medication may not be ordered or indicated, and is a temporary solution, while limit setting will modifybehavior.Incorrect: If limit setting is implemented upon admission, seclusion may not be needed.

Set non-punitive limits requiring consistent, appropriate behavior. Minimize the situation by ignoring the client's inappropriate actions. Administer an extra dosage of an antipsychotic medication. Seclude the client in a quiet room to regain control.

38. A school nurse plans to educate a ninth-grade class about substance abuse. Which factor is the most important for the nurse to consider related to the usage of drugs byadolescents?

Incorrect: Lack of responsible parenting and discipline may be a contributing factor in teen drug use. Adolescents who experience a lack of emotional support and boundaries for behavior and activity may be at anincreased risk for drug abuse. Developmentally, peer influence and pressure for conformity is one of the most powerful forces in the life of an adolescent and may account for drug-taking behavior. Correct: Peer pressure for

conformity is one of the strongest factors resulting in drug usage during adolescence. Erikson's developmental stagefor adolescence, identity versus role confusion, involves the task of mastery for personal identity. The peer relationship, individuation from the family, development of sexual maturity and direction for adult ambitions are the primary tasks of this stage. Incorrect: Sexual maturation occurs during the adolescent period, however, the primary influence for illicit drug-taking behaviors is related to peer influence and pressures for conformity. Incorrect: The media has an influence on the development of social mores and behavior of all ages, adolescents in particular. However, the peer group plays a more significant role in the behavior, choices, and development of the adolescent. Erikson's developmental stage for adolescence, identity versus role confusion, involves the task of mastery forpersonal identity. The peer relationship, individuation from the family, development of sexual maturity and direction foradult ambitions are the primary tasks of this stage.

Lack of responsible parenting Strong peer pressure Increased sexuality Influence of the media

39. A client receiving anger management therapy describes his interactions with his family as aggressive and hostile. The client believes that the family continually takesadvantage of him. Which concept regarding anger is most accurate?

Incorrect: Anger is a temporary emotional response and not a permanent state. Anger may result in poor interpersonal relationships. Correct: Anger is a negative emotional response, which is temporary. Anger and aggression are responses to perceptions of threat or loss of self-control, and clients often demonstrate anxiety priorto exhibiting anger. Incorrect: Individuals in the grieving process commonly experience feelings of anger. However, the most common etiologic factor is vulnerability and uneasiness. Various situational factors may provoke the emotional response. Incorrect: Anger is not a sustained emotional state, but occurs in response to perceived injury and/or fear and sense of powerlessness.

Permanent emotion resulting in dysfunctional relationships due to loss of selfcontrol Temporary emotion exhibited as a result of vulnerability and uneasiness Episodic emotion dependent on the type and level of grief and individualexperiences If sustained, may lead to more violent behaviors, such as murder and rape

40. A nurse works in the psychiatric unit. The nurse's neighbor is admitted to the unit. After returning home from work each day, the nurse receives several calls from neighborhood friends inquiring about the client's status. One person demands to know what is wrong and asks if it will be safe to live next door to the client after the client isdischarged. Which response by the nurse is most appropriate?

Incorrect: The nurse is making an assumption about the client-neighbor relationship. This response is inappropriate. Correct: Confidentiality is a right and responsibility in the nurse-client relationship. It is highly unethicalfor the nurse to divulge any information about the client to the neighbors. Incorrect: It is inappropriate for the nurse togive advice. Incorrect: Only the client may choose visitors while hospitalized. It is inappropriate for the nurse tobecome personally involved with the relationship between the client and neighbor.

"It sounds as though you are afraid of your neighbor now; is that right?" "All information about your neighbor is confidential; I cannot discuss this withyou." "The best advice I can give you is for you to do is to talk to your neighbordirectly." "I'll ask your neighbor if you can come to the hospital for a visit; you need to getthings straightened out before your neighbor comes home."

41. The driver of an automobile escaped a severe accident with minor injuries. However, the passenger in the car, the driver's sister, died at the scene. Since the accident, the client cannot recall any events of the automobile accident. Which is the clientexperiencing?

Incorrect: Distortion is an automatic unconscious defense mechanism that is used to decrease anxiety and restore emotional well-being. Correct: Dissociative amnesia is a condition where an individual is unable to retrievepreviously acquired information. The condition is usually triggered by a traumatic event and painful emotional state. The memory loss is abrupt. Nor organic disease is associated with dissociative amnesia. Hypnosis may be effective

as a first step in determining the buried memory. Psychotherapy can then be implemented to cope with the difficult issues and depression that may ensue. Incorrect: Denial is a commonly used defense mechanism. Denial of a fact orthe significance of a fact can alter an individual's perception of reality. In the scenario, the individual does not deny the details of the accident. The client has pushed the traumatic event into her unconscious mind to block it from conscious awareness. Denial shuts out painful awareness. Incorrect: Displacement means the transfer of feelingsfrom a threatening person to one that is less threatening.

Distortion Dissociative amnesia Denial Displacement

42. The nurse teaches the client to avoid tyramine-rich foods when taking whichmedication?

Correct: Nardil is a MAO inhibitor. The client must not eat foods containing tyramine while taking this medication. Tyramine may cause a hypertensive crisis related to the severe vasoconstriction, which results in a rise inblood pressure. This reaction is life-threatening.Incorrect: Elavil is an antidepressant and is not affected by tyraminerich foods.Incorrect: Eskalith is an anti-manic medication with no restriction of tyramine-rich foods.Incorrect: Prolixin isa long-acting antipsychotic agent. It is not affected by foods containing tyramine.

Phenelzine sulfate (Nardil) Amitriptyline hydrochloride (Elavil) Lithium carbonate (Lithobid) Fluphenazine decanoate (Prolixin)

43. A client, preparing for discharge from the inpatient mental health facility, tells the nurse on the day shift, "I could not sleep last night because the cats were meowing. I wasup all night rolling toilet paper around the room." Which is the best response of thenurse?

Incorrect: This response of the nurse may block communication with the client. Correct: Reflective listening promotes open communication and explores the topic with the client to obtain further information. Incorrect: Although the response may reflect the client's feelings, it is not appropriate for the nurse to prematurely interpret the client'scommunication instead of listening and gathering assessment data regarding the mental health status. Incorrect: Invalidation of the client's feelings or experience may block the line of communication between the nurse and theclient.

"There are no cats here in the hospital; we do not allow animals here." "You heard cats meowing, and rolled toilet paper? Tell me about this." "Do you think you heard the cats because you are worried about leaving the hospital? "

"You need to forget all about this; remember, your hospital discharge is today."

44. Which nursing diagnosis is most appropriate for the client diagnosed with borderlinepersonality disorder?

Incorrect: Clients diagnosed with borderline personality disorder may be socially isolated but often they have intense interpersonal relationships, in spite of inappropriate expressions of anger. Incorrect: This is not the priority nursing diagnosis. Even though the client may have mood changes, it is not the primary problem. Incorrect: The clientwith borderline personality disorder has difficulty in maintaining stable relationships due to intense, changeableemotions and moods. Depression may occur at times concurrent to feelings of loneliness and alienation. Correct: Theclient diagnosed with a borderline personality disorder has a profound identity disturbance regarding self and bodyimage, sexual identity and meaningful relationships. Behavior is impulsive and moods are labile. The client is at risk of engaging in harmful activities such as self-mutilation. Much of the client's disturbed behavior is rooted in fear ofabandonment.

Social isolation related to episodes of inappropriate anger Delusional thinking related to extreme mood changes Ineffective individual coping related to depressed affect Personal identity disturbance related to fear of abandonment

45. The nurse involved with the mental health care of the client with bulimia nervosa

understands which diagnostic criterion to define the condition?

Incorrect: The fear of gaining weight may be present, but the diagnostic criteria are binging and purging. Incorrect: The client with bulimia ingests large amounts of food, typically highly concentrated carbohydrates and fat. The adequacy of nutritional status is related to the frequency of the binge-purge cycle. Correct: Bulimia nervosa ischaracterized by the client who eats large amounts, induces vomiting or uses laxatives for the purging of food. Incorrect: The gross distortion of body image with the intense fear of fat is characteristic of anorexia nervosa. The individual with bulimia nervosa engages in the binge-purge eating cycle.

Intense fear of gaining weight through the severe restriction of food intake Sense of powerlessness and refusal to provide adequate nutrition with inducedvomiting Recurrent episodes of binge eating of large amounts of food in a short timeperiod with induced vomiting Distortion of body image with the delusion of being fat when actually very thin.

46. A client believes that members of organized crime are involved in a conspiracy with aliens to abduct and transport him to another planet. Of which is this an example?

Incorrect: Hallucinations are sensory experiences that usually reflect a stress response in the environment.

The perceptions often are frightening and intense. Auditory hallucinations are more frequently associated with psychotic disorders; visual and tactile hallucinations result from physiologic causes such as organic brain syndromeor drug and alcohol toxicity. In this situation, no hallucination is evident because the client does not report seeing or hearing members of those groups. Correct: A delusion is a false belief, not related to logic or reality, arriving out of a perceived threat to the ego. This situation exemplifies a delusion of persecution; the client believes that others are seeking to hurt him. Incorrect: Illusions are misperceptions of reality, usually visual or auditory. Incorrect: Displacement is the transfer of feelings from one person to a less threatening person.

Hallucination Delusion Illusion Displacement

47. The nurse planning the care for a client experiencing the manic phase of a bipolardisorder makes the diagnosis of "altered thought process related to flight of ideas." Which outcome criterion indicates improvement? The client will:

Incorrect: Relevant outcome criteria for the manic phase of a bipolar disorder typically do not address issues related to self-perception. Incorrect: The client experiencing the manic phase of a bipolar disorder is usually eager to

verbalize feelings and express thoughts and opinions. Incorrect: As the manic phase of bipolar disorder continues, the client usually has increased hostility, periods of irritability, and may talk often about sexual and obscene topics. Correct: Flight of ideas often occurs in the manic phase of a bipolar disorder. Typically, the speech is rapid with frequent topic or idea changes and few coherent sentences.

report feeling in control. verbalize feelings upon request. express positive statements about self. speak slowly in coherent sentences.

48. The client most likely to benefit from group therapy is one who is diagnosed withwhich condition?

Incorrect: The client with Alzheimer's disease is usually unable to benefit from group therapy because of the lack of orientation, and in the later stages, the wandering and communication problems. Incorrect: The client with dementia can usually participate only in reminiscence therapy or simple occupational therapy. Correct: The client withanxiety is oriented to reality and can benefit from community and group therapy. Incorrect: The client with delirium is not a candidate for group therapy because delirium is an acute condition that subsides when the underlying physicalcause is treated.

Alzheimer's Dementia Anxiety Delirium

49. A man wants to admit his spouse to an acute care mental health facility under an involuntary admission. Under which condition is involuntary admission justified?

Incorrect: Chronic illness and aggressiveness are not criteria for involuntary admission. Incorrect: Agreement of the need for treatment is a criteria for voluntary admission for treatment of mental illness. Correct: Danger to self orothers is a criterion for an involuntary admission for mental illness. Incorrect: An acute care facility is not an appropriate site for respite care. The client with a mental health disorder cannot be mandated to receive mental health care unless a danger to self or others exists. A family may perceive the need or desire to have a break in the care, however, involuntary admission is not permissible for that reason.

The client is chronically ill and becomes aggressive. The client and family agree that the client needs treatment. The client is a danger to self or others due to a mental disorder. The client's family needs respite care for the client.

50. A client insists that she hears people laughing and making fun of her, telling her that she would be better off dead. Which thought process disturbance is the clientexhibiting?

Incorrect: Individuals with delusions of persecution believe that someone is seeking to hurt or cause damage. There is no indication that the client believes the people are trying to harm or kill her. Incorrect: A delusion ofreference means the client believes he/she is someone else. Correct: A client with an auditory hallucination hears sounds that are not present in reality. The client in the scenario hears people ridiculing her with suggestions. Incorrect: Hypnagogic hallucinations are false sensory perceptions during twilight.

Delusion of persecution Delusion of reference Auditory hallucination Hypnagogic hallucination

51. A client diagnosed with terminal breast cancer is informed that she has a short time to live. At her request, the client is discharged from the hospital and returns home. She receives Hospice nursing care prior to death. When visiting the client, the Hospice nurseassesses that the client refuses to stay in bed. Instead, the client remains in a reclinerrocker and tries not to sleep. The client's behavior best illustrates which common feelingabou

Incorrect: Although the client may experience anxiety regarding many unsettled financial affairs, the most

common or intense experience that the dying person has is fear of the unknown. Correct: Fear of death is related tothe unknown questions regarding what happens following death. Incorrect: Saying goodbye to loved ones is part ofthe process of dying. The fear of the unknown is more distressing. Incorrect: The fear of the unknown is more distressing than the fear of separation from the family. The process of separation and closure in interpersonal relationships occurs over time.

Anxiety regarding the state of her financial affairs Fear of the unknown after death Fear that she will die alone Anxiety related to separation from her family

52. A client seeks mental health care due to extreme anxiety about airplane flight. The fear of flying interferes with work and leisure plans. Which disorder is demonstrated?

Incorrect: The client is not exhibiting signs of obsessive-compulsive disorder. An obsession is a repetitive or persistent thought, idea, impulse or image; a compulsion is the repetitive act that the client performs following anobsession. The behavior is performed in a specific order or manner and is intended to reduce the anxiety related to the obsession. Incorrect: Post-traumatic stress disorder does not apply to this situation. Persons with this disorderexhibit intense and exaggerated responses, usually including terror, helplessness and physiologic responses related to intense fear. The maladaptive reaction is triggered by overwhelming events, such as natural disaster, combat

experience, fire, motor vehicle accident, rape or other catastrophic situation. Incorrect: A characteristic of panic disorder is the sudden onset of the episode. Panic attacks usually last minutes and do not occur in direct exposure tothe definable stimulus. The physiological symptoms experienced during a panic attack include: heart palpitations, sweating, shortness of breath, chills, dizziness, trembling, chest pain, fear of dying, numbness, and fear of doingsomething uncontrollable during the episode. There is no indication in this situation that the client is experiencing these symptoms. Correct: A phobia is severe and persist fear of a specific object (spiders, dogs,) or situations (fear offlying) that triggers intense anxiety that interferes with daily life. Although the person may recognize that the fear is excessive or irrational, he/she has an inability to alter the feeling associated with the phobic object or situation.

Obsessive-compulsive disorder Post-traumatic stress disorder Panic disorder Phobic disorder

53. The nurse works with the parent and 8-year-old child with attention deficithyperactivity disorder (ADHD). The nurse makes a diagnosis of "impaired socialinteractions." Which outcome criterion is most appropriate? The child will:

Correct: Children with ADHD typically have difficulties establishing relationships and social interactions. This outcome is specific, observable, and appropriate for the nursing diagnosis of impaired social interactions. Incorrect:

Children with ADHD may have decreased self-esteem, but this outcome best fits the diagnosis of self-esteem disturbance, not impaired social interactions. Incorrect: Children with ADHD cannot stay on task or work independently. Children with ADHD frequently employ attention-getting behaviors. This outcome best fits the diagnosis of self-esteem disturbance. Incorrect: Children with ADHD often display inappropriate behaviors. The children may act-out frustration with bossiness, intrusiveness or aggression. This outcome best matches the diagnosis of risk for violence directed at others.

demonstrate increased ability to play in groups without aggressiveness. demonstrate increased self-esteem and feelings of self-confidence. use less attention-seeking behaviors to meet self-esteem needs. express feelings of anger and frustration by using age-appropriate behaviors .

54. A client diagnosed with dissociative amnesia is unable to remember important personal information. With which condition is dissociative amnesia usually associated?

Incorrect: It is unlikely that a conflict in an interpersonal relationship is traumatic enough to result in dissociative amnesia. Dysfunctional relationships generally involve chronic communication problems and otherfeatures that develop insidiously. A dissociative amnesic event is triggered by an acute, traumatic situation. Incorrect:

Dissociative amnesia occurs after an acute, precipitating event that is traumatic or emotionally disturbing. Chronicanxiety is not likely to result in amnesia. Incorrect: Clients with posttraumatic stress disorder experience flashbacksrelated to intense fear, helplessness or horror. With flashbacks, the person behaves as though he/she is experiencingit for the first time. Correct: Dissociative amnesia is marked by the sudden inability to access previously learned orintegrated information. The condition is preceded by an extremely traumatic event or events. No evidence of organic brain disease is evident.

Interpersonal relationship conflict Chronic anxiety Post traumatic stress disorder Traumatic event involving fear and horror

55. The nurse conducting an assessment for a client in the middle phase of Alzheimer's disease is most likely to document which finding?

Incorrect: The individual in the late phase of Alzheimer disease is incapable of independent living. The ability to communicate is severely impaired. The client essentially lives in a vegetative state. Correct: Alzheimer disease is primary dementia that follows a predictable pattern in the early, middle and late stages. The early stage is characterized by amnesia. In the middle stage, the client has dementia that is evidenced by difficulty in completion ofactivities of daily living without assistance. The individual with Alzheimer has impaired judgment, creating safety

issues. Depression and social isolation are common. In the late stage, the individual experiences severe changes incognition, memory, reasoning, gross motor function and expressive and receptive language. Incorrect: The individual in the early phase of Alzheimer disease experiences short and long-term memory loss, although, the ability to function independently remains intact. Communication is relatively unchanged in the early phase of the disease; aphasia is characteristic of the late phase. Incorrect: In the late phase of Alzheimer disease, the individual experiences severe changes in cognitive function, including loss of global memory, perceptual, problem-solving and abstract reasoning abilities. Personality and behavioral changes are marked by aggression and agitation, loss ofinhibitions, and aphasia. Bowel and bladder incontinence is common in the late stage.

Inability to complete activities of daily living; severe communication impairment Difficulty in completing activities of daily living; poor judgment regarding safety Loss of some short and long-term memory; performance of activities of dailyliving Personality and behavior changes; bowel and bladder incontinence

56. The nurse provides care for an individual newly diagnosed with acquiredimmunodeficiency syndrome (AIDS). The client experiences anticipatory grieving relatedto the loss of health status and probable death. Which expected outcome is mostappropriate for the psychosocial care of the client with AIDS? The client will:

Incorrect: Complete acceptance of the disease and eventual outcome is unrealistic for the client newly diagnosed with AIDS. Resolution of the grieving process is acceptance. However, most individuals do not move

through the phases of grief in a potentially terminal disease, particularly one with the social stigma as AIDS, in a rapidmanner. Incorrect: Discussion of the causative factors and preventative measures may increase feelings of regret, guilt, remorse, sorrow, and fear. These feelings are not therapeutic in the grieving process. Correct: Grief is a normal response to various types of loss: health, life, relationships, disease, and future plans. Some strategies for the nursing care of the client with AIDS include: acknowledging the stage of grief and support and use of therapeutic communication. Incorrect: Pharmacologic treatment for depression related to AIDS may or may not be indicated. The nurse's role for the care of the individual with AIDS is to implement therapeutic communication or strategies to facilitate effective coping.

accept the disease process and associated losses. demonstrate understanding of the cause of AIDS and preventative measures. express feelings, concerns and fears about the disease and treatment. receive pharmacologic treatment for depressive symptoms related to AIDS.

57. In order to prevent a drug-drug interaction in a client receiving diazepam (Valium), which drugs should be avoided?

Incorrect: Caffeine and tobacco are stimulants and diazepam is a central nervous system depressant.

Incorrect: There is no contraindication for the use of either serotonin re-uptake inhibitors (SSRI) or propranolol with diazepam. SSRI's are as effective as tricyclic antidepressants and likely to be safer. Fluoxetine is the most commonly used drug for depression. In contrast, to tricyclic antidepressants, SSRI produces central nervous system excitationrather than sedation. Correct: Diazepam is an anti-anxiety medication with CNS depressant properties. It is also usedto relieve tension headaches, treat seizures and produce muscle relaxation. The client should never ingest alcohol or other CNS depressants while taking diazepam because a toxic interaction can result. Incorrect: Diazepam may beused with

antipsychotics. Caffeine and tobacco SSRI's and propranolol CNS depressants and alcohol Antipsychotics and tyramine

58. The mother of a 9-year-old girl has just learned that her spouse has been sexuallyabusing their daughter for the past several months. The mother brings the child to theclinic for treatment. When assessing the child's response to the sexual abuse by herfather, which behavior is the nurse most likely to expect in the child?

Incorrect: Although suicidal feelings may result from sexual molestation, guilt and shame are the most common initial responses of a child to sexual abuse. Correct: The victim of incest often feels guilty and responsible for the abuse. The shame and embarrassment prohibits the victim from seeking help. A true accounting of the nature

of the abuse may be difficult to obtain due to the perceived isolation and helplessness. Incorrect: Guilt and shame areusually the most prevalent responses to sexual abuse. A secondary issue that may develop may be the fear of a severed relationship between the child and caregiver (parent) or between the partners in the domestic relationship. The child is emotionally and financially dependent on the abuser. The child may fear isolation from the family as well. Incorrect: The most common responses that occur as a result of a sexually abusive relationship or incest activity are usually guilt and shame. The child may fear isolation within the family, rejection or judgment. However, the fear of pain, injury or report of the activity of the abuser is more commonly greater than the fear of the mother's response. The child is financially and emotionally dependent on caregivers for survival. At age nine, a fantasized run-away experience is not usually implemented.

Contemplation of suicide due to the abuse by her father Sense of responsibility and guilt for not stopping the abusive activity Fear that her parents will divorce because of the abusive behavior Plans to run away from home because she fears her mother's anger

59. Which signs and symptoms should alert the nurse that the client may have aparanoid personality disorder?

Incorrect: Manipulation, often referred to as splitting, is a typically behavior exhibited by a person with a borderline personality disorder. Self-mutilation is another hallmark sign of the client with borderline personality disorder. Correct: The essential feature of the client with paranoid personality disorder is a pattern of mistrust of others and suspiciousness of people's intentions. The person with this disorder is hyper-vigilant and controlling with an inflated sense of self-importance. In addition, individuals are preoccupied with doubts about the loyalty of othersand maintain malevolent feelings regarding people they believe to have insulted or injured them. Many perceivenegative meaning into benevolent comments. Persons with paranoid personality disorder often believe spouses or sexual partners are disloyal and sexually promiscuous. Incorrect: Ideas of reference and egocentric behavior arecharacteristic of the client with schizotypal personality disorder. The salient feature of this personality disorder is the detachment from social relationships and restricted interpersonal communication. They prefer solitary activities, maintain little interest in activities, and have no close friends other than first-degree relatives. The person with schizoid personality disorder are indifferent to others and have a flat affect with emotional detachment. Incorrect: Grandiosity and need for admiration are typical of the client with a narcissistic personality disorder. A preoccupation with thoughts of extreme power, wealth, beauty, love, talents or success is common. Many persons with this personality disorder have an exaggerated sense of entitlement. They lack empathy for the situations, feelings or needs of others.

Manipulation and self-mutilation

Hypervigilance and exaggerated sense of importance Ideas of reference and eccentric behavior Pattern of grandiosity and the need for admiration

60. Which client statement best demonstrates symptoms of spiritual distress?

Incorrect: This answer does not convey spiritual distress but rather a desire to receive spiritual guidance. Incorrect: Rather than spiritual distress, this describes a longing to participate in spirituallyrelated activities. Correct: The client is expressing feelings of hopelessness, which usually occur in spiritual distress. Incorrect: This answer conveys feelings of regret and possibly depression, not spiritual distress.

"My minister is coming by to see me this afternoon." "I really miss my church family, activities and prayer time." "I know I'm not going to get better; God has forgotten about me." "If only I could get well, what a wonderful life I could be living now."

61. Which prejudicial attitudes toward mentally ill persons are common in society?

Incorrect: Many pharmacologic as well as therapeutic modalities exist to treat a variety of mental health

disorders. Incorrect: Although mental illness is a societal burden, prejudicial attitudes are related to cultural views. Correct: Many misconceptions and myths are present in society concerning mental illness and persons with mentaldisorders. Incorrect: The definition of mental illness is varied among cultural and sub-cultural groups. Behavior, attitudes, ideas, and mental health issues may be considered normal in a particular setting or within a certain groupbut deemed abnormal or inappropriate within another. Many of the mores of previous generations are dissimilar to those of today.

No effective treatment exists for mental illness Mental illness is an economic and societal burden Myths and misconceptions are held regarding mental illness Mental illness is similar in most cultures

62. An adolescent reports feelings of exhaustion, change in appetite, weight loss, andheadaches after the break-up with his girlfriend. His teachers document withdrawnbehavior and note his recent graphic sketches containing violent subject matter. The nurse planning the care makes a nursing diagnosis of "risk for self-inflicted violence." Which is the most appropriate expected outcome? The adolescent will:

Correct: The client clearly is at risk for suicide. Safety is the primary goal for care of the client who is at risk for self-inflicted violence. Incorrect: Demonstrating new coping mechanisms best matches the nursing diagnosis ofineffective individual coping. Incorrect: Family interaction is best related to ineffective family coping. Incorrect:

Identifying personal strengths relates best to the diagnosis of self-esteem disturbance.

name two people that he will tell if thoughts of suicide recur. demonstrate two new coping mechanisms to deal with stress. interact more effectively with family members and peers. identify three personal strengths or talents.

63. A client, unconscious and unresponsive, is brought to the emergency department. The client's sister reports that the client has been taking alprazolam (Xanax) and has been drinking heavily during the past week. Which treatment should the nurse expect toimplement initially?

Incorrect: The neurological and vital signs will be monitored for central nervous system depression; however, the most important, primary treatment is removal of the toxic substance per gastric lavage and activated charcoal. Correct: Gastric lavage and intravenous fluids are used to flush the toxic substances out of the client's system. Incorrect: Oxygen saturation and pulse oximetry monitoring may be needed for the client with respiratory depression; suctioning is performed for the client with decreased swallowing and pooling of oropharyngeal secretions. However, the gastric lavage for the removal of the toxic substance and intravenous fluids for hydration are the most urgenttreatments. Incorrect: The client may require psychiatric treatment after consciousness is regained and stabilized.

Blood alcohol levels are likely to be obtained. However, gastric lavage for removal of the toxic combination of the alprazolam and alcohol and intravenous fluids for rehydration are essential for survival.

Frequent monitoring of neurological and vital signs Gastric lavage with activated charcoal and intravenous fluids Constant oxygen administration with oximeter and suctioning Psychiatric referral and blood alcohol levels

64. The client is talking to the nurse, and says, "I am the daughter of the King of theUniverse and I talk to him almost every day. Right now, I am mad, so I'm not talking to him." Which is the most appropriate response of the nurse?

Incorrect: This response indicates a confrontation to the client's delusion and may provoke anxiety. Incorrect: This response invalidates the client's feelings, even though it correctly does not support the delusion. Correct: This response does not support the delusion but opens communication to obtain further information. Incorrect: This response is not therapeutic, and the doctor may already be aware of the delusion.

"Surely you don't believe that you are the daughter of the King of the Universe." "I find it difficult to believe that you are the daughter of the King of the Universe." "Tell me more about your belief that you are the daughter of the King of the

Universe." "Have you told your doctor that you believe that you are the daughter of a king?"

65. The nurse instructs the client taking amitriptyline hydrochloride (Elavil) about themedication side effects. Which client statement indicates an understanding of themedication side effects?

Incorrect: Elavil is an antidepressant. Antidepressants do not affect the body temperature. Incorrect: Shortness of breath would not be related to use of an antidepressant such as (Elavil). Incorrect: Auditory hallucinations occur with psychotic disorders, such as delirium or schizophrenia. Visual or tactile hallucinations may result from alcohol or drug toxicity or other physiologic problems, such as organic brain syndrome. Medication can beused to reduce the frequency or intensity of hallucinations. Correct: Drowsiness is one of the most common sideeffects of Elavil and the client must be aware of the possible impact on driving and other tasks requiring alertness.

"I may have a low body temperature." "I may feel shortness of breath" "I may start to see things that aren't there." "I may have extreme drowsiness."

66. The nurse documents the following assessment findings for a newly hospitalizedclient: incoherent statements, marked loosening of associations, and bizarremannerisms with inappropriate affect. Based on the client's assessment findings, thenurse concludes that the client is demonstrating symptoms suggestive of which type ofschizophrenia?

Incorrect: Residual means left-over and may apply to symptoms that are not corrected after treatment. These symptoms are not seen in residual schizophrenia. Correct: These symptoms are generally present in disorganized schizophrenia. Disorganized schizophrenia usually begins in adolescence or young adulthood. Incorrect: The client in a catatonia state is typically rigid, mute and inactive. Incorrect: Suspicion and accusations of harm are most often seen in clients with paranoid schizophrenia.

Residual Disorganized Catatonic Paranoid

67. A client and nurse are in the working phase of the therapeutic relationship. Which behavior should the client demonstrate to reflect progress in therapy?

Incorrect: The goal of therapy is anger management in a healthy and effective manner. Incorrect: Feedback regarding the facilities and staff are not appropriate topics for the client in therapy. It is common for clients receiving therapy to displace feelings of anger or resentment to the care providers. Correct: Insight and problem solving areappropriate tasks of the working phase of a therapeutic relationship.

Incorrect: It may not be realistic for the client toexpect full autonomy to occur during the working phase of the therapeutic relationship.

Expression of feelings of anger over personal situations Feedback regarding feelings about mental health facilities and staff Insight regarding perception of reality and problem-solving Expectations of complete autonomy at end of therapy

68. An angry client says to the nurse, "You did not bring my medicine on time this morning, and you made me late for group therapy." Which defense mechanism is the client demonstrating?

Incorrect: Reaction formation is used when a thought, feeling or impulse is unacceptable to the conscious mind. This is not demonstrated by the client's statement. Incorrect: Displacement transfers emotions from one personto another person who is less threatening. The client's statement does not demonstrate this defense mechanism. Correct: Projection attributes blame to another for one's own faults. Incorrect: Denial is a defense mechanism inwhich the individual refuses to acknowledge unacceptable feelings or denies the existence of an actual situation.

Reaction-formation Displacement

Projection Denial

69. The nurse assesses the client's ability for abstract thinking. Which is the bestinterrogative or statement to obtain this assessment data?

Incorrect: This example of finding a stamped envelope on the street demonstrates concrete thinking, which is defined as thinking that is characterized by immediate experience rather than abstraction. Attention is made to detail rather than generalities. Incorrect: This math problem is an example of concrete thinking. Incorrect: Drawing clock and adding hands is an example of concrete thinking. Correct: The comparison of the traits of bananas and oranges is an example of abstract thinking which is defined as the ability to conceptualize ideas or finding meaning inproverbs.

"What would you do if you found a stamped envelope on the street?" "Please subtract 7 from 100 as many times as you can." "Draw a clock with numbers and hands on it." "Tell me how bananas and oranges are alike."

70. Which is the main difference between delirium and dementia?

Incorrect: Dementia describes a set of chronic, cognitive disorders with a slow onset and irreversible outcome. Correct: The onset is the essential differentiating factor in the assessment of the client. Dementia has a slow, progressive onset, and delirium has a rapid onset. Incorrect: Delirium describes a set of acute forms of cognitivedysfunction with a rapid onset, and with prompt intervention, organic brain damage is usually avoided. Incorrect: Delirium has a rapid onset, and is usually reversible with prompt intervention and treatment.

Dementia has a rapid onset and is reversible. Dementia has a slow onset, while delirium has a rapid onset. Delirium has a rapid onset with organic brain damage. Delirium has a slow onset and may be irreversible.

71. Which classification of drugs is used most often during the acute detoxificationperiod of alcohol withdrawal?

Correct: Anti-anxiety medications, especially the benzodiazepines, are most commonly prescribed for alcohol withdrawal as well as sleep promotion, muscle spasm relief, seizure management, and anxiety reduction. Incorrect: Antidepressants are not usually prescribed during the first 24-48 hours of acute detoxification. Incorrect: Anticonvulsants are used for seizure disorders, not used during detoxification. Incorrect: Antimanic drugs are used

for bipolar disease, not detoxification and alcohol withdrawal.

Anti-anxiety Antidepressants Anticonvulsants Anti-manic

72. A client has symptoms related to alcohol withdrawal. Which symptoms is the client most likely to exhibit?

Correct: Agitation, tachycardia and sweating are classic symptoms of alcohol withdrawal. Incorrect: Euphoria and increased speech may occur with slight alcohol intake, but euphoria is not a symptom at any stage of alcoholwithdrawal. Incorrect: The symptoms of alcoholic intoxication include changes in gait, poor coordination and speech difficulties. Incorrect: Symptoms of excessive alcoholic consumption includes drowsiness, falling asleep and lethargy.

Agitation, increase of pulse rate, and diaphoresis Euphoria, rapid speech, and increased energy Unsteady gait, muscular incoordination, and slurred speech Relaxed manner, sleepiness, and decreased activity

73. The nurse providing education to the child and family with oppositional defiantdisorder recommends a therapeutic plan for care including: structure in the daily routine, teaching of coping and problem-solving skills, and consistent, loving limits for behavior. Which behaviors may indicate to the nurse and family that further intervention isneeded?

Correct: Oppositional defiant behavior is a diagnosis given to children who act in an intentionally disruptive manner, such as: provocative, argumentative, and defiant behavior. These children are easily angered and retort with aggression and destructiveness. It is believed that an oppositional defiant is rare in a home with parent(s) whoprovide consistency in discipline and expectations for family life. Incorrect: Inattentiveness, impulsivity, and hyperactively are characteristics of attention-deficit hyperactivity disorder. Incorrect: Separation anxiety disorder is a type of anxiety disorder whereby the child has excessive and developmentally inappropriate anxiety concerningseparation from the home or attachment figure. Persistent reluctance or refusal to go to school is common. Repeatednightmares regarding the separation occurs. Others fear harm or loss of the attachment figure during the period ofseparation. Incorrect: Past and present suicidal ideology is atypical for the child with oppositional defiant behaviordisorder.

Provocative behavior Inattentiveness Excessive distress with separation History of suicide attempt

74. A client with hypochondriasis makes frequent calls and visits to the primary care

provider with various physical complaints, particularly related to heart palpitations, breathing, and digestion. Which nursing diagnosis is most appropriate for the clientdiagnosed with hypochondriasis?

Incorrect: Clients who engage in denial tend to minimize somatic symptoms and often delay seeking health care. Conversely, those clients with hypochondriasis seek excessive health care for minor or imagined physical problems. Incorrect: There is no indication that the client is experiencing pain. However, clients with hypochondriasis have preoccupation with the symptoms of illness. Incorrect: The client does not exhibit personal identity disturbance, which is exemplified by difficulty in decision-making and confusion about the sense of self. Correct: Clients withhypochondriasis isolate family and friends due to the preoccupation with bodily functions and physical ailments. Personal resources, such as time and money, are spent on medication, health care, and treatment.

Ineffective denial related to feelings of anxiety Pain related to physiological symptoms Personal identity disturbance related to loss of body function Social isolation related to preoccupation with illnesses

75. Chloral hydrate (Noctec) is ordered for an elderly, hospitalized client. The administration of this medication is contraindicated in which condition?

Incorrect: Paranoid behavior is a side effect of chloral hydrate. Correct: Chloral hydrate is contraindicated for patients with renal or hepatic impairment. The drug is metabolized in the liver and excreted in the urine. Incorrect: Chloral hydrate is not used to treat chronic psychosis. Incorrect: Chloral hydrate is often used for alcohol withdrawal.

Paranoid behavior Renal impairment Chronic psychosis Alcohol withdrawal

ATI. FUNDAMENTAL

1. Which client is most at risk for impaired skin integrity?

Incorrect: This client may be at risk because of the implied skeletal traction related to the fractured femur, but this client is not the highest risk.Incorrect: Just because a client has diabetes does not make him at risk for skinproblems, although skin breakdown may occur if proper hygiene is not followed.Correct: The woman has two factors that make her at risk: age and immobility.Incorrect: This client is not at risk for skin breakdown. A 60-year-old female who sustained a fractured femur in a car accident

A 70-year-old male who has recently been diagnosed with diabetes An 85-year-old woman who is bedfast related to a stroke A 55-year-old male who was admitted to the ICU with a heart attack

2. When administering a medication, the nurse must follow the "five rights." Which isNOT one of the "five rights?" Incorrect: The five rights include the right drug, the right dose, the right route, the right time, and the right patient. The question reads--which is NOT one of the five rights. Proper identification of the patient is correct and is one of the five rights.Correct: The five rights include the right drug, the right dose, the right route, the right time, and the right patient. The question reads--which is NOT one of the five rights--Documentation of the medication is not considered one of the five rightsIncorrect: The five rights include the right drug, the right dose, the right route, the right time, and the right patient. The question reads--which is NOT one of the five rights--the right route is one of the five rights.Incorrect: The five rights include the right drug, the right dose, the right route, the right time, and the right patient. The question reads--which is NOT one of the five rights--Right time is one of the five rights.

Identification of the client by using the wrist band Documentation of the medication prior to administration

Administration of the drug via the prescribed method Administration of the medication within the time frame it is ordered

3. Which client is at greatest risk for a nosocomial infection? Incorrect: Even though the client has a NG tube, they usually are not the cause of nosocomial (hospital acquired) infections.Incorrect: The mouth contains microorganisms. Suctioning usually does not lead to infections.Incorrect: If a client is admitted with pneumonia, then he can't acquire it in the hospital.Correct: This client has had surgery and had a catheter placed. Both are invasive procedures. The client is a very high risk for acquiring a nosocomial infection. A client who has a NG tube connected to low, intermittent suction A client who requires frequent suctioning of the mouth A client who is admitted to the hospital with pneumonia A client with an indwelling catheter inserted during surgery

4. A client recovers from abdominal surgery. Which can the nurse expect regarding the client's bowel sounds on the third post-operative day? Incorrect: Peristalsis should resume by the second postoperative day.Correct: These are the normal bowel sounds for the third postop day as peristalsis resumes.Incorrect: This is an abnormal bowel sound.Incorrect: Highpitched bowel sounds are an indication of a complication and are abnormal.

Absent Normal gurgling Hyperactive High-pitched

5. A client is to receive a lumbar puncture. In which position should the client's back be placed? Incorrect: Extension is the straightening of a body part and is not appropriate in this situation.Incorrect: The client would have to arch his back for this position to be possible and this is not the proper position for a lumbarpuncture.Correct: Bending the back so that the chin rests on the knees is the proper position for this test. This increases the space between the vertebrae providing easier access for spinal fluid withdrawl. Care must be taken tomonitor respiratory function during the test.Incorrect: This means turning side to side which is not an appropriate position. Extension Hyperextension Flexion Rotation

6. A client with an eye infection receives instructions regarding proper administration of the eye ointment. Which method is essential for preventing eye infections when usingeye ointment from a multiple dose applicator? Incorrect: This is a correct step in medication administration but has nothing to do with preventing

infection.Incorrect: Ointment should be applied from the inner to the outer canthus of the eye, and does not pertain toprevention of eye infections.Incorrect: Washing the eyelids may rid the lashes of crust and drainage, but does not prevent infection. Different parts of the wash cloth should be used for each eye.Correct: The applicator must remain sterile. Subsequent use of the ointment would reintroduce organisms back into the eye. Place a thin ribbon of medication in the lower eyelid. Apply the ointment from outer canthus toward the midline. Wash the eyelids with warm water before applying the ointment. Avoid touching the applicator to the eye or lids.

7. A client in the immediate post-operative recovery period has urinary retention. At which point should the nurse notify the physician?

Incorrect: If the client has urinary retention, the client should void within 8 hours. At this point, (3-4 hours) the nurse should continue to monitor urine output.Incorrect: The time is too soon to evaluate urinary retention.Correct: If the client has not voided within an 8 hour period the physician needs to be called. Catheterization may be ordered. Overstretching of the bladder can lead to nerve damage.Incorrect: The time is too long. Damage may have occurred already.

After three to four hours After one to two hours After six to eight hours After ten to twelve hours

8. A client dying of cancer states to the nurse, "I really don't know why this is happening to me. Why can't it happen to someone else?" Which stage of death and dying does thisreflect? Incorrect: There is no indication that the client is depressed.Incorrect: Bargaining is characterized by the need to bargain for more time. This client is not asking for more time.Incorrect: In this stage the client refuses to participate in treatment or discussions because he doesn't believe it is real. There is no indication that this is occuring.Correct: Anger is looking for a cause or placing blame on someone else. Why me statements are very common in this stage.

Depression Bargaining Denial Anger

9. The nurse plans to obtain a urine specimen from a client with an indwelling catheter. Which technique is most appropriate for collection of this specimen? Incorrect: Disconnecting the catheter should never be done because of the risk of contamination. This is a closed sterile system.Incorrect: Puncturing the catheter itself with a large gauge needle will cause it to leak, increasing the risk for infection.Correct: This is the correct technique.Incorrect: Specimens should NEVER be collected from the drainage bag because the urine may be hours old and contaminated.

Disconnect the catheter from the drainage tubing and drip the urine into a sterile container. Withdraw the specimen from the catheter using a large gauge needle. After clamping the tubing, collect the specimen from the special port with a sterile needle. Collect the specimen from the urine collected in the drainage bag.

10. Family members have gathered at the bedside of a dying client who is unresponsive, cool to touch, and mouth breathing. A family member asks the nurse what the family can do to keep the loved one comfortable during the last hours of life. Which comfortmeasure is most appropriate? Incorrect: It is unlikely that a family member would suction his loved one. Oral secretions dry when death is imminent.Incorrect: As death draws closer the body temperature will continue to drop. The client is already cool to touch. Several blankets are more appropriate.Incorrect: Turning and repositioning the client in a comfortable position is an important intervention for comfort, skin integrity, and respiratory care.Correct: Mouth breathing causes the lips and tongue to become dry and cracked. Oral secretions decrease as one approaches death. A moist swab promotes comfort and prevents cracking and bleeding of the lips.

Suction the mouth when necessary. Cover the client with a light sheet. Maintain the client in a supine position. 11. In the immediate postoperative period, the nurse can expect the client's wounddrainage to be: Correct: Sanguineous drainage is bright red or bloody.Incorrect: Serous drainage is clear, pale yellow, and is the result of clotted blood.Incorrect: Purulent is pus, which is a sign of infection. Pus may be green, tan or white incolor and may be foul-smelling.Incorrect: Serosanguineous is pinkish red and is not seen until several days after surgery.

sanguineous. serous. purulent. serosanguineous.

12. Which of the following are nutritional requirements for the older adult? Incorrect: Carbohydrates and minerals are not increased in the older adult.Incorrect: Calories are usually less because of the decreased metabolism.Incorrect: Proteins and vitamins are not increased for the older adult.Correct: Older adults are prone to dehydration and constipation. Therefore, increasing fiber and fluid will help prevent these conditions. Increased carbohydrates and minerals Increased calories and fat Increased protein and vitamins Increased fiber and fluid

13. A male client fractured his lower leg in a car accident and is hospitalized related to his injuries. Which nursing intervention is most appropriate when providing care for aclient with a newly applied cast? Incorrect: Petals should be applied to prevent skin breakdown, but this is not the most appropriate intervention.Incorrect: The chemical reaction of the plaster in the newly applied casts makes them warm, but as

drying occurs, they are cold. Monitoring temperature (esp. with the elderly) is important but not the most importantintervention.Correct: Frequent checks for circulation and nerve function are very important for a client with a newlyapplied cast. Also because the client is hospitalized, frequent assessments are needed.Incorrect: Measuring the client for crutches is not a priority. Cover the cast edges with petals. Monitor the client's environmental temperature. Perform frequent neurovascular checks. Measure the client for crutches.

14. When providing care for a client in Buck's skin traction, which nursing intervention ismost appropriate? Incorrect: Countertraction should never be released unless there is a physician's order.Incorrect: One indication for Buck's traction is to provide immobilization for fractured hips prior to surgical repair. Turning is not appropriate.Correct: Assessment should be done to provide a baseline for further care.Incorrect: Massaging the extremitiy is contraindicated because of the risk of dislodging a clot. Release countertraction for five minutes every eight hours. Turn the client from side to back every two hours. Assess for nerve and circulatory disturbances every four hours. Massage the extremity in traction every four hours.

15. Which is a major reason for or advantage of giving medications orally? Correct: Oral medications are more economical and more convenient to take. These are major

considerations, but the most consideration should be given to the fact that the oral route is usually thesafest.Incorrect: Oral medications are the slowest to absorb. Absorption depends on age (slower for elderly), the type of medication (liquid vs tablets), and GI function.Incorrect: Oral medications, if not properly administered, may causetissue damage (ulcers).Incorrect: Oral medications are absorbed erratically. Absorption depends on age, type of

medication, and GI function, (renal function and liver function also have a profound effect on the effectiveness or toxicity of the drug).

Oral medications are more economical. Oral medications are absorbed rapidly Oral medications do not cause tissue damage Oral medications are absorbed more consistently.

16. A nurse discovers a fire on a busy medical unit. Which action should be given priority? Incorrect: This is an appropriate action but done after seeing to the clients' security.Incorrect: This is appropriate but only after the clients are cared for.Correct: When there is a fire, the clients who are in the immediate vacinity of the fire MUST be removed first. The safety of the clients is the top priority.Incorrect: This is appropriate but only after the safety of the clients.

Sound the fire alarm to notify hospital personnel. Close the doors to all client rooms. Move any clients who are in close proximity to the fire. Telephone the operator and report the fire.

17. When donning sterile gloves, the fingers of the second glove inadvertently brush theoutside edge of the glove wrapper. Which should the nurse do next? Incorrect: The glove has been contaminated and should be discarded.Correct: One glove is already on. It is very difficult to open packages with one hand (unsterile). The likelihood of contamination to the sterile glove is great. Starting over is wise.Incorrect: Unless someone is in the room, the nurse will have to walk out of the room to getassistance. The risk of contamination to the gloved hand is high.Incorrect: In order to get the contaminated glove on, it would have to come in contact with the sterile glove, thereby contaminating both gloves.

Continue to don the gloves. Discard the gloves and get a new pair. Ask someone to assist with donning a single glove. Designate one hand as sterile and the other as unsterile.

18. The physician orders heparin subcutaneously for a client scheduled for surgery. Which is the preferred method of administration? Incorrect: Heparin can be given only subcutaneously or by IV. Firm pressure is contraindicated. Injecting into the gluteal muscle indicates an IM injection, which is not the appropriate routeIncorrect: Heparin should neither beinjected into a muscle nor massaged afterwards.Correct: Heparin may be given subcutaneously into the fatty tissueabout two inches from the umbilicus using a small needle.Incorrect: Heparin is not given IM and should not beaspirated.

Inject into the thigh muscle followed by firm pressure for 30 seconds. Inject into the deltoid muscle, massaging afterward. Inject into the fatty tissue of the abdomen using a 27 gauge needle. Inject into the gluteal muscle, aspirating after the needle has penetrated the skin.

19. A 70-year-old client is admitted to the hospital after suffering from vomiting and diarrhea for three days. The client reports weakness and leg cramping. The client'sserum electrolyte report revealed that the potassium is 2.5 mEq/L and sodium is 135 mEq/L. Based on the client's clinical profile, which can the nurse conclude? The clienthas: Incorrect: Hyperkalemia indicates a potassium level that is too high. Normal potassium levels range from 3.5-5.0 mEq/L.Incorrect: Hyponatremia is a sodium level that is too low. The sodium level in this question is normal. Normal sodium levels range from 135-145 mEq/L.Incorrect: Hypernatremia is a sodium level that is too high.Correct: A potassium level less than 3.5 mEq/L indicates hypokalemia.

hyperkalemia.

hyponatremia. hypernatremia. hypokalemia.

20. A female nurse is employed by a large hospital and uses her password to look up data about her neighbor who was recently admitted to the intensive care unit (ICU). Howshould the nurse's action be interpreted? Incorrect: Defamation is where untrue information harms a person's reputation, which did not occur in this situation.Incorrect: Malpractice is a lawsuit alleging that negligence occurred--this is not the case.Correct: The nurse violated the client's rights to confidentiality.Incorrect: Libel is a damaging written statement that harms a person's reputation.

As a defamation of character As malpractice As an invasion of privacy As a condition of libel

21. A client with a below the knee amputation receives instruction regarding how to wrap the stump. Which instruction regarding stump bandaging is most accurate? Incorrect: The joint should be in correct anatomical position. If not, contractures may occur.Incorrect: Circular

turns will cut off the circulation and hinder the healing to the stump incision.Incorrect: Compression should be the same tightness.Correct: Distal (distance) means further away, while proximal means closest to the trunk. The stump should be wrapped from bottom to top. Wrap the joint in the extended position. Wrap the stump with circular turns to promote circulation. Wrap the bandage with compression tighter at the top of the stump. Wrap the stump distal to proximal to promote venous circulation.

22. Postural drainage is ordered for a client for 20 minutes, three times per day. When should the nurse plan to perform the postural drainage? Incorrect: This is too soon after the meal and may cause indigestion and discomfort for the client (especially if the client is elderly because of slower digestion).Incorrect: This is too close to meal time and the client may be expectorating secretions or be too tired to eat.Correct: The appropriate time frame for postural drainage is 1 1/2 to 2 hours after meals when digestion has occurred.Incorrect: This is not the appropriate time frame, especially in the morning before breakfast when the client would have to be awakened.

45 minutes after each meal 15 minutes before each meal 90 minutes after each meal

2 hours before each meal

23. Which situation requires an incident report? Incorrect: This situation does not indicate that an unusual event has occurred.Correct: An incident report is a written account of an unusual event that involves a client, employee or visitor that has the potential for being injurious. Incident reports are used to prevent hazardous situations or for future litigation. Falls are injurious and should bedocumented on an incident report.Incorrect: This situation does not require an incident report.Incorrect: This situation does not require an incident report.

A client complains of insomnia for three nights while hospitalized. A client with dementia falls from the bed onto the floor. A family member complains that the care at the facility is substandard. An employee is habitually late arriving for the assigned shifts.

24. A nurse suspects that a client has dysphagia. Clients may be diagnosed with dysphagia if which condition occurs? Incorrect: Slurred speech is not indicative of dysphagia but of a stroke.Incorrect: This is not an indication of dysphagia.Correct: Signs and symptoms of dysphagia (difficulty swallowing) include drooling and coughing.Incorrect: This is a sign of dysphasia which is difficulty speaking (note the difference in spelling).

Slurred speech Protrusion of the tongue Excessive drooling Inability to form and pronounce words

25. A client experiences persistant nausea because of cancer treatment. The nurse provides instructions for the client to ensure that adequate nutritional requirements aremet. Which instruction is most appropriate? Incorrect: The technique of chewing food slowly will not benefit the client with persistant nausea.Incorrect: Foods should be appetizing. Blending foods together would not help the client's nausea and most likely would causevomiting because of the unappealing sight.Correct: Frequent amounts of clear, cool beverages such as white soda pop, or ginger ale may settle the stomach allowing the opportunity for small nutritious meals.Incorrect: Increasing thefluids at every meal will decrease the solid food intake and contribute to a feeling of fullness.

Chew food slowly.

Blend foods together after cooking. Drink clear, cool beverages. Increase the amount of fluids consumed with every meal.

26. Which method of medication administration is the most rapid? Incorrect: Intradermal injections (like TB injections) are given just under the skin, therefore the absorption time is slow.Incorrect: Intramuscular injections penetrate the muscular layer, and therefore, IM medications are slow to absorb.Correct: This route provides the most rapid absorption because medications are injected directly into the bloodstream by the needle located in the vein.Incorrect: Medications are injected into the fatty tissue and thereforeabsorb slower than the IV route.

Intradermal Intramuscular Intravenous Subcutaneous

27. A diabetic client has a shallow pressure ulcer on the left heel with distinct edges and drainage. The ulcer is surrounded by an area of warmth, erythema, and edema. This ischaracteristic of which stage? Incorrect: Stage 4 ulcers are characterized by deep craters, because of damage to the muscle and bones.

Infection is most always present.Incorrect: Stage 3 ulcers involve the subcutaneous tissue and form deepcraters.Correct: These signs and symptoms are most indicative of a Stage 2 ulcer.Incorrect: Stage 1 ulcers involve the top layer of skin and are characterized by redness that does not go away after two hours.

Stage 4 Stage 3 Stage 2 Stage 1

28. The nurse prepares to change a sterile dressing and drops sterile dressing suppliesonto the sterile field. One of the 4 X 4 gauze pads falls 1/2 inch from the edge of thesterile field. Which action is indicated in order to maintain the sterility of the dressing? Incorrect: It is not necessary to discard the entire field when only the gauze is contaminated.Correct: The gauze should not be used because it fell outside the one inch boundary. The outside inch is considered

contaminated.Incorrect: The gauze is contaminated and must not be used.Incorrect: Moving the dressing will further contaminate the supplies and the sterile field.

Discard the entire sterile field. Avoid using the 4 X 4 gauze. Use the 4 X 4 gauze as the dressing. Move the 4 X 4 gauze pad to the center of the field with gloved hands.

29. Which foods are high sources of carbohydrates? Incorrect: Hot dogs are a source of protein. Although fries are made from potatoes (a starch), the fat content is so high that this food selection is considered primarily a fat. Ice cream contains a high fat gram count.Incorrect: Thefood selection of olives and peanuts contain a high fat load. Tuna is primarily a low-fat, high-protien food.Correct: Lettuce is a fiber food which is considered a complex carbohydrate. Cake is a simple carbohydrate. A milk sugar, lactose, is a component of skim milk.Incorrect: Liver is a food high in protein. Rice is a complex carbohydrate.

Hot dogs, french fries, and ice cream Olives, peanuts, and tuna Cake, skim milk, and lettuce Liver, onions, and rice

30. Which position is most appropriate for a client who is having difficulty breathing? Incorrect: Lateral is side-lying and will not help breathing.Incorrect: Sims positioning is a left lateral side-lying position with the knee close to the chest. This position would not be beneficial for breathing.Correct: This position is a seated, upright position that eases breathing by causing better lung expansion. The abdominal organs are not pressing on the diaphragm restricting diaphragmatic movement and lung expansion.Incorrect: Supine is lying flat onthe back. This position does not benefit breathing.

Lateral Sims High Fowler's Supine

31. A physician orders physical restraints for a client. Which is the main reason physical restraints are necessary? Incorrect: OSHA monitors worksites and job related injuries.Incorrect: This is one reason for the use of restraints, but not the main one.Correct: Restraints are devices that restrict movement and are used for protection ofthe client, others or property.Incorrect: While nurses want to prevent falls and accidents, this is not the main reason for use of restraints.

To meet the requirements for the Occupational Safety and Health Administration (OSHA) To prevent confused clients from wandering outside the facility To prevent injury to the client, to others or to property To improve safety on the unit

32. A home health nurse investigates reasons for a 76-year-old female client's incontinence. The client tells the nurse that she is unable to reach the bathroom in time because her wheelchair is hard to manipulate through the door of the bathroom. Whichtype of incontinence should the nurse document? Incorrect: This type of incontinence is a result of dribbling related to sneezing, coughing, or lifting.Correct: Functional incontinence is the inability to get to the bathroom in time related to environmental barriers.Incorrect: This type of incontinence is related to damage to motor and sensory tracts in the nervous system.Incorrect: This type of incontinence is a result of the perception of the need to frequently void.

Stress Functional Reflex

Urge

33. A client receives gastrostomy tube feedings. Which nursing intervention will help prevent aspiration? Incorrect: Gastric residuals should be checked every four hours, before feedings or per doctor's orders.Incorrect: Elevating the head of the bed is correct, but 90 degrees is in a sitting position which is toohigh.Correct: This method allows a slow, continuous infusion of formula allowing for more complete digestion. Bolus feedings on the other hand have a high probability of aspiration due to rapid infusion.Incorrect: Bolus feeds are givenwithin a short time and have a higher possibility of aspiration than continuous feeds.

Measure gastric residuals once per day. Elevate the client's head of bed 90 degrees after feedings. Use a continuous method of formula delivery. Give bolus feeds at 40 cc's per minute.

34. "Impaired skin integrity related to stomal drainage" is a nursing diagnosis for a clientwith a colostomy. Which nursing intervention is most appropriate for this nursing diagnosis? Incorrect: Skin should be washed with soap and water and patted dry. Betadine is not needed.Incorrect: The appliance should remain on for as long as the bag remains intact--this may be up to a week. Replacing the appliancedaily will put the client at risk for skin breakdown.Incorrect: The faceplate should be the same size as the

stoma.Correct: Skin barrier will help prevent irritation and promote skin integrity.

Cleanse the skin around the stoma with betadine swabs daily. Replace the appliance every day. Trim the opening in the faceplate 1/4 inch smaller than the stoma. Apply a protective skin barrier around the stoma before replacing the appliance.

35. A client is scheduled for surgery. When may the nurse safely document that the clienthas informed consent about the surgery? Incorrect: The surgeon and anesthesiologist may have explained the procedure, but this does not indicate that the client understands it (anxiety, confusion).Correct: Informed consent must be given by the client/family prior to any procedure or surgery. Informed consent requires three parts: the ability of the client to make a rational decision, comprehension of the risks and alternatives, and the consent was signed without intimidation or coercion.Incorrect: This is an understanding of the teaching not the procedure itself.Incorrect: The operative permit is different from the informed consent.

When both the surgeon and anesthesiologist have discussed the procedure with the client. When the client can give an explanation of the surgery, the risks, and alternative

treatments. When the client can repeat post-operative instructions regarding breathing exercises and diet changes. When the client has read and signed the operative permit.

36. The nurse is obtaining a wound culture of a client's infected pressure ulcer using aculturette tube with an enclosed swab. Which technique is most appropriate whenobtaining the culture? Incorrect: The outside of the culturette will be handled by several people. This is not important when obtaining the culture.Incorrect: Wearing sterile gloves is not necessary. Clean gloves are required to obtain a culture and to maintain standard precautions.Correct: In order to get an accurate report of the causitive organism, the insideof the culturette and the lid must be kept sterile to prevent outside contamination.Incorrect: Documenting the colorand amount is important, but not necessary when obtaining the culture itself.

Keep the outside of the tube as uncontaminated as possible. Wear sterile gloves when collecting the specimen.

Keep the inside lid and the tube sterile. Document the color and amount of the drainage obtained.

37. A diabetic client receives Humulin N insulin. When should the nurse expect the peak action to occur? Incorrect: Humulin N insulin is an intermediate acting insulin with peak action between 8-12 hours This time is too long.Incorrect: Intermediate acting insulin has a peak action of 8-12 hours. Therefore, this time is too short.Correct: Humulin N insulin is an intermediate acting insulin with peak effects seen within 8-12 hours.Incorrect: Intermediate acting insulins have peak effects between 8-12 hours. The time frame here is too short.

Eighteen to twenty-four hours One to two hours Eight to twelve hours Four to six hours

38. Which instruction should be given to a client who is drawing up short and long actinginsulins in the same syringe? Correct: Regular insulin (clear, short acting) contains no additional proteins and therefore is drawn up first so that it is not contaminated by insulin that contains proteins (cloudy, long acting).Incorrect: Clear insulin is drawn up

first.Incorrect: Drawing up from the fullest bottle first makes no difference.Incorrect: Insulin need not be refrigerated but must be protected from temperature extremes.

Draw up the clear insulin first, then the cloudy insulin. Draw up the cloudy insulin first, then the clear insulin. Draw up the insulin from the fullest bottle first. Draw up the insulin that is room temperature first, followed by the one that is cold.

39. A male client has recently been diagnosed with hypertension and was told by the physician that he must limit his salt intake, lose fifty pounds, and exercise four times perweek. The nurse overhears the client state, "I don't like eating foods that don't have salt. That can't make that much difference in my blood pressure, and I hate to exercise. Itmakes my blood pressure go up when I work that hard. My brother weighs more than Ido, and he doesn't have high blood pressure." Which is the most appropriate nursingdiagnosis for this client? Incorrect: This diagnosis may be accurate, but is not the most appropriate.Correct: The client clearly does not understand the correlation between diet, exercise and blood pressure and needs further teaching.Incorrect: This is not the priority diagnosis at this time.Incorrect: To this point there is no indication that the client has been noncompliant. The risk for noncompliance is high, however.

Denial of the disease and diet treatment Knowledge deficit of disease process and therapeutic regimen Ineffective individual coping of disease and treatment Noncompliance with therapeutic treatment regimen

40. Which is the main rationale for elevating a newly applied cast on pillows as it isdrying? Incorrect: The cast normally does not crack during drying.Incorrect: Elevation of the extremity on pillows may reduce the edema which causes pain, but this is not the main reason for the use of pillows.Correct: Positioning a wetcast on a firm surface will cause the cast to be flat, which could place pressure on the leg and interfere with bonealignment.Incorrect: Plaster flakes will occur whether or not pillows are present.

Prevent the cast from cracking related to rapid drying Reduce pain and provide comfort for the client as the bone becomes realigned Prevent the changing of cast shape and interference with bone alignment

Prevent the accumulation of plaster flakes inside the cast and bed

41. An older adult is hospitalized because of dehydration. The nurse assesses theclient's skin turgor and gently pinches the client's skin over the sternum. Which shouldthe nurse expect? Incorrect: Normally clients with dehydration have hot, dry skin.Correct: Tenting of the skin is often an indication of dehydration.Incorrect: Good skin turgor is an indication of adequate hydration.Incorrect: The skin of the client with dehydration is often flushed.

Skin will feel cool and clammy to touch. Skin will return slowly back into position. Skin will return rapidly back into position. Skin and mucous membranes will be pale.

42. A elderly client falls and fractures her arm. Which information regarding fractures in the older adult is most important for the nurse to consider? Incorrect: Older adults have a decreased pain tolerance.Incorrect: There is no evidence that older adults have an increased number of allergic reactions to casting materials.Correct: The loss of subcutaneous fat and the fragile nature of the skin, causes irritation that may be more troublesome in older adults.Incorrect: Greenstickfractures occur most frequently in children.

Older adults have an increased tolerance to pain related to the fracture. Older adults have a higher number of allergic reactions related to the casting materials. Older adults are more susceptible to skin irritation from the cast. Older adults have a higher incidence of greenstick fractures.

43. A client returns from the operating room with a colostomy. Which is the expected color of the stoma in the immediate post-operative period? Incorrect: A few weeks after surgery the colostomy should be pink and moist, but immediately after surgery, the color should be red. Incorrect: Pale white indicates a circulatory problem to the tissue. Correct: Due to themanipulation of the bowel, the color should be bright red. It is not unusual for there to be a scant amount of blood inthe appliance bag directly after surgery. The stoma color will change to pink a few weeks after surgery. Incorrect: Thisis a serious problem and is related to tissue necrosis due to poor circulation.

Pink Pale white Beefy red Bluish purple

44. The nurse encourages a post-operative client to cough, turn, and breath deeply. Which post-operative complication can these exercises prevent?

Incorrect: Thrombus (clot) formation usually begins in the lower extremities, breaks loose and travels to the lungs. Other methods are used to prevent clot formation.Incorrect: Pulmonary edema is not a common complicationduring the post-anesthesia period.Incorrect: Pleurisy is an inflammation of the lining of the lungs and is not a commonpost-anesthesia complication.Correct: Pulmonary infection or pneumonia is a common problem for the client(especially the elderly) in the postanesthesia period related to pain and immobility. Breathing exercises promote the removal of respiratory secretions and inflate the alveoli.

Pulmonary embolism Pulmonary edema Pleurisy Pulmonary infection

45. Which method is most appropriate when administering a extended-release tablet?

Incorrect: Timed release tablets should never be chewed.Incorrect: The tablet coating controls the slow release of the medication and would be disrupted if the tablet was broken in half.Correct: The only way to give a coated tablet that supports the slow release of medication is whole with the coating intact.Incorrect: Crushing the tablet would interrupt the coating and controlled release of the drug would be lost. Instruct the client to chew the tablet before swallowing. Break the tablet in half and administer it in 2 pieces. Administer the tablet whole. Crush the tablet.

46. Three days after returning home from having abdominal surgery, the client coughs and feels a pulling of the incision followed by green drainage. The client calls the clinicand speaks to the nurse who suspects wound dehiscence. Which nursing response ismost appropriate? Incorrect: This intervention will not address the main problem of wound infection/dehiscence.Correct: Wound dehiscence is considered a medical emergency and the client should have immediate attention.Incorrect: This intervention may be an option if the towel were clean and the drainage heavy but the client still needs immediatehelp.Incorrect: Fever is a sign of infection but is not appropriate in this situation.

"Wipe away the drainage and lie down." "Come to the clinic right away." "Put a towel over the wound." "Take your temperature."

47. A client had abdominal surgery three days ago. Which assessment finding should be reported to the surgeon? Incorrect: This is a normal sign of wound healing.Incorrect: Pink wound edges are normal unless they are accompanied by warmth, edema and drainage.Incorrect: Bruising often occurs around the incision site and thereforeis not considered abnormal unless the location becomes hard. It needs to be documented, however.Correct: This is an indication that wound infection is present. The signs of wound infection include: redness, edema, drainage and lack of approximation of the wound edges.

The incisional edges are closely approximated. The incisional edges are pink and cool to touch. There is bruising surrounding the incision. The skin around the incision is warm and edematous.

48. The nurse educates a client with a transverse colostomy about the normalconsistency of the stool following a colostomy. Which consistency should the client expect? Incorrect: A function of the large intestine is to absorb water. In soft-formed stools, water has been absorbed which molds the feces to the diameter of the bowel.Correct: The closer the colostomy is to the cecum (first part of the large intestine) the more liquid the stool because thewater has not had a chance to be absorbed.Incorrect: Too much water has been absorbed for a

transverse colostomy.Incorrect: Stools should be liquid if there is a transverse colostomy. Soft formed Liquid to semisolid Dry, stone-like Hard formed with large water loss

49. Which technique is most appropriate when suctioning a client who has atracheostomy? Incorrect: Wall suction should be adjusted to 100-140 mm Hg.Incorrect: Suctioning is considered a sterile procedure and instilling normal saline is not indicated unless mucus is tenacious.Correct: This is appropriate.

Rotating the catheter prevents injury to the mucosa of the trachea.Incorrect: Administering oxygen is not necessary during suctioning. Tracheostomy collars are used to administer oxygen.

Adjust wall suction to 150-170 mm Hg of pressure. Don clean gloves prior to suctioning and insert 10 mL of normal saline into tracheostomy. Rotate the suction catheter using suction as it is being removed from the tracheostomy. Administer oxygen at 2 L/min per nasal cannula while suctioning the tracheostomy.

50. A physician orders the following: Morphine sulfate 10 mg q 3-4 hours IV PRN for pain. The medication is available as 1/4 grain per 1 mL. Which dosage should the nurseadminister? Incorrect: The dosage is incorrect.Incorrect: The dosage is incorrect.Incorrect: This is an incorrect dosage.Correct: The conversion is 60 mg/ grain (15 mg/1/4 grain) or 15 mg in 1 mL. 1.0 mL .4 mL 2.1 mL .6 mL

51. Which landmarks should be identified if the nurse plans to give an intramuscularinjection in the dorsogluteal area? Incorrect: This landmark is used for injections into the deltoid muscle.Incorrect: This method is used for injections in the ventrogluteal site.Incorrect: This landmark is used if the injection is in the vastus lateralis site.Correct: This is the appropriate landmark for an injection in the dorsogluteal area. The lower edge of the acromion process and the axilla The greater trochanter of the femur and the anterior superior iliac spine The greater trochanter of the femur and the top of the thigh The posterior iliac spine and the greater trochanter of the femur

52. Which type of injection requires that the needle be inserted at a 15 degree angle? Incorrect: Intramuscular injections should be given at a 90 degree angle.Correct: This is the correct angle for this type of injection.Incorrect: IV needles are inserted at skin level.Incorrect: Subtaneous injections should be givenat a 45 degree angle. Intramuscular Intradermal Intravenous Subcutaneous

53. Which complication of a colostomy is generally considered a long-termcomplication? Incorrect: Infection can occur at any time.Correct: Stomal prolapse is caused from increased pressure within

the abdominal cavity related to coughing or sneezing or because the surgical incision is too large for thestoma.Incorrect: Necrosis is a sign that circulation is impaired.Incorrect: Bleeding may occur if there is irritation to themucous membranes but is not considered a complication unless it becomes severe. Infection Stomal prolapse Necrosis Bleeding

54. Which is the main purpose of the state Nurse Practice Act?

Correct: The Nurse Practice Act protects the public by defining the scope of nursing in that state, providing rules for disciplinary actions and regulating licensure requirements.Incorrect: This is only one component of the NursePractice Act and is not the main purpose.Incorrect: The Nurse Practice Act does set standards for practice, but this is not the main purpose.Incorrect: The State Board of Nursing regulates licensure requirements, but there are other purposes to its function.

Protect the public Disciplinary action for those with violations Describe nursing standards Issue a license

55. When performing post-mortem care, which nursing responsibility should be carriedout first? Incorrect: The family may not wish to stay with the body. Also in some facilities, such as longterm-care, the physician does not pronounce the person dead.Incorrect: This is a responsibility, but not initially.Correct: Providing emotional support to the family is priority-- also cleaning up the body for family viewing is necessary.Incorrect: This is a nursing responsibility, but not an initial one.

Arrange for the family to stay with the body until the physician pronounces the person dead.

Contact the clergy of the family's choice. Care of the body and care of family members. Gather legal documents from the client's chart before calling the funeral home.

56. A physician's order reads: Instill one drop OS BID. The nurse should administer the drop in: Incorrect: The abbreviation for the right eye is OD.Correct: This is the correct abbreviation for the left eye.Incorrect: This order does not include the abbreviation for the left ear.Incorrect: The abbreviation for both eyes is OU.

the right eye. the left eye. left ear. both eyes.

57. A client admitted to the hospital is found to have scabies, a contagious skin disease caused by a mite. Which type of isolation should be provided for the client? Incorrect: This type of precaution is not needed for scabies. No drainage is present.Incorrect: This type of precaution is not needed to prevent the spread of scabies.Incorrect: Scabies is not transmitted by air.Correct: Contactisolation requires gowning and gloving to prevent the spread of the scabies.

Drainage Wound Airborne Contact

58. The physician orders cortisone acetate 60 mg. IM stat. Which does the term "stat" mean? Incorrect: Stat orders should be carried out immediately. Correct: The term stat means give immediately. Stat orders usually indicate that the condition is life threatening and treatment is urgent. Incorrect: Stat orders aregiven at once. Incorrect: Stat orders should be given priority and not given several hours later.

Give no later than thirty minutes after the order is written. Give as quickly as possible. Give when convenient. Give with the next prescribed dose.

59. A 72-year-old female client is admitted into the hospital. The nurse takes a baseline set of vital signs and documents the following: temperature 99.3 degrees Fahrenheit, pulse 88, respirations 22, and blood pressure 160/92. Which vital sign should the nursereport to the physician? Incorrect: A pulse rate of 88 is within normal limits of 60-100. Incorrect: Normal temperature ranges are from 96.6-99.3 degrees Fahrenheit. Correct: The blood pressure is elevated from normal limits. The normal range is 120/80. These values rise slightly as one ages, but this value is still considered high. Incorrect: The respiratory rate is only slightly elevated. The nurse should reassess the respiratory rate in about 30 minutes. Pulse Temperature Blood pressure Respirations

60. A post-operative client has a nasogastric (NG) tube attached to low, intermittent suction. Which nursing intervention will help prevent skin breakdown associated with theNG tube? Incorrect: This is inappropriate unless ordered by the physician. This also will not prevent skin breakdown. Incorrect: This will cause irritation to the nose and throat. Correct: Removing the tape and repositioning the tube every eight hours will relieve pressure on the nares, and providing nares care stimulates circulation. Incorrect: Taping the tube to the client's forehead will put additional pressure on the tip of the nose causing skin breakdown.

Discontinue the low, intermittent suction for fifteen minutes every eight hours. Remove and reinsert the NG tube every other day, alternating nares. Provide care to the nares and reposition the tube every eight hours. Position the tube at the tip of the nose and tape the tube to the client's forehead.

61. A female client returns from the operating room and the nurse assesses her pain. Which statement made by the nurse is most helpful in learning more about the client'spain? Incorrect: This response leads to a closed ended question with the client answering either yes or no.Incorrect: Nurses manage a client's pain but the statement does not assess the pain level. The client may not know what to tell the nurse to do.Incorrect: This is a closed ended question that does not assess pain.Correct: This statement promotes open communication regarding the client's location, duration, characteristics and intensity of thepain and those feelings associated with it. "Is your pain worse now than usual?" "Can I do anything to help you cope with your pain?" "Do you know what causes your pain to be so severe?" "Tell me about the pain you are having now."

62. Which statement is most true regarding the sleep patterns for the older adult? Incorrect: REM and NREM sleep decrease for the older adult.Correct: Older adults have shortened sleep

patterns, especially REM episodes and a decrease in stages 3 and 4 of NREM sleep.Incorrect: Sleep disordersincrease for the older adult.Incorrect: The alteration of sleep patterns is related to a decrease in REM sleep. Older adults do adjust to changes because adjustment is gradual. There is an increase in REM and NREM sleep for the older adult. There is a decrease in the length and amount of REM sleep. Sleep disorders decrease for the older adult. Older adults do not adjust to new sleep pattern changes as readily as youngeradults.

63. To facilitate effective communication with a client who speaks very little English, which intervention is most therapeutic? Incorrect: Explaining detailed actions may be confusing to clients that do not speak English.Incorrect: Taking notes as a client is speaking is not effective communication and conveys the idea that the nurse is not listening.Correct: Active listening is the most therapeutic intervention for effective communication.Incorrect: Sitting at eye level may be therapeutic but is not the most effective technique for communication between person with a language barrier. Explain all nursing actions in detail. Make a written notation on the client's major concerns. Listen attentively for phrases made by the client. Sit at eye level near the client for a few minutes.

64. A physician orders the following: Demerol 75 mg q 4h PRN for pain. Which component of the medication order should be clarified with the physician? Incorrect: The frequency is every 4 hours.Incorrect: The time is PRN.Correct: The route is missing and should be clarified before administration. Demerol usually is given IM or IV.Incorrect: The dosage is 75 mg. Frequency Time Route Dosage

65. Which method is most effective to prevent thrombophlebitis in a client recoveringfrom surgery? Incorrect: The test for thrombophlebitis (inflammation of the vein caused by a clot) is the Homan's sign (pain when the foot is dorsiflexed).Incorrect: Turning the client does not prevent clot formation but does prevent pneumonia.Correct: Antiembolism stockings prevent stagnation of venous blood and promote venous return to the heart, which prevents clot formation. These stockings should be measured prior to surgery, and are put on in surgery or in the immediate postoperative period.Incorrect: This method will prevent pneumonia but does not prevent clot formation. Perform the Allen's test every four hours. Assist the client to turn every two hours. Apply antiembolism stockings immediately after surgery. Encourage coughing exercises every shift.

66. A client is newly diagnosed with dysphagia related to a brain attack. Which food is best for chewing and swallowing? Correct: All fluids should be thickened for easier swallowing.Incorrect: Dry crackers are hard to swallow.Incorrect: Sticky foods such as bananas should be avoided initially.Incorrect: Toast is hard to swallow even though it has butter on it. Thickened juice Dry crackers Banana

Buttered toast

67. A client is receiving a cleansing enema prior to surgery. During the administration of the enema, the client states, "I am having a lot of cramping." Which is the mostappropriate nursing action initially? Incorrect: Slowing the amount of fluid infusing will not stop the cramping. Cramping prevents all of the solution to be administered and therefore, the results may not be as effective.Incorrect: Assisting the client to the bathroom may be important to expell the fluid, but is not a priority action.Correct: Stopping the flow of solution willstop the cramping.Incorrect: Discontinuing the enema may not be appropriate. Enough of the solution needs to beadministered to be effective.

Slow the amount of fluid infusing. Assist the client to the bathroom. Stop the enema solution until the cramping ceases. Discontinue the procedure.

68. A nurse reviews the laboratory report for a client who has just been admitted into the hospital. The client's blood sugar is 106 mg/dL. Which do these results indicate? Correct: Normal fasting blood glucose levels range from 65 to 110 mg/dL. Normal postprandial blood glucose levels range from 65 to 139 mg/dL. So, either way, this is a normal finding.Incorrect: Low blood sugars range from 40 to 60 mg/dL.Incorrect: High blood sugars range from 140 to 500 mg/dL.Incorrect: A value of 106 mg/dL is normal. In any case, however, diabetes is not diagnosed based on one blood sugar result. A normal blood sugar A low blood sugar A high blood sugar A diabetic condition

69. Which is the most effective technique to prevent the transmission of pathogenicmicroorganisms? Incorrect: Although the process of autoclaving using extremely high temperature is effective in significantly reducing microorganism colony counts, this is not the most effective technique in reducing the transmission of

infectious organisms among clients. To reduce the spread of viruses and bacteria by contact, hand hygiene with thorough handwashing using soap and water for at least one minute has been the standard of practice for decades. The Center for Disease Control has issued guidelines promoting the use of fast-drying, alcoholbased, antisepticproducts as the primary method to control infection.Correct: Although handwashing has been promoted as the single, most effective way to prevent the spread of microbes, studies show compliance with proper technique is often poor. Many times health care workers do not take one full minute for vigorous handwashing. The Center for Disease Control now promotes the use of fast-drying, alcohol products between contacts with clients and before and afterexposure to infectious materials.Incorrect: Very few clients are isolated with infectious diseases. This method does prevent the transmission, however, it is not the main technique for preventing the spread of nosocomial infection.Incorrect: Standard precautions must be used for all clients. Autoclave all contaminated equipment, supplies, and linen Use of alcohol-based, antimicrobial product before and after client contact Contact isolation of clients with infectious diseases Standard precautions for clients who are immunosuppressed

70. A client with a breathing problem is receiving instructions from the nurse regarding the use of oxygen at home. Which information should be included regarding the hazardsassociated with oxygen usage? Correct: Oxygen is an essential component if a fire occurs. An oxygen source near a spark or small flame will facilitate the fire and encourage combustion of materials.Incorrect: Oxygen itself is not flammable. Oxygen maybe stored at room temperature.Incorrect: Smoking should not occur if a

client is on oxygen. Oxygen should not beinterrupted unless ordered by the physician.Incorrect: Oxygen cylinders contain large amounts of pressue and cancause damage if dropped. Oxygen supports combustion and can cause a spark to become a flame. Oxygen is a flammable material which must be stored in a cool environment. Oxygen should be shut off when someone near the client is smoking. Oxygen cylinders contain minimal pressure.

71. An older adult in a long-term-care facility requests an evening snack prior to bedtime. Which snack would be best for the client based on an understanding of age-relatedchanges and gastrointestional functioning?

Incorrect: This snack contains too may calories and is too much food to have the client digest it. Cheese is constipating.Incorrect: High carbohydrate foods do not promote gastric motility.Correct: These fresh fruits are high in fiber and contain water. Fiber foods promote GI motility which prevents constipation, and the high water content is necessary for homeostasis of body fluids.Incorrect: Low residue snacks do not promote gastric functioning. Jello with fruit cocktail and a turkey and cheese sandwich Toast with butter and jelly Fresh peaches and strawberries Vanilla pudding and a cookie

72. A 75-year-old female client weighs 92 pounds and is confined to bed. She is confusedand incontinent. Which nursing interventions will aid in the prevention of skin breakdownfor this client? Correct: Inspecting the skin to assess for redness and intervening with actions to prevent skin breakdown include: turning the client every two hours, ensuring adequate nutrition, massage, lotion, refraining from use of frictionto the area when moving the client, and use of no or very mild soaps.Incorrect: Alcohol has a drying effect. The client's skin because of aging is already dry. Dry skin is prone to ulcer formation.Incorrect: Pressure ulcers are not massaged because additional damage can occur.Incorrect: The client's diet should have an increase in protein to counteract the effects of negative nitrogen balance related to immobility. Reposition the client and inspect the skin for redness every two hours. Apply alcohol to heels and elbows to toughen the skin.

Massage the reddened areas every four hours. Provide a diet high in carbohydrates, vitamins, and minerals.

73. The nurse needs to reposition a client confined to bed. Which body mechanicstechnique is most appropriate to accomplish the task? Correct: The nurse should always face the direction the client is to be moved. Not only is this good technique, but it enables clients with visual and hearing deficits to receive directions more accurately. Incorrect: The nurses' feet should be at least 18 inches apart to provide a broad base of support and balance. Incorrect: Placing adraw sheet under the client is appropriate only if there is another person to assist. The question does not indicate a second person is present. Incorrect: Placing a pillow near the headboard is appropriate, but is not a body mechanicstechnique. Face the head of the client's bed. Move the feet about four inches apart. Position a draw sheet under the client. Place the pillow near the headboard.

74. An elderly client is admitted to the hospital from a long-term-care facility. The client isdepressed, has lost weight, and refuses to eat. The client states to the nurse, " I don'tknow why I can't just die and be through with everything." Which response by the nurseis most therapeutic? Correct: This promotes open communication. The nurse has restated the client's words and has asked the client to give more information as to her feelings. Incorrect: This statement is judgmental and reprimanding. Incorrect:

This statement makes the client defensive, is judgmental, and belittling. Incorrect: Changing the subject ignores theclient's feelings. "Die and be through with all of this? Tell me about it." "You don't really mean that, do you?" "Why do you want to die? It can't be that bad." "Let me give you a bath. You'll feel better after that."

75. The nurse in the emergency department assesses a client for hypoxia. Whichphysical symptom should the nurse expect?

Incorrect: Hypotension usually does not occur as a symptom of hypoxia.Incorrect: This is not seen in clients with hypoxia.Incorrect: This is not a symptom of hypoxia.Correct: The heart rate increases to compensate for the lack of oxygen and the heart works harder to pump the blood to the tissues. Other signs and symptoms of hypoxiainclude: dyspnea, cyanosis, apnea, restlessness, anxiety, and confusion.

Hypotension Chest pain Bradycardia Tachycardia

76. A female client has a nasogastric (NG) feeding tube inserted to meet her nutritional needs following a brain attack. Which nursing intervention is most appropriate indetermining the proper positioning of the tube prior to each feeding? Incorrect: Measuring the tubing left outside the nose cannot give an indication of the placement in the stomach.Correct: Testing the gastric pH is a more reliable way to determining the location of the feeding tube than auscultation over the stomach for insufflation of air. Althought injecting 15-30 ml of air into the tube while listening for a shooshing sound may indicate correct placement, this method is not always consistent and should not be used as aprimary method of determing the placement of a feeding tube.Incorrect: Chest x-rays may be obtained initially, but obtaining an x-ray with every feeding is inappropriate.Incorrect: This should be done as the feeding is infused (not prior to) and changes in respiratory function may indicate aspiration.

Assess the length of tubing present outside the nares. Aspirate the stomach contents to check gastric pH. Obtain a chest x-ray. Monitor the client's breathing patterns and color.

77. A client recovering from surgery is receiving a clear liquid diet. Which is the purpose of this type of diet? Correct: Clear liquid diets are usually used after surgery to stimulate peristalsis. Icechips are usually started first, and then proceed to jello, broth, and clear juice.Incorrect: Clearliquid diets do not contain many calories.Incorrect: Clear liquids do not contain manynutrients.Incorrect: Clear liquid diets provide some glucose but most clients have an IVcontaining dextrose that keeps the blood sugars normal until the diet can be resumed. To stimulate peristalsis To replenish calories that were lost because of NPO status To provide proteins, vitamins and minerals needed for metabolism To keep blood sugars from falling

78. An 82-year-old hospitalized client tells the nurse that he is not able to sleep at night. Which measure to promote sleep should the nurse implement LAST? Incorrect: The client's sleep patterns should have been documented upon admission. The nurse should have an idea of these patterns prior to providing care.Incorrect: This could be a cause of the client's insomnia and should

be implemented initially or in conjunction with the backrub.Correct: Non-pharmacological methods for sleep shouldalways be provided before medicating the client.Incorrect: Backrubs should be implemented prior to givingmedication.

Ask the client about his sleep patterns. Listen to the client's concerns about his illness and hospitalization. Give the medication ordered for sleep. Give a backrub using lotion.

79. Fecal impaction is a complication of long-term constipation. Which symptomindicates that a fecal impaction is present? Incorrect: Occult or hidden blood is not an indication of impaction.Incorrect: This is not an indication of impaction but if an impaction remains untreated causing a complete bowel obstruction, nausea and vomiting could

occur.Correct: This is one sign of an impaction. The others are: expelling hard, dry feces, pain in rectal area, abdominal fullness and bloating.Incorrect: This is not an indication of an impaction.

Stool is positive for occult blood. Nausea and vomiting Oozing of liquid around the impaction Foul-smelling stool

80. A hospitalized older adult develops constipation. Which of the following is the contributory factor for developing constipation? Incorrect: Pain itself does not cause constipation, however, some of the pain medications such as morphine slow peristalsis and predispose a client to constipation.Correct: Reduced activity slows peristalsis leading toconstipation.Incorrect: Lack of privacy may contribute to constipation but is not the main cause.Incorrect: NPO statusitself does not contribute to constipation unless it is prolonged. Pain Reduced activity Lack of privacy NPO status

81. A physician orders an indwelling catheter for a female client. As the nurse advances the catheter into the urethra, urine begins to flow into the tubing. Which nursing action isindicated next?

Incorrect: The catheter port is used to obtain a sterile urine specimen.Incorrect: The catheter balloon must be inflated before the bag is connected or the catheter may become dislodged during connection.Correct: Advancing the catheter an inch will ensure that the entire balloon is in the bladder rather than just the tip of thecatheter.Incorrect: The balloon is inflated with sterile water. Insert 10 cc's of sterile water into the catheter port using a needle and syringe. Connect the drainage bag and secure the catheter to the client's leg. Advance the catheter one inch and inflate the balloon with sterile water. Inflate the balloon with air.

82. A nurse opens a sterile tray preparing to change the dressing on a client's surgical wound. Which technique is most appropriate in order to maintain a sterile field? Incorrect: This technique will contaminate the sterile glove.Incorrect: Pouring a solution should be done slowly to avoid splashing.Correct: Unsterile items should NEVER be passed over the sterile field. If this happens, thetray is considered contaminated.Incorrect: The top of the package wrapper should be opened first. When donning a sterile glove, touch the outside surface of the glove with thenondominant hand. Pour the sterile solution quickly into a container holding the bottle over the edgeof the field. Avoid moving unsterile items over the sterile field. Open the top outermost flap of the package wrapper last.

83. A physician orders a client's drain insertion site to be cleansed every eight hours witha betadine swab. Which technique used by the nurse is most appropriate for cleansingdrain sites?

Incorrect: This technique is not appropriate for drain sites but may be used for incisions.Correct: This circular technique prevents transferring of microorganisms back onto a cleaned area.Incorrect: Usually the drain itself is notcleansed on a regular basis unless it is covered with drainage. Betadine may cause the drain to become dry andbrittle.Incorrect: This technique is appropriate for an incision, but not for a drain.

Vertical strokes, first on one side of the drain and then the other Circular, making the circle wider as the strokes go outward

Cleanse the drain itself first, prior to the skin around the drain Horizontal strokes extending one inch from the drain

84. A client has abnormal breath sounds with crackles and rhonchi, and a weak, dry cough. The client's respiratory rate is 28 and he experiences dyspnea and fatigue uponexertion. Which nursing diagnosis is most appropriate? Incorrect: There is no indication that perfusion is altered.Correct: Abnormal breath sounds (rales and rhonchi) are major characteristics of this diagnosis, especially since a weak cough is present.Incorrect: The major problem is the inability of the client to clear the airway.Incorrect: While activity is a problem, it is not the major diagnosis.

Altered tissue perfusion Ineffective airway clearance Impaired gas exchange Activity intolerance

85. A nurse applies a heat pack to a client's swollen knee joint. Which is the rationale for the nurse's action? Incorrect: Heat causes vasodilation which increases circulation to an area.Incorrect: Cold decreases pain by

numbing the nerve endings.Correct: Heat decreases stiffness and improves mobility.Incorrect: Ice prevents swelling.

Heat constricts blood vessels. Heat numbs nerve endings. Heat decreases stiffness. Heat prevents swelling.

86. A client receives oxygen per nasal cannula. How much oxygen can safely be delivered via nasal cannula? Incorrect: This rate is too high for a nasal cannula but may be delivered by mask.iIncorrect: This rate is too high for a nasal cannula but may be delivered by a mask.Correct: Nasal cannulas deliver 1-6 liters of oxygen/ min.Incorrect: This rate is too high to be delivered by nasal cannula. A mask may be used for this level of oxygen.

15 L/min 7-9 L/min 1-6 L/min 10-12 L/min

ATI. MATERNAL-NEWBORN

1. Two days after delivery, a postpartum client prepares for discharge. What should the nurse teach her about lochia flow? Incorrect: Lochia does change color but goes from lochia rubra (bright red) on days 1-3, to lochia serosa (pinkish brown) on days 4-9, to lochia alba (creamy white) days 10-21.Incorrect: Numerous clots are abnormal andshould be reported to the physician.Incorrect: Saturation of the perineal pad is considered abnormal and may indicatepostpartal hemorrage.Correct: Lochia normally lasts for about 21 days, and changes from a bright red, to pinkishbrown, to creamy white.

The color of the lochia changes from a bright red to white after four days Numerous large clots are normal for the next three to four days Saturation of the perineal pad with blood is expected when getting up from thebed Lochia should last for about 3 weeks, changing color every few days

2. A nurse monitors fetal well-being by means of an external monitor. At the peak of the contractions, the fetal heart rate has repeatedly dropped 30 beats/min below thebaseline. Late decelerations are suspected and the nurse notifies the physician. Which isthe rationale for this action? Incorrect: A nuchal cord (cord around the neck) is associated with variable decelerations, not late decelerations.Incorrect: Variable decelerations (not late decelerations) are associated with cord compression.Incorrect: Late decelerations are a result of hypoxia. They are not reflective of the strength of maternal contractions.Correct: Late decelerations are associated with uteroplacental insufficiency and are a sign of fetalhypoxia. Repeated late decelerations indicate fetal distress. The umbilical cord is wrapped tightly around the fetus' neck The fetal cord is being compressed due to rapid descent of the fetal head Maternal contractions are not adequate enough to deliver the fetus The fetus is not receiving adequate oxygen and is in distress

3. Which preoperative nursing interventions should be included for a client who isscheduled to have an emergency cesarean birth? Incorrect: Monitoring O2 saturations and administering pain medications are postoperative interventions.Incorrect: Taking vital signs every 15 minutes is a postoperative intervention. Instructing the client regarding breathing exercises is not appropriate in a crisis situation when the client's anxiety is high, becauseinformation would probably not be retained. In an emergency, there is time only for essential interventions.Correct: Because this is an emergency, surgery must be performed quickly. Anxiety of the client and the family will be high. Inserting an indwelling catheter helps to keep the bladder empty and free from injury when the incision ismade.Incorrect: The nurse should have assessed breath sounds upon admission. Breath sounds are important if the

client is to receive general anesthesia, but the anesthesiologist will be listening to breath sounds in surgery in that case. Monitor oxygen saturation and administer pain medication. Assess vital signs every 15 minutes and instruct the client about postoperative care. Alleviate anxiety and insert an indwelling catheter. Perform a sterile vaginal examination and assess breath sounds.

4. Which nursing instruction should be given to the breastfeeding mother regarding careof the breasts after discharge? Incorrect: Engorgement occurs on about the third or fourth postpartum day and is a result of the breast milk formation. The primary way to relieve engorgement is by pumping or longer nursing. Giving a bottle of formula will compound the problem because the baby will not be hungry and will not empty the breasts well.Incorrect: Applying lotion to the nipples is not effective for keeping them soft. Excessive amounts of lotion may harbor microorganisms.Correct: In order to stimulate adequate milk production, the breasts should be pumped if the infant isnot sucking or eating well, or if the breasts are not fully emptied.Incorrect: Using soap on the breasts dries the nipplesand can cause cracking. The baby should be given a bottle of formula if engorgement occurs. The nipples should be covered with lotion when the baby is not nursing. The breasts should be pumped if the baby is not sucking adequately. The breasts should be washed with soap and water once per day.

5. A client in preterm labor is admitted to the hospital. Which classification of drugs should the nurse anticipate administering? Correct: Tocolytics are used to stop labor. One of the most commonly used tocolytic drugs is ritodrine (Yutopar).Incorrect: Anticonvulsants are used for clients with pregnancy-induced hypertension who are likely to seize.Incorrect: The glucocorticoids (e.g., betamethasone and dexamethasone) are used for accelerating fetal lung maturation and production of surfactant. They are commonly used if the membranes are ruptured or labor can not be

stopped.Incorrect: Antiinfectives are used if there is infection. Preterm labor may or may not involve ruptured membranes with its accompanying risk of infection.

Tocolytics Anticonvulsants Glucocorticoids Antiinfectives

6. Which of the following are probable signs, strongly indicating pregnancy? Incorrect: The presence of fetal heart sounds is a positive sign of pregnancy; quickening is a presumptive sign of pregnancy.Incorrect: These are presumptive signs. They may indicate pregnancy or they may be caused by other conditions, such as disease processes.Correct: These are probable signs that strongly indicate pregnancy. Hegar's sign is a softening of the lower uterine segment, and Chadwick's sign is the bluish or purplish color of thecervix as a result of the increased blood supply and increased estrogen. Ballottement occurs when the cervix istapped by an examiner's finger and the fetus floats upward in the amniotic fluid and then falls downward.Incorrect: These are presumptive signs that might indicate pregnancy, but they might be caused by other conditions, such as disease processes. Presence of fetal heart sounds and quickening Missed menstrual periods, nausea, and vomiting Hegar's sign, Chadwick's sign, and ballottement Increased urination and tenderness of the breasts

7. Two hours after delivery the nurse assesses the client and documents that the fundus is soft, boggy, above the level of the umbilicus, and displaced to the right side. The nurseencourages the client to void. Which is the rationale for this nursing action? Correct: Bladder distention can lead to postpartum hemorrhage. A full bladder displaces the uterus causing it not to contract properly. Emptying the bladder allows the uterus to contract more firmly.Incorrect: A distended bladder rises out of the abdomen, causing the uterus to be displaced and increasing the risk of hemorrhage. It does not affectthe perineum.Incorrect: Bladder distention can lead to urinary stasis and infection. This, however, does not relate to the soft, boggy uterus or the potential for hemorrhage.Incorrect: Massaging is uncomfortable regardless of whetherthe bladder is full or not. A full bladder displaces the uterus causing it not to contract properly, which may lead to postpartal hemorrhage. A full bladder prevents normal contractions of the uterus. An overdistended bladder may press against the episiotomy causing dehiscence. Distention of the bladder can cause urinary stasis and infection. It makes the client more comfortable when the fundus is massaged.

8. Which site is preferred for giving an IM injection to a newborn? Incorrect: Ventrogluteal muscles are located in the hip area. It is not the preferred site for injections in the newborn because of lack of muscle mass.Correct: The middle third of the vastus lateralis is the preferred site for injections.Incorrect: Ventrogluteal muscles are located in the hip area. It is not the preferred site for injections in the

newborn because of lack of muscle mass.Incorrect: Newborns do not receive injections in the dorsogluteal site(gluteus maximus) due to decreased muscle mass. Ventrogluteal Vastus lateralis Rectus femoris Dorsogluteal

9. During the first twelve hours following a normal vaginal delivery, the client voids 2,000 mL of urine. How should the nurse interpret this finding? Incorrect: Urinary tract infections are common during pregnancy and in the postpartum period. Urinary frequency is a common finding. However, voiding large amounts of urine is not a sign of a UTI.Incorrect: High output renal failure occurs with injury/trauma to the kidneys. There has been no damage to the kidneys.Incorrect: Most women do receive some IV fluids during labor and delivery, however the IV rates are carefully calculated according to

weight.Correct: During pregnancy the circulating blood volume increases by about 50%. In order to get rid of theexcess fluid volume after delivery, the woman experiences an increased amount of urine output during the first few hours.

Urinary tract infection High output renal failure Excessive use of IV fluids during delivery Normal diuresis after delivery

10. If a pregnant client diagnosed with gestational diabetes cannot maintain control ofher blood sugar by diet alone, which medication will she receive? Incorrect: Glucophage is an oral hypoglycemic. Oral hypoglygemics cross the placenta and can cause damage to the fetus. They are not used in gestational diabetes for that reason.Incorrect: Glucagon is a hormone usedto raise blood sugar and manage severe hypoglycemia. Clients with gestational diabetes havehyperglycemia.Correct: Insulin is the drug of choice for gestational diabetes. Insulin lowers the client's blood sugarwithout harming the fetus.Incorrect: DiaBeta is an oral hypoglycemic drug. Oral hypoglycemic agents cross theplacenta and can cause damage to the fetus. They are not used for gestational diabetes for that reason. Metformin (Glucophage) Glucagon Insulin Glyburide (DiaBeta)

11. Which assessment finding indicates that placental separation has occurred duringthe third stage of labor? Incorrect: There is usually an increase in bleeding (a sudden gush of blood) when the placenta separates.Incorrect: Contractions continue in an attempt to expel the placenta. The contractions may not be as intense, but they do not stop. Also fundal massage helps contract the uterus preventing postpartal bleeding.Incorrect: Shaking and chills occur about 10-15 minutes after the delivery of the baby, but are not related to the placental detachment. They are a result of the release of pressure on pelvic nerves and the release of epinephrine during labor.Correct: As the placenta detaches, the cord that has been clamped becomes longer as it slides out of the vagina. Decreased vaginal bleeding Contractions stop Maternal shaking and chills Lengthening of the umbilical cord

12. The nurse midwife is concerned about a pregnant client who is suspected of having aTORCH infection. Which is the main reason TORCH infections are grouped together? They are: Incorrect: Most TORCH infections can cause mild flu-like symptoms for the mother. Death may or may not occur in the fetus.Incorrect: TORCH is an abbreviation for Toxoplasmosis, Other (syphilis, HIV and Hepatitis B), Rubella, Cytomegalovirus, and Herpes simplexnot all of these are sexually transmitted.Correct: All TORCH

infections have the capability of infecting the fetus or causing serious effects to the newborn.Incorrect: A vector is a carrier of the disease such as a mosquito. Not all of the TORCH infections are carried by vector. benign to the woman but cause death to the fetus. sexually transmitted. capable of infecting the fetus. transmitted to the pregnant woman by a vector.

13. During the postpartum period, a hospitalized client complains of discomfort related toher episiotomy. The nurse assigns the diagnosis of pain related to perineal sutures. Which nursing intervention is most appropriate during the first 24 hours following anepisiotomy?

Incorrect: Petroleum jelly will harbor bacteria, which may hinder healing.Incorrect: The client should practice Kegal exercises to increase bladder tone, but these exercises would add to the client's discomfort during the first 24hours.Incorrect: Taking a warm sitz bath is recommended after the first 24 hours.Correct: Ice packs will decrease edema and discomfort, and prevent formation of a hematoma. Instruct the client to use petroleum jelly on the episiotomy after voiding. Encourage the client to practice Kegal exercises. Advise the client to take a warm sitz bath every four hours. Apply ice packs to the perineum.

14. A client asks the nurse about the benefits of breastfeeding. Which response by the nurse provides the most accurate information? Incorrect: Breastfeeding does not help speed up weight loss. The lactating mother requires more calories, but usually has an increased appetite to accommodate that need.Incorrect: Protein amounts are greater in formulaand cow's milk.Correct: Breast milk is easier to digest because of the type of fat and protein in the milk.Incorrect: Breastfeeding does not prevent to woman from getting pregnant because it does not prevent ovulation. Most womenovulate within the first 6 weeks after delivery. Breastfeeding helps women lose weight faster. Breast milk contains a greater amount of protein. Breast milk is easier to digest than formula. Breastfeeding is a good method of contraception.

15. Which physiological change takes place during the puerperium?

Incorrect: The puerperium is the first 6 weeks after delivery. The client will experience lochia for the first few weeks, and hormone levels will stabilize. Menstruation cannot occur until ovulation occurs.Incorrect: This occurs in stage three of labor.Correct: The uterine changes are called involution. The uterus should return to its pre-pregnancy state within 6 weeks after delivery.Incorrect: This describes the labor process, not the puerperium. The endometrium begins to undergo alterations necessary for menstruation. The placenta begins to separate from the uterine wall. The uterus returns to a pre-pregnant size and location. The uterus contracts at regular intervals with dilation of the cervix occurring.

16. A client delivered two days ago and is suspected of having postpartum "blues." Which symptoms confirm the diagnosis? Correct: These are signs of the postpartum blues, which typically diminishes within three-four days after delivery. Postpartum blues, a transient period of tearfulness, is a result of hormonal shifts. Other symptoms of the blues include: sadness, anxiety about the health of the baby, insomnia, anorexia, anger, feelings of anticlimax.Incorrect: Postpartum blues, a transient period of tearfulness, is a result of hormonal shifts. Depressionand suicidal thoughts are signs of postpartal depression, not the blues and should be followed up with psychiatrictreatment.Incorrect: Excess anxiety and the inability to care for the family are signs of postpartal depression, not theblues. Postpartum blues, a transient period of tearfulness, is a result of hormonal shifts.Incorrect: Nausea andvomiting are psychosomatic symptoms of postpartal depression and require psychiatric treatment. Postpartum blues, a transient period of tearfulness, is a result of hormonal shifts.

Uncontrollable crying and insecurity Depression and suicidal thoughts Sense of the inability to care for the family and extreme anxiety Nausea and vomiting

17. Shortly after delivery, the nursery nurse gives the newborn an injection of phytonadione (Vitamin K). The infant's grandmother wants to know why the baby got ashot in his leg. Which response by the nurse is most appropriate? Incorrect: Calcium is needed for bone and muscle growth, not Vitamin K.Incorrect: Vitamin K is used to promote clotting, and does not affect digestion.Incorrect: The B vitamins are responsible for carbohydrate metabolism

and the energy derived from glucose, not Vitamin K.Correct: Vitamin K is given to prevent bleeding until the intestinal bacteria can start to produce it. The intestines of a newborn are sterile until it starts to feed. Vitamin K helps with the clotting factors necessary to control bleeding.

"Vitamin K promotes bone and muscle growth." "Vitamin K helps the baby digest milk." "Vitamin K helps stabilize the baby's blood sugar." "Vitamin K is used to prevent bleeding."

18. At 10 weeks gestation, a primagravida asks the nurse what is occurringdevelopmentally with her baby. Which response by the nurse is correct? Incorrect: Wrinkles do not form until late in the pregnancy. Fat stores usually do not form until the third trimester.Incorrect: The eyelids are fused until about 26 weeks.Correct: The kidneys are making urine, which is excreted by the fetus into the amniotic fluid.Incorrect: The heart is already formed and beating at 8 weeks. "The skin is wrinkled and fat is being formed." "The eyelids are open and he can see." "The kidneys are making urine." "The heart is being developed."

19. A nurse in the clinic instructs a primagravida about the danger signs of pregnancy. The client demonstrates understanding of the instructions, stating she will notify thephysician if which sign occurs?

Incorrect: White vaginal discharge is a normal occurrence during pregnancy due to increased amounts of estrogen and increased blood supply to the cervix and vagina. It is not a danger sign.Incorrect: Backache iscommon in pregnancy due to the alteration of the woman's center of gravity; it is not a danger sign. Backachesbecome worse as the uterus enlarges.Incorrect: Frequent, urgent urination is a common discomfort; it is not a dangersign. The pressure of the enlarging uterus causes frequency and urgency.Correct: Abdominal pain is a danger sign and can be indicative of an abruptio placenta. It is important for a physician to evaluate this symptom. It is one ofseveral danger signs, including: headache, rupture of membranes, vaginal bleeding, edema, epigastric pain, elevatedtemperature, painful urination, prolonged vomiting, blurred vision, change in or absence of fetal movement.

White vaginal discharge Dull backache Frequent, urgent urination Abdominal pain

20. An hour after delivery, the nurse instills erythromycin (Ilotycin) ointment into the eyesof a newborn. The main objective of the treatment is to prevent infection caused bywhich organism? Incorrect: Erythromycin (Ilotycin) is an antibiotic ointment used to prevent blindness related to gonorrhea. Antibiotics are effective against bacteria. Rubella is a virus.Correct: Ilotycin, an antibiotic, is used for the prophylaxic treatment of gonorrhea and chlamydia. If left untreated, it could result in blindness.Incorrect: Ilotycin, an antibiotic, isnot effective in combating syphilis infections.Incorrect: HIV is a virus. Antibiotics are effective against bacteria. Ilotycinis an antibiotic ointment and therefore not effective against HIV.

Rubella Gonorrhea Syphilis Human immunodeficiency virus (HIV)

21. A woman in active labor receives a narcotic analgesic for pain control. If the narcotic is given a half an hour before delivery, which effect will the medication have on the infant? It will cause the infant's:

Incorrect: Narcotic analgesics cause respiratory depression and do not affect the infant's blood sugar.Correct: Narcotic analgesics can cause respiratory depression for the infant and also for the mother. This is evidenced by low Apgar scores (apnea and bradycardia) in the infant. If respiratory depression occurs, a narcotic antagonist (Narcan) is usually given.Incorrect: Narcotic analgesics, if given too close to delivery, can cause bradycardia, not tachycardia.Incorrect: Narcotics, such as Demerol, cause CNS depression, not hyperactivity.

blood sugar to fall. respiratory rate to decrease. heart rate to increase. movements to be hyperactive.

22. For a client in the second trimester of pregnancy, which assessment data support a diagnosis of pregnancy-induced hypertension (PIH)? Incorrect: A decrease in hemoglobin is indicative of anemia, while uterine tenderness may indicate abruptio placenta.Incorrect: Polyuria and weight loss are signs of gestational diabetes.Correct: PIH is characterized by twocomponents: elevated blood pressure and proteinuria. Vasospasm in the arterioles leads to increased blood pressureand a decrease in blood flow to the uterus and placenta. This results in a questionable outcome for the fetus due to placental insufficiency. Renal blood flow is affected, ultimately resulting in proteinuria.Incorrect: Elevated blood glucose is a sign of gestational diabetes. Hematuria may indicate a U.T.I.

Hemoglobin 10.2 mg/dL and uterine tenderness Polyuria and weight loss of 3 pounds in the last month Blood pressure 168/110 and 3+ proteinuria Hematuria and blood glucose of 160 mg/dL

23. A 35-week gestation infant was delivered by forceps. Which assessment findings should alert the nurse to a possible complication of the forceps delivery? Correct: A weak, ineffective suck could be a result of facial paralysis which is a major complication of forceps deliveries. Scalp edema is another complication and should subside within 2-3 days. Other complications of forcepsdeliveries include: cephalohematomas, intracranial hemorrhage (especially in premature infants) and excessivebruising, which increases the risk for hyperbilirubinemia.Incorrect: Molding of the head is a common occurrence withvaginal deliveries. Jitteriness is a sign of low blood sugar, not forceps delivery.Incorrect: A shrill, highpitched cry and tachypnea are signs of drug withdrawal, not a complication of forceps delivery.Incorrect: Hypothermia is not a complication of forceps deliveries. The hemoglobin level is quite low (should be about 15-16 g/dL), but unless there isexcessive bleeding, the hemoglobin level should be unaffected by the forceps delivery. Weak, ineffective suck, and scalp edema Molding of the head and jitteriness Shrill, high pitched cry, and tachypnea Hypothermia and hemoglobin of 12.5 g/dL

24. In which position should the nurse place the laboring client in order to increase theintensity of the contractions and improve oxygenation to the fetus? Incorrect: This position is contraindicated because the fetus creates pressure on the mother's vena cava.Incorrect: Squatting widens the pelvic inlet, but does not improve contractions or fetal oxygenation.Correct: This prevents vena cava compression and, therefore, improves fetal oxygenation; at the same time, it provides a restfulposition between contractions.Incorrect: High Fowler's (sitting upright) will assist with the intensity of the contractionsbecause of gravity, but it will not help with fetal oxygenation. Supine with legs elevated Squatting Left side-lying High Fowler's

25. A woman enters the birthing center in active labor. She tells the nurse that her membranes ruptured 26 hours ago. The nurse immediately takes the client's vital signs. Which is the rationale for the nurse's actions? Incorrect: Pulse rates increase due to pain, not because of rupture of membranes.Incorrect: The woman is not reporting pain and ruptured membranes do not cause pain. Lack of fluid (ruptured membranes) has no influenceon respiratory rates.Incorrect: Blood pressure is not affected by prolonged rupture of membranes.Correct: The membranes are a protective barrier for the fetus. If the membranes are ruptured for a prolonged period of time, microorganisms from the vagina can ascend into the uterus. The longer the membranes have been ruptured, the greater the risk for infection. Pulse rates rise the longer the membranes are ruptured Respiratory rates decrease due to lack of fluid in the uterus Prolonged rupture of membranes can lead to transient hypertension Infection is a complication of prolonged rupture of membranes

26. A new client's pregnancy is confirmed at 10 weeks gestation. Her history reveals thather first two pregnancies ended in spontaneous abortion at 12 and 20 weeks. She has a4-year-old and a set of 1-year-old twins. How should the nurse record the client's current gravida and para status? Incorrect: Gravida includes the number of times the woman has been pregnant. She has been pregnant 5 times. A parity of 3 would be obtained by incorrectly counting the 20-week spontaneous abortion as a viable infant.Incorrect: The woman has been pregnant 5 times, including the present pregnancy. The abortions count as

pregnancies, but not in the parity.Correct: Gravida is the number of times a woman has been pregnant, including the present pregnancy. Para is the number of pregnancies carried past 20 weeks' gestation, regardless of the number of fetuses delivered. The woman has been pregnant five times, including this pregnancy, and has had two pregnancies that have exceeded 20 weeks. Even though she delivered two children as a result of one of those pregnancies, thepara for her twin pregnancy remains at 1. The pregnancy after which she delivered her four-year-old child makes her a para 2.Incorrect: A para of 4 would be obtained by incorrectly counting the 2 spontaneous abortions as viable at delivery. Gravida 2, para 3 Gravida 4, para 2 Gravida 5, para 2 Gravida 5, para 4

27. A 16-year-old client reports to the school nurse because of nausea and vomiting. After exploring the signs and symptoms with the client, the nurse asks the girl whethershe could be pregnant. The girl confirms that she is pregnant, but states that she doesnot know how it happened. Which nursing diagnosis is most important? Incorrect: Although this addresses the client's nausea and vomiting, it is not the most important diagnosis at this time. There are no data to indicate that the client actually has a nutritional deficit. Because nausea and vomiting place her at risk for nutritional deficit, a diagnosis of risk for altered nutrition. . . would be appropriate. The

knowledge diagnosis is an actual problem and should be addressed at this contact with the client; the nutritionproblem will be ongoing during the pregnancy.Incorrect: This diagnosis does not address the reason for the lack of client knowledgeshe may be at risk for poor parenting, but this is not the priority because there will be time toaddress that issue as the pregnancy progresses.Incorrect: There is no clear evidence of the denial of pregnancy nor of the lack of coping skills.Correct: This client clearly has a knowledge deficit about the causes of pregnancy and the physiological changes associated with it. It is important for teaching to begin immediately because her understandingis essential to her compliance with suggestions for a healthy pregnancy. Altered nutrition: less than body requirements related to nausea and vomiting Risk for altered family processes related to the client's age Ineffective individual coping related to denial of pregnancy Knowledge deficit related to the client's developmental stage and age

28. A client is admitted to the hospital for induction of labor. Which are the main indications for labor induction? Incorrect: These are contraindications for labor induction.Correct: Induction of labor is the stimulation of contractions (usually by the use of Pitocin) before they begin on their own. Maternal indications for induction of laborinclude: pregnancy induced hypertension, chorioamnionitis, gestational diabetes, chronic hypertension and prematurerupture of membranes. Fetal indications include intrauterine growth retardation, post-term dates and fetaldemise.Incorrect: These are contraindications for labor induction.Incorrect: These are contraindications for labor induction. They are indications for a C-section.

Placenta previa and twins Pregnancy-induced hypertension and postterm fetus Breech position and prematurity Cephalopelvic disproportion and fetal distress

29. A client in active labor receives a regional anesthetic. Which is the main purpose of regional anesthetics? Incorrect: This choice describes general anesthesia.Correct: Regional anesthetics provide numbness and loss of pain sensation to an area. The most common regional blocks are: local, pudendal, epidural, and spinal.Incorrect: Pain sensations travel to the central nervous system not away from it.Incorrect: This choice describes the action for narcotic medications, not regional anesthetics. To relieve pain by decreasing the client's level of consciousness

To provide general loss of sensation by blocking sensory nerves to an area To provide pain relief by blocking descending impulses from the central nervous system To relieve pain by decreasing the perception of pain leading to the pain centers in the brain

30. The nursery nurse reviews a newborn's birth history and notes that the Apgar scores were 5 at one minute after birth, and 7 at five minutes after birth. How should the nurseinterpret these scores? The infant: Incorrect: Usually babies that only need suctioning of the mouth and nose have Apgars that are 8 or 9.Incorrect: If intubation is required, it means that the baby's heart and respiratory rates are not stable, and Apgars would be lower than 5.Incorrect: Apgar scores are used to quickly assess the well-being of the baby. Apgar scores range from 0-10. A score of 0 indicates that the baby is dead. An Apgar score of 5 indicates that the baby needs assistance.Correct: Apgar scores of 5 and 7 indicate that the heart rate was below 100, the respiratory effort was irregular, there was little muscle tone, the baby was pink with blue extremities, and there was a grimace. These scores indicate that the baby needed stimulation in order to breathe, and oxygen to increase its oxygen saturation. needed brief oral and nasal suctioning. required endotracheal intubation and bagging with a hand-held resuscitator. was stillborn and required CPR. required physical stimulation and supplemental oxygen.

31. With routine prenatal screening, a woman in the second trimester of pregnancy is confirmed to have gestational diabetes. How may the nurse explain the role of diet andinsulin in the management of blood sugar during pregnancy? Correct: Insulin is given to gestational diabetic clients because their insulin requirements cannot keep up with the metabolic needs of the fetus in the last trimester. Insulin decreases the blood sugar.Incorrect: Oral hypoglycemic agents are not given to clients with gestational diabetes because they cross the placenta and areharmful to the fetus.Incorrect: The client will need frequent follow-up after delivery and into the postpartum period, butshe should not need insulin after delivery because in gestational diabetes, blood glucose usually returns to normalafter delivery.Incorrect: Clients with gestational diabetes need to eat three balanced meals and three snacks daily. The glucose load is best when maintained at a steady level throughout the day to avoid periodic overproduction ofinsulin. The last snack of the day should contain protein to stabilize the energy production during the night.

"Insulin lowers an elevated blood sugar during pregnancy to meet the increased metabolic needs of the baby." "You will need to take an oral hypoglycemic, which is a pill to lower your blood sugar." "There is a good possibility you will be taking insulin for the rest of your life." "You should eat three large meals per day to maintain steady glucose load."

32. A breastfeeding mother complains of cramping. Which is the main cause of the client's afterpains? Incorrect: Infection of the suture line can cause pain and discomfort, but is not the cause of afterpains. Afterpains are postpartum uterine contractions.Incorrect: Constipation and bloating do occur in the postpartum periodas peristalsis resumes, but constipation does not cause afterpains, which are uterine contractions.Correct: Afterpains are caused by uterine contractions that occur for the first 2-3 days postpartum. Breast-feeding mothers have moreafterpains due to the release of oxytocin stimulated by the nursing baby. Oxytocin strengthens uterine contractions and compresses blood vessels, preventing blood loss.Incorrect: Trauma is not the cause of afterpains. Afterpains are postpartum uterine contractions.

Infection of the suture line Constipation and bloating Contractions of the uterus Trauma during delivery

33. A client who is 37 weeks gestation comes to the office for a routine visit. This is the client's first baby and she asks the nurse how she will know when labor begins. Whichsigns indicate that true labor has begun? Incorrect: These signs describe Braxton-Hicks contractions, which occur throughout pregnancy and increase in intensity and frequency as labor grows closer.Incorrect: True labor pains start in the lower back and sweep to the front in waves.Incorrect: These signs occur with lightening, usually 10-14 days before labor begins.Correct: These aretrue signs of labor, along with the rupturing of the membranes and cervical dilatation.

Contractions that are irregular and decrease in intensity when walking Abdominal pain that starts at the fundus and progresses to the lower back Increased pressure on the bladder and urinary frequency Expulsion of pink-tinged mucous and contractions that start in the lower back

34. A multiparous woman with a history of all vaginal births is admitted to the hospital in labor. After several hours, the client's labor has not progressed and she is getting tiredand restless. The decision is made to proceed with cesarean delivery. The nurse recognizes the client's knowledge deficit regarding the surgical delivery and care afterbirth. Which is the appropriate expected outcome for correction of the client's knowledgedeficit? The client will: Incorrect: This expected outcome does not address the client's knowledge deficit. Instead, this is an expected outcome for the nursing diagnosis of ineffective individual coping.Incorrect: This choice does not address

the client's knowledge deficit, but instead addresses a problem with interrupted bonding.Correct: Goals/outcomesshould reflect resolution of the stated nursing diagnosisin this case, knowledge deficit. Verbalization of reasons for the surgery would indicate resolution of the knowledge deficit. If interventions for knowledge deficit are effective, otherproblems (e.g., anxiety, ineffective coping) may be prevented.Incorrect: This choice addresses the anxiety that will occur because of the unknown, but does not address the stated problem, knowledge deficit.

demonstrate appropriate coping mechanisms needed to get through the surgery. accept that the type of delivery will not affect the bonding with the baby. verbalize understanding about the reason for the unplanned surgery. demonstrate decreased anxiety and fear of the unknown.

35. The physician performs an amniotomy for a woman in labor. Which nursing action should follow the procedure? Incorrect: Maternal oxygenation is not affected by an amniotomy.Incorrect: Maternal pulse and blood pressure are not affected by an amniotomy.Incorrect: Assessing the perineum should be done after an episiotomy, notafter amniotomy.Correct: An amniotomy, or artificial rupture of membranes (AROM), is used to speed up labor. The nurse must document the color, amount, character and odor of the fluid, and assess for fetal well being.

Check the client's capillary refill and oxygenation. Monitor the maternal pulse and blood pressure. Inspect the perineum for lacerations, bleeding, and hematoma. Assess the fluid for color, odor, and amount.

36. For a pregnant adolescent who is anemic, which foods should the nurse include inthe client's dietary plan to increase iron levels? Incorrect: Milk does not contain iron and it interferes with iron absorption.Correct: Orange juice enhances the absorption of iron. Apricots are a good source of iron.Incorrect: Chicken does contain iron, but cottage cheese, a dairy product, does not.Incorrect: Pickles contain large amounts of salt, not iron. Peanut butter sandwiches do notcontain much iron.

Milk and fish

Orange juice and apricots Chicken and cottage cheese Pickles and peanut butter sandwiches

37. Which condition must occur in order for identical (monozygotic) twins to develop? Incorrect: Usually only one ovum is released per month; one sperm cannot fertilize two ova.Incorrect: This is the case in fraternal (dizygotic) twins. There are two placentas, two chorions, and two amnions. The twins may be thesame or different sex.Correct: One sperm fertilizes one ovum, and then the zygote divides into two individuals with one placenta, one chorion, two amnion and two umbilical cords. These twins are always the same sex.Incorrect: The enzyme on the head of the sperm dissolves the coating of the ovum so eventually only one sperm penetrates one

egg.

One sperm fertilizes two ova Two sperm fertilize two ova One sperm fertilizes one ovum Two sperm fertilize one ovum

38. Which fetal structure is responsible for carrying oxygenated blood from the placentato the fetus?

Incorrect: The ductus arteriosus is a shunt that connects the lungs to the aorta, allowing the blood to bypass the lungs.Incorrect: Except in the case of fetal circulation, arteries do carry oxygenated blood; but during pregnancy, the two umbilical arteries carry unoxygenated blood from the fetus to the placenta, where reoxygenationoccurs.Incorrect: The portal vein carries blood from the intestine to the liver.Correct: The umbilical vein carries oxygenated blood from the placenta to the fetus. The direction of blood flow is toward the fetal heart.

Ductus arteriosus Umbilical artery Portal vein Umbilical vein

39. A client at 33 weeks gestation is admitted for suspected abruptio placenta. Which factor in the client's history supports this diagnosis? The client states that she: Incorrect: Drinking alcohol is not usually associated with abruptio placenta.Incorrect: Clients with abruptio placenta do not have contractions that can be releived by walking. Usually the pain is quite intense.Incorrect: Intercourse should not cause an abruptio placenta, although it is contraindicated in clients with placentaprevia.Correct: The use of crack cocaine is associated with the separation of the placenta and the bleeding/

hemorrhage that results. Cocaine use is not usually an isolated incident, so the nurse should ask the client about thefrequency/amount of the drug usage.

drinks two glasses of wine before dinner every night. has intermittent contractions that are relieved by walking. had intercourse with her partner last night. used crack an hour before the symptoms began.

40. Which explanation is most appropriate when describing physiological jaundice to theparents of a newborn? Incorrect: Pathological jaundice, not physiological jaundice, occurs within the first 24 hours and is a result of an ABO incompatibility or Rh incompatibility.Correct: Physiological jaundice is the result of the breakdown of excessive amounts of red blood cells that are not needed after birth. Physiological jaundice is also related to theinability of the immature liver to rid the body of bilirubin, which occurs as the red blood cells are broken down. The bilirubin accumulates in the blood causing it to be yellow.Incorrect: Jaundice related to breast milk occurs after the first 7 days, not within the first three. It is not the cause of physiological jaundice.Incorrect: Hepatitis B may have beenacquired during delivery and may cause jaundice, but it is not the cause of physiological jaundice, which this case represents. "The baby has an minor incompatibility of the blood." The baby is breaking down the extra red blood cells that were present at birth. The baby is getting too much breast milk, but this is not dangerous.

The baby may have gotten exposed to hepatitis B during the delivery.

41. A woman at 42 weeks gestation enters the hospital for induction of labor. Since the infant is postterm, which complications should the nurse anticipate when planning forthe delivery? Incorrect: Cephalopelvic disproportion is seen in large-for-gestational age infants, not postterm infants. Hypothermia occurs in premature and small-for-gestational age infants.Correct: Asphyxia is a result of chronic hypoxia in utero because of the progressive degeneration of the placenta. Meconium stained amniotic fluid is a resultof the relaxation of the anal sphincter and the passage of meconium into the fluid related to hypoxia. If the meconiumstained fluid is aspirated into the infant's lungs at delivery, pneumonia (and possibly death) will result. If there is meconium stained fluid, the infant's mouth and throat are suctioned as soon as the head is delivered.Incorrect: Intraventricular hemorrhage occurs as a major complication in premature infants, not postterm infants. Dry, cracked skin is a normal finding of postterm infants and is not considered a complication.Incorrect: Hyperbilirubinemia is not acomplication of postterm infants at birth. Hypocalcemia is a complication in small-for gestational age infants Cephalopelvic disproportion and hypothermia Asphyxia and meconium aspiration Intraventricular hemorrhage and dry, cracked skin Hyperbilirubinemia and hypocalcemia

42. Which method of temperature regulation would safely and effectively prevent coldstress in a newly delivered infant?

Incorrect: The baby should be wrapped snuggly with a warm blanket in order to preserve heat loss.Incorrect: It helps to cover the feet, of course. However, because the scalp is so vascular (and the blood is close to the surface) and because the head makes up a large portion of the baby's surface area, most heat loss occurs via the head initially. Peripheral circulation is sluggish at first, so not much blood would be cooled by circulating through cold feet.Correct: Newly delivered infants lose a great deal of heat as the amniotic fluid evaporates from the surface of theskin. To prevent rapid heat loss, the baby's face and head should be dried and a hat placed on the baby's head.Incorrect: Infants should NEVER be placed on a heating pad because of risk for burns. Wrap the baby loosely with a blanket. Be sure the baby's feet are covered. Cover the baby's head with a hat. Position the baby on a heating pad.

43. The nurse performs Leopold's maneuvers for a client admitted in labor. Which is the main goal of Leopold's manuevers? Incorrect: Sterile vaginal exams are used to assess the dilation of the cervix.Incorrect: Leopold's maneuvers are not used to assess contraction frequency or intensity. However, some nurses do place their hands on the abdomen to palpate the intensity and frequency of the contractions.Incorrect: Leopold's maneuvers are not used to assess membrane rupture. Sterile vaginal exams may assess this if membranes are intact.Correct: Leopold's maneuvers are a method of determining fetal position by abdominal palpation. It assesses the position, presentationand engagement of the fetus. It also assists in the location of fetal heart sounds. To determine whether the client's cervix has dilated To assess the frequency and intensity of the contractions To assess whether membranes have been ruptured To determine the presentation and position of the fetus

44. Immediately after birth, the nurse places the newborn under a radiant warmer. Which is the primary rationale for the nurse's action? Correct: Temperature regulation is the priority for the newborn. Infants who are cold stressed are at risk for respiratory complications or death.Incorrect: Placing the infant in the warmer does assist the nurse with easieraccess, but temperature regulation is the main priority.Incorrect: Most infants are not connected to the cardiac monitor unless the Apgar scores are low.Incorrect: The warmer does provide easy access for the family, but this is notthe main reason for its use. To facilitate an efficient means of thermoregulation

To facilitate initial assessment by the nurse To permit the use of the cardiac monitor To permit close observation by the family members

45. A client, gravida 1, para 0, in active labor, is becoming increasingly anxious. Which statement by the nurse will block therapeutic communication with the client? Incorrect: Since this is the client's first baby, there will be concerns/anxiety because of the unknown expectations. This response is appropriate, and will help decrease anxiety by allowing identification and ventilation of fears.Incorrect: This response will encourage the client to talk and will foster good communication.Correct: This is an example of meaningless reassurance and will block therapeutic communication because the needs of the client arenot being met.Incorrect: This response will facilitate communication, not block it. "What concerns are you having now?" "Tell me how you are feeling." "Everything is going just fine." "You seem a little nervous."

46. A nurse prepares to teach a class regarding postpartum care and includes infections in the teaching plan. Which is the main cause of mastitis in the postpartum client? Correct: Poor breast-feeding technique and improper positioning of the baby are the main reasons for mastitis. Improper release of the baby's suction can lead to sore, cracked nipples, creating a portal of entry forpathogens.Incorrect: Poor hand washing is not the main reason that a woman

gets mastitis but can be a contributingcause. For example, if the woman touches her perineal pad and then the breast, the bacteria on the hands can causean infection.Incorrect: Systemic infections such as flu or cold are not the cause of mastitis, which is a localizedinfection.Incorrect: Prolonged nursing by itself does not cause mastitis. Often babies engage in nonnutritive sucking. Poor breast feeding technique Inadequate hand washing Systemic maternal infection Prolonged nursing

47. A postterm infant is delivered by cesarean section because of fetal distress and meconium-stained amniotic fluid. The nursery nurse frequently monitors the baby's

respiratory rate, observing for tachypnea. Which is the reason for the nurse's actions? The infant may:

Incorrect: Respiratory depression does not result in tachypnea but in apnea.Correct: This infant is a risk for meconium aspiration pneumonia related to postmaturity, meconium staining, fetal distress and being delivered by csection.Incorrect: Infants with respiratory distress (tachypneic) are usually cold stressed and hypothermic, nothyperthermic.Incorrect: A pneumothorax usually is seen in premature infants who lack surfactant. experience respiratory depression from the medications used during delivery. develop meconium aspiration pneumonia. have an elevated temperature. have a pneumothorax related to delivery.

48. The nurse notices a variable deceleration on a fetal monitor strip. Which nursingaction is appropriate? Incorrect: Hyperventilation is not the cause of the variable decelerations.Incorrect: Hypertonic uterine contractions refer to a labor with very painful but not necessarily effective contractions. The uterus does not relax between contractions. This leads to fetal distress and results in late decelerations, not variable decelerations.Correct: Variable decelerations are a result of cord compression. Turning the client onto her left side may improve fetal oxygenation by relieving pressure on the cord.Incorrect: Variable decelerations are a result of fetal cord compression.

Decreasing the fluids will not relieve cord compression. Instruct the mother to breathe slowly because this is a sign of hyperventilation. Decrease the amount of Pitocin because this is a sign of hypertonic uterinecontractions. Turn the woman onto her left side to relieve pressure on the umbilical cord. Reduce the oral and IV fluids to decrease circulatory overload.

49. The nursery nurse delays the first bottle feeding of a newborn. Which is the mostcommon reason for the nurse's actions? The infant has: Incorrect: One method of increasing an infant's low blood sugar is by feeding him.Correct: Bottle feeding of an infant who is tachypneic (resp. rate > 60) is contraindicated due to risk of aspiration.Incorrect: Acrocyanosis (blue hands and feet) is a normal finding for the first 24 hours.Incorrect: It is not unusual for the nurse to hear a heartmurmur shortly after birth. a blood glucose of 45 gm/dL. a respiratory rate above 60. blue hands and feet. a heart murmur.

50. During active labor, after a sudden slowing of the fetal heart rate, the nurse assesses the woman's perineum and observes a prolapsed cord. Which nursing action is mostappropriate? Correct: With a sterile gloved hand, the nurse should push the presenting part away from the cord, thus preventing cord compression. The cord supplies the fetus with oxygen and nutrients. The fetus is already showing

signs of distress because of the slowing of the heart rate. In addition, the nurse should prepare for immediatedelivery.Incorrect: Since the head is not engaged (which is why the cord prolapsed), it will be very difficult to insert a scalp electrode.Incorrect: Trendelenburg position places the client with her head lower than her feet. Reverse Trendelenburg places the client with the head higher than the feet. Due to gravity, this will place additional pressure on the cord.Incorrect: Covering the cord with a dry gauze will not help the situation. The gauze will get wet in a matterof seconds. There is a risk that the gauze will be lost internally. Hold the presenting part away from the cord. Insert a scalp electrode for an internal fetal monitor. Place the client in reverse Trendenlenburg position. Cover the cord with a dry, sterile gauze.

51. A client is in the latent stage of labor. Which nursing intervention is most appropriate? Correct: Latent stage is an early stage of labor, which begins with the onset of contractions and ends when the cervix is dilated to 4 cm. Walking adds gravity to the force of the contractions, promotes fetal descent, and relieves backache. Once the membranes rupture, bed rest may be indicated, for example if the fetal head is notengaged.Incorrect: Pushing is not indicated until full cervical dilation.Incorrect: This type of breathing pattern is used late in labor when pushing begins.Incorrect: Once labor begins fluids and ice chips are preferred. Nausea andvomiting are common as labor progresses. During labor, peristalsis stops. Therefore having food in the stomach is not advisable. Encourage the client to walk in the hall until membranes rupture. Instruct the client to place her head on her chest and push with the contraction. Teach the client to use the pant-blow method of breathing. Advise the client to eat a light meal consisting of carbohydrates.

52. Which conditions create a risk for uterine atony in the immediate postpartum period? Incorrect: Breastfeeding causes uterine contractions due to the release of oxytocin. Uterine atony is not related to the delivery of a child with chromosomal defect.Incorrect: Uterine atony is not a result of posttermpregnancy or amniotomy.Incorrect: Gestational diabetes in and of itself does not cause uterine atony. However, clients with gestational diabetes do have babies that are large for gestational age (> 4000 grams). Pregnancyinduced hypertension is associated with vasospasm, which does not result in uterine atony.Correct: Uterine atony is

the inability of the uterus to contract, which leads to hemorrhage. Clients who have had more than one delivery havedecreased muscle tone in the uterus. Clients with twins or triplets are at risk for overdistention of the uterus, whichmay lead to uterine atony and hemorrhage. Breast feeding and delivery of an infant with chromosome defects Postterm birth and an amniotomy during labor Gestational diabetes and pregnancy-induced hypertension Multiparity and multiple gestation

53. A client at ten weeks gestation tells the nurse that she has been having morning sickness. The nurse advises the client to eat foods that are easy to digest and low in fat. Which is the rationale for the nurse's instruction? Incorrect: Low fat diets do not stimulate peristalsis. On the contrary, high fat foods can lead to bloating, increased peristalsis and diarrhea.Correct: Foods containing a high fat content stay in the digestive system longer. Decreasing the amount of fat causes faster gastric emptying, which leaves less in the stomach to bevomited.Incorrect: Fluid and electrolyte imbalance is not a cause of nausea and vomiting related topregnancy.Incorrect: Relaxation of the cardiac sphincter, causing heartburn, is a result of increased progesterone. It causes heartburn, not nausea and vomiting. A low fat diet increases peristalsis, which reduces the food volume in the stomach A low fat diet is digested faster and leaves less in the stomach that can be vomited Easily digested foods provide a better balance of fluids and electrolytes, resultingin less nausea and vomiting Easily digested foods are less likely to cause relaxation of the cardiac sphincter,

which causes regurgitation and vomiting

54. Which information is most important for the nurse to gather when a client is admittedto the unit in labor? Incorrect: This is useful information, but the priority information is that regarding medical conditions which may create serious risks to the fetus and mother.Correct: Asking the client about any medical problems should be the priority because it provides a quick assessment for risks to the fetus and mother.Incorrect: Fluids are given in the latent phases of labor, but gathering this information at the initial admission interview is not as important as obtaining information about medical conditions which may create serious risks to the fetus and mother.Incorrect: This is not

important unless the client has PIH or a cardiac condition. Even then, the initial assessment would be to find out if theclient actually has PIH or cardiac condition (e.g., by checking the history), not to diagnose it.

Name of the support person Medical problems or complications Fluid preferences Amount of weight gained during the pregnancy

55. The nurse conducting a physical assessment notes that a 1-day-old newborn withdark skin has a bluish-gray discoloration over the lower back, the buttocks, and thescrotum. How should this assessment finding be documented? Incorrect: Bruising usually does not involve the scrotum, and is not usually gray.Correct: Mongolian spots are the result of increased pigmentation over parts of the baby. They are most commonly found in infants of Asian, Indian, African-American or Mediterranean descent. They are harmless and fade during the first two years of life.Incorrect: Nevus flammeus is a dark red lesion called a port wine stain. It does not blanch when touched, anddoes not fade with age. This type of hemangioma usually is seen on the face or thigh rather than the back.Incorrect: Acrocyanosis, a normal finding, is a bluish discoloration of the hands and feet (not the back or buttocks), and isrelated to sluggishness of the peripheral circulation. Extensive bruising Mongolian spots Nevus flammeus Acrocyanosis

56. A small-for-gestational-age infant is irritable and jittery, and has hyperreflexia and clonus. He is jaundiced, has temperature instability, and spitty after feedings. The nurse suspects the infant is displaying signs of passive addiction during pregnancy. When planning for the infant's care at home, which nursing assessment is most important forthe infant experiencing neonatal abstinence syndrome? Correct: In cases of maternal drug addiction, it is very important that the home situation be assessed because infant abuse and neglect are common in homes where there is drug/alcohol abuse.Incorrect: While this maybe important information to know, it does not address the infant or its care.Incorrect: Assessing whether or not the mother has money enough to afford treatment for her addiction is not as important as the infant's safety.Incorrect: Drug withdrawal is not measured in degree of severity. The baby is withdrawing, and that is all that is important. The mother's ability to provide a safe environment The extent of addiction of the mother The mother's ability to obtain treatment The severity of the infant' s withdrawal

57. A woman in active labor is admitted to the labor and delivery unit, accompanied by her partner. As labor progresses, the nurse notes he is not interacting with the woman and sits in the corner, looking out the window. How may the nurse understand the man's actions?

Incorrect: Other factors such as culture, personality, and language should be considered before assessing the inability to cope due to overwhelming concern for the woman. At this point there are no data to indicate overwhelming concern.Correct: These factors must first be considered along with ability to speak the language. Keeping in mind that there are individual and cultural differences in expressing concern will enable the nurse to make unbiased assessments.Incorrect: Embarrassment may be a reason for the man's actions, but is not the first consideration. It is important to first consider that there individual and cultural differences in expressing concern. This will enable the nurse to make unbiased assessments.Incorrect: If a man's religious beliefs prohibited him fromviewing a birth, he is not likely to be in the room durint the active phase of labor.

He is likely to be very concerned about the woman's health to the point that hisability to cope with the situation is compromised.

His actions reflect personality or cultural differences, which do not necessarily indicate a lack of concern. Due to his embarrassment and discomfort regarding the woman's expressions of pain, he withdraws from the situation. His religious beliefs regarding participation in the birth experience affect his interactivity and communication in this situation.

58. A client is admitted to the hospital with severe pregnancy-induced hypertension (PIH). The physician orders magnesium sulfate. Which nursing intervention is importantwhen administering this drug? Correct: Because hypertension is a sign of PIH, the client's BP must be monitored. The client's respiratory rate should be monitored because one sign of magnesium sulfate toxicity is a respiratory rate under 12/min.Incorrect: Assessing blood glucose levels does not pertain to PIH but to gestational diabetes.Incorrect: A side effect of magnesium sulfate is a decrease in blood pressure, which might cause orthostatic hypotension; however, the client with severe PIH will be on strict bed rest and not allowed to walk.Incorrect: Magnesium sulfate may be used forpreterm labor to slow contractions, but this does not pertain to PIH. Assess blood pressure and respiratory rate every fifteen minutes. Monitor blood glucose levels every eight hours. Evaluate for orthostatic hypotension when getting the client up to walk.

Observe for premature labor every shift.

59. A 27-week gestation infant is taken to a newborn intensive care unit 150 miles away. Initially, which emotion should the nurse expect the mother to display after the transfer? Incorrect: Usually denial is seen when the mother fails to recognize the severity of the situation. Denial would probably have occurred before the transfer, when the mother first learned about the baby's critical status.Incorrect: The mother may display frustration but it is aimed at not being able to follow the baby to the intensivecare unit, and lack of knowledge about the child's condition.Correct: The mother feels a great deal of guilt for not having a perfect baby and perhaps for causing the baby pain and discomfort. The mother may also feel that she could have done something to prevent the early delivery. This is the primary emotion to expect.Incorrect: Anger is not usually seen initially. It occurs later in the grieving process. Denial Frustration Guilt Anger

60. A 38 week gestation newborn weighs 4020 grams, is sluggish, and has limp muscle tone. The baby experienced a broken clavicle during delivery. Based on this information, which can the nurse conclude about the baby? Incorrect: Normally infants who are withdrawing from drugs are hyperactive and jittery, not lethargic and

limp.Correct: These symptoms indicate a large-for-gestational-age (LGA) infant. LGA infants typically have diabetic mothers and have respiratory problems and difficulty with stabilization of blood sugar.Incorrect: The baby's signs andsymptoms are reflective of large-for-gestational age, not a heart defect.Incorrect: Respiratory depression may causethe infant to be limp, but this does not account for the baby's elevated weight. Neonatal abstinence symptoms Large for gestational age Congenital cardiac defect Respiratory depression

61. Which assessment finding suggests thrombophlebitis in a postpartum client? Incorrect: These signs and symptoms are indications of pulmonary embolism.Incorrect: These are signs and symptoms of a pulmonary embolism. Pulmonary emboli may occur as a result of clot formation in the calf.Correct: These signs and symptoms are common for clients with thrombophlebitis. Thrombopheblitis occurs because of

changes in the blood volume and coagulation factors that result after delivery. Although eliciting a Homan's sign coulddislodge a thrombus, it is considered a positive sign.Incorrect: These signs and symptoms do not relate to thrombophebitis.

Dyspnea, tachypnea, and apprehension Chills, hypotension, and abdominal tenderness Positive Homan's sign, calf warmth, and pain Dizziness, loss of consciousness, and chest pain

62. A client comes to the clinic to confirm that she is pregnant. Her last menstrual period was January 31st. According to Naegele's rule, when should the client expect to deliver? Incorrect: Seven days have not been added.Incorrect: Only two months have been subtracted.Correct: When using Nagele's rule to estimate delivery dates, the nurse takes the client's last menstrual period (LMP), adds 7days, and then subtracts 3 months. Adding 7 days to the LMP of January 31st makes it February 7th. Subtracting 3 months then makes the due date November 7th.Incorrect: Seven days have been subtracted instead of added to theLMP. November 31 December 7 November 7 December 24

63. Which procedure should be avoided for the client known to have a placenta previa?

Incorrect: Non-stress tests are necessary to monitor the well-being of the fetus. Non-stress tests are usually performed if the client returns home after a bleeding episode.Incorrect: Performing a catheterization has nothing to dowith placenta previa.Correct: In placenta previa, the placenta covers all or part of the cervical opening. Therefore vaginal exams are contraindicated because of risk of bleeding or infection. Hemorrhage is the main complication ofplacenta previa.Incorrect: Abdominal ultrasounds are non-invasive and are commonly performed upon admission to the hospital to locate the position of the placenta. A non-stress test A urinary catheterization A sterile vaginal exam An abdominal ultrasound

64. A woman in the first trimester comes to the clinic with vaginal bleeding. The physician determines that the fetus has died and that the placenta, fetus, and tissues stillremain in the uterus. How should the findings be documented? Incorrect: A complete abortion occurs when all products of conception are expelled.Incorrect: Stillborn is a lay term that means the baby has died. This does not address the products of conception such as the placenta or tissues.Correct: Prolonged retention of the products of conception (placenta/tissues) after the fetus has died is knownas a missed abortion. Infection and coagulation defects are common complications.Incorrect: An incomplete abortion occurs when some, but not all, of the products of conception have been expelled. Complete abortion Stillborn abortion Missed abortion

Incomplete abortion

65. A woman in the transition stage of labor is using paced breathing to relieve pain. She complains of blurred vision, numbness, and tingling of her hands and mouth. Whichcondition is indicated by these signs and symptoms? Incorrect: Anoxia/hypoxia results in restlessness, nasal flaring, and cyanosis of the lips and nailbeds. The signs and symptoms listed in the question are not related to anoxia.Correct: These signs and symptoms are a result of hyperventilation. The nurse should have the client breathe slower and into a paper bag to counteract the signs and

symptoms.Incorrect: Anxiety usually causes rapid heart rate and muscle tenseness, not the symptoms listed in the question.Incorrect: While hypertension often affects vision, it is not the reason for this cluster of signs and symptoms.

Anoxia Hyperventilation Anxiety Hypertension

66. Which data support a diagnosis of abruptio placenta in a pregnant woman? Correct: These are classic signs of an abruptio placenta. Other signs and symptoms include: dark, red vaginal bleeding, fetal distress, signs of hypovolemic shock.Incorrect: These are signs of placenta previa, not abruptioplacenta.Incorrect: These have nothing to do with abruptio placenta.Incorrect: Bright red blood loss is a sign of placenta previa. Hypertension may occur in abruptio placenta, however. Uterine rigidity and abdominal pain Painless bleeding with soft abdomen Premature rupture of membranes and uterine contractions Bright red blood loss and elevated blood pressure

67. A women in her first trimester contracts rubella. How is the fetus likely to be affected? Incorrect: Rubella is usually associated with hearing, vision and cardiac defects.Correct: The rubella virus

usually causes mild illness in the mother, but has devastating effects on the fetus, including cataracts, heart defects (patent ductus arteriosus and pulmonary stenosis are the most common), deafness, mental and motor retardation, growth retardation and clotting disorders.Incorrect: Spinal cord defects are a result of the inability of the vertebrae tofuseit is a congenital problem and not related to rubella.Incorrect: Polydactyly, the presence of extra digits (fingers or toes), and club feet are not usually seen in fetuses with rubella. Reproductive and urinary defects Heart defects and cataracts Spinal cord and skeletal defects Polydactyly and club feet

68. An hour after delivery, a 4000 gram infant exhibits pallor, jitteriness, a blood sugar level of 40 gm/dL, irritability and periodic apnea. Which maternal condition could be thecause of the newborn's symptoms? Incorrect: Jitteriness and irritability may indicate a drug withdrawal problem, but the large birth weight and the low glucose levels indicate an infant of a diabetic mother.Incorrect: Jitteriness, irritability, and pallor are classic signs of hypoglycemia in the infant with a history of gestational diabetes. Infants born to mothers with pregnancyinduced hypertension may be small for gestational age due to uteroplacental insufficiency.Incorrect: TORCH infections do affect the baby, but the symptoms described do not indicate a TORCH infection.Correct: These signs and symptoms are classic of an infant of a diabetic mother. Drug addiction Pregnancy-induced hypertension TORCH infection

Gestational diabetes

69. A client delivered vaginally six hours ago. Which assessment finding can be interpreted as normal? Correct: A slight elevation in temperature during the first 24 hours post delivery may be a result of dehydration. Temperature elevations after 24 hours are considered abnormal.Incorrect: A reading of 140/90 may indicate hypertension, which is a serious complication.Incorrect: A respiratory rate of 10 is not normal and could be a result of medications/narcotics given during labor.Incorrect: After delivery the pulse rate is usually slightly lower than normal (usually 60-70 bpm) because of the fluid shifts and diuresis.

Temperature 100.0 degrees F Blood pressure 140/90

Respirations 10 Pulse 90

70. A new mother receives instructions about care of her newborn son's circumcision. Which statement made by the mother indicates that further teaching is needed? Incorrect: The doctor should be notified if there is prolonged, excessive bleeding or signs of infection.Correct: This statement indicates that the client does not understand about the care of the circumcision. The yellowish mucous is normal and is from accumulated yellow serum. It helps in normal healing and should not bewashed away.Incorrect: Vaseline gauze or betadine ointment should be applied with each diaper change to prevent the penis from sticking to the diaper.Incorrect: Sponge baths should be given to circumcised babies for the first 7 to 10 days. If a plastibell is used, tub baths may be given when the bell falls off (usually 7 10 days). "I will call the doctor if my baby's penis starts to bleed." "I should wash off any yellowish mucous on my baby's penis." "I will put vaseline on his penis every time I change his diaper." "I should give my baby a sponge bath for the first week."

71. A 17-year-old client delivered her first baby 8 hours ago. Which of the following is an indication that appropriate bonding is occurring? The client: Correct: Making eye contact is a sign of positive attachment. Other signs include: speaking or singing to the infant, talking about the physical characteristics of the baby, (big feet, little nose etc.), calling the baby by name,

stroking or massaging the baby to quiet it.Incorrect: Asking why the baby cries is not a sign of attachment and it may be a sign of rejection or neglect.Incorrect: This may indicate the lack of caretaker responsibility. Participation in infant care increases bonding.Incorrect: Asking if the baby is cute is not an indication that an attachment is occurring. The nurse's response must be carefully thought out to avoid blocking expressions of concern by the client. makes eye contact with the baby. wonders why the baby cries so much. asks the nurse to help change the baby' s diaper. asks the nurse if the baby is cute.

72. A new mother is crying in her room. She tells the nurse that her new baby boy has enlarged breasts and she thinks that there is something wrong. How should the nurserespond? Incorrect: This statement would increase the client's worry and anxiety by confirming her fears that something is wrong. The baby's symptom is completely normal.Correct: Enlarged breasts are common as a result ofhormonal withdrawal. Breast enlargement usually subsides within the first few weeks after delivery. This response provides the mother with information, which should decrease her anxiety.Incorrect: This may be an appropriate response, but should not be the nurse's first response because it suggests the possibility that something is wrongwith the baby. The baby's symptom is completely normal.Incorrect: This statement is an example of meaningless reassurance. "You should ask your doctor about that."

"Enlarged breasts are common for both boys and girls. It will go away." "Let me look at the baby for you." "Everything is going to be just fine. Your baby is healthy."

73. During the active phase of labor, the membranes rupture and the nurse notes green amniotic fluid. Which nursing action should be initiated immediately? Incorrect: Green amniotic fluid is an indication of meconium staining, which may indicate fetal distress. The physician should be notified but not before assessing the status of the fetus.Incorrect: This is a comfort measure. It can wait until after the nurse assesses for fetal distress.Incorrect: Testing the fluid usually differentiates amniotic fluid from urine.Correct: Any time the membranes rupture, the nurse should immediately assess fetal heart rate, especiallywhen the fluid is meconium stained, as this may indicate fetal distress.

Call the physician.

Replace the soiled underpad. Test the fluid with pH (Nitrazine) paper. Assess fetal heart rate.

74. At 28 weeks gestation, a woman enters the hospital in preterm labor and receives atocolytic medication to stop labor. Which assessment findings should be reported immediately to the physician? Incorrect: Fetal heart rate of 160 is considered normal. Incorrect: These represent Braxton Hick contractions, not true labor. Contractions should be monitored closely for intensity, frequency and duration. Incorrect: The vital signs are within normal limits. Correct: Ferning is an indication of amniotic fluid, which indicates that the membranesare ruptured. This should be reported immediately because delivery may be imminent. With ruptured membranes, theclient should be monitored for infection.

Fetal heart rate averaging 160 beats/min Irregular contractions every 15-20 minutes that last 30 seconds before stopping Maternal temperature 98.8 degrees F, pulse 84, respiratory rate 22, BP 130/70 Ferning pattern of vaginal discharge under a microscope

75. A labor and delivery nurse suspects that a client is in the transition stage of labor. Which information supports this conclusion? The client is: Incorrect: These are typical signs of the latent or early phase of labor. Correct: These are typical signs of the transition phase of labor. In addition to irritability and the inability to focus, the client may exhibit anger, loss of control,

anxiety, mood swings, rectal pressure, and increasing amounts of pain. Incorrect: These occur in the second stage of labor, just prior to birth. Incorrect: These are signs that the client is in the latent or early phase of labor.

walking around the unit and talking with her partner. irritable and needs frequent repetition of directions. expelling feces and the fetal head is crowning. reading a magazine and talking on the phone.

ATI. CHILD CARE 2.0

The parents of a 5-month-old infant state that their infant seems to eat very little. Most of thefood comes out of the infant's mouth and onto his clothes. Which of the following explanationsshould the nurse give to the parents?

Trying to introduce food after the intake of a bottle formula is usually not recommended because the infant is satiated and has no inclination to try something new.Solid foods should be offered at 4 to 6 months. The gastrointestinal tract has matured enough to handle more nutrients and is less sensitive to potentially allergenic foods.This deprives the infant of the pleasure oflearning new tastes and developing a discriminating palate. It may cause problems with poorchewing because of lack of experience.Due to the extrusion (protrusion) reflex, the

infant'stongue pushes the food out of the mouth. It is most helpful to suggest using a longhandledspoon and placing the food in the back of the infant's mouth to avoid the reflex. "Give the baby a bottle of formula before solid food to assure adequate caloricintake." "Stop the solid foods and try again when the baby is 12 months old." "Put the cereal in a bottle and feed the baby through a nipple with a large hole." "Place the food in the back of the baby's mouth using a long-handled spoon." 2

A nurse smells an odor identified as marijuana coming from a room. Which of the following clientfindings would confirm inhalation of the substance?

All are findings of a client who has smoked/inhaled cannabis/marijuana. These clients are typically euphoric or somewhat mildly intoxicated. They have poor coordination with bloodshot (red) eyes and may laugh inappropriately.These findings are more commonly due to of the effects of depressants.These findings are more commonly due to the effects of opiates.These findings are more commonly due to the effects of cocaine. Poor coordination, red eyes, and euphoria Slurred speech, confusion, and combativeness Loss of consciousness, respiratory depression, and coma Hypertension, tachycardia, and hyperflexia3

A nurse is checking children at an orthopedic outpatient setting. Which of the following should the nurse expect to see as manifestations of scoliosis?

Lumbar curvature is a manifestation of lordosis.These are manifestations of scoliosis. Often parents observe that a child's skirt doesn't hang straight or the pant legs areuneven.Tenderness is a general symptom that may indicate something is wrong in an underlying organ. A nurse could not see changes such as swelling of the spine.These symptomscould be associated with other orthopedic problems but are not characteristic of scoliosis. Pain and an exaggerated lumbar curvature Uneven shoulder heights and poorly fitting slacks

Tenderness and swelling of the spine Limited range of motion of the back and a limp4

A nurse is providing client/patient education to the mother of an 8-year-old child diagnosed with B-hemolytic streptococci infection (strep throat). The nurse emphasizes the importance of promptly starting and completing the entire course of antibiotics. The mother asks why this is important. The nurse states that the antibiotic will

Pain may interfere with oral intake, but this is not the priority concern with promptdiagnosis and care of strep throat. Cool fluids or ice chips may be comforting. Relief to the neckmay be provided by the application of cold or warm compresses to the area. Warm saline gargles may also relieve throat discomfort.Sinusitis and abscess formation on the pharyngealand peritonsillar areas are complications that can develop with a strep throat infection, but thesecomplications are not of the greatest concern with this infection.Anterior cervicallymphadenopathy is a symptom of a streptococcal infection resulting in pharyngitis and tenderlymph nodes. This usually subsides in 3 to 5 days if uncomplicated.Antibiotics should be initiated as soon as possible and taken as prescribed to quickly and completely eliminate thestreptococcal organism, which can lead to acute rheumatic fever, glomerulonephritis, and acute renal failure. alleviate painful swallowing to avoid complications of dehydration andmalnutrition. prevent sinusitis or abscess formation on the pharyngeal or peritonsillar areas. reduce the risk of anterior cervical lymphadenopathy. eliminate organisms that might initiate acute renal failure or rheumatic fever. 5

A nurse is reinforcing teaching about accidental poisoning to a parent during a routine well-child visit. The nurse asks the parent, "What would be your first response if your child accidentally took an overdose of acetaminophen (Tylenol)?" Which of the following statements by the parent would indicate a correct understanding?

Syrup of ipecac is no longer recommended as a routine home treatment of poisoning. Giving syrup of ipecac might possibly be appropriate, but certain substances that are corrosivewould make using this measure contraindicated because it would increase the damage to themucosa lining.Placing the child into a side-lying position is an appropriate measure to preventaspiration.Calling the Poison Control Center is the best initial response to an accidentalpoisoning because each case needs to be dealt with by getting prompt medical attention toinitiate the appropriate emergency treatment actions.Giving the child one sip of water, not a full glass, is appropriate to dilute the ingested poison. However, this is not the first action that should be taken. "I will give my child a dose of ipecac." "I will place my child on her back." "I will call the Poison Control Center." "I will get my child to drink a full glass of water."

A nurse is caring for a 23-month-old child with iron-deficiency anemia. The parents indicate theyhave been taught about the diagnosis, but are concerned that they are not doing all that theyneed to do. Which of the following should the nurse include when reinforcing teaching?

Cow's milk contains substances that bind with iron and interfere with its absorption. Ironshould not be given with milk or milk products.There are no food limitations or suggestions whenchildren are taking oral iron preparations. Foods with vitamin C, such as citrus fruits, enhancethe absorption of iron.Oral iron supplements do not cause GI bleeding or ulcers.Liquid iron maystain the teeth, so the nurse should instruct the parents to give it through a straw placed in theback of the child's mouth to avoid staining the teeth. Give the oral iron supplementation with a glass of cow's milk to prevent stomachproblems. Provide diet instructions including limiting citrus fruits in favor of more vegetables. Provide information about complications of iron including gastrointestinalbleeding and ulcers. Give liquid iron through a straw placed in the back of the mouth. 7

A nurse is reviewing discharge teaching with the parents of a child who has pediculosis. Which of the following should the nurse include in the teaching?

Children should not share combs, hair ornaments, hats, caps, scarves, coats, and otheritems used on or near the hair.Pets are not carriers of lice.Clothes should be dried in a hot dryerfor at least 20 min to kill the lice.Lice need a blood source to survive. Placing the nonwashableitems in a sealed plastic bag for 14 days will kill the lice. "Children can share scarves and coats, but not hats or combs." "Household pets can carry and transmit lice to people." "After washing clothing, hang clothes outside to dry." "Seal nonwashable items in plastic bags for 14 days." 8

A nurse is caring for a toddler who is in an oxygen tent. Which of the following actions should the nurse take in order to promote comfort while maintaining the child's safety?

Not all toys are safe to put inside an oxygen tent. Vinyl or plastic toys that do not absorb moisture are suitable to put inside the tent. Stuffed animals absorb moisture and are difficult to dry. High levels of oxygen are a source of sparks, so mechanical or electrical toys are a potential fire hazard.The moisture inside an oxygen tent will make the child cold and the child'sclothes moist. Therefore, the nurse should try to keep the child warm and dry by changing bedding and clothes, which will enhance the child's comfort without compromisingsafety.Oxygen is heavier than air; therefore, oxygen loss will be greater at the bottom of the tent. The tent should be tucked snugly without open edges to prevent oxygen loss.Some tents areopened at the top. Oxygen is a heavy gas and most of it will stay at the bottom of the tent. This measure does not promote the child's comfort while in the oxygen tent. Give the child a stuffed animal and car with rubber wheels to play with. Change the bedding and the child's clothing frequently or as often as needed. Tuck the bottom of the tent under the mattress on three sides, leaving one side open so the child can look out. Cover the opening on the roof of the tent with a blanket to prevent the child frombecoming chilled. 9

A nurse is reinforcing teaching with the parent of a child with a urinary tract infection. Which of the following statements made by the parent indicates understanding of how to prevent futureinfections?

Children should be encouraged to void frequently, especially before long trips or other circumstances in which toilet facilities may not be available for an extended period of time. Urinethat is held can harbor bacteria that can result in a urinary tract infection.Cotton underwearallows for more air flow to the perineal area and reduces the risk of urinary tractinfections.Wiping from back to front increases the risk of feces entering the urethra and causinga urinary tract infection.Bubble baths and perfumed perineal products can irritate the urethraand lead to a urinary tract infection. These should be avoided, especially for girls. "I will bring my child to the bathroom before we leave for extended trips." "I need to switch my child from cotton underwear to nylon underwear." "I should teach my child to wipe from back to front after urinating." "I will have my child soak in a bubble bath once or twice a week." 10

A nurse is reviewing discharge instructions with the parent of an infant who has acute laryngotracheobronchitis (croup). Which of the following statements made by the parentindicates a need for further teaching?

This is a correct intervention. Corticosteroids have an anti-inflammatory effect that decreases subglottic edema. This will make breathing easier.This is a correct intervention. Clearing the nasal passages decreases the amount of secretions in the upper and lowerairways.Dry air will exacerbate the child's croup. Cool temperature therapies are advocated forthis condition. Cool mist constricts edematous blood vessels. A cool air vaporizer can be used athome to maintain high humidity and provide relief. Warm mist from warm running water such as

a hot shower in a closed bathroom may be beneficial.It is essential that children withlaryngotracheobronchitis (croup) be allowed and encouraged to drink any fluids they like toincrease fluid intake.

"I will give my child the corticosteroids prescribed by the doctor." "I will clear the child's nasal passages with a bulb syringe to aid in breathing." "I will place a dehumidifier in my child's room." "I will encourage my child to take plenty of fluids over the next several days."

11

A 15-year-old client visits the clinic to get medical clearance to play a sport. The nurse reviews measures to prevent athlete's foot with the client. Which of the following statements by the client indicates that the instructions were understood?

Many people believe tinea pedis is transmitted via showering in the same location as someone who is infected. However, transmission of tinea pedis to other individuals is rare.Ointments have not proven to be successful in treating tinea pedis. Application of antifungal powder containing tolnaftate or tolnaftate liquid is a treatment measure. Medication is not usually recommended as a preventative measure.The client should avoid heat and perspiration by wearing light socks.Wearing well-ventilated shoes and clean, lightweight socks is encouraged in order to prevent heat and perspiration conditions. Occlusive shoes should be avoided. "I will avoid showering at the gym." "I can apply an antifungal cream daily."

"I should wear dark-colored socks." "I should wear well-ventilated shoes." 12

A nurse is collecting data on a 3-year-old child with eczema in an outpatient center. The parent asks whether any changes can be made at home to prevent the recurrence of eczema. Which of the following is an appropriate response by the nurse?

Clients with eczema should avoid any material that produces heat, as this can cause perspiration and itching.Bubble baths and harsh soaps cause drying of the skin and can further irritate the eczema.A room humidifier or vaporizer may be helpful for keeping moisture in the air and keeping the skin from drying.Woolen clothing or blankets cause itching and should not be used. "Cover the crib mattress with a plastic cover." "Give the child a bubble bath for 20 min each day." "Place a humidifier in the child's room." "Dress the child in warm wool clothing in cold weather." 13

A nurse is caring for a 7-year-old child who is admitted with an asthma exacerbation. This is the third admission since diagnosis 6 months ago. Which of the following topics should be

reinforced with the parents and child in order to prevent future readmissions?

Monitoring the child's oxygen saturation and respiratory rate provides information about how well the child is oxygenating, but does not prevent future attacks.Allergen control is aimed at the prevention of exposure to airborne allergens and irritants that can trigger an asthma attack. Preventing exposure to allergens does reduce the risk for future attacks.Peak flow readings allow parents to make educated decisions regarding asthma management. However, these measurements will not prevent future asthma attacks or hospitalization.Upright positioning

is important to help with lung expansion during an asthma attack, but it is not a preventative measure. Monitoring oxygen saturation and respiratory rate daily Identification and avoidance of factors that trigger symptoms Monitoring peak flow measurements regularly Positioning the client upright in a position of comfort14

Which of the following physical manifestations of a client with anorexia nervosa best indicatescompliance with the treatment plan of care?

Effectiveness of nursing interventions includes weight gain or no further weight loss. Measuring weight is routinely completed to determine the effectiveness of the plan of care. This is the best indicator of compliance with the treatment plan.Return of soft bowel movementsindicates that the client is not using laxatives or enemas to speed up the intestinal passage offood. This is a good indication but is not the best indicator of compliance.This is a good indication of weight gain and normalizing of body function, but it is not the best indication oftreatment plan success.Improvement of the oral mucosa indicates that nutritional deficienciesare improving. This is a positive sign, but not the best indicator of compliance with the treatmentplan of care. A weekly weight gain of 1 kg (2.2 lb) Daily bowel movements that are soft Return of regular menstrual periods Improvement of the oral mucosa15

An assistive personnel (AP) is caring for a child diagnosed with leukemia and undergoingchemotherapy. In which of the following clinical situations should a nurse intervene?

Chemotherapy can damage gastrointestinal mucosal cells. Using a soft toothbrush willprovide mouth care and will be gentle on the mucous membranes in order to preventulceration.Hair loss is a common side effect of chemotherapy. Children often feel better if their heads are covered so no one can see that they have lost their hair. A soft cap is most comfortable and won't increase perspiration or cause itching as do othermaterials.Chemotherapy will put children at risk for infection secondary to immunosuppression, so all visitors with infections are restricted.The rectal area is prone to ulceration from variousdrugs, feces, and urine. Urine and feces must be removed immediately and the perianal areawashed. Using rectal temperatures is avoided to prevent trauma. The AP offers a soft toothbrush for oral care. The AP applies a soft cotton cap to the child's head. The AP maintains a restriction of all visitors and health personnel with infections. The AP prepares to take a rectal temperature.

16 A nurse is preparing to administer an intramuscular (IM) injection to a 2-month-old infant. Which of the following is the preferred injection site?(PICTURE HERE)

The vastus lateralis is the preferred site for IM injections in infants.The deltoid muscle isnot the preferred site for IM injections in infants.It is recommended that the ventrogluteal site notbe used until infants begin walking. 17

A nurse is reinforcing teaching with the parent of an infant who has club feet with bilateral casts. Which of the following statements should be included in the teaching?

If a cast is too tight, circulation will be impaired and the toes will swell.Serialmanipulation and casting allows for the gradual stretching of skin and accommodates the rapidgrowth in early infancy, and is performed every week for 8 to 12 weeks. If normal alignment is not achieved by 3 months, surgical intervention is indicated and will take place at about 6 to 12months of age.It can take 24 to 48 hr for the cast to dry completely. A regular fan or cool-air hair dryer to circulate air may facilitate drying when humidity is high. Heated fans and dryers shouldnot be used because they can cause the cast to dry on the outside but remain wet on the inside. They may also cause burns from the conduction of heat from the cast to the underlyingtissue.Pain is not a problem associated with casting for club feet. "Check the toes for any swelling or discoloration." "Monthly recasting should be scheduled with the orthopedist." "Use a heated fan or dryer to facilitate the drying of the cast." "Give the baby Tylenol every 4 hr to help with pain." 18

A nurse is caring for a child with measles. Which of the following actions is appropriate supportive care?

Photophobia accompanies rubeola; therefore, diversional activities with bright lights arecontraindicated. Dimming the room lights is soothing for the child.Isolation should be until

day 5of the rash. The period of communicability is from 4 days before the appearance of the rash until5 days following the appearance of the rash.An elevated temperature is common. Overheating, which increases itching, should be avoided. The child should wear lightweight, loose, and nonirritating clothing, and keep out of the sun. Antipyretics should also be administered.Vitamin A supplementation reduces the morbidity and mortality in children with the measles. Children with measles should be given vitamin A supplements. Nurses need to instruct parents on safe storage and administration of vitamin A to prevent excessive administration and possible toxicity. Provide diversional activities such as video games. Maintain isolation for 48 hr after the rash resolves. Keep the child warm with adequate undergarments and bedding. Administer vitamin A supplements as prescribed. 19

A nurse is caring for a 14-year-old client diagnosed with diabetes mellitus. The nurse is discussing the ongoing monitoring needed with this diagnosis. Which of the following should beincluded in the discussion?

When children are ill their fluid intake should be monitored. They often drink less, leading to dehydration. When children are hyperglycemic, dehydration from illness leads to increasedhyperglycemia and requires extra fluid intake.Exercise results in increased movement ofglucose into the cells and decreased blood glucose levels. The client should have a snack, not additional insulin.There is poor correlation between glycosuria and blood glucose. Bloodglucose monitoring is much more accurate than urine glucose monitoring.Children with diabetesshould

increase the amount of whole grains, fruits, and vegetables, which contain complexcarbohydrates, in their diets. Concentrated sweets are avoided to prevent hyperglycemia. The illness requires careful attention to fluid balance since hyperglycemiacontributes to dehydration. Exercise requires additional insulin since glucose will be released from the cellsduring activity.

Urine glucose must be monitored because there is a correlation betweensimultaneous glycosuria and blood glucose concentrations. The diet needs to include fewer complex carbohydrates because they quicklyraise blood glucose. 20

A nurse is reinforcing teaching with the parent of a 4-year-old child with influenza. Which of the following should the nurse include in the teaching?

Influenza is spread by direct contact. This means it can be spread from one person to another or by touching an object that has been contaminated by nasopharyngeal secretions.Themost infectious period for influenza is 24 hr before and after the onset of symptoms.There is apossible link between aspirin and Reye syndrome, so children with influenza or other viralillnesses should not be given aspirin. Most cases of Reye syndrome follow a common viralillness such as chickenpox or influenza.The immunization vaccine can be given at the sametime as other vaccines, but must be given in a separate syringe and at a different injection site. Influenza is transmitted by airborne means, so handwashing will not preventtransmission. Children are not infectious after 12 hr from the onset of influenza symptoms. Aspirin should not be given to children with influenza for relief of discomfort. The influenza vaccine may not be given at the same time as otherimmunizations.

21

A nurse is discussing nutrition with an adolescent who is pregnant. The adolescent's parent is inthe room. Which of the following statements made by the parent indicates a need for furtherdietary instruction?

This statement needs clarification. Snacks containing sugar are often eaten by theadolescent who is pregnant, but are not a good source of calories for energy and nutrition forthe developing fetus.Whether pregnant or not, an adolescent's nutritional needs include anincrease in calcium, protein, and iron.Nutritious between-meal snacks are a good source ofenergy. Complex carbohydrates of wheat and whole grains and fruits are appropriate snacks.This is a good suggestion because the adolescent does need additional calories in thesecond and third trimester. "I told my daughter that any calories ingested are a source of energy andnutrition." "I try to provide foods with an increased amount of calcium, protein, and iron." "I encourage between-meal snacks that are complex carbohydrates and fruits." "I have planned meals and snacks for additional calories in the second and thirdtrimester." 22

A nurse is caring for a 14-year-old child with appendicitis who has a pain rating of 8 on a scale of 1 to 10. The child has just returned to the unit after a computed tomography (CT) scan of the abdomen and tells the nurse the pain just stopped. Which of the following should the nurse dofirst?

Even though continued pain assessment is important, this is not the first priority with thesudden relief of pain.The child's vital signs will need to be taken before surgery, but this is not

the priority at this time.The sudden cessation of pain in a child with appendicitis should cause the nurse to suspect a ruptured appendix. The primary care provider should be notified immediately since the client is at increased risk for developing peritonitis, which can cause death if appropriate interventions are not immediately taken.The nurse would assess bowel sounds as the child is prepared for surgery, but this is not the priority at this time. Continue with the pain assessment. Take the child's vital signs. Notify the primary care provider. Auscultate the child's bowel sounds. 23

A nurse is monitoring a 9-year-old child on the first postoperative day following abdominal surgery. The nurse notes the child grimacing and guarding her abdomen. Which of the following pain assessment tools should the nurse use based on its acceptance by children?

The poker chip scale is used by children, but is not rated as the most preferred by that age group.The FACES pain rating scale is the best choice for a 9-year-old child because it includes visual face, numerical correspondence, and text stating feelings. This is the most preferred scale for children.The visual analog scale allows a child to mark a line stating the length of pain; however, this scale is difficult to determine and clarification is needed.The numerical 1 to 10 rating scale is used for adults. It has too many options and often requires clarification when used with children.

Poker chip tool FACES rating scale Visual analog scale Numerical 1 to 10 rating scale 24

A nurse is reinforcing teaching to a group of parents about preventing accidental poisoning in preschoolers. Which of the following should the nurse include?

This does not prevent accidental poisoning.Parents should caution against eating inedible items such as houseplants.Never remove labels from containers of toxic substances. Store toxic agents in a locked cabinet.All potentially toxic agents should be placed out of reach in a locked cabinet. Toddlers may be able to climb onto the sink and get into the cabinet. Have syrup of ipecac available in the home. Explain to preschool children that plants can be eaten only after they are cooked. Keep labels on containers of toxic substances and never remove them. Place medications in a cabinet above the sink. 25

A nurse is caring for a 4-month-old infant with thrush (candidiasis) who is breastfed. Which of the following is an appropriate nursing action?

To prevent relapse, therapy with nystatin should be continued for at least 2 days after the lesions disappear.No change in feeding is needed as part of the care for a client with thrush.No change in feeding is needed as part of the care for a client with thrush. Sucking may be painful, but changing to a formula or another method of feeding will not alleviate the discomfort.Oral candidiasis cannot be removed with a tongue blade and attempting to do so will cause bleeding and discomfort for the infant. Administer the prescribed nystatin (Mycostatin) for 2 to 3 days after the lesions disappear.

Place the infant on a soy-based formula to supplement breastfeeding until thrush is resolved. Discontinue breastfeeding and resume 48 hr after the last lesion disappears. Scrape off the white patches of thrush from the oral mucous membrane with a tongue depressor.

26

A nurse is caring for an adolescent with inadequate weight gain. Which of the following nutritional considerations is important to reinforce when talking with the client about appropriate nutrition?

Generally, adolescents obtain or exceed the recommended carbohydrate consumption.Limiting fat consumption is important to overall health.Adolescence is a time of accelerated physical growth, which can include doubling the adolescent requirements of calcium, iron, zinc, and protein. Maximum bone mass is acquired during adolescence making calcium intake during these years essential.Just increasing calories will not ensure adequate consumption of the necessary vitamins and minerals necessary to assure good health. Identify food preferences high in complex carbohydrates. Identify food preferences high in saturated and unsaturated fats. Identify food preferences high in calcium and protein.

Identify food preferences high in calories. 27

A primary care provider prescribes amoxicillin suspension to a child with otitis media who weighs 22 lb. The prescription reads: 30 mg/kg/day in divided doses every 8 hr. The pharmacy carries the medication as 200 mg/5 mL. How may mL should the nurse give in one dose? Answer in mL.

mL First, convert the client's weight to kg: 2.2 lb = 1 kg, so 22 lb = 10 kg.

24 hr/day = 3 doses/day 8 hr/dose

30 mg/kg/day = 10 mg/kg/dose 3 doses

10 mg x 10 kg = 100 mg every 8 hr

The desired dose is 100 mg. The medication is available as 200 mg/5 mL.

100 mg/x mL = ()5 mL 200 mg/5 mL

5 mL = 2.5 mL/dose &nbsp2 First, convert the client's weight to kg: 2.2 lb = 1 kg, so 22 lb = 10 kg.

24 hr/day = 3 doses/day 8 hr/dose

30 mg/kg/day = 10 mg/kg/dose3 doses

10 mg x 10 kg = 100 mg every 8 hr

The desired dose is 100 mg. The medication is available as 200 mg/5 mL.

100 mg/x mL = ()5 mL200 mg/5 mL

5 mL = 2.5 mL/dose&nbsp228

Which of the following approaches is the most accurate way to measure the heart rate of a 10monthold infant?

The apical heart rate is auscultated and is the most accurate measurement for aninfant.The radial pulse is not palpable in an infant.The ulnar vein is deep in the arm and is notpalpable.The brachial pulse is palpable in an infant. It provides a quick check of circulationstatus, but it is not the most accurate approach. Apical Radial Ulna Brachial 29

A nurse is caring for a 4-year-old child diagnosed with leukemia who is admitted with myelosuppression. Which of the following actions should the nurse take?

High carbohydrates will not provide the proper nutrients for protection against infection. The child needs a diet high in protein for defense against infection.Chemotherapy can causeinjury to mucous cells lining the rectal area, making this area prone to ulceration and tears. Rectal temperatures should be avoided to prevent trauma to this fragile tissue.Lemon andglycerine swabs are abrasive and can irritate tissue.A break in the skin is a potential source of infection. Provide a diet high in carbohydrates. Monitor rectal temperature every 4 hr. Use lemon or glycerine swabs for oral care. Inspect the skin daily for lesions. 30

The parents of a 4-year-old child state that they had an infant die 2 months ago duringchildbirth. They are concerned about their 4-year-old child's response to the infant's death. Which of the following statements by the parents indicates an expected response about deathfrom the 4-year-old child?

This expresses a more adult understanding of death. Preschoolers tend to think that thesibling is still alive.This kind of question would be appropriate for a school-age child.Theseresponses are most characteristic of adolescents who have the most difficulty coping with

death.Young children often feel guilty and responsible for a sibling's death, or may view illness or injury as a punishment for their thoughts about the sibling. "Our child wants to go to the cemetery to be with his sister." "Our child asks many questions about what happened to the baby's body." "Our child is not sleeping, eating, or playing lately and we are worried." "Our child blames himself for the baby's death because he said he didn't want a baby brother or sister."

31

A nurse is preparing a room for the admission of a client with sickle cell anemia who is in vasoocclusive crisis. Which type of equipment should the nurse place in the client's room?

Wheelchairs are used to decrease energy expenditures; however, this is not the most important equipment needed at this time.It would be nice to have these available in the room, but these are not priority measures at this time.It would be nice to have these available in the room, but these are not priority measures.Exchange transfusion is an important part of the treatment for vaso-occlusive crisis. One of the main objectives when managing a sickle cell crisis is blood replacement to treat anemia and hydration to reduce the viscosity of the sickled blood.

Wheelchair with adjustable leg rests A radio and age-appropriate reading materials Extra blankets and pillows Blood transfusion equipment 32

An 8-year-old child is admitted to a pediatric unit with a fractured femur and is placed in skeletal traction. Which of the following nursing interventions is the most appropriate?

Weights should hang freely to promote the forward force of traction.High-fat meals are not recommended. When a client is immobile, a high-fiber diet may be recommended to keep stools soft and prevent complications.The pulses on the side that are in traction are compared to the pulses on the contralateral side to assure that circulation in the affected side is not compromised. Color of the skin and nailbeds can also be assessed to observe for any neurovascular changes.The child's position should be changed at least every 2 hr to relieve frictional pressure on the bed and minimize skin breakdown. Passive, active, or activewithresistance exercises of the uninvolved extremities and joints should be performed to maintain strength and range of motion. Dependent upon the type of traction, varying degrees of position changes can be made without interfering with the traction. Position the weights securely against the foot of the bed. Provide small, frequent, high-fat meals to the child.

Compare pulses on affected site to contralateral side. Provide diversional activities to minimize the child's movement. 33

A nurse on a pediatric unit is assigned to care for a child with Reye syndrome. Which of the following is the most serious clinical manifestations for which the nurse should monitor?

Reye syndrome results in children secondary to a mitochondrial insult sustained following a viral infection and the administration of aspirin. It is not associated with an anaphylactic reaction.Cerebral edema with increased intracranial pressure presents the most

significant threat to life. Vital functions and neurologic status need to be monitored by the nurse.Secondary to related liver dysfunction, coagulation impairment with prolonged bleedingtime is a sign of Reye syndrome. This complication is not the most serious clinical manifestationof Reye syndrome.The possibility of hypovolemic shock is a constant threat in children withcontrolled fluid intake and osmotic diuresis. Monitoring of intake and output is necessary foradjusting fluid volumes to prevent dehydration. Anaphylaxis Cerebral edema Impaired coagulation Hypervolemia34

A nurse is preparing to take a rectal temperature on a 7-month-old infant. Which of the following should the nurse keep in mind when preparing to take the temperature?

A rectal thermometer should be well-lubricated prior to insertion, but the maximum depth of tip insertion is 2.5 cm into the rectum, not inches.Rectal temperatures are approximately 1 Chigher than an oral temperature. To convert from Fahrenheit to Celsius: (5/9)(x-32)= C. For example, a rectal temperature of 99.6 F is converted to Celsius by performing (5/9) (99.6-32)=(5/9)(67.6)=338/9=37.5 C. Next, the oral temperature of 97.7 F is converted toCelsius by performing (5/9)(97.7-32)=(5/9)(65.7)=328.5/9=36.5 C. The rectal temperature of 37.5 C is 1 higher than the oral temperature of 36.5 C. Therefore, a rectal temperature of 99.6 F is equivalent to an oral temperature of 97.7 F.Rectal temperatures are more accurate, but infants should have their temperature taken by axillary or tympanic membrane because ofthe danger of damaging rectal mucosa. Rectal temperatures should only be obtained on infantsand children when no other route can be used.Rectal thermometers should be left in place for 4min, but the use of mercury thermometers is losing favor. Mercury thermometers should not be

used because if broken, inhaled vapors from the mercury can cause significant toxicity. A well-lubricated thermometer tip should be inserted a maximum of 2.5 in into the rectum. A rectal temperature of 99.6 F is equal to an oral temperature of 97.7 F. Infants should have temperatures taken rectally for accuracy andthermoregulation. Mercury thermometers are the thermometers of choice to obtain the rectaltemperature, holding it in place for 4 min. 35

A nurse is talking to the parents of a 3-year-old child about water safety precautions. Which of the following statements made by the parents indicates a need for clarification?

Toddlers are curious and can fall headfirst and be trapped.Toddlers can drown in bathtubs while being left unattended.Toddlers can drown in unusual places due to their curiousness and fascination with water. All standing amounts of water should be drained.Teaching swimming and water safety are beneficial, but insufficient for protection against drowning. When the child is near the swimming pool, the parent needs to be supervisingthe toddler. Statistics show that 90% of all drownings occur in the swimming pool. Water is fascinating to children and drowning can occur quickly. "We keep the toilet seat down at all times." "We don't answer the phone during bath time." "We empty all buckets filled with water." "We have our child in swimming lessons."

36

A nurse is caring for a child with Wilms' tumor. The parents ask why the sign "Do not palpate theabdomen" has to be placed on their child's bed. Which of the following is the correct responseby the nurse?

This is not the reason the abdomen should not be palpated. Wilms' tumor ischaracteristically nontender.Palpation will not cause the tumor to grow. Tumors grow from multiplication of cells, not manipulation of the mass.Palpation of the abdomen is avoidedbecause manipulation of the mass may cause cancer cells to spread to other sites.The tumor ison the kidney, not the bladder. Palpation will not cause urinary leakage.

"Any manipulation of the abdomen can result in pain for your child." "Palpation of the abdomen could cause the tumor to grow." "Palpation of the abdomen could result in some of the tumor cells breaking loose, causing it to spread." "Any manipulation of the abdomen will put pressure on the bladder and causeurine to leak." 37

A nurse is caring for a child with muscular dystrophy. Which of the following priority actions should the nurse include in the care of this child?

Maintaining function of muscles is the goal of treatment for muscular dystrophy.

Stretching, range-of-motion exercises, and strength and muscle training should be performed tohelp maintain function. Children who remain active can delay the eventual confinement in awheelchair for a great length of time.Duchenne muscular dystrophy occurs from mother-tosontransmission of the defective gene. It is inherited from an X-linked trait. Therefore, genetic counseling is an important aspect of supportive family care and it is recommended for theparents, female siblings, maternal aunts, and their female offspring. This is not the priority nursing action.Flu and pneumococcal vaccines are encouraged as well as the avoidance ofpersons with respiratory infections because children with muscular dystrophy are at anincreased risk for respiratory infections.Incentive spirometer use and breathing exercises shouldbe performed daily to increase and maintain vital lung capacity. Limit physical activity and plan frequent rest periods to avoid overexertion andexhaustion of muscle groups. Recommend genetic counseling for parents, male siblings, and paternal unclesand their male offspring. Advise against flu and pneumococcal vaccines due to a compromised respiratory system. Have the child use an incentive spirometer and perform breathing exercisesroutinely. 38

A nurse is caring for a child with acute glomerulonephritis. The child has edema, hypertension, and gross hematuria. Which of the following is the most appropriate nursing intervention?

Children with glomerulonephritis require frequent monitoring of vital signs, but oxygensaturation is not necessary.For children with hypertension and edema, moderate sodium and fluid restrictions may be instituted. Foods high in potassium are restricted during oliguricperiods.Due to the edema present in the disease process, the child is weighed and fluid

balanceis monitored daily to check the fluid balance.This is not the first priority in the child's care. Most

children recover completely. However, health supervision following hospitalization should be continued weekly and then monthly for evaluation and urinalysis. Monitor the oxygen saturation every 4 hr. Teach the parents dietary restrictions regarding protein. Weigh the child daily and record intake and output. Counsel the parents about the need for follow-up. 39

A nurse is performing a routine physical examination on an adolescent client who asks, "Why do I have to use a condom if my girlfriend is on the pill? I thought the pill was enough protection against pregnancy." Which of the following is the most appropriate response by the nurse?

Using two forms of birth control may be effective against pregnancy, but this response does not explain why one form must be a condom.Having both partners share responsibility for birth control is a positive situation, but this is not the reason the client should use a condom along with birth control pills.When used correctly, contraceptives are as effective in adolescents as in adults.Condoms are the only birth control method that protect against sexually transmitted diseases. "You need to use two forms of birth control so if one fails you have a second form of protection against pregnancy." "Using a condom allows you to share the responsibility for birth control." "Oral contraceptives are less than 99 percent effective in adolescents. Therefore,

a second form of contraception is needed." "Oral contraceptives are highly effective in preventing pregnancy but do not prevent sexually transmitted diseases." 40

A nurse is preparing to admit a 15-year-old client with HIV/AIDS. Based on the client's diagnosis, which of the following nursing actions is appropriate?

HIV/AIDS is transmitted through blood and body fluids. The precautions necessary for blood and body fluid transmission are standard precautions. The client and the client's family should be educated regarding the transmission of infectious disease. Basic information about standard precautions should be presented in a manner that is age-appropriate and considers educational levels for the client and the client's family.The combination of hot water and detergents used in hospital dishwashers is sufficient to decontaminate dishes, glasses and cups, and eating utensils. Disposable dishes are not necessary.Airborne precautions require a negative pressure room. Tuberculosis, not HIV, is a disease that would require this precaution.Visitors do not need to wear either a gown or mask. A mask and gown are required during procedures and client care activities that are likely to generate splashes or sprays of blood, body fluids, secretions, or excretions to provide protection from contact transmission of pathogenic organisms. Contribute to planning client education on standard precautions in ageappropriate manner.

Contact the dietary department to request foods be delivered on disposable dishes. Prepare for infection control in a negative pressure room for this client. Instruct visitors to wear gowns and masks when entering the client's room.

41

A nurse is reinforcing home care instructions with the parents of a 5-year-old child who has acute bronchitis. In order to prevent the transmission of the virus, which of the following shouldthe nurse include in the instructions?

Acute bronchitis is generally caused by a virus. Transmission is via direct contact; therefore, isolation is not required.Careful handwashing is important when caring for childrenwith respiratory infections. They should be taught to use a tissue to cover their nose and mouth when they cough or sneeze and to wash their hands. Bronchitis is transmitted via articlescontaminated with nasopharyngeal secretions.The virus will not live if dishes are washedproperly. The combination of hot water and detergent is sufficient to decontaminate dishes, glasses, cups, and eating utensils.Clients wear masks when they are immunocompromised anda health care professional is trying to prevent the client from acquiring a secondary infection. Aclient with bronchitis is not considered immunocompromised. Isolate the child in a bedroom separated from the rest of the family. Teach the child to wash his hands after coughing secretions into a tissue. Serve food to the child on disposable dishes with plastic utensils. Have the child wear a mask whenever leaving the bedroom. 42

A nurse is monitoring a 6-month-old infant who is diagnosed with pneumonia. The nurse observes an absence of respirations and peripheral cyanosis. After determining unresponsiveness, which of the following is the next nursing action?

This should be done after establishing an open airway.This is not the next nursing action.Following cardiopulmonary resuscitation (CPR) techniques, the nurse should determineunresponsiveness and reposition the infant to dislodge an obstruction or open the airway.For infants, help is called after 1 min of CPR. Look, listen, and feel for normal breathing. Give two rescue breaths. Position the infant to open the airway. Immediately call for assistance. 43

Which of the following emotional manifestations demonstrates an improvement in a 7-montholdinfant diagnosed with nonorganic failure to thrive?

The absence of separation anxiety and fear of strangers is a clinical manifestation offailure to thrive. Most infants develop separation anxiety and fear of strangers from 6 to 8months of age.Radar scan of the environment with a wide-eyed gaze is a clinical manifestationof failure to thrive.Infants with nonorganic failure to thrive are detached and show less interest insocial interactions. Signs of nonorganic failure to thrive include the infant being passive, sleepy, and lethargic.Infants with nonorganic failure to thrive do not like to be held or touched, so thiswould be a sign of improvement. Infant has no fear of strangers. Infant scans environment with wide-eyed gaze. Infant is passive and sleeps well. Infant likes to be held and touched. 44

During an outpatient clinic visit a 13-year-old client is diagnosed with infectious mononucleosis. The nurse should expect which of the following to be included in the client's plan of care?

A simple nonnarcotic analgesic is usually sufficient to relieve the headache, fever, and malaise of mononucleosis.Gargling alleviates the pain from sore throat. Warm water is soothing to the inflamed throat and rinses the pharynx of secretions.Sometimes a short course ofpenicillin is prescribed for sore throat, but ampicillin is contraindicated because it frequentlytriggers a maculopapular rash.The child and family should be advised to limit exposure topersons outside of the family, especially during the acute phase to prevent secondary infection. Take acetaminophen (Tylenol) with codeine as prescribed for pain. Encourage gargling with warm water to alleviate pain. Start a short course of ampicillin. Encourage social activity to prevent depression. 45

A nurse is reinforcing teaching with the parents of an infant diagnosed with recurrent otitis media. Which of the following is appropriate teaching to include?

The use of steroids, decongestants, and antihistamines to treat acute otitis media is notrecommended.The upright position prevents formula from draining into the middle ear throughthe eustachian tube.Equalizing tubes are used to treat otitis media, not tonsillectomy or anadenoidectomy. These have not been found to be effective treatments.An ice compress placed over the affected ear may provide comfort and reduce edema and pressure. "Give the child an over-the-counter antihistamine when the symptoms begin." "Hold the child in an upright position while feeding." "Talk with the primary health care provider about performing a tonsillectomy." "Apply a warm compress over the affected ear to provide comfort."

46

A nurse is caring for a child who has hemophilia. The nurse should expect abnormal results in which of the following diagnostic tests?

Tests that measure serum fibrinogen level are all normal in persons with hemophilia.Hemoglobin serves as the vehicle for transportation of oxygen and carbon dioxide. Itis not a test that relates to the cause of hemophilia.PT measures prothrombin activity and bypasses the intrinsic-extrinsic mechanism. It detects deficiencies in factor V, VII, X, and fibrinogen as well as prothrombin.This laboratory test reveals that a client with hemophilia has aprolonged PTT. PTT measures the activity of thromboplastin, which depends on intrinsic clottingfactors. Factor VIII and IX are needed for the formation of thromboplastin, and it is factor VIII orIX that is deficient in hemophilia or hemophilia B respectively. Fibrinogen Hemoglobin level Prothrombin time (PT) Partial thromboplastin time (PTT) 47

A nurse is caring for a 4-year-old child who is prescribed an intravenous medication preoperatively. Which of the following therapeutic play techniques is most appropriate when reinforcing the teaching for this procedure?

Role playing is best when it involves the child and enables the child to handleequipment. This option has the nurses role playing with passive involvement of the child.Stories can be helpful to introduce the topic; however, this does not diminish the anxiety of seeing the equipment for the first time.Movies may scare a preschool child, especially if the child in the

movie cries during the procedure.Allowing the child to see, hold, and collect the suppliesfamiliarizes the child with the frightening aspects of the procedure. Instruction can be based onthe child's questions in a nonthreatening environment. The child can gain an understanding of the procedure by pretending to start an IV on a doll. Role play with another nurse the technique of IV placement and how themedication is infused. Read a story that explains the basics of how IVs are placed. Watch a movie narrated by nurses and children about IV placement. Explain the basic procedure and give the child IV supplies to play with, minus theneedle. 48 A nurse is monitoring a child whose parents are suspected of child neglect. Which of the following is an expected finding of neglect?

Physical neglect involves the deprivation of necessities such as clothing, food, shelter, supervision, medical care, and education.Lack of parental education is not correlated withphysical neglect.Socioeconomic group is not a factor in child neglect.If the child is clean, fadedclothing with large shoes may be a sign of financial difficulties and not a sign of physical neglect. Lack of required immunizations Parental lack of education Lower socioeconomic group Faded clothing with large shoes49

A nurse is initiating a plan of care for a toddler who is hospitalized. Which of the following instructions is important to communicate to the nursing assistant?

Toddlers are able to undress themselves, but do not have the fine motor development skills required for dressing.Allowing the child to feed himself provides opportunities forautonomy and motor skill development.Toddlers view everything in relation to self only and are involved in parallel play.One way of dealing with negativism is to decrease opportunities for "no" answers. Have the toddler dress himself. Offer the toddler finger foods for snacks. Provide opportunities to share toys with others. Ask the child simple yes or no questions. 50

A nurse is caring for a 3-year-old child with strabismus. Which of the following actions should the nurse advise the parents to implement to help prevent amblyopia?

Biconcave lenses are used to correct myopia.While trauma should be avoided to preventeye damage, this is not an implementation to prevent amblyopia (impairment of vision orblindness) from strabismus.Strabismus, or cross eye, is when one eye deviates from the point offixation. If the misalignment is constant, the weak eye becomes lazy and the brain eventuallysuppresses the image. If not corrected by the age of 4 to 6 years, blindness from disuse oramblyopia may result. Treatment includes covering the strong eye to strengthen the muscles in the weak eye.Dry eyes are not a manifestation of strabismus. Wear corrective biconcave lenses.

Prevent trauma to the eyes. Patch the strong eye.

Instill artificial tears. 51

A nurse is caring for an infant with a history of vomiting due to gastroenteritis. Which of the following nursing interventions is considered the priority?

Maintaining the infant's airway is of the highest priority. A child who is vomiting should be positioned on the side or in a semi-reclining position to prevent aspiration.Administration offluids and electrolytes is important to prevent or correct dehydration and electrolyte imbalances, but is not of the highest priority.Antiemetic medications are administered as prescribed if necessary. This is not the first priority.Of major importance is avoiding ketosis. A dietary intake high in carbohydrates spares the body protein and avoids ketosis which can result fromexhaustion of the glycogen stores. This is not the highest priority. Place the infant in a side or semi-reclined position. Administer oral rehydration and electrolyte therapy. Administer antiemetic medications as prescribed. Maintain a high-carbohydrate intake to prevent ketosis. 52

A nurse has reinforced teaching to the parent of a 9-month-old infant who has redness in the diaper area and inner thighs. Which of the following statements by the parent indicates a correctunderstanding of this teaching?

Hair dryers can burn the skin and thus are not recommended.Powder should not beused. It tends to cake when the skin is wet, and there is also the danger of inhalation. Applying askin cream barrier is much more effective.Rubber pants should not be used because they do not allow air to circulate. Thus, they promote skin irritation and breakdown.Exposing the skin allows it to air dry completely which helps prevent breakdown. "I can use a hair dryer on the reddened skin to help with the drying." "I can use powder after diaper changes to absorb excess moisture." "I can use cloth diapers with rubber outer pants until the rash clears." "I can keep the diaper off to expose the skin to air." 53

A 6-year-old child is brought to the emergency department after falling down the outdoor steps. The parent's account of the incident appears different than the neighbor's account of the incident. Upon questioning the child, the nurse should recognize which of the following as ausual pattern of behavior exhibited by an abused child?

The child would answer questions but would not contradict the parent's story.The typical reaction of the child is to repeat the same story as the parent. Children rarely betray the parenteven when the parent is abusive. The child will even defend the parent.While children have creative imaginations, they do not make up stories in this scenario. Stress of the situation andthe fear of losing what security they have with the parent keeps children who have been abusedclinging to the parent's story.Children are afraid of losing the parent, so they do not implicate the parent in the abusive behavior.

The child refuses to answer questions. The child repeats the same story as the parent. The child will fabricate an obviously false story. The child tells what really happened at the time.

54

A nurse is providing care for a 2-day-old neonate with a cleft lip and palate. The nurse evaluatesthe parents' understanding of correct feeding methods. Which of the following observationsindicates a need for further teaching?

Large, soft nipples reach back further in the mouth. The large holes make sucking less of an effort, and thus these types of devices appear to work well for nipple feedings after a cleft lip and palate repair.When breastfeeding, the nipple is positioned back in the oral cavity so that the action of the tongue can make the expression of milk easier.When an infant has trouble with a nipple feeding, a rubber-tipped eye dropper makes feeding easier. The rubber tip extends the length of the feeding device and has a larger hole so the infant doesn't have to create as muchsuction.Infants with a cleft lip and palate have difficulty creating the suction required to get theformula from the bottle. A "gravity flow" nipple with a squeezable bottle allows the formula to be deposited directly into the mouth. Uses a long, soft nipple with a cross-cut opening attached to a bottle When breastfeeding, positions the nipple toward the front of the mouth Uses an eyedropper with a piece of rubber tubing on the tip Obtains a "gravity flow" nipple and attaches it to a squeezable plastic bottle55

A nurse is holding an infant during a lumbar puncture for a suspicion of meningitis. The infant is in a sitting position with the buttocks at the edge of the table and the neck flexed, and the nurseis immobilizing the infant's arms and legs. Which assessment takes priority during theprocedure?

This is important to note anytime a spinal tap is done, but it is not the priorityassessment.This is important to note anytime a spinal tap is done, but it is not the priorityassessment.Based on the child's position, the nurse should be concerned about limiting chestexpansion and movement of the diaphragm. Based on the bending of the neck, the nurseshould be concerned about the infant's soft, pliable trachea, which may collapse.This isimportant to note anytime a spinal tap is done, but it is not the priority assessment. Circulation checks of the lower extremities Heart rate and crying pattern Chest expansion and diaphragm excursion Clarity of spinal fluid and level of consciousness

56

A nurse is reinforcing teaching given to the parent of a 1-year-old child who has had a high temperature, vomiting, and diarrhea for 48 hr. The child has sunken eyes and cracked lips. Which of the following should the nurse tell the parent?

This is contraindicated because this diet has little nutritional value (it is low in energy andprotein) and is high in carbohydrates and low in electrolytes.These soups contain excessivesodium and inadequate carbohydrates.Infants and children with acute diarrhea and dehydrationshould be treated first with oral rehydration solutions. These help the child replace the sodium and water that are being lost through the vomiting.Water has no electrolytes and will not help compensate for the diarrhea and dehydration.

"Give the infant applesauce and rice cereal because these have been found tohave high nutritional value."

"Encourage the child to take sips of chicken or beef broth because they willreplace the fluid losses your child is experiencing." "Give the infant oral rehydration solutions that are available commercially. They replace some of the electrolytes lost through vomiting." "Give the child nothing by mouth for 4 hr. Once the vomiting has decreased you can introduce sips of clear water." 57

A nurse is caring for a 4-year-old client with full-thickness burns. Which of the following nursing actions are essential for the care of this child? (Select all that apply.)

Monitor level of consciousness is correct. Symptoms of confusion or seizures can result fromalterations in the electrolyte balance. Disorientation is one of the first signs of sepsis or mayindicate inadequate hydration.

Maintain intravenous fluids is correct. Fluid shifts that occur after a burn injury make intravenousfluids very important. Intravenous fluid therapy compensates for loss of water and sodium, reestablishes electrolyte balance, and corrects acidosis. IV therapy restores circulating volume, provides sufficient perfusion, and improves renal function.

Document vital signs is correct. Management of pulmonary and cardiovascular status is apriority, especially in the acute phase of burn injury treatment. The respiratory system is

monitored for burn involvement and if suspected or evident, then 100% oxygen is administered. An endotracheal tube may need to be inserted to maintain the airway. Blood gas values including carbon monoxide levels are obtained. Heart rate helps to determine the adequacy offluid resuscitation.

Provide a low-calorie, high carbohydrate diet is incorrect. Clients who have suffered burns should have a high-protein, high-calorie diet. This helps to avoid protein breakdown as the body's metabolism increases after a burn injury.

Monitor urinary output is correct. Urinary output helps to determine the adequacy of fluidresuscitation. Urine output and specific gravity help to establish adequate hydration and guidethe rate of fluid administration.

Administer morphine subcutaneously for pain is incorrect. Morphine sulfate is the preferredmedication for severe burn injuries. It is administered continuously by IV infusion. The unstable circulatory status, edema, and tissue damage make intramuscular and subcutaneous injectionscontraindicated in burn injuries.

Monitor level of consciousness is correct. Symptoms of confusion or seizures can result fromalterations in the electrolyte balance. Disorientation is one of the first signs of sepsis or mayindicate inadequate hydration.

Maintain intravenous fluids is correct. Fluid shifts that occur after a burn injury make intravenousfluids very important. Intravenous fluid therapy compensates for loss of water and sodium, reestablishes electrolyte balance, and corrects acidosis. IV therapy restores circulating volume, provides sufficient perfusion, and improves renal function.

Document vital signs is correct. Management of pulmonary and cardiovascular status is a priority, especially in the acute phase of burn injury treatment. The respiratory system is monitored for burn involvement and if suspected or evident, then 100% oxygen is administered. An endotracheal tube may need to be inserted to maintain the airway. Blood gas values including carbon monoxide levels are obtained. Heart rate helps to determine the adequacy of fluid resuscitation.

Provide a low-calorie, high carbohydrate diet is incorrect. Clients who have suffered burns should have a high-protein, high-calorie diet. This helps to avoid protein breakdown as the body's metabolism increases after a burn injury.

Monitor urinary output is correct. Urinary output helps to determine the adequacy of fluid resuscitation. Urine output and specific gravity help to establish adequate hydration and guide the rate of fluid administration.

Administer morphine subcutaneously for pain is incorrect. Morphine sulfate is the preferred medication for severe burn injuries. It is administered continuously by IV infusion. The unstable circulatory status, edema, and tissue damage make intramuscular and subcutaneous injections contraindicated in burn injuries. Monitor level of consciousness. Maintain intravenous fluids. Document vital signs.

Provide a low-calorie, high-carbohydrate diet. Monitor urinary output. Administer morphine subcutaneously for pain. 58

A nurse is caring for a 7-month-old infant with acute bronchiolitis. The infant has a persistent, dry, hacking cough that worsens at night, tachypnea, and weakness. Which of the following actions should the nurse implement?

Cough suppressants may be useful to allow rest but can interfere with clearance of secretions. They have not proven to be of benefit for this condition.Bronchiolitis is caused by a virus and is transmitted via direct contact. Therefore, contact precautions are required rather than droplet precautions.Antibodies and corticosteroids are not effective in uncomplicated bronchiolitis.Fluids by mouth may be contraindicated to prevent aspiration if the child has tachypnea, weakness, and fatigue. Therefore, IV fluids are preferred to maintain hydration and dilute secretions. Administer prescribed cough suppressants as needed. Place the child on droplet precautions. Administer antibiotics and corticosteroids as prescribed. Provide intravenous fluids as prescribed. 59

A nurse is caring for an infant with hypospadias. Which of the following is an expected finding?

This defect describes epispadias, a condition in which the meatal/urethral opening is on the dorsal/back surface of the penis.With hypospadias, the urethral opening can be anywhere on the underside/ventral surface of the penile shaft or the perineum.Fluid in the scrotal sac is

referred to as hydrocele.Testes that are not palpable within the scrotal sac are an indication of cryptorchidism. This is a failure of one or both testes to descend through the inguinal canal. The meatal opening is on the dorsal surface of the penis. The urethral opening is on the underside of the penis. Fluid is present in the scrotal sac containing the testes. The testes are not palpable within the scrotal sac. 60

A nurse is caring for a 3-year-old child who is diagnosed with a urinary tract infection (UTI). The parent is concerned about recognizing the signs and symptoms of future UTIs. Which of the following statements made by the parent indicates a correct understanding of the manifestations of a UTI?

A child who has frequent urination and exhibits strong-smelling urine should be evaluated for a UTI.These are signs of glomerulonephritis, not UTI.These symptoms are seen in acute renal failure and are not signs of a UTI.Hematuria, not abdominal pain, is a sign of a UTI. "I should look for more frequent urination and strong-smelling urine." "My child would have tea-colored urine and puffiness around the eyes." "I should observe for episodes of nausea and less frequent urination." "My child would have pale-colored urine and abdominal tenderness and pain."

ATI.CHILD CARE 1.0

A child diagnosed with asthma begins corticosteroid treatments. The nurse explains to the parents that the purpose of corticosteroid treatment is to produce which therapeutic effect?

Incorrect: Dilation of the bronchial airways is common in treating asthma. Albuterol is a common medication.Incorrect: Bronchospasms are usually reduced by B-2 agonists andbronchodilators.Incorrect: Infections are treated by antibiotics but not indicated in the treatmentof asthma unless lung congestion is noted.Correct: Corticosteroid usage is common fordecreasing inflammation of the bronchial airways. Dilation of bronchial airways Decrease bronchospasms Prevention of infection Anti-inflammatory effect 2

Which is the recommended treatment for moderate to severe lead poisoning?

Incorrect: IV fluids are typically not used in the treatment of lead poisoning. IV fluids area conservative treatment regimen and are not indicated for treatment of lead poisoning; a moreradical therapy is needed to remove the lead from the body.Incorrect: Treatment with antiemetics is not effective in the treatment of lead toxicity because the heavy metal is absorbedinto the body. Lead ingestion usually occurs more than one time.Correct: The heavy metal antagonist, edetate calcium disodium, is frequently the drug of choice for the removal of the leadtoxin from the body. Chelating agents inactivate the toxicity of the lead and cause excretion through the urine. Others drugs may treat the symptoms of toxicity rather than remove the leadfrom the body. Untreated lead toxicity can lead to a wide array of neurobehavioral problems include: attention deficit-hyperactivity disorder, reduced cognitive performance, irritability or lethargy, aggressiveness, and hearing impairment. The most serious and irreversible side effect

of lead poisoning is encephalopathy, which is associated with lead levels > 100 mg/ dL.Incorrect: Antibiotics have no effect on the removal of the toxin. IV fluids Antiemetics Heavy metal antagonist Antibiotics 3

Which treatment is a nursing priority when providing care for an infant diagnosed with bacterialmeningitis?

Incorrect: Cardiorespiratory monitoring is standard for care of the child with bacterialmeningitis as a means of establishing the baseline parameters for vital signs. The infant with meningitis may have a low baseline heart rate, tachypnea or fever. This however, is not the priority nursing intervention.Incorrect: The initiation of IV fluids for hydration and nutrition is a primary concern for the care of the infant with bacterial meningitis. However, the most important intervention is starting antibiotic therapy.Incorrect: Meningococcal meningitis is the only type of meningitis spread through air-born droplets and therefore, respiratory precautions need to beinitiated as soon as possible. Respiratory isolation is important for the control of transmission ofthe disease after the child receives the first doses of antibiotics.Correct: The first nursing priorityis the implementation of antibiotic therapy, which prohibits the microbial damage to the neurologic system through the cerebral spinal fluid. Bacterial meningitis has a high rate of infantmorbidity (illness) or mortality (death). Immediate treatment with antibiotics can prevent: death, deafness, reduced cognitive ability, attention deficit-hyperactive disorder, seizures and various other complications.

Initiate cardiorespiratory monitoring. Initiate intravenous fluids. Observe respiratory isolation. Administer antibiotic therapy. 4

The dosage of a pediatric medication is 120mg/kg/day to be give t.i.d. The patient weighs 12 pounds. What is the correct dose for the nurse to administer?

Incorrect: The dose of 120 mg is half the indicated dose. The erred dosage represents a failure to divide the total daily dose by the number of individual dosages required per day. The failure to use the weight in the calculation is evident.Incorrect: The dosage of 480 mg is an excessive dose for the child. The calculation error is likely a failure to convert pounds to kilograms.Correct: The patient weighs twelve pounds. This weight converts to kilograms by dividing 12 by 2.2 (1 kg. = 2.2 lb.). In this example, the child's weight converts to 5.4 kg. The daily dose of 120 mg is given t.i.d: each individual dose is 40 mg/kg. Then multiply the weight in kilograms by the individual dose (40mg). The individual dose is 218 mg.Incorrect: The dose of 650 mg is too large of a dose. The weight of the child when converting from pounds to kilogramsis 5.45 kg. The dose is ordered to be given t.i.d.. Therefore, the daily dose of 120 mg/kg/day is divided by 3 to yield an individual dose of 40 mg/kg/dose. The error is this dosage was likely a failure to divide the total daily dose by the number of doses required per day. 120 mg

480 mg 218 mg 651 mg5

In a child diagnosed with Tetralogy of Fallot, which of the following is a compensatory mechanism to decrease venous return to the heart?

Correct: Squatting is a compensatory mechanism that decreases venous return(deoxygenated blood) to the heart. The clinical sign is commonly seen in young children with Tetrology of Fallot (a type of cyanotic heart disease). The signs associated with cyanotic heart disease include hypoxia, poor growth, low tolerance for physical exertion, cardiomegaly, murmurand acute, intermittent blue spells that occur after crying or feeding (tet spells).Incorrect: Clubbing is found in children with chronic respiratory disease and cyanotic heart disease. However, this finding is rare in young children.Incorrect: Shortness of breath, retractions and increased respiratory effort occur with lung dysfunction. Generally, the child with impaired oxygenation due to a cardiac lesion does not exhibit signs of respiratory distress.Incorrect: Polycythemia is common in children with hypoxia due to respiratory or cardiac dysfunction. This compensatory mechanism increases the oxygen-.carrying capacity in the body. The effect is not related to the venous return of unoxygenated blood to the heart. Squatting Clubbing Shortness of breath Polycythemia

A 1-year-old receives routine health maintenance care at the pediatric clinic. The child receives an MMR immunization. The mother asks the nurse, "When will my child get the next dose of

MMR vaccine?" Which is the correct response by the nurse?

Incorrect: The DPT vaccine is routinely given in six months.Incorrect: An additional dose of MMR vaccine is needed in the middle school years to maintain full immunity from thediseases.Incorrect: The first dose of Hepatitis B vaccine (HBV) is given in the hospital prior to discharge home. A follow-up HBV is given in 1-2 months and followed up in 6-12 months following the second does. The schedule does not coordinate with the routine immunization schedule for MMR.Correct: A second MMR, often called a booster, will be needed when the child enters middle school at age eleven or twelve years of age. In six months with the next DPT No further vaccination needed With the Hepatitis B series After the child is 10 years of age7

Which is a major difference in the clinical manifestation of adolescents with anorexia nervosa compared to bulimia?

Incorrect: Binge eating is a common manifestation of both disorders.Incorrect: Purgingcan be associated with both disorders.Correct: The major difference between adolescents with anorexia nervosa and adolescents with bulimia is body image distortion. Clients with anorexiasee themselves as being overweight no matter how underweight they become. Clients withbulimia see their weight realistically but have psychological problems that manifest in an eatingdisorder.Incorrect: Decreased self-esteem is often a catalyst of both disorders.

Binge eating Purging

Body image distortion Decreased self esteem 8

Which is the most common factor associated with non-organic failure to thrive?

Incorrect: A cool, drafty sleeping area is not a comfortable environment for sleep, but is unrelated as a cause of failure to thrive.Correct: The most significant factor associated with nonorganic failure to thrive is typically a disturbance in the mother/child relationship. A situation involving dysfunctional family relationships is often complex; characterized by marital discord, economic pressures, and parental immaturity with a low stress tolerance.Incorrect: Lack ofinterest in the surroundings is a symptom of failure to thrive but not an etiologic factor.Incorrect: The financial hardship related to the expense of infant formula is not usually a primary cause ofnon-organic failure to thrive. The Women's Infant and Children program (WIC) provides infantformula at a low or no cost with eligibility. Therefore, the disorder is not likely to be related to obtaining adequate nutrition but instead related to the ability of the infant to consume, digest, and utilize the nutritive source. A cool, drafty area for sleeping Disturbance of mother/child relationship Lack of interest in the surroundings Financial hardship causing poor nutritional care9

An adolescent recovering from substance abuse is diagnosed with hepatitis B. Which nursinginstruction should be included when planning the client's care?

Incorrect: During periods of acute hepatitis, the client needs plenty of rest.Correct: The mode of the transmission in hepatitis B is via the parenteral route through blood products, intravenous injection and maternal-fetal transplacental transmission. It is extremely important forthe nurse to educate the family about the relationship between intravenous exposure and thetransmission of the disease.Incorrect: Fluid intake is not limited in the treatment of hepatitisB.Incorrect: Eating a diet high in fat is contraindicated because of the liver involvement. Increase exercise. Avoid sharing needles. Limit fluid intake. Eat a diet high in fat. 10

Changes in the growth and development of the preschooler are characterized by:

Incorrect: Continued rapid physical growth is not common for this developmentallevel.Incorrect: Major changes in weight, height and head circumference have usually alreadytaken place prior to this developmental level.Correct: Physical growth slows in the preschoolyears. Preschoolers enjoy social contacts. Preschoolers are gaining control of their muscles andparticipate in vigorous activities with other children.Incorrect: Improvement in motor ability israpid at this developmental level as the children are gaining control of their muscles. Preschoolers' gait resembles that of an adult. Rapid physical growth and a persistent curiosity.

Major changes in weight, height and head circumference. A slowing of physical growth and expansion of social contacts. A slow improvement in motor ability.

11

A teacher asks the school nurse to assess the behavior of a child with attention deficithyperactivity disorder (ADHD). Which situation best facilitates an effective nurse/child interaction?

Incorrect: Playground time for the discussion is desirable because the child would notmiss classroom instruction. However, recess is an appropriate time for the child to expel energy. The playground offers many opportunities for distraction and the child may have difficulty concentrating on the discussion in this environment.Incorrect: A discussion between the nurse and student in the classroom would be brief and impersonal due to the presence of otherstudents. At the end of the day, this environment is noisy, chaotic and rushed and may be one ofthe least desirable locations for a meaningful dialogue between the nurse and student. The child will most likely be distracted.Correct: A characteristic of attention deficit-hyperactivity disorder is distractibility and impulsivity. A quiet and calm environment is necessary to engage the child in afocused discussion.Incorrect: Although a physical education class provides the structured environment for the release of energy in an appropriate manner, this is not a place for the nurse to provide education to the child with ADHD. This setting also lacks the privacy necessary for a confidential exchange of information. Playground during recess In the classroom at the end of the day Nurse's office before school Physical education class12

The health care provider orders 60% oxygen to be administered with a partial rebreather maskand bag reservoir. Which error regarding the oxygen delivery system requires correction?

Incorrect: Moisture collecting in the mask is the result of humidification of the air. Oxygen can be very drying to the tissue and alveoli; damage can result to the airways withouthumidification.Incorrect: To optimize the delivery of oxygen via mask, a snug head strap is necessary.Incorrect: The mask covering the nose and mouth is a correct application of the mask.Correct: The reservoir bag on the non-rebreather mask should remain partially filled during inspiration to provide positive end expiratory pressure (PEEP). If the bag collapses theequipment may be faulty. Moisture collects inside the mask The strap around the head is snug The mask covers the nose and mouth The reservoir bag collapses during inspiration13

A toddler is admitted to the hospital for treatment of acute gastroenteritis and dehydration. The mother states that she must go home to make arrangements for the care of her other children. To reduce the child's separation anxiety, which nursing intervention is most appropriate?

Incorrect: Placing the child in the crib may make the child feel more alone and afraid. At

this developmental stage, the child is not likely to be comfortable enough with the surroundingsto adjust and begin to play.Correct: Anxiety is the child's predominant emotion with theseparation from a parent. Activities that calm and comfort the child are appropriate. Often a toddler will fall asleep in the nurse's arms due to the stress of a parent's leaving the child alonein the hospital.Incorrect: The video may serve as a distraction, but does not provide the security

of human contact.Incorrect: The nurse's station may allow an opportunity for social contact. Many toddlers may be overwhelmed or frightened by the activity at this central location. Place the child in the crib with toys.. Rock the child in a rocking chair.. Turn on an age-appropriate video.. Take the child to the nurse's station.. 14

Which technique is most appropriate when assessing the circulation of a child's leg in traction?

Incorrect: The movement of the toes is a neurological assessment and does not relate to circulation.Correct: The best way to assess circulation is to palpate the dorsalis pedal pulse located on the top of the foot. If a peripheral pulse is not palpable, a Doppler may be necessary to ascertain loss of circulation and pulse.Incorrect: Assessing pain sensation in the lower extremities is a neurological assessment and does not relate to circulation.Incorrect: Range of motion to the affected area is usually contraindicated while in traction. Determine if the child can wiggle the toes. Palpate the dorsalis pedis artery. Assess for pain sensation in the lower extremity. Perform range of motion in the lower extremity. 15

Initially, which solid food is generally recommended for an infant's diet?

Correct: Rice cereal is bland, easily digested and fortified with iron. Rice cereal is the first food introduced into the diet at approximately six months.Incorrect: Strained vegetables are introduced after the infant tolerates rice cereal. The order that various foods are initiated is controversial and dependent on regional, generational, cultural and personal factors.Incorrect: Strained fruits are introduced generally after the infant tolerates strained vegetables.Incorrect: Meats are introduced between 8-10 months of age. Infant meats are generally denser in texture and less preferred by many infants. The coordination of the muscles of the tongue and pharynx must be more developed for the introduction of solid meat. Infant rice cereal Strained vegetables Strained fruits Infant meats

16

A boy diagnosed with hemophilia falls while roller-blading and injures his knee. The nurse is most likely to assess which physical finding?

Correct: Hemophilia is a group of bleeding disorders in which there is a deficiency of one of the clotting factors. After a child sustains a traumatic injury to a joint, hemarthrosis is likely to result.Incorrect: Thrombocytopenia, is a decreased number of platelets in the circulating blood, and is not related to hemophilia.Incorrect: Petechiae are pinpoint non-raised, purplish spots on the skin, which are characteristic of low platelets.Incorrect: Neutropenia, which is the diminished number of neutrophils, is not associated with hemophilia. Hemarthrosis Thrombocytopenia Petechiae

Neutropenia17

A toddler is admitted to the emergency department following a febrile seizure. Which informationdoes the nurse provide to the family regarding febrile seizures?

Correct: There is little chance that future seizures will occur as a result of a febrile seizure. Febrile seizures are the body's reaction to an immature thermoregulation system. This reaction is not indicative of an epileptic disorder.Incorrect: There is no familial tendency noted in febrile seizures.Incorrect: Neurological defects are not commonly seen in children with febrileseizures.Incorrect: Children who experience a febrile seizure are not at risk of developing aseizure disorder. There is no relationship to seizure disorders. There must be a familial tendency toward seizures. The child most likely has a neurological defect or brain tumor. The child is likely to develop a seizure disorder as he grows. 18

The nurse assesses Koplik spots on the lingual and buccal mucosa of a 4-year-old. Whichdisease is likely to appear within the next two to three days?

Correct: Koplik spots are small red spots with bluish white centers on the lingual andbuccal mucosa. These spots are characteristic of an outbreak of measles. The measles rash

usually erupts a day or two after the appearance of the Koplik spots.Incorrect: Mumps are anacute viral disease characterized by the swelling of the parotid glands.Incorrect: Varicella virus (chickenpox) is a contagious viral disease characterized by vesicular eruptions on theskin.Incorrect: Pertussis is a highly contagious respiratory disease characterized by paroxysmalcoughing with dyspnea on inspiration. Rubeola (measles) Mumps (epidemic parotitis) Varicella (chickenpox) Pertussis (whooping cough) 19

Which is the most appropriate pain scale to use for a Spanish speaking 5-year-old child whocommunicates very little in English?

Incorrect: The Poker Chip Tool uses four red poker chips to indicate the degree of pain. The child must be able to count and have some concept of numbers for correct use of thetool.Incorrect: The Eland Color Tool directs the child to use colored markers to fill in an outline of the body with a color that describes their pain. This scale is used for children over four years of age.Correct: The FACES Pain Rating Scale is intended for use in the child ages 1-7 years. This scale uses a pictorial face that represents the child's level of pain. The smile face indicates that the child has no pain and the tearful face represents the highest level of pain. This scale is useful to the non-English speaking child because words are not necessary for reporting painlevel.Incorrect: The Word Graphic Rating Scale uses descriptive words to rate the intensity of

pain (no pain to worst possible pain) and aids the child in answering various questions about thenature and degree of pain. It would not be appropriate for a child who speaks little English andmay not yet read. Poker Chip Tool Eland Color Tool FACES Pain Rating Scale

Word Graphic Rating Scale 20

If a child with Type I diabetes mellitus takes regular insulin at 0800, which time during the day should the parents be taught to expect the peak action of regular insulin?

Incorrect: At 0830 the onset, not the peak, of the regular insulin should be occurring.Correct: Regular insulin peaks in two to four hours. The onset of the insulin begins in 30 minutes. A snack should be planned to avoid any symptoms of hypoglycemia.Incorrect: At 1300 the peak time of the insulin has already passed.Incorrect: At 1500 the peak effect of insulinis achieved. 0830 1100 1300 1500

21

The nurse evaluates the effectiveness of care for the school-aged child with juvenile rheumatoidarthritis (JRA). Which clinical outcome does the nurse expect the child to demonstrate afternursing care interventions are implemented?

Incorrect: JRA is treated with non-steroidal anti-inflammatory drugs (NSAIDs), such as ibuprofen, aspirin, and naproxen, to reduce inflammation to the joints. Slower-acting antirheumatic drugs may be added. Cytotoxic drugs are reserved for the child with severe, debilitating disease. Antibiotics are not used for JRA.Correct: The nurse observes the movements of the child and uses pain assessment tools to determine the intensity of pain. Nonpharmacologic modalities and anti-inflammatory and analgesic medication are provided topromote comfort and relieve pain associated with JRA. By modifying pain perception, jointmobility is likely to improve with reduced discomfort.Incorrect: Children are encouraged tomaximize their efforts for self-care and activities of daily living. Exercise enhances the mobility and strength of the supporting muscles, which is necessary in pain prevention. However, overexertion should be avoided.Incorrect: Children with JRA, like many with chronic illness ordisability, develop personality traits including: manipulativeness, hostility, and passive aggressiveness. Although it is the goal for care of the family to promote an understanding of the child's disease and altered lifestyle and have compassion for the situation, it is important thatfamily members do not enable the negative behaviors to persist. Efforts need to be made to intervene early and prevent permanent ineffective coping techniques. The symptoms will subside with use of antibiotics. The child is able to move with minimal or no discomfort. The child limits his own physical activity to prevent pain. The family copes with the child's manipulative behaviors. 22

The nurse providing care to the child with pediculosis capitus (head lice) educates the familyabout the condition, transmission, and treatment. Which condition is necessary for survival ofthe louse on the host?

Correct: Survival of the louse is dependent upon blood that is extracted from thehost.Incorrect: The louse feeds on the blood of the human host. Warmth is not necessary for survival.Incorrect: The louse feeds on the blood of the human host. Moisture is not necessary

for survival.Incorrect: The louse feeds on the blood of the human host. Mucous is not necessaryfor survival. Blood Warmth Moisture Mucous 23

An adolescent with Type I diabetes mellitus asks her mother for permission to go with friends to get pizza and ice cream. Which response by the adolescent's mother indicates that previousnursing instruction has been effective?

Incorrect: It is not necessary to reinforce to the adolescent that he/she is unable to eatsimilar to the friends. Peer groups and being accepted are very important at this age. The adolescent who can manage the insulin needs in accordance with the diet is allowed to have avariety of foods.Incorrect: It is not realistic or therapeutic to expect the friends to know thediabetic diet. This type of expectation may alienate the child from the peer group. The adolescent typically values the similarity to the peer group.Incorrect: By avoiding the situation, the adolescent does not directly deal with the underlying task of adapting the chronic illness intothe lifestyle. An important part of developing relationships among adolescents is spending leisure time together. The child alienates him or herself and suppresses feelings of anger towards self, others and/or the disease.Correct: The standard diabetic diet and appropriate nutritional education are flexible and incorporates many preferred foods at various times. The dose of insulin will need to be adjusted for this altered schedule, type and amount of food. Closeglucose monitoring is very important to the safety of the diabetic child whose metabolic needsare variable.

"It is important for you to spend time with friends but you can not eat what theyare eating." "Your friends need to learn that there are certain foods that you can not have. They will understand that you can't go with them." "You must stay away from those foods. It is easier for you to avoid the situation. You can go with your friends another time." "It is important for you to spend time with your friends. I will help you select yourfood and determine your next insulin dose." 24

When obtaining a health history, which significant event may precede a diagnosis of rheumatic fever?

Incorrect: Chickenpox is caused by varicella virus. Rheumatic fever is a complication ofgroup A beta hemolytic streptococcal pharyngitis.Correct: There is evidence that rheumatic feveris associated with group A beta hemolytic streptococci, which is a common cause of pharyngitis.Incorrect: The presence of a heart murmur is not reason enough to diagnose rheumatic fever. Rheumatic fever is associated with streptococcal infections.Incorrect: Vomiting and diarrhea are frequently caused by intestinal viruses, not bacteria. The primary symptoms of group A beta hemolytic streptococcal pharyngitis include: fever, malaise, dysphagia, lymphadenopathy and occasionally diarrhea in the young child. Rheumatic fever is acomplication of strep throat that can cause cardiac damage. Exposure to chickenpox Recent severe sore throat Presence of a heart murmur

Vomiting and diarrhea 25

Which intervention is most appropriate when providing nursing care for the child diagnosed withDuchenne's muscular dystrophy?

Incorrect: Limitation of physical activity may accelerate the process of musculardeterioration and atrophy.Incorrect: Increased weight gain becomes more likely as the activity level diminishes. The care of the child with a progressive, incapacitating disease becomes increasingly more demanding for the caregivers at home. As a loss of mobility and independence occurs, the child will require more lifting, dressing and physical care. Excessiveweight would aggravate the situation.Correct: The most important way for the nurse to impact the family of the child with Duchenne's muscular dystrophy is to assist the child and family incoping with the progressive, incapacitating and incurable disease. As muscular weakness progresses, wasting and contractures develop. A loss of ambulance occurs usually be 9-11 years of age. Difficult issues for the family to deal with pertain to loss of independence, mobilityand self-care and, eventually, death.Incorrect: No effective pharmacologic treatment exists for Duchenne's muscular dystrophy. Corticosteroid use has been reported beneficial in improving the muscle strength and size in some children. However, there is no conclusive evidence that steroid use has palliative or curative function. The goal for treatment is to maintain as much muscular function for as long as possible. Limit physical activity. Increase caloric intake. Assist the family to cope.

Administer steroids.

26

An adolescent comes to the clinic with a fever, sore throat, and fatigue. Physical assessment findings reveal enlargement of the spleen and lymph nodes. Mononucleosis is diagnosed. Which is the nurse's priority in planning the care for the child at home?

Correct: The patient with splenomegaly is cautioned to avoid heavy lifting, trauma to the abdomen or vigorous athletics. Splenic rupture is a concern and requires immediateattention.Incorrect: Limitation of visitors is important for the promotion of client rest, however, isolation or visitor restriction is not needed.Incorrect: Maintenance of adequate fluid volume is aconcern but not a main priority. Bland cool liquids that are not irritating to the throat are encouraged.Incorrect: Due to the fatigue, getting plenty of rest is necessary but not the mainpriority for instruction. Avoid vigorous athletics. Limit visitors. Maintain adequate fluids. Obtain plenty of rest. 27

The presence of which classic cell provides data for the definitive diagnosis of Hodgkin's disease?

Correct: The Reed-Sternberg cell, which is seen on microscopic examination of lymph node tissue, contains two nuclei and is diagnostic of the disease.Incorrect: Sickling of the redblood cell is seen in sickle cell anemia, not Hodgkin's disease.Incorrect: T-cells are not

commonly elevated with autoimmunity diseases and not definitive for the diagnosis of Hodgkin's disease.Incorrect: Epstein Barr is a virus and not related to Hodgkin's disease. Reed-Sternberg cell Sickle shaped red blood cell Floating T-cell Epstein Barr cell28

Which is the most common complication of acetaminophen toxicity in the toddler?

Incorrect: Respiratory distress is not a complication related to acetaminophen toxicity. In the latter stage, hepatic impairment, occurs which is characterized by jaundice, confusion andstupor, pain in the upper right quadrant, and coagulation abnormalities.Incorrect: Nausea and vomiting occur as a result of acetaminophen toxicity. Dehydration is possible in the toddler with frequent vomiting and may be accompanied by electrolyte disturbances.Incorrect: Due to themetabolism of acetaminophen in the liver, the most common complication is liver impairment.Correct: Acetaminophen is metabolized in the liver, therefore, hepatic damage is a major concern. Respiratory distress Fluid overload Renal failure Hepatic damage29

During the acute phase of glomerulonephritis in a child, which intervention is the mostappropriate?

Incorrect: Although the child with acute glomerulonephritis is more susceptible to infection, protective isolation procedures are not indicated. Careful handwashing and avoidanceof known or likely exposure to infectious organisms is reasonable and prudent.Incorrect: Duringthe oliguric phase of glomerulonephritis, the potassium intake should be limited. The risk for hyperkalemia is increased if a high potassium intake accompanies decreased urinary outputand excretion of potassium.Incorrect: Bedrest is often maintained in the acute phase. Childrenhave malaise and fatigue with glomerulonephritis and usually restrict their own activities. Although rest and sleep are important, the most important intervention is focused on theprevention of serious complications, such as malignant hypertension.Correct: Neurologiccomplications, such as seizures and diminished level of consciousness may occur because ofsevere hypertension associated with acute glomerulonephritis. The child with edema, hypertension and gross hematuria may be subject to neurologic complications. Observe protective isolation procedures. Encourage increased potassium intake. Encourage bedrest with appropriate diversional activity. Assess the child for signs of neurologic complications. 30

An 18-year-old female diagnosed with systemic lupus erythematosus (SLE) comes to therheumatology clinic for a follow-up visit. The nurse assesses the client's skin and reviews the client's BUN and creatinine levels. Which is the rationale for the nurse's actions?

Correct: SLE is a chronic inflammatory disease characterized by injury to the skin, joints, kidneys, nervous system and mucous membranes. Clients often seek medical help for relief offever, weight loss, joint pain, butterfly rash, pleural effusion and nephritis. Because of the kidney

damage, the blood pressure will rise and protein in the urine may be evident. Edema results.Incorrect: Kidney damage is common, but does not result in dehydration or dry skin.Incorrect: The characteristic rash in SLE is on the face, not generalized over the body, and is in the shape of a butterfly.Incorrect: Urinary frequency is not a characteristic of SLE. Instead hematuria and decreased urine output are common. A "butterfly rash" and kidney damage are common characteristics of the disease. The client is prone to dry, scaly skin and dehydration related to kidney dysfunction. The generalized rash may lead to a secondary infection affecting the kidneys. The disease process is complicated by urinary frequency and a papular rash.

31

At an unscheduled clinic appointment, the mother of a 9-month-old states that she is concerned about her baby's small size and frequent crying. The mother has limited support systems and poor role modeling for parenthood from her own childhood. Which initial physical assessment data is most important for the nurse to obtain at this time?

Incorrect: The measurement of head and chest circumference can provide data indicating the presence of hydrocephalus, microcephaly or neurological defects. Although these growth parameters also indicate the patterns of growth, the height and weight are more specific

measures of overall growth.Incorrect: Heart rate and breath sounds are important measures for the physical assessment of the cardiorespiratory status but do not indicate growth patterns.Correct: Excessive crying may indicate a wide variety of physical or emotional problems in infancy. The nurse who suspects that the infant is failing to thrive in the home environment first obtains the data regarding the infant's pattern of growth: the height and weight.Incorrect: The suck reflex, present at birth, is vital for infant nutrition. By nine months of age, however, the child should be eating solid foods, chewing soft foods and teething. The assessment for presence of the suck reflex is most appropriate during the newborn assessment. Chest and head circumference Heart rate and breathe sounds Height and weight Sucking reflex 32

Which information regarding suspected episodes of child abuse should the nurse include in the documentation?

Incorrect: Summative statements regarding the events of potential child maltreatment or sexual abuse are inappropriate. Direct quotes from interviewees are recommended to reduce bias and premature judgment.Correct: The documentation of events related to potential child abuse needs to be an objective, factual and concise. Direct quotations from interviewees are recommended to reduce personal bias, interpretation or judgment.Incorrect: Generalizations

regarding the nature of actions leading to harm in a child are inappropriate. Clear, concise, and concrete information is absolutely necessary for the documentation of the events in question.Incorrect: Interpretative statements do not have an appropriate role in the delivery of care to the child with suspected maltreatment or sexual abuse. When allegations are made regarding the actions leading to harm to a child, clear, concise and factual information needs to be documented. Summative statements

Exact quotes regarding the events Generalized description of events Statements related to causative factors 33

A toddler is diagnosed with impetigo and the nurse gives the toddler's mother instructions about skin care. Which statement by the mother indicates a need for further education?

Incorrect: The transmission of impetigo occurs from direct contact with infected skin surfaces. The disease is highly contagious.Incorrect: Impetigo contagiosa is highly communicable in the toddler and preschool child. The skin is colonized with staphylococcal organisms that cause impetigo and therefore, toddlers and preschoolers are susceptible tobacterial infections from their own skin.Correct: Impetigo is a staphlococcal infection that ishighly contagious. The impetigo lesions should be cleaned three to four times a day with Burrow's solution 1:20 to remove the crusts. Usually, the application of a topical bacteriocidal ointment (Bactroban) follows the wound debridement. With proper wound care, lesions are notlikely to scar unless a secondary infection occurs.Incorrect: Cleaning the lesions three to fourtimes daily is a correct treatment regimen. "Impetigo can be spread from one body surface to another." "Toddlers are susceptible from the bacteria on their own skin." "The crusted areas should be allowed to fall off without treatment." "The lesions need to be cleaned three to four times daily." 34

The nurse provides nutritional education for the mother of a toddler. Which information in the toddler's health history indicates a problem regarding nutrition? The toddler:

Incorrect: The developmental stage of a toddler is often characterized by ritualistic behavior regarding many daily routines, including mealtime. The toddler in his effort to control the environment and create a predictable and secure life may insist on various details regardingthe meal.Incorrect: Playing with food and dishes is a normal developmental finding for toddlers. Toddlers do eat some of the food they play with. Therefore, foods should be nutritious and appropriate for the age group.Incorrect: Imitating eating patterns of others is a normal finding fortoddlers.Correct: The variety and volume of food in a toddler's diet is increased and the volume of milk is lessened. Cow's milk contains little iron and displaces the hunger for solid foods. The most common cause of iron deficiency anemia is related to excess cow's milk in the diet. is particular about the arrangement of food on the plate. likes to play with the food and dishes. imitates the eating habits of an older sibling. drinks 42 ounces of cow's milk per day. 35

During the mental health examination of a troubled teen, the nurse assesses for the risk ofviolence. Which nursing response is most appropriate?

Correct: Safety for the teen is important. By asking a direct question, the youth canverbalize feelings instead of acting out the behavior against oneself or others.Incorrect: Asking aquestion about the past does not focus on the problems in the present. This question does not address violent behavior currently.Incorrect: Discussing impulse control is important but does not address suicidal or homicidal behavior that is at risk with violent teens.Incorrect: While family patterns are often passed from generation to generation, the priority is focusing on theclient's violent behavior and not the family's pattern of behavior.

"Do you feel like hurting yourself or anyone else?" "Have you responded to stress with aggressive behavior in the past?" "Have you ever had a problem with impulse control?" "Tell me how your family deals with anger."

36

A physician orders gentamicin (Garamycin) one drop OS four times daily for a 3-year-old child. Which method of medication administration is most appropriate?

Incorrect: Instilling drops into the right eye is incorrect as the doctor ordered the drops beinstilled into the left eye. OD is the abbreviation for right eye.Correct: OS is the correctabbreviation for the left eye. The appropriate procedure is to pull the left lower eyelid down forming a cradle and then instilling the drop.Incorrect: Pulling down the pinna of the ear is not anappropriate technique for instilling eye drops.Incorrect: Pulling the pinna of the ear upward is notthe appropriate procedure for instilling eye drops. Pull the right lower eyelid down, instill drops, and then repeat with the other eye. Pull the left lower eyelid down, forming a cradle, and instill the drops. Pull the left pinna down and back to instill the drops. Pull the right pinna upward and back and instill the drops. 37

A school-age client receives a blood transfusion. The nurse assesses shortness of breath,

bulging neck veins, and a moist cough. These findings are indicative of which complication?

Incorrect: An allergic reaction has signs and symptoms of uticaria, flushing, wheezing and laryngeal edema.Correct: The findings of shortness of breath, neck vein bulging and a moist cough indicate fluid overload.Incorrect: An air embolism would present with symptoms of difficulty breathing, a sharp pain in the chest and apprehension.Incorrect: A hemolytic reaction may present with symptoms of chills, fever, nausea/vomiting, headache, pain in the chest, not dyspnea, and moist cough. An allergic reaction Fluid overload An air embolism A hemolytic reaction 38

The nurse plans the preoperative care of the infant with pyloric stenosis. In feeding the infant, which measure should be implemented until surgery?

Incorrect: An increase in the frequency and amount of the feedings will increase the volume of the stomach, which is already having difficulty emptying, resulting in overload withinthe stomach. Projectile vomiting is a common symptom.Incorrect: Burping any infant isimportant. The infant with pyloric stenosis is not burped any more frequently than any other infant. Care is taken that the infant is handled gently during the burping.Incorrect: The Breck

feeder is used for infants with cleft lip and palate.Correct: Pyloric stenosis is a narrowing of thepyloric sphincter at the outlet of the stomach. The infant should be allowed to rest after the feeding. Handling the infant should be kept to a minimum so the feeding can advance down thedigestive tract. Increase the frequency of the feedings. Burp the infant between feedings.

Feed the infant with a Breck feeder. Let the infant rest after the feeding. 39

When providing instructions to a day care provider about the transmission of chickenpox, whichstatement by the day-care worker reflects a need for further education about the infectiousphase of this disease?

Incorrect: Varicella virus is transmitted through the respiratory route in the droplet form.Incorrect: Varicella virus is transmitted through direct or indirect contact.Correct: Chickenpox is a highly contagious disease caused by a primary infection with varicellazostervirus. The characteristic feature is the generalized, vesicular rash that itches. The mode of transmission is direct contact with persons infected with the varicella and herpes zoster viruses. Respiratory spread by droplet also occurs. The disease is most contagious in the incubation period prior to or including the time of onset of prodromal symptoms and the first crop of therash. The lesion dries and the crust falls off within 5 to 20 days. By the time the lesions scab over, children are no longer infectious and may return to the daycare setting. Seizures are not associated with chickenpox.Incorrect: Varicella lesions with drainage are contagious; the child should remain at home until all vesicles have dried and crusted. Immunosuppresses personsshould not be exposed to the virus. "Chickenpox is spread through the respiratory tract." "Chickenpox is transmitted by direct contact." "When the rash first appears, we should watch for seizures." "Children that have seeping pox should remain at home." 40

Digoxin (Lanoxin) is used in the treatment of a client diagnosed with a congenital heart defect. Which is the mechanism of action?

Incorrect: The mechanism of action for digoxin is increased cardiac contractility. Nitroglycerine, not digoxin, is an example of a cardiac medication that is used primarily for theeffect of vasodilation of the coronary arteries.Correct: Digoxin is used to increase the contractility of the heart and improve the cardiac output. By increasing the effectiveness of the heart's pumping action, the blood supply to the body is improved. The increased tissue perfusion leads to improved oxygen delivery to the organ sites.Incorrect: Digoxin is primarilyused to increase the cardiac output by improving the contractility of the heart.Incorrect: Adiuretic is often used for congestive heart failure to reduce systemic overload associated withcongenital heart defects. Dilates the coronary arteries Improves contractility of the heart Reduces venous return to the heart Decreases systemic overload

41

The nurse provides care for the child diagnosed with glomerulonephritis and collects a urinesample for urinalysis. Which urine color suggests the presence of red blood cells?

Correct: The presence of blood in the urine gives the urine a smoky color.Incorrect: Acloudy appearance is commonly associated with the presence of white blood cells.Incorrect: Bright orange urine occurs as a result of the administration of phenazopyrodine hydrochloride

(Pyridium). This medication reduces the symptomatic relief or urinary burning, itching, frequencyand urgency with urinary tract infection or following urologic procedures. This medication stains clothing.Incorrect: Dark yellow urine is an indication of concentrated urine with a high specificgravity. Smoky Cloudy Bright orange Dark yellow42

A 2-month-old baby is diagnosed with cystic fibrosis. Which statement most accurately defines this disorder?

Incorrect: Cystic fibrosis is not a dominant disorder and does not lead to fatty depositson the liver.Incorrect: Cystic fibrosis is not linked on a dominant gene and is not produced by thelungs.Incorrect: Structural changes of the heart are not produced by cystic fibrosis.Correct: Cystic fibrosis is a recessive disorder that is inherited from both parents. Cystic fibrosis is adisorder of the exocrine glands causing the glands to produce abnormally thick mucoussecretions. The glands most affected are those in the pancreas, respiratory system and sweat glands. Dominant disorder in which an enzyme deficiency leads to fatty deposits on theliver Dominant disorder in which secretions produced in the lungs plug the airway andinduce respiratory distress Recessive disorder in which structural changes occur in the heart muscle Recessive disorder in which abnormal amounts of secretions are produced bythe exocrine glands43

Initial treatment for unilateral (talipes equinus) clubfoot includes which intervention?

Incorrect: Passive range of motion may be used in later stages of healing but is not theinitial treatment.Correct: Casting is implemented as soon as the diagnosis is made. The cast is changed regularly as the child grows.Incorrect: A soft brace is used to add support to an extremity but does not change the true alignment.Incorrect: The hip spica cast is used for congenital dysplasia of the hip and not for clubfoot. Passive range of motion exercises to the ankle and toes four times daily Application of a cast to the affected foot as soon as diagnosis is made A soft brace to hold the foot in the proper alignment until surgery The use of a spica cast until the newborn is ready for surgery44

A 10-year-old is seen in the allergy clinic. The child describes itchy, watery eyes and nasal congestion after spending time outdoors. The skin is noted to be dry and scaly in patches on theback of the arms. Skin testing is ordered. Which measure by the nurse promotes the accuracyof the testing?

Incorrect: Skin testing is commonly repeated with different variations of allergens included. It is still important to note past sensitivities on past medical histories, but the use ofantihistamines are still more important for the test accuracy.Incorrect: Assessment of lung fields and nasal mucosa is a precautionary measure. This indicates a reaction to past allergens and

could be enhanced following the skin testing.Correct: Antihistamine use suppresses skin test

reactivity and should be withheld for five days prior to testing.Incorrect: The upper forearm and upper back, not the abdomen, are appropriate sites for allergy testing. Determining a sitedepends upon the location of choice, cooperation of child and specifics of skin. Assessing past medical history for skin testing that was previously positive Assessing the lung fields for wheezing and nasal mucosa for irritation Reviewing medications over the past five days for antihistamine use Using the child's abdomen instead of upper forearm for skin testing45

A school-age child falls from a bicycle and sustains head trauma. Upon arrival to the emergencydepartment, the nurse identifies signs indicating increased intracranial pressure. Which signshould the nurse document?

Incorrect: A rapid response of the pupils to light is an appropriate neurological response. Pupils that are fixed and dilated or sluggish to react to light and accommodation indicateincreased intracranial pressure. Retinal hemorrhages and papilledema occur with braininjury.Incorrect: Widened pulse is the increased discrepancy between the systolic and diastolic blood pressure. The pulse pressure typically widens with increased intracranial pressure. This isa late finding, occurring more often in adults.Incorrect: Elevated body temperature sometimesoccurs as an indicator of increased intracranial pressure.Correct: Classic signs of increasedintracranial pressure in the school-age child include: altered mental status, agitation, vomitingwithout nausea, diplopia (double vision), elevated body temperature and widened pulsepressure. The behavioral changes of a child are the most reliable indicators of acute intracranialpressure change. Rapid response of the pupils to light Narrowed pulse pressure Decrease in body temperature Mounting agitation

46

According to Erikson's theory of child development, the normal school-aged child masters whichpsychosocial stage for development of healthy personality?

Incorrect: Autonomy versus shame and doubt is Erikson's stage of psychosocialdevelopment that occurs during the toddler period. The child learns of his or her ability to predictably control own actions that also have a direct effect on the reaction and behavior of others. During toddlerhood, areas of conflict regarding autonomy are typical. The toddler's will to control exists with an immature lack of understanding about natural consequences toactions.Incorrect: Identity versus role diffusion emerges during the pubescent period. A sense ofgroup identity precedes the development of personal identity. Adolescents first engage in mastering the task of finding their place within the peer group prior to resolving issues relating towho they are in relation to the inner self, family and society.Incorrect: Initiative versus guilt is a psychosocial, developmental task of the preschool child. Children at this age begin to play andlearn about the world through their own endeavors. They take pride in the new-found abilities to engage in new activities and produce some outcome. The demands of a task for a preschool child may exceed the maturity or skills and produce some degree of remorse, anxiety orguilt.Correct: The child in the middle years develops a fundamental attitude toward work. Duringthis stage of accomplishment, the child masters various skills that enable him/her to participatein the family or community in a meaningful way. The child who is not prepared or capable to

accomplish or assume the responsibilities associated with the stage of industry may develop asense of inferiority Autonomy versus shame and doubt Identity versus role diffusion Initiative versus guilt Industry versus inferiority47

A preschool child is brought to the primary-care clinic because of anal itching at night. The child is diagnosed with pinworm infestation and mebendazole (Vermox) is prescribed. Which instruction to the family is most important regarding follow-up care?

Incorrect: Washing the sheets in hot water is advised for hygiene measures. However, the control of the infestation occurs with the pharmacologic treatment (Vermox).Incorrect: Isolation of the child is unnecessary after treatment with Vermox. The first dose of medication is highly effective in controlling the infestation.Incorrect: Showering is recommended rather than tub-bathing during the two week period between the initial dose of Vermox and the follow-up dose. Other measures to prevent re-infestation to the child or others include: thorough handwashing after toileting, keeping the child's fingernails short to minimize the ova collecting underthe nail, and dressing the child in one piece sleep attire to minimize itching at night.Correct: All family members are treated with one dose of the medication. After treatment with the single Vermox dose, the pinworm infestation is eradicated. A second dose two weeks after the initial therapy is necessary to prevent re-infestation. All sheets must be washed in hot water. Contact with other children should be avoided for 7-10 days.

Daily tub bathing is recommended during infestation. The entire family must be treated with Vermox. 48

A nurse assesses the growth and development of a 3-month-old. Which activity is undeveloped at this age?

Incorrect: Holding a rattle is an appropriate motor activity for a three-month old infantdue to the intact grasp reflex. The infant frequently drops a rattle because the grasp is reflexive and not purposeful.Incorrect: Attempting to roll over is an appropriate developmental function at three months of age. The infant is beginning to coordinate the movements of the body.Correct: The three-month old infant does not have the ability to use the pincher grasp. Usually this finemotor skill occurs at about eight months of age.Incorrect: Typically, smiling in response to a mother's voice and social behavior occurs at two months of age. Holds a rattle and places it in the mouth Attempts to roll over Picks up a small object using finger and thumb Smiles in response to mother's voice 49

Which diagnostic procedure is used to ascertain if a female client has gonorrhea?

Incorrect: Blood specimens are not an indicator of gonorrhea.Correct: The diagnostic test for gonorrhea is a vaginal culture with microscopic examination.Incorrect: Pus in the urinemay be indicative of a urinary tract infection but not gonorrhea.Incorrect: Determining the lastmenstrual period is not helpful with the diagnosis of gonorrhea. Take a blood specimen.

Obtain a vaginal culture. Test the urine for pus. Determine the last menstrual period. 50

The nurse assesses the development of a 3-year-old child at a routine clinic visit. The nurse should expect the mastery of which developmental task?

Correct: A three-year old child with normal development is able to identify five body parts.Incorrect: The dexterity necessary to copy a square does not normally develop until six years of age.Incorrect: The fine motor skills that are necessary for tying shoelaces do not normally appear until five or six years of age.Incorrect: The school-aged child, not the preschooler, is normally able to hop on one foot. Identifying five body parts Copying a square Tying shoelaces Hopping on one foot51

The nurse providing nutritional instruction to parents of a child diagnosed with cystic fibrosisrecommends which type of diet?

Correct: The child with cystic fibrosis lacks the pancreatic enzyme necessary for adequate digestion and growth. A well-balanced, high-calorie, high protein diet, as much as 50% above normal, should be encouraged. The impaired intestinal absorption can lead to poor nutritional status. Additionally, the child should take supplementary pancreatic enzymes to increase nutritional availability and reduce the waste product transit time through thebowel.Incorrect: Fat is expelled in stool as a result of poor fat absorption in the intestinaltract.Incorrect: A high fiber diet is not a primary concern in the nutritional counseling for the childwith cystic fibrosis. A well-balanced diet that is high in protein and calories is necessary to achieve growth.Incorrect: A vegetarian diet is not encouraged unless it contains a high caloric density, high protein and high fat components to meet the needs of the child with cystic fibrosis. High calorie Low fat High fiber Vegetarian 52

Which nursing assessment indicates bleeding in the postoperative phase after a tonsillectomy?

Correct: Frequent swallowing is an indication that the operative site is bleeding. Surgicalintervention or surgical packing may be needed.Incorrect: Complaints of throat pain arecommon after surgery. Analgesic/antipyretic drugs, such as acetaminophen or non-steroidal anti-inflammatory drugs (NSAID), are commonly administered to promote comfort and reduceinflammation.Incorrect: An increase in the pulse rate is common due to pain, stress, and physiologic response to surgery.Incorrect: Dark brown blood is commonly seen in the vomitus,

secretions of the nose, and between the teeth related to the blood loss during the surgicalprocedure. Active bleeding is evidence by bright red blood or pink mucous. Oral intake is restricted until there are no signs of hemorrhage. Frequent swallowing Complaints of throat pain Increased pulse rate

Dark brown blood in the emesis 53

The school nurse provides an educational program for a group of preadolescents regarding theuse of birth control pills. Which is the mechanism of action for oral contraceptives?

Incorrect: Oral contraceptives do not kill sperm.Correct: Oral contraceptives inhibitovulation. If the ovum is not released from the ovary, it cannot be fertilized and pregnancy cannot occur.Incorrect: Oral contraceptives do not change the genetics of sperm.Incorrect: Oral contraceptives do not interfere with the implantation of the fertilized ovum in the uterus. Kill sperm Inhibit ovulation Cause sperm dysfunction Prevent ovum implantation54

The mother of a school-aged child diagnosed with a terminal illness asks the nurse, Howshould I explain this to my child? Which nursing instruction is most appropriate?

Incorrect: The child's physical condition may not determine how much information thechild understands or wants to know.Incorrect: Although the school-aged child is in the Piaget's stage of concrete-operational thought, photographs of other children in the latter stage of thedisease may provide too much detail and frighten him/her. This level of detail may be useful for

health care professionals but not for an ill child.Incorrect: A full description of the disease process and contributing causes is probably more than the child is interested in learning. The school-aged child is egocentric and the explanation needs to pertain directly to the personalsituation. Only information that is relevant to the child will be retained.Correct: The child's age, cognitive and emotional development set the boundaries regarding the type and extent ofinformation and the manner for delivery. Provide an explanation of the illness depending on the child's physical condition. Show photographs of the children in the latter phase of the disease. Give a description of the disease process and contributing causes. Explain the illness honestly at an age-appropriate level. 55

The nurse prepares to administer an intramuscular injection to a 4-year-old child. Which site ismost appropriate?

Incorrect: The ventrogluteal is not a recommended site for young children. The sciatic nerve can be injured from a long needle injected posteriorly and medially.Incorrect: The vastus lateralis is an acceptable site for intramuscular injection in preschool children. However, often the site becomes sore and interferes with walking. The vastus lateralis site is ideal for intramuscular injection in infancy and can be accessed in a variety of sitting, lying or sidelyingpositions. This muscle can tolerate larger volumes of fluid because of the development of the muscle mass in infancy.Incorrect: The dorsogluteal is site that is frequently used for intramuscular injections in older children and adults. The site is contraindicated in children

walking less than one year.Correct: The deltoid muscle is adequately developed in the preschool child and is likely to tolerate the fluid in an intramuscular injection. From adevelopmental perspective, the deltoid may be a more acceptable site for injection due to thesense of privacy that begins in the preschool age group. Ventrogluteal Vastus lateralis

Dorsogluteal Deltoid

56

Which is the peak age for the occurrence of sudden infant death syndrome (SIDS)?

Incorrect: Two to four weeks of age is early for most cases of SIDS. Unexplained death at this time could be related to many types of congenital abnormalities, such as persistent, lateapnea of prematurity that exists beyond 44 weeks post-conceptual age.Correct: The peak time period for the incidence of SIDS is two to four months of age with 95% of the cases occuring bythe age of six months. It is more common in low birth weight babies, males, crowded livingconditions, environmental cigarette smoke, and during the winter months. Research indicatesthat infant positioning during sleep may be a factor in the incidence of SIDS. Supine positioningfor sleep in healthy infants is recommended to reduce the risk of SIDS.Incorrect: By ten monthsof age, the risk for SIDS is markedly reduced. The infant has good head, neck and back control. If an obstruction of the airway occurs as a result of head positioning, the infant at 6-10 monthsof age can rescue him/herself.Incorrect: After ten months of age, infant death is most likely to bethe result of an accidental injury. Prior to four weeks Two to six months Six to ten months After ten months 57

An adolescent seeks health care at a free clinic and states she is sexually active and concernedabout AIDS. Which nursing instruction regarding transmission of sexually transmitted diseases is most appropriate?

Incorrect: Diaphragms do not prevent the transmission of infection. A diaphragm serves as a barrier for the prevention of sperm penetration to the uterus. The diaphragm with spermicidal agent is an effective method to reduce the chance of pregnancy, not the spread of infection.Incorrect: The lack of symptoms of AIDS is unrelated to the infectious status as the virus may lie dormant in the body for an extended period of time prior to the onset of symptomsof infection.Correct: Condoms are the best protection against the spread of sexually transmitteddisease (STD) if used correctly.Incorrect: Spermacidals do not protect the body from the exposure to the virus. It is essential that you wear a diaphragm to prevent cross infection with yourpartner. If your partner has no symptoms of sexually transmitted disease then you do notneed to worry. A condom is the best protection, except for abstinence, to prevent sexually transmitted disease. A spermicidal is used to prevent the spread of sexually transmitted disease in both partners. 58

An adolescent girl is diagnosed with scoliosis and her mother is reluctant to pursue treatment. The nurse educates the mother that untreated scoliosis may ultimately have which outcome?

Incorrect: Scoliosis is not likely to correct over time.Incorrect: Scoliosis of the thoracicregion does not affect bladder control.Incorrect: Scoliosis of the thoracic region does not affect

gastrointestinal function.Correct: Scoliosis is the lateral curvature of the spine, which may becongenital, idiopathic, or a result of paralysis. Scoliosis is at the level of the chest, therefore, ifuntreated, heart and lung function may be compromised. Correct itself over time Compromise bladder control Reduce gastrointestinal function Affect heart and lung performance 59

When assessing the heart rate for infants and small children, which pulse point should the nurseuse?

Correct: The apical pulse is recommended for infants and children and is heard at the apex of the heart for one full minute.Incorrect: The carotid pulse is located in the neck and is notused to assess heart rate in children.Incorrect: Brachial pulses are used to assess the absenceof a pulse when performing CPR.Incorrect: Apical pulses are taken in children under one because the radial pulse rate is too faint. Apical Carotid Brachial Radial 60

A hospitalized 18-month-old is diagnosed with gastroenteritis. When providing discharge

teaching for the child's parents, which food should the nurse suggest be reintroduced last?

Incorrect: Bananas can be introduced into the diet early and are part of the BRAT diet. Bananas provide potassium, which may have been lost through vomiting and diarrhea.Correct: Following an acute episode of gastroenteritis, foods are reintroduced slowly. The glucose and milk within the pudding would be introduced later into the diet after the body is able to toleratethe BRAT (Bananas, Rice, Applesauce and Toast) diet. The high sugar and milk content found inthe pudding may trigger further diarrhea.Incorrect: Applesauce is a part of the BRAT diet and can be introduced into the diet early. It is bland and easy to digest.Incorrect: Rice is also part of the BRAT diet, is easily digested and can be introduced into the diet early. Rice provides carbohydrates needed for energy. Bananas Pudding Applesauce Rice

61

A child with Type I diabetes mellitus experiences polyuria, polydipsia, and polyphasia, and has ahistory of diabetic ketoacidosis. The family should be instructed that ketones can appear in the urine when the blood glucose reaches which level?

Incorrect: 120 mg/dl is a normal blood glucose reading.Correct: The normal blood

glucose varies between 110-150 mg/dl. It is not until 250 mg/dl that blood glucose will spill over into the urine.Incorrect: 180 mg/dl is above normal limits but not far enough above normal limitsfor the appearance of glucose in the urine.Incorrect: 150 mg/dl is on the upper boundaries ofnormal but would not have any blood glucose spilling to the urine. 120 mg/dl

250 mg/dl 180 mg/dl 150 mg/dl62

A hospitalized preschool child is recovering from surgery and is in the playroom. Which behaviorshould the nurse expect to observe?

Incorrect: Playing a board game with another child is a typical activity for the schoolaged child. They enjoy the interacting or competing with peers. The preschool child generally does not have the cognitive abilities to understand the goals, strategies and directions of boardgames to have success and enjoy them. The social skills are immature and turn-taking is not fully functional in children of this age group. Attempts at play with board games usually require the interaction with adult or mature players.Correct: An imaginary friend is not uncommon for this age. The creative imagination of a preschool child is rapidly developing and acts as a way the child can practice social relationships in a non-threatening environment.Incorrect: throwing aball into the air and catching it usually requires the coordination of an older child, usually afterage six.Incorrect: Parallel play is the independent play activity among toddlers. Playing a board game with another child Talking to an imaginary friend Throwing a ball into the air and catching it Playing with a toy beside other children63

A 4-week-old infant with an atrial septal defect (ASD) is monitored for signs of congestive heart

failure (CHF) prior to surgery. The nurse measures intake and output to determine the fluid status of the child. Which is the best method to obtain an approximation of the volume of urineoutput?

Incorrect: Simply weighing the wet diaper does not take into account the weight of thediaper.Incorrect: The correct method to measure urine output is to weigh the diaper after urination and subtract the weight of the dry diaper.Correct: The correct procedure for the determination of an accurate urinary output is to weigh the diaper prior to and followingurination. Subtract the difference. Cloth diapers vary in weight; a baseline measurement will be necessary. However, because the weight of disposable diapers is stable with very little discrepancy among diaper weights, each diaper does not need to be weighed but the baselinefor that type and size is necessary information. The conversion for volume to weight measurement is: 1cc = 1 gm.Incorrect: The correct method to measure urinary output is to weigh the diaper prior to and following urination. Weighing the infant before and after urination istheoretically a plausible method for determining an approximate urinary volume. Practicallyspeaking, this approach is inconvenient and may disrupt the infant's sleep patterns. It is notlikely that the care provider will be able to weigh the infant immediately after urination. The clothing needs to be removed for an accurate weight, which is cumbersome and timeconsuming. An easier, more feasible method is to simply weigh the diaper, not the infant. Record the weight of the wet diaper directly on the graphic flow sheet. After urination, weigh the wet diaper and divide by the birthweight. Weigh the diaper before and after urination and subtract the difference. Weigh the infant before and after urination and record the volume. 64

A 12-week-old infant requires the insertion of a ventriculoperitoneal (VP) shunt for the treatment of hydrocephalus. Which is the most serious complication of VP shunts?

Incorrect: Shunt migration may occur initially but is not of significant harm to theclient.Incorrect: Occasionally obstruction of the shunt may occur from a thrombus. Medicationsare given to dissolve the thrombus.Incorrect: Mechanical obstruction may occur from a kink inthe tubing or tubing separation. Careful assessment of the client's condition is needed.Correct: Shunt infection is the most serious complication. Any type of invasive procedure provides a risk of infection. Shunt infections are particularly grave since they may compromise the intellect ofthe child. Shunt migration Obstruction of the shunt Mechanical malfunction Shunt infection 65

A mother brings her toddler into the clinic because of fussiness, fever, and pulling of the right ear. The mother states that the symptoms are the same as when the child had a previous ear infection. Which is the most appropriate nursing response?

Incorrect: Deafness rarely results from otitis media; however, repeated damage to the eardrum may cause transient hearing impairment.Incorrect: The failure of the previous ear

infection to resolve may be an incorrect assumption. In most cases, the completion of the fullantibiotic course is crucial for the eradication of the infection and prevention of secondaryinfection with resistant strains of organisms.Incorrect: Otitis external is an infection in the outerear canal. Otitis media is an infection of the inner ear.Correct: The nurse explains the structure of the infant's ear and addresses the reason for the infant's repeated ear infections. "It is good that you are concerned because frequent ear infections may lead todeafness." "It is probable that the previous ear infection did not completely resolve." "Otitis media is an infection in the outer ear canal and is common in earlychildhood." "Children are more prone to ear infections due to the shorter, straighter ear canal."

66

An 11-year-old receives an allergy shot during a clinic visit. Which symptom indicates the early stage of anaphylactic shock?

Incorrect: 100/60 is an appropriate blood pressure for an eleven-year old child.Correct: Acorrelation has been found to relate reports of impending doom and anxiousness andanaphylactic reactions.Incorrect: A heart rate of 90 beats per minute is normal for a child that is eleven years old.Incorrect: Abdominal cramping is not a common early symptom of an anaphylactic reaction. Abdominal cramping may be seen as the reaction progresses. A blood pressure reading of 100/60 A sense of impending doom A heart rate of 90

Complaints of abdominal cramping67

When initiating a plan of care for the child with leukemia, the nurse informs the family toanticipate which diagnostic test?

Incorrect: Leukemia is a disease affecting white blood cells, not red cells.Incorrect: Positron Emission Tomography (PET) is a scan used to measure the functions in the brain such as: blood flow, metabolic activity and biochemical change.Correct: The test commonly used to diagnose leukemia is a bone marrow aspiration. Leukemia is the uncontrolled production ofwhite blood cells, and is the most frequently occurring type of cancer in children. The bone marrow of the client is characterized by a high white blood cell count (leukocytosis), low plateletcount (thrombocytopenia) and immature white blood cells (blast cells.)Incorrect: Blood gasanalysis is used to determine the oxygenation, ventilation, and pH status of the child. Arterial, venous, or capillary samples may be used although the value ranges differ slightly depending on the site the blood was obtained. Red blood cell count Positron Emission Tomography (PET) Bone marrow aspiration Blood gas analysis68

The administration of aspirin has been discouraged in pediatric use because of the associationof which condition?

Incorrect: Erb palsy is a manifestation of brachial plexus injury that results fromexcessive stretching or pulling of the arm away from the shoulder head. Paralysis of the affectedarm occurs. This condition has no relation to aspirin administration.Incorrect: Hodgkin's disease

is a malignant disorder of the lymphatic tissue and has no relation to aspirinadministration.Incorrect: Kaposi sarcoma is a malignant, multifocal neoplasm associated withHIV.Correct: Aspirin should not be given to children with influenza virus or chickenpox due to theassociation with Reye's Syndrome. This disorder is characterized by encephalopathy and liver dysfunction. Erb palsy Hodgkin's disease Kaposi sarcoma Reye's syndrome69

A child diagnosed with sickle cell disease experiences an acute, vaso-occlusive crisis. Which nursing intervention is the priority?

Incorrect: Dehydration aggravates the sickling process. Oral and parenteral fluidsenhance the transportation of cells without clumping, thereby, inhibiting thrombus formation. However, the top priority for care is to supply the cells and tissues with needed oxygen to prevent ischemic tissue damage and acidosis.Incorrect: Prevention of infection is a concern dueto a dysfunctioning spleen in children with sickle cell disease. The first priority for care of the child with acute vaso-occlusive crisis is to supply oxygen to the ischemic areas. The altered tissue perfusion occurs as a result of impaired arterial blood flow.Incorrect: Pain relief is obtained with narcotic analgesia. Vaso-occlusive crisis is extremely painful and the child is oftenunder-medicated. The physiologic response to pain is increased oxygen consumption, increased heart rate and blood pressure. Control of pain is an important factor in reducing theoxygen demand in the tissues. The first priority for the care of the child in sickle cell crisis, however, is to meet the body's need for oxygen with supplementary sources as needed.

Resolution of tissue and cellular ischemia is necessary to prevent acidosis and tissue damage.

Local heat source may be soothing to the affected area. Bed rest is usually necessary during the acute phase of the disease.Correct: The priority for the care of the child with acute vasoocclusive crisis is oxygen therapy. Sickled cells cause obstruction in the blood vessels leading to tissue ischemia and infarction. Maintain hydration. Prevent infection. Provide analgesia. Administer oxygen. 70

The nurse provides discharge instructions to the parent of a child who has a newly applied cast to the leg. Which statement by the parent indicates the need for further nursing instruction?

Incorrect: The best method for setting the cast is air-drying. Blow dryers will dry the external surface of the cast; leaving the interior moist and incompletely set.Correct: The palms of the hands should be used when handling the wet cast to prevent depressions to the external surface.Incorrect: The assessment of color, perfusion and temperature of the extremity is important for the detection of circulatory impairment.Incorrect: The position of the extremity slightly above the level of the heart reduces venous congestion and prevents edema. "The new cast will need to air dry." "I should use my fingertips when handling the wet cast."

"I need to check the toes for temperature and color." "I will place the leg above the level of the heart."

71

Which is the main difference between the male and female anatomy related to urinary tract infections?

Incorrect: The male urethra is curved which inhibits the progression of the pathogen toward the bladder.Correct: The urethra is shorter and straighter in females, therefore, pathogens can enter the bladder more readily. Due to the proximity of the rectum, the female urethra may be more easily contaminated with bacteria. The nurse can provide education to the child and/or caregiver to practice hygiene measures to reduce the spread of organisms to the genitourinary tract.Incorrect: Both the male and female tracts are lined with mucous membranes.Incorrect: The diameter of the urethra is quite similar in both males and females. The male urethra is straighter, allowing more rapid elimination of pathogens. The female urethra is shorter so the pathogens can enter the bladder more quickly. The male urethra is lined with mucous membranes to trap microorganisms. The female urethra is larger in diameter, allowing for the rapid entrance of pathogens.

72

A teenage boy commits suicide. Which is the main cause of suicide?

Incorrect: Suicide can be from an act of defiance or an I'll show you attitude but this is not the most common theory.Incorrect: Suicide can be a psychotic act during a period of mental instability, but this is not the most common theory.Correct: Suicide is most commonly referred to as destructive aggression turned inward against the self.Incorrect: Suicide is not usually a form of manipulation. An act of defiance

A psychotic act Destructive aggression turned inward A form of manipulation 73

Which signs indicate IV infiltration in a child?

Incorrect: Fever, chills and pain are not common with IV infiltration.Correct: Erythema (redness), pain, edema and a streaking at the vein site are indications that IV fluids may beaccumulating in the interstitial tissue and not going through the circulation. Also these signs mayindicate an irritation of the vein that could cause IV compromise.Incorrect: Limited ability tomove the extremity may be caused by tape or placement of the arm on an armboard. It mayalso be caused by IV infiltration.Incorrect: The backflow of blood at the site and to the tubing may be caused by the position of the arm or solution or may occur from the IV controller beingpaused. This is not a sign of infiltration. Fever, chills, and pain Erythema, edema, and streaking at the vein site Limited ability to move the extremity Backflow of blood at the IV site and in the IV tubing74

Following the surgical repair of a toddler's cleft palate, which method of administering fluid ismost appropriate in the post-operative recovery period?

Incorrect: A sippy cup is plastic and the tip of the lid is placed inside the mouth; the child may want to suck on the end to obtain the fluid inside. The vacuum cups prevent spilling, however, a good deal of negative pressure is needed for use of the cup. The tip may irritate the newly corrected surgery.Incorrect: A drinking straw is discouraged. A child has to suck out of the straw to obtain fluid and the negative pressure may stress the suture line. Additionally, the firm surface of the straw's end could potentially cause damage to the incisional site.Correct: Nothingshould be placed inside of the mouth as palate sutures may be damaged. A cleft palate is typically repaired between 12-18 months of age. It is important to offer the cup in a manner that is appropriate for the skills of toddler age.Incorrect: A Breck feeder is a device that is placed inside an infant's mouth for specialized feeding efforts. It is a large syringe with rubber tubing that extends to the back of the mouth to reduce the chance of regurgitation through the openpalate and nasal cavity. The Breck feeder is used most often with infants. Sippy cup Drinking straw Regular cup Breck feeder 75

The nurse planning the care for a child that has partial and full thickness burns understands thatan early complication with extensive burns is

Incorrect: In the initial phase of severe burn injury, cellular damage occurs, causing fluid shifts. When bodily fluid shifts rapidly out of the intravascular compartment, shock can result.

The nurse providing care to the child with severe burns monitors the vital signs, providesintravenous infusion and assesses closely for signs indicating hypovolemia.Incorrect: Infectionis not a common manifestation of severe burn injury in the early stages. The primary concern in the initial phase is the identification and treatment of fluid and electrolyte imbalances that maybe potentially life-threatening. Although, infection may occur later, the primary concern initially

for the child with extensive burns is the development of hypovolemic shock.Correct: The primary emphasis during the initial phase of burn care is the prevention of hypovolemic shock.Incorrect: Because the child's blood volume is affected initially, the renal function is threatened. High output renal failure does not occur. The average urine output should be 1 mL per kg/hour. dehydration. infection. hypovolemic shock. high-output renal failure.

ATI. MANAGEMENT 2.0

A nurse has been encountering resistance from a subordinate to whom she has been delegatingtasks. Which of the following is the most appropriate initial response by the nurse to thisproblem?

This will foster open communication, personal growth, and positive effects of selfesteem. Although this is an appropriate action, it is not the most appropriate action for the nurse to take initially.Insubordination can be caused by a variety of factors. The subordinate may feel overwhelmed being delegated to by more than one nurse. The subordinate may lack the selfconfidence or knowledge needed to perform the task. The subordinate may have an inherent

resistance to authority. Overdelegation with overly specific instruction may stifle the subordinate's creativity, independent thinking, and motivation. Once the cause is determined, the nurse is better able to attempt to eliminate the cause if possible. For example, the nurse canprovide education on a particular task for which the subordinate lacks knowledge orconfidence.Immediate confrontation using formal authority will most likely be intimating to thesubordinate and will not get to the root cause of the problem. This will reduce the chance of finding an effective solution.Failure to allow the subordinate to be challenged will cause the subordinate to become disinterested and fail to internalize responsibility or accountability for theassigned tasks. Provide the subordinate with consistent, constructive feedback, both positive andnegative. Ascertain why the task is not being accomplished and attempt to remedy the cause. Use formal authority to confront the subordinate in private at the proper time. Only delegate uncomplicated tasks that do not require independent thinking. 2

A nurse is caring for a client who is a well-known member of the community. The nurse receives a call from an individual who states that he is the client's brother and would like information on the client's condition. Which of the following should be the nurse's next action?

It is impossible for the nurse to verify that information over the phone.Health InformationPortability and Accountability Act (HIPAA) guidelines prevent the sharing of information over the phone.This action acknowledges to the caller that the person is a client in the facility and, therefore, is a Health Information Portability and Accountability Act (HIPAA) violation.This is in

violation of the client's privacy. Verify the caller's relationship to the client.

Refuse to give any information about the client to the caller. Refer the individual to the client for information. Answer the individual's questions in a clear, calm, thorough manner. 3

Which of the following actions is an example of a nurse's advocacy role?

The nurse would provide a list of agencies, and the client/family would select theagency.Unless specifically requested by the client, the nurse is imposing a personal ethic or value on the client.Advocacy is based on mutual understanding between nurses and clients, taking into account common needs and common results. Within the advocacy role, specificresponsibilities have been identified. The nurse assists the client in making autonomous and informed decisions by being sure the client has relevant information.The role of the nurse is toenable clients to make their own choices. Selecting a home care agency in preparation for discharge Giving the client a list of skilled nursing facilities with ministerial services Providing all relevant information about medications and medical equipment Directing the client to purchase supplies from a specific vendor4

Which of the following considerations should a nurse use as a guide when delegating tasks toassistive personnel (AP)? (Select all that apply.)

How stable are the conditions of the clients involved? is correct. The acuity and stability of the client's condition are important to delegating tasks. How complex is the task involved? is correct. The tasks should be within the ability and education level of the AP. How well is the AP liked by others? is incorrect. This is a subjective assessment that should not be used as a guide for delegation of tasks. It is more important to assess the skill level of theAP and the needs of the client. How predictable are the client responses to the assigned tasks? is correct. The predictability of the client's condition should determine if the task can be delegated. What is the potential for harm to clients? is correct. The nurse has a responsibility to keep clients safe at all times.How stable are the conditions of the clients involved? is correct. The acuity and stability of the client's condition are important to delegating tasks. How complex is the task involved? is correct. The tasks should be within the ability and education level of the AP. How well is the AP liked by others? is incorrect. This is a subjective assessment that should not be used as a guide for delegation of tasks. It is more important to assess the skill level of theAP and the needs of the client. How predictable are the client responses to the assigned tasks? is correct. The predictability of the client's condition should determine if the task can be delegated. What is the potential for harm to clients? is correct. The nurse has a responsibility to keep clients safe at all times. How stable are the conditions of the clients involved? How complex is the task involved? How well is the AP liked by others? How predictable are the client responses to the assigned tasks? What is the potential for harm to clients?

A client with a fracture of the right femur has a nursing diagnosis of impaired physical mobility. Which of the following actions should be the lowest priority?

This is a priority for clients with a nursing diagnosis of impaired physical mobility. The client has the right to be free from pain, and ensuring that the client is free from pain willincrease the client's activity tolerance.Monitoring circulation in the right leg is a high priority forthis client. The client is at high risk for thrombosis or impaired tissue perfusion.This is a foundational nursing action for client safety. The nursing diagnosis of impaired physical mobility makes it an especially high priority for this client.Adequate nutrition is important, but it is a lowpriority for a client with a fractured femur and a nursing diagnosis of impaired physical mobility. Administer pain medications prior to planned activity. Monitor the circulatory status of the right leg. Place the call light and bedside table within the client's reach. Remind the client about the importance of adequate nutrition for bone healing.

Which of the following tasks associated with caring for a client with pulmonary edema can bedelegated to assistive personnel (AP)?

This task is included in the AP training.AP are not trained to assess for fluid retention. This task requires special knowledge and training.AP are not trained to instruct the client in

deep breathing exercises. This is a task that requires special knowledge and training.AP are nottrained to monitor respiratory response to activity. This task requires special knowledge and training. Record client fluid intake and output. Check for fluid retention. Instruct the client in coughing and deep breathing exercises. Monitor respiratory response to activity. 7

Which of the following activities is the safest to assign to an assistive personnel (AP) who justcompleted orientation?

An AP may be allowed to change sterile dressings in some organizations when specially trained. Traditionally, sterile dressing changes are nursing actions.This task should only be performed by an AP who has had special training and testing.The client could experience dysphagia (difficulty swallowing).The AP would be trained and checked off on this task during orientation. Changing a sterile dressing on a client who has a decubitus ulcer Printing client telemetry rhythm strips Assisting with a meal for a client with a recent cerebrovascular accident Obtaining vital signs on a client who has a fever8

A nurse is working on a unit that is undergoing significant change. Which of the following

behaviors indicates her support of unit change?

Understanding how change affects the nurse and other staff as individuals motivates the nurse to adopt the change.Speaking openly with leadership will assist the leadership inevaluating the change and minimizing the resistance to change.Maintaining the status quoworks against change instead of supporting it.Discussing the negatives of the change with otherunit staff lowers unit morale and shows a lack of support for unit change.

She understands how change affects people as individuals. She avoids discussion of the change. She assists in maintaining the status quo. She discusses the negatives of the change with other unit staff. 9

A nursing unit has been modified to provide rehabilitation to clients who are incarcerated. The nurse notes that several nursing assistants are not providing care to these clients. Which of the following actions should the nurse take first to have all staff participate in the clients' care?

This is a later step, and it would be the responsibility of the nurse manager.There may be civil consequences, but there is no evidence of criminal activity.The staff should be provided with information that withholding of care is client abandonment.This does not address the client abandonment by the staff. The budget would probably not support this. Arrange a meeting with the human resources department to remind staff of job responsibilities. Notify the state department of health of the staff's criminal intent. Provide a session to remind the staff of the ethical and legal issues in withholding care. Contact an external staffing agency for extra personnel. 10

An off-going nurse reports that a client admitted with vomiting and abdominal pain is experiencing increasing pain despite more frequent administration of narcotic analgesics. He reports that he notified the primary care provider of the client's pain status 6 hr ago and that he is still waiting for a call back. With intervention, it is determined that the client has a perforated bowel. Which of the following actions is the primary legal responsibility of the oncoming nurse?

Identifying education needs is the responsibility of the nurse's manager.Contacting the ethics committee would be the responsibility of the manager/supervisor. The incident would be reported to the risk management department.The episode may be a civil matter, but it is not a criminal matter.A reasonable, prudent nurse would be persistent in notification. Malpractice judgments can involve nurses' responsibility to seek care for clients. Report the incident to the education department because the prior nurse needs more training. Report the incident to the hospital ethics committee because the prior nurse is unethical. Report the incident to local law enforcement because the prior nurse's care was criminal. Report the incident to the nursing supervisor because the prior nurse's care was negligent.

11

A licensed practical nurse (LPN) is a charge nurse in a long-term care facility. When making assignments for assistive personnel (AP), the LPN should

The charge nurse should delegate by the appropriateness of the task.It is important that AP understand the expectations of assignments.Every employee has different experiences and education, even within the same job description. They may not all know how to do all tasks.The person to whom the task is delegated is responsible for completing it. assign the tasks that nurses do not have time to do.

clearly communicate expectations and desired results. expect employees with the same job description to have the same ability. accept responsibility for all activities delegated to others. 12

Which of the following is a characteristic of effective nurse mentoring?

Overdependence of the mentee on the mentor will not help the mentee to grow.The relationship between the mentor and mentee occurs over an extended period of time and will hopefully last for years.Effective nurse mentoring focuses on career development.Mentoring is a reciprocal relationship between the mentor and mentee. Encourages the mentee's reliance on the mentor Facilitates the quick development of the relationship Focuses predominantly on career development Provides for a one-way dissemination of knowledge 13

A staff member of the hospital staff makes sexual statements and jokes to a nurse. Which of the following actions should the nurse take first?

This is a subsequent step in dealing with harassment in a hostile work environment.This is a later step in the process of dealing with workplace harassment if initial actions fail to stop the harassment.The initial action is to tell the staff member to stop the harassing behavior. When investigated, one of the first questions is whether the staff member was informed that the communication was perceived as offensive.Harassment occurs if the recipient perceives sexual harassment. The claim would be investigated. Filing a grievance would not be the initial step. Notify the staff member's immediate supervisor. Notify the security department. Tell the staff member to stop immediately. File a grievance. 14

Which of the following actions is an example of cost effective nursing care?

It would be preferable to open each individual pill pack at the bedside and pour into a single medicine cup. The client can then swallow the medications, either one at a time or all together, per the client's preference.Supplies are the next most significant cost secondary to personnel costs. Staff leaving with forgotten supplies in their pockets (e.g., gauze wraps, tape, syringes, penlights) can take a significant portion out of the operating budget.This is an unacceptable practice that can lead to contamination of the medications.Unnecessarily using packs or trays for procedures and disposing of unused supplies constitutes significant costs in

lost supplies. Saving unused supplies from packs and trays or using individual items when possible can provide cost savings. The nurse places pills in separate paper medicine cups to administer. The nurse empties her pockets of all supplies prior to going home. The nurse holds medications in her hand to deliver and administer them. The nurse uses packs or trays for procedures whenever possible. 15

One way a licensed practical nurse (LPN) can appropriately contain costs at the bedside is to This is not within the scope of the position description.This is within the scope of theposition description.This has no relevance in this situation.This is inappropriate.

delegate skin care assessment to assistive personnel (AP). delegate client hygiene care to AP. only perform skills related to Maslow's Hierarchy of Needs. reuse dressing supplies if they are not soiled.

16

A nurse notes on admission that a client has bruises on the arms and back. The client, a resident of a group home, has mobility with the use of a motorized wheelchair. Which of the following nursing interventions is legally required?

This is not a nursing role. Advocates for the physically challenged or a case worker would become involved.It is important for the nurse to note the skin condition when the client isadmitted. The nurse must report suspected abuse. The nursing supervisor will follow agency protocol for notification.This is not the legally mandated course of action to be taken by thenurse.Diagnostic tests are prescribed by the provider. Assisting the client to seek legal representation related to potential abuse Documenting the location and type of injury and notifying the nursing supervisor Notifying the group home administrator and inquiring into the nature of the client'sbruises Ordering serum specimens to check for clotting abnormalities17

An LPN is delegating client care to an assistive personnel (AP). Which of the following isappropriate for the LPN to perform in regards to the delegation? (Select all that apply.)

Monitoring and follow-up is correct. The delegating nurse must provide supervision by following up and monitoring the AP's performance. Transferring responsibility is incorrect. Responsibility for the care of the client is not transferred from the LPN. There is a risk of liability with delegation related to the AP's skills and experience, type of task, regulations and Nurse Practice Acts, and policies and procedures. Receiving and providing feedback is correct. This is part of providing direction and understandable expectations regarding the activity or activities to be performed. Evaluating performance is correct. It is the nurse's responsibility to evaluate the AP's performance to assure the proper completion of delegated tasks. Assuring clear documentation is correct. The nurse is responsible for assuring documentation by APs if delegated tasks are clear and accurate. Monitoring and follow-up is correct. The delegating nurse must provide supervision by following up and monitoring the AP's performance. Transferring responsibility is incorrect. Responsibility for the care of the client is not transferred from the LPN. There is a risk of liability with delegation related to the AP's skills and experience, type of task, regulations and Nurse Practice Acts, and policies and procedures. Receiving and providing feedback is correct. This is part of providing direction and understandable expectations regarding the activity or activities to be performed. Evaluating performance is correct. It is the nurse's responsibility to evaluate the AP's performance to assure the proper completion of delegated tasks. Assuring clear documentation is correct. The nurse is responsible for assuring documentation by APs if delegated tasks are clear and accurate.

Monitoring and follow-up Transferring responsibility Receiving and providing feedback

Evaluating performance Assuring clear documentation18

Which of the following tasks can a licensed practical nurse (LPN) delegate to assistivepersonnel (AP)?

It is within the scope of practice for an AP to measure Foley catheter output.State laws and organizational policies assign the responsibility of applying client restraints tonurses.Administering client medications is a nursing action. This task requires special training and knowledge.This task is part of the nursing process and requires special knowledge. Measuring urine output from a Foley catheter Putting restraints on a combative client Applying prescribed medicated cream to a skin infection Monitoring a client's nutritional status19

A client in the emergency department has died from injuries sustained from a motor vehicle crash. Which of the following is the most important action for the nurse at this time?

In most states, a qualified healthcare provider must ask all clients over 18 years of ageabout organ donation at the time of admission. If the answer is "yes," a copy of the documentshould be placed in the client's medical record.This is helpful for the family; however, it is not themost important intervention at this time.The family must be approached about organ donationfirst by the nurse and healthcare team in the department.It is too soon to seek closure.

Reviewing the medical record for information regarding organ donation Helping the family prepare for funeral home arrangements Calling the nearest organ procurement agency Introducing the police officer at the accident scene to facilitate closure20

A nurse has received report from the night shift nurse. Which of the following clients should the nurse monitor first?

This client needs to have level of pain monitored, but no immediate action isindicated.This client needs to be monitored, but no immediate action is required.This client isthe highest priority due to the symptoms of active gastrointestinal bleed. The client's status could worsen quickly, and immediate intervention is required.This client needs wound monitoring, but no immediate action is anticipated. A client with frequent reports of back pain for the past 24 hr A client who is scheduled for a cardiac catheterization this morning A client who vomited bright red emesis at 0600 A client who has a stage II pressure ulcer on the sacral area

21

A licensed practical nurse (LPN) tells a client, "If you don't stop pushing that call bell, I'm going to disconnect it." Shortly after, the client uses the call bell for the fourth time in an hour, and the

LPN disconnects the call bell from the wall in the client's room. Which of the following types oftort has the LPN committed?

Battery is intentional touching.False imprisonment is restraining a person against his orher consent.Invasion of privacy is unwanted intrusion into a client's private affairs.Assault is a

verbal threat or threatening gesture, but no physical contact occurs. It produces fear andapprehension. Battery False imprisonment Invasion of privacy Assault 22

A couple was filmed by a television crew while attending an in vitro fertilization program. They had been assured there would be no publicity. The couple files a lawsuit after the newscast. Which of the following legal terms best describes this situation?

Negligence is conduct that falls below the standard of care. The standard of care is established by law for the protection of others against an unreasonably great risk ofharm.Defamation is the publication of false statements that result in damage to a person'sreputation.The tort of invasion of privacy protects the client's rights to be free from unwantedintrusion into his or her private affairs.Battery is intentional touching without consent. Negligence Defamation of character Invasion of privacy Battery23

A nurse notices a decline in a colleague's work performance, which has resulted in unsafe clientsituations. When inconsistencies in the narcotic count are found, the nurse suspects that

thecolleague has a substance abuse problem. Which of the following is the most appropriateaction?

The nurse should be confronted privately by the manager.If the nurse is chemically impaired, she needs professional help, and client safety is at risk while the nurse is stillpracticing.The unit manager will need as much evidence as possible in order to address theproblem effectively.The nurse should be confronted, even if there are only suspicions. Immediately confront the nurse with one witness or more present. Try and protect the colleague from being fired. Collect as much evidence as possible to give to the unit manager. Do nothing until suspicions can be proven. 24

The role of the licensed practical nurse (LPN) in obtaining informed consent is to

The provider is responsible for explaining the purpose. The person obtaining consent would usually be the provider performing the procedure.The provider, usually a physician, has the full knowledge of therapeutic effects and side effects.The role of the nurse is to ensure that the client has given informed consent. A nurse can seek expressed consent from a client by witnessing a signature on a standard consent form.The provider, usually a physician, can offer options on alternative treatments. Other resource personnel may be needed to provideinformation to clients on the cost of therapy. explain the purpose of the treatment or procedure. discuss risks, benefits, and side effects.

witness the client's signature. relate the costs or alternatives. 25

In which of the following ways does a preceptorship program best benefit students?

The student receives more supervision by the hospital nurse, and the hospital nursecollaborates in the evaluation of the student with the faculty member.This is not necessarily true. Employment opportunity may depend on student performance during the preceptor period andjob availability on the assigned unit.The clinical hours requirement should remain the same.Thisfacilitates student learning and helps with hospital nursing and faculty shortages. Students receive less supervision by the course professor. Students have a greater opportunity for employment at the hospital aftergraduation. Students spend less time in the clinical setting. Students receive instruction by experienced nurses.

26

A nurse is assigned to a client in isolation. To best contain costs and use time effectively, the nurse should

To eliminate the need for multiple trips and repeated gowning and gloving when entering the room of a client placed in isolation, the nurse should attempt to complete as many actionsas possible in one trip.Scheduled visits are not necessary.An intercom should be used to answer the client's call light.The client should be informed of the need to keep traffic out of theroom and to limit the number of visits. However, it will be necessary to enter the client's room more than once a shift. cluster client care or provide complete care at one time.

schedule visits to the room every 2 hr. don a gown, mask, and gloves each time to answer the call light. visit the client once each shift to monitor physical and psychological needs. 27

A client falls in his room, and his roommate calls the nurse. The nurse finds the client on the bathroom floor with a knee abrasion. He states he has no injury. In addition to client demographics, date, time, vital signs, and place, which of the following documentations isappropriate?

The nurse did not document the abrasion. The nurse would not document the client statement related to time.Since the nurse did not observe the fall, she cannot document thereason. The nurse would note the area of abrasion.The nurse does not mention the abrasion. The nurse should not comment on negative findings because she did not witness the clienttripping due to the condition of the floor.This is objective documentation, including all of the specifics. "Client fell; no injury; found on floor. Roommate witnessed fall and states client tripped. Client states he is fine. He states he had the call light on for 30 min." "Client found on bathroom floor; abrasion present. Fell due to carelessness." "Client fell on floor; feels fine; no injuries. Loose tiles noted on bathroom floor." "Client found on bathroom floor in room 842; abrasion on left knee. Roommatepresent and observed the fall. Floor dry and free of obstacles." 28

A nurse is speaking with a client about smoking cessation. Which of the following is an appropriate motivational strategy to effect change?

Timing of discussions is important. Communication will not be motivational or effective while the client is anxious.The spouse can be helpful in supporting the client's decision to stopsmoking.All comments toward the client should be nonjudgmental and factual. This comment does not promote open communication between the nurse and the client.Motivation may begained by presenting factual information documenting the need for change. This is a rationaleempirical strategy of effecting change. Discuss smoking cessation while the client is experiencing anxiety from nicotinewithdrawal. Ask the spouse to leave the room before speaking with the client. Explain to the client that some people do not view smoking as socially acceptablebehavior. Present the client with the latest research on cancer and smoking. 29

A nurse is caring for a client who is receiving the second of 2 units of packed red blood cells. Which of the following tasks is appropriate to delegate to assistive personnel (AP)?

AP are not trained to assess for blood transfusion reactions. This task requires special knowledge and training.AP are trained to record client vital signs.AP are not trained to provide client instructions. This task requires special knowledge and training.AP are trained to measure urine but not to assess it. This task requires special knowledge and training. Monitor for skin reaction to blood transfusion. Record the client's vital signs during blood transfusion. Instruct the client to report any difficulty breathing. Assess urine output and color.

30

The introduction of a new bed system in a hospital has led to an increase in client falls. Which ofthe following actions should be taken for correction of the problem?

This would not be the best initial course of action and would not be done by unitpersonnel.The purpose of the risk management department is to identify and eliminate potentialrisk factors.It is not a realistic expectation for all clients to call for assistance.This action will notprevent falls since clients are still leaving the bed and ambulating. Notify the manufacturers of the bed system. Contact the facility risk management department. Require clients to call for assistance prior to ambulation. Place clients on bedrest with bathroom privileges.

31

Which of the following conflict resolution strategies is most effective for a nurse to use when dealing with a client who does not want to participate in therapy?

The actual problem is usually not solved in this win-lose situation.The conflict remains tooften re-emerge at another time.This approach is used when one person pursues what he orshe wants at the expense of others, leaving the loser angry and frustrated.Each party must giveup something. To avoid a lose-lose situation, both individuals must give up something of equal value.

Accommodation Avoidance Competition

Compromise32

When organizing client assignments, which of the following is within the scope of practice forlicensed practical nurses? (Select all that apply.)

Performing urinary catheterization is correct. Urinary catheterization is considered directbasic care, which is within the LPN scope of practice. Creating a nursing care plan is incorrect. The LPN may implement or modify a nursing care planbut may not develop one. Administering intramuscular medication is correct. Medication administration is considereddirect basic care, which is within the LPN scope of practice. Developing a client discharge plan is incorrect. It is not within the LPN scope of practice todevelop a client discharge plan. Performing sterile suctioning is correct. Sterile suctioning is considered direct basic care, whichis within the LPN scope of practice. Implementing a teaching plan is correct. The LPN may implement or modify a teaching plan but may not develop one.Performing urinary catheterization is correct. Urinary catheterization isconsidered direct basic care, which is within the LPN scope of practice. Creating a nursing care plan is incorrect. The LPN may implement or modify a nursing care planbut may not develop one. Administering intramuscular medication is correct. Medication administration is considereddirect basic care, which is within the LPN scope of practice. Developing a client discharge plan is incorrect. It is not within the LPN scope of practice todevelop a client discharge plan. Performing sterile suctioning is correct. Sterile suctioning is considered direct basic care, whichis within the LPN scope of practice. Implementing a teaching plan is correct. The LPN may implement or modify a teaching plan but

may not develop one. Performing urinary catheterization Creating a nursing care plan Administering intramuscular medication Developing a client discharge plan Performing sterile suctioning Implementing a teaching plan33

A client who has terminal cancer and is receiving chemotherapy and radiation therapy tells the nurse that she is only continuing treatment for her family's sake. Which of the following is thenurse's best response?

A meeting with the provider, family, and client will enable all options to be explored and treatment options to be set forth. The family can be better informed about the client's true feelings, and the client's wishes can be honored if all understand the situation. There can also be a compromise on the client and family's part, which would be appropriate.The social workeris not the best choice at this point. The social worker does not know the medical options and treatments available. The social worker would be a good choice after a decision is made about what care will continue.The chaplain is not the best choice at this point. The chaplain may not have the medical knowledge or know the options available to the client for an informed decisionto be made. However, the chaplain may be consulted later.The nurse, in this case, is ignoring

the problem and not facing the ethical issue at hand. In this case, the nurse needs to act as an advocate for the client. "Would you like to meet with your family and physician about this matter?" "Would you like to talk to a social worker about your situation?" "Would you like to speak to a chaplain about your thoughts and feelings?" "I know you are tired of this treatment, but you need to have hope and try and fight this." 34

A nurse is caring for a client who has uncontrolled diabetes mellitus as a result of noncompliance with a prescribed diabetic diet. When acting as a change agent, which of the following is the most appropriate action by the nurse to minimize the client's resistance to diet changes?

Telling the client that diet changes are easy and enjoyable is forcing the nurse's own feelings on the client, and those opinions may not be true for this client.Change is most effective when the client accepts responsibility and incorporates it into daily living.Open communication about dietary habits will give the nurse the opportunity to identify the client's barriers to diet changes.Greater compliance with change is achieved when the client is allowed to make changes gradually and in stages. Encourage the client that making diet changes is easy and enjoyable. Encourage the client's spouse to control the client's diet at home.

Encourage the client to speak openly about her dietary habits. Encourage the client to make the needed diet changes all at once. 35

A nurse enters a client's room and finds the client unresponsive on the floor beside the bed. Which of the following should be the nurse's first action?

The code blue button should not yet be activated. Other interventions should occur first.Before moving the client to the bed, the nurse should perform other interventions.The priority action is to determine if the client has a pulse and respirations.The rescue position is an inappropriate action for this client. Press the code blue button. Move the client to the bed. Check the client for a pulse and respirations. Roll the client into the rescue position.

36

Before administering a preoperative medication to a conscious client, the nurse should

JCAHO recommends two sources of client identification (not to include the room number).Proper identification must include two forms of verification (preferably including verbal verification of name).Verifying the name band with the medical record is correct, but having the client state his name is more plausible than having him state his medical record number.Two forms of identification must be verified. verify the client's name band with the medical record and verbally confirm the client's name. verify the client's name band with the medical record, and then place the name band on the front of the chart.

verify the client's name band with the medical record and ask the client toverbalize his medical record number. verbally confirm the client's name, and then place the name band on the front ofthe chart. 37

A nurse is contributing to the review of clinical pathways for clients on a unit. In reviewing the clinical pathways, which of the following indicates a need for nursing intervention?

Effective planning requires the nurse to monitor and evaluate outcome-based criteria. Aclient unable to give a return demonstration of a self-injection of insulin will need furtherreinforcement of client teaching until able to do so.It is not uncommon for a client to requirepremedication with an opioid analgesic prior to physical therapy in order to maximizeparticipation in therapy.A client who is 1 day postoperative may not yet have the return of bowel sounds or have passed flatus. Continued monitoring is indicated.A moist, pink, granulating wound bed is a good sign of wound healing. A client with diabetes mellitus who is fearful of self-injecting with insulin following diabetic teaching A client who was in a motor vehicle crash requiring premedication with an opioid analgesic prior to physical therapy A client who is 1 day postoperative with absent bowel sounds and is unable to pass flatus A client with a wound bed that is moist, pink, and has granulating tissue 38

A licensed practical nurse (LPN) on a medical-surgical unit is concerned with the staff-to-client ratio at the beginning of his assigned shift. The staffing level is below the level determined by the client classification tool. The LPN notifies the supervisor, who is unable to resolve the staffing issue. The LPN feels forced to accept an assignment that will not allow him to provide competent client care. Which of the following courses of action should the LPN take forprotection against liability?

This does not provide the LPN with protection against liability and may still constituteabandonment of some clients.Written protest notifying the nursing administrators that the nursesare being required to care for more clients than is reasonable shows that the nurses wereattempting to act reasonably. Knowledge of a potential problem by the administrators shifts some of the responsibility to the institution in the event that a client suffers injury due to inattention caused by a staffing shortage.This would constitute client abandonment andexacerbate the inadequate staffing situation for the shift, placing the clients at an even greaterrisk.The LPN should accept the assignment and provide the best care possible, but this alonedoes not provide protection against liability. Assume care for a number of clients that meets the nurse's comfort level. Notify the nursing administrators in writing and keep a copy. Do not accept the assignment and leave the unit. Accept the assignment and provide the best care possible. 39

When implementing new orders for a client with chronic obstructive pulmonary disease (COPD), which of the following should the nurse complete first? This task is the lowest priority.This task should be done, but it is not the highest priority.The correct oxygen rate is essential for clients with COPD. Ensuring proper oxygenation

of the client is the highest priority. Clients who are hypoxemic and have hypercarbia require lower oxygen rates, usually 1 to 2 L/min.This task is not the highest priority. Schedule pulmonary function tests. Sit the client up in a chair for 1 hr twice a day. Decrease oxygen flow rate to 2 L/min by nasal cannula. Collect sputum sample for culture and sensitivity. 40

A client diagnosed with chronic obstructive pulmonary disease has oxygen prescribed. Upon entering the client's room, the nurse observes the client smoking a cigarette. Which of the following responses should be the highest priority?

Extinguishing the cigarette will eliminate the threat of fire. Due to oxygen's flammable nature, smoking is prohibited near the use of oxygen. To prevent reoccurrence of this situation, client teaching should be reinforced after the threat of explosion is gone.Oxygen should not be interrupted unless there is an immediate threat.Alerting other personnel is time consuming and unnecessary if other appropriate steps are taken.Client teaching is important, but it is not the highest priority intervention at this time. Extinguishing the cigarette Reducing oxygen flow Alerting other healthcare personnel Instructing the client not to smoke

41

Which of the following agencies requires that healthcare facilities have an infection control plan?

NLN is an accrediting agency for educational institutions.JCAHO requires each healthcare facility to have an effective infection control plan to qualify for accreditation.Each state has a governing agency responsible for licensure and enacting individual state laws related to healthcare.The National Council is responsible for administering the licensure exam and validating entry-level competency. National League of Nursing (NLN) Joint Commission for Accreditation of Healthcare Organizations (JCAHO) State Board of Nursing National Council for Licensure Examination 42

A client admitted to the hospital says, "I feel that I am not being taken care of. People are talking about me and I want out of here." Which of the following is the most appropriate response by the nurse?

Client reports should be investigated and directed.The nurse should gather data about the client's perceptions and provide information.This does not address the client's concern.This dismisses and devalues the client's statements. Ignore the client's statements and redirect the communication. Speak with the client about information on the Patient's Bill of Rights. Move the client to another unit with different personnel. Explain to the client that he is receiving excellent care. 43

A hospital has implemented a peer evaluation system for its nursing staff. Evaluations are basedupon overall performance rather than isolated incidents. Which of the following evaluationstrategies ensures that the peer evaluation process is impartial and fair?

This ensures fairness and uniformity in the peer evaluation process.This reflects apunitive approach based upon isolated events.This is not necessary if the hospital uses anappropriate method of evaluating performance.Nurses are obligated to participate in the peerevaluation process. Use the same objective measurement tool for all nurses. Review personnel files for counseling forms and verbal and written warnings. Use the same nurses on peer evaluation committees. Excuse nurses from the peer evaluation process if they are not comfortable. 44

Two staff nurses request the same weekend off. The nurse manager posts the schedule assigning both to work. This is an example of which of the following conflict resolution strategies?

A win-win strategy results when both sides gain some benefit.A win-lose strategy results in one side gaining a benefit and the other side losing a benefit.Lose-lose strategies result whenboth sides lose some benefit, as is the case in this scenario.No one has won or lost, but the problem still exists. Win-win Win-lose

Lose-lose Win-draw 45

A licensed practical nurse receives a transfer from the intensive care unit, and the client is in stable condition. The client is post motor vehicle crash and had an open reduction and internal fixation. Two hours after being transferred to the unit, the client turns on the call light and reportssudden difficulty breathing and chest pain. Which of the following interventions should beperformed first?

The airway must be secured according to the ABC (airway, breathing, and circulation) principles.This is important, but not the first priority.Assessment data is important, but not the first priority.This is the lowest priority of the interventions listed. Assure patency of the airway. Obtain the client's vital signs. Assess heart and lung sounds. Monitor pain level and skin.

46

A licensed practical nurse (LPN) smells alcohol on the breath of an assistive personnel (AP) who is working on the unit during the night shift. The AP poses a potential hazard to client safety. The LPN should

To protect client safety, the AP should not be allowed to return to the work area.The APmay be impaired and cause injury to self or others if allowed to drive. Arrangements should be made to safely transport the chemically impaired AP home.This issue needs to be handled immediately so as not to compromise client safety.Although this may help long term, it does not address the situation at hand.

remove the AP from the unit and privately confront. direct the AP to leave the unit immediately and return home. refer the matter to the manager on the day shift. refer the AP to Alcoholics Anonymous. 47

A nurse is caring for a woman who is pregnant and thinking about putting her infant up for adoption. The nurse should recommend referral of the client to

Social workers assist clients and family members with psychosocial issues, including those associated with AIDS, legal needs, financial issues, and the adoptive process.Family planners assist families with planning and do not assist in adoptions.Clinical nurse specialists are clinical experts within a specific discipline.It would be inappropriate for the nurse to directly contact an adoption agency. a social worker. a family planner. a clinical nurse specialist. an adoption agency. 48

In the event of a bioterrorism attack, client data gathered during the response phase of an

emergency management plan are critical in identifying whether clients' symptoms are typical of an endemic disease or an unusual event. Which of the following community presentations is more typical of an unusual event and should raise concern?

A bioterrorism-related outbreak would have a rapidly increasing incidence of disease in a population that is normally healthy.In a bioterrorism attack, there is a great likelihood that there will be a lower incidence of exposure and disease in clients who have been indoors.In the event of a bioterrorism attack, clients presenting with particular signs and symptoms will most likely present in clusters from a single locale.A bioterrorism attack would most likely show large numbers of rapidly fatal cases. A gradual increase in the incidence of similar signs and symptoms in the community A lower incidence of disease among clients who have been indoors Presentation of clients with the same symptoms from a variety of locations A consistent level of fatalities among people seeking care 49

Which of the following clients would most benefit from a client care conference?

This client is meeting outcomes with pain controlled by using a PCA pump. This is not

the most appropriate answer.This client may benefit from nutrition counseling but is not the most appropriate client for a client care conference.This client is not meeting outcomes and will need care from multiple specialities. This client will also require planning to control cost and ensure the quality of care provided.This client has a chronic illness but does not have an acute illness that requires a client care conference. A client with sickle cell anemia whose pain is controlled with a patient-controlled analgesia (PCA) pump A client who is 4 days post gastric bypass surgery and is preparing to go home on a special diet A client who is 2 days post transplant surgery and is experiencing multi-organ failure

A client with kidney failure who has been on hemodialysis for 5 years 50

Which of the following strategies taken by an LPN will best help to minimize burnout amongassistive personnel (AP)?

Human behavior is frequently motivated by a goal an individual wants to achieve. Identification of AP values and goals can help to align personal and organizational values and goals. Fostering the attainment of personal AP goals can motivate the AP and contribute to the organizational goals.Human beings are motivated differently because of the uniqueness of eachindividual. Listening and providing support and encouragement can help to provide anenvironment that maximizes the development of each AP's potential.Overmanaging APs, failure to give APs adequate decision-making power, and making responsibilities too narrow can inhibit AP motivation.Feedback that is corrective in nature can often be demotivating. Positive reinforcement is a much more powerful motivator that is frequently underused. Positivereinforcement can result in increased productivity and benefit the organization by motivating theperson to accomplish again. Assist the APs to focus predominantly on the organizational values and goals. Listen, support, and encourage any discouraged APs while acting as a role model. Provide well-defined parameters of responsibility and AP assignments. Provide corrective feedback to quickly rectify the situation with any dissatisfiedAPs. 51

Which of the following strategies helps to build an effective nursing team?

A preceptor is an experienced nurse who provides knowledge, emotional support, and clarifications of role expectations.This would prevent a one-on-one relationship with each newgraduate and decrease the amount of assistance that each new graduate would receive. This would be frustrating for the preceptor and the new graduates.The new graduate is not fullyprepared to deal with the frustration that originates from conflicts with others. The new graduate needs education to prepare him for his professional role.The preceptors should be experiencednurses who have proven themselves to be role models to other nurses and who have beeneducated on mentoring. Implement a preceptor program for new graduates. Assign all new graduates to one preceptor who is the most experienced. Encourage the new graduates to work through issues of conflict on their own. Let last year's new graduates serve as preceptors for this year's new graduates. 52

A primary care provider writes a prescription for meperidine (Demerol) 300 mg to be given stat. The nurse receives the prescription and checks for appropriateness. The drug reference states that the normal dose is 100 mg. Which of the following actions should the nurse take next?

The nurse is accountable for administering the correct dose, and 300 mg is too high of adose.This is the nurse's responsibility, not the supervisor's.This does nothing to alleviate the

client's pain.The nurse administering the medication is accountable for knowing whatmedications are prescribed, their therapeutic and nontherapeutic effects, and the proper dosage. The nurse should question the prescription if the dose seems unusually low or high. Give the medication as prescribed. Report the situation to the nursing supervisor.

Document that the medication was not given. Contact the provider for clarification. 53

An assistive personnel (AP) approaches a licensed practical nurse (LPN) and says that he is unfamiliar with the transfer equipment in the client's room. Which of the following actions by the LPN would be most helpful to the AP?

This transfers the problem to another team member.Since the learner is only observing and not actively involved, he is not likely to learn the task at hand.This does not address the immediate problem.Active participation facilitates learning. When the learner is actively involved, his mind is more likely to be engaged. Learning is retained longer when it is put to immediate use and reinforced. Tell him to seek assistance from another AP. Take the AP to the client's room and have him watch while the LPN performs the transfer. Request an inservice by the staff education department. Demonstrate the procedure with the AP and have him return the demonstration. 54

A nurse is assigned to care for four clients. Which of the following clients should the nurse monitor first?

Monitoring airway and breathing are normally the priority concerns, but there is nothing about this client that suggests respiratory distress.This client is not the highest priority.The client scheduled for discharge should be stable and would be the lowest priority.Circulation concerns are the second priority after airway. This client would be the highest priority since none of the other clients has a compromised airway. A stable client 2 days post thoracotomy with a chest tube A client with diabetes scheduled for morning insulin A client scheduled for discharge to a long-term care facility A client with nonpalpable pedal pulses in the left foot 55

According to guidelines for medication-related documentation, which of the following orders requires clarification?

There are no errors in this order. It is important to place the zero before the decimal point.U (unit) is on the JCAHO official "Do Not Use List." "Unit" should be written out. S.Q. should be "Sub-Q," and q.d. should be written out as "daily."There are no errors in this order.

The standard method for the documentation of time in healthcare is 0900.There are no errors in this order. PRN is a standard abbreviation for as needed. Digoxin 0.125 mg by mouth daily Heparin 2000 u S.Q. q.d. Aspirin 325 mg orally at 0900 Morphine 2 to 4 mg IV PRN every 4 hr

56

A fire starts in a client's room in the wastebasket. The first action the nurse should take is to This does not remove the immediate danger and would not be the first action.This wouldnot be the first action.This would not be the first action.This is the first priority in this case.

According to the pneumonic RACE, the priorities for a fire are as follows: remove the client; signal the alarm or call for help; confine the fire by closing the doors or turning off the oxygen; and extinguish the fire with an extinguisher. seal off the room. call for help immediately. put out the fire with an extinguisher. remove the client from the room at once. 57

A new licensed practical nurse (LPN) is orienting on a hospital unit and is told that a team nursing approach is used on the unit. The nurse should understand that which of the following is a characteristic of this type of nursing approach?

Completion of tasks is not a team nursing approach.This is primary care nursing, in which one nurse assumes the total care for the clients to whom the nurse is assigned.Team nursing is when nursing personnel are led by a registered nurse to provide care for a group of clients.Case management is an interdisciplinary healthcare delivery system that promotes an appropriate use of hospital personnel and material resources. It does not necessarily dictate whether a primary care or team approach is used. A task-based approach is used to deliver care to the clients. Each nurse is responsible for the total care of her assigned clients. A registered nurse leads nursing personnel in providing care to a group of clients.

Case management is used to maximize healthcare delivery. 58

A nurse is caring for a group of clients on a medical-surgical unit. Which of the following client findings requires the most immediate action?

The blood pressure is slightly elevated and may require treatment, but it would not require immediate action.This finding is not abnormal and would not require immediate action.The client is in psychological distress but does not have a physical need that requires immediate action.Client airway and oxygenation are the highest priority and require immediate action. A client's blood pressure is 145/90 mm Hg, and his heart rate is 90 beats/min. A client's dressing at the thoracentesis site has 1 cm of bloody drainage. A client is crying and states, "I cannot go on like this much longer." A client has new crackles in the lung bases, and her oxygen saturation is 89%. 59

A client with a spinal cord injury is having difficulty coping with lifestyle changes. The client, who was employed as a construction worker, cannot return to work and is anxious about finances and mounting medical bills. As an advocate, the nurse's most appropriate action is to

recommend a client referral to

The unemployment office does not specialize in clients with disabilities.Referrals are written to address the client's needs. A social worker is the most appropriate referral to help the client find beneficial community resources.The billing department will not assist the client with a long-term solution to financial difficulties or unemployment.The client may benefit from a support group, but the most appropriate referral for assistance is vocational rehabilitation. an unemployment office. a social worker. the hospital billing department.

a support group. 60

A provider left orders for the nurse to call immediately with the client findings from an emergency test. Which of the following is the best way to increase the probability that the clientfindings will be communicated clearly?

The listener may not be able to understand all that is said if the results are spokenquickly. The nurse should speak slowly and clearly.The nurse should know what to say before speaking with the provider to minimize errors in relaying the message.It is not the responsibilityof the unit manager to report the results, and the unit manager may not be immediatelyavailable.Communication should be clear and concise and should not contain extraneous information. Relay the results in a quick and urgent manner. Read the whole report before speaking with the provider. Ask the unit manager to notify the provider of the emergent test results. Remind the provider of the client's history and assessment findings prior torelaying the test results.

ATI. MANAGEMENT 1.0

Which staffing schedule commonly leads to physical stress, complaints of fatigue, diminishedjob performance, and reduced role satisfaction?

Incorrect. Permanent shifts allow staff members to participate in selecting the shifts that best suits their personal life. Staff members can plan ahead and participate in social activities, which increases job satisfaction and reduces absenteeism and turnover.Correct. Alternating androtating shifts affect the health of nurses and the quality of their work. The shift of work schedules causes stress, and nurses complain of restlessness, anorexia, digestive problems, fatigue, and proneness to making errors.Incorrect. Block, or cyclical scheduling, involves thesame schedule repeatedly. This type of schedule is associated with reduced sick time and fatigue since nurses are not working too many consecutive days. Furthermore, block schedulinghelps establish stable work groups and decrease floating that promotes team spirit and jobsatisfaction.Incorrect. In the Baylor plan, nurses have the option to work a variety of 12hourshifts. Flexible staffing often results in increased employee satisfaction, reduced attrition, andimproved coverage of undesirable shifts. Although 12-hour shifts are often correlated with fatigue, total time off is increased and nurses report having more time to relax on their consecutive days off. Permanent shift Alternating shifts Block schedule Baylor plan2

The nurse manager is deciding who should take charge on the day shift. Duties will includeevaluating the performance of a staff member who has been performing below expectation.

Using McClelland's basic needs theory, the nurse manager should assign the person who has a need for:

Correct. People who have a high need for power often take leadership roles. They are primarily driven to exert influence over others and produce change.Incorrect. Individuals with ahigh need for affiliation may be less effective in a situation requiring conflict resolution. They want to be respected and desire to enhance morale and group cohesion. They avoid situations where decisions might alienate peers.Incorrect. People with high achievement needs desire tomake a contribution and to succeed. They are eager for responsibility, but function best when objectives and tasks are well defined. The person with high internal motivation may not be inclined to modify the behavior of others.Incorrect. Productivity is not identified specifically asone of the basic needs in McClelland's theory. power. affiliation. achievement. productivity. 3

A female is scheduled to have her first mammogram. Which type of health care service does she receive?

Correct. This is an example of primary care, which focuses on health promotion and

early detection.Incorrect. Secondary care facilities focus on diagnosis and treatment and includeacute care settings such as hospitals and emergency rooms.Incorrect. Tertiary care includes rehabilitation and long-term care. Examples of a tertiary agency are a hospice unit, nursinghome, and rehabilitation center.Incorrect. Elementary is a synonym for primary, but it is not a type of health care service. Primary Secondary Tertiary Elementary4

Which standard is an example of a Standard for Professional Performance document from theAmerican Nurses Association?

Incorrect. This is an example of a process standard that addresses methods of providing services. Protocols and procedures are examples of process standards.Incorrect. This is an example of an outcome standard that is used to evaluate client outcomes.Correct. Standards forprofessional performance focus on the nurse's responsibilities as a member of the discipline ofnursing. This is Standard I of the Standards of Professional Performance as written by the American Nurses Association.Incorrect. This represents a structural standard that supports the provision of health care and includes number of personnel, their educational backgrounds, andassigned responsibilities. Postoperative vital signs are to be checked every hour for four hours. By day three, uses overhead trapeze every two hours to strengthen muscles inupper limbs. The nurse systematically evaluates the quality and effectiveness of nursing practice.

All nurses administering chemotherapy agents must be certified. 5

A nurse is feeling overwhelmed at the beginning of the shift. How does the nurse proceed?

Incorrect. The best time management is to receive reports first, assess clients, then prioritize care.Incorrect. Review of medications is not always the first priority, although this activity is often one of the first things nurses do.Correct. Any work is best accomplished if some time is invested in planning. The first step in sound planning is gathering needed data, which includes reports from the previous shift and current client data. Shift report should generallyalways occur prior to any other activity (unless an emergency occurs) so that the nurse is armedwith current information during the interaction. Mistakes can be made if actions are not basedon current information.Incorrect. A shift report should precede any direct care of the client to avoid errors in decision making. This choice did not include an essential element of time management, which is planning. Assess clients, prioritize care, receive report Review medications, receive report, plan care Receive report, assess clients, plan an approach Conduct a brief assessment on all clients, receive report, begin working

Factors affecting the move toward a change in the way health care is delivered in the 21st century include:

Correct. Advances such as organ transplantation and computerized technology have

allowed people to live longer.Incorrect. Actually, there have been more problems with access to health care due in part to the number and type of providers, eligibility for government programs, and ability to get insurance and health care benefits.Incorrect. Increased specialization hasbecome evident over the past 30 years, owing to increases in research and knowledge. Specialization has led to fragmentation of care.Incorrect. Uneven distribution of heath care isevident in rural areas where there are insufficient professionals and services. Some states havean insufficient number of health care providers per capita as compared to other states. increased technological advances and longer life. better access to care but at a higher cost. reduced access to specialized services. improved distribution of services. 7

Which client care task is appropriate for the nurse to delegate to a newly hired nursingassistant?

Incorrect. A nursing assistant may be trained to properly and safely feed a dysphagic client; however, because the assistant is new, the RN would not have had adequate opportunity to assess the competency level of this individual.Incorrect. The Nurse Practice Act stipulates that certain acts should NEVER be delegated to nursing assistants. These include assessment, evaluation, and any task requiring nursing judgment.Incorrect. It is inappropriate for any nursingassistant to assess for bladder distention, since nursing assessment should not bedelegated.Correct. Taking vital signs is within the job description of a nursing assistant; furthermore, the nurse assigned clients in stable condition to the new nursing assistant. Until thenurse can evaluate the competencies of the nursing assistant, this approach to delegation isappropriate. Feeding a dysphagic client

Evaluating the effects of an analgesic

Assessing urinary output and bladder distention Taking vital signs on stable post-operative clients 8

A manager decides to use travel budget money to send staff members to local workshops ratherthan to pay for one or two individuals to attend a national seminar. Which ethical or moral theorydoes this approach represent?

Correct. Utilitarianism is a consequence-based moral theory that encourages decisionmaking based on what provides the greatest good for the greatest number of people.Incorrect. Relationship-based theories emphasize commitment and caring to promote the common goodof the group rather than the individual.Incorrect. Principles-based theories are based on thebelief that decisions should be made because of a sense of duty, obligation, or a person's rights. For example, the individual who believes the client should be told the truth regarding a terminaldiagnosis is using a principles-based framework.Incorrect. Caring-based theories fall into thesame category as relationship-based theories where stress is placed on the need to maintainrelationships and promote the welfare of the group. Consequence-based Relationship-based Principles-based Caring-based9

An elderly client angrily tells the nurse to stop during the middle of the morning bath. The decision to continue bathing the client could result in which type of liability?

Incorrect. Assault is a threat to an individual that creates reasonable fear of being touched in an injurious manner by another person.Correct. This situation is an example of an intentional tort referred to as battery: actual touching of a person or their clothing withoutpermission.Incorrect. Negligence is an unintentional tort that involves below standard care thatplaces an individual at risk for harm.Incorrect. False imprisonment involves the unlawful restraintof a client (i.e., physical restraints), or detaining the person against his/her will. Assault Battery Negligence False imprisonment10

Efficiency of nursing care is the relationship between:

Incorrect. Safety and quality are components of effectiveness (doing the right thing).Correct. The efficiency with which nursing care, or any service, is provided is defined as the relationship between the outcomes (results of care) and the resources used to deliver clientcare. Efficiency is a measure of effectiveness in terms of both quality and cost.Incorrect. If nursing can validate that a given number of nursing care hours result in a given level of qualityand quantity of care, the ratio between required and actual staffing could be one measure ofnursing care output. However, the ratio of required to actual staffing is only one component of efficient care. The best definition of efficiency is the relationship between client outcome and theresources used to reach the outcome.Incorrect. Prioritization of care and time management aredimensions of quality nursing care; however, efficiency refers to the relationship between the outcomes and the resources when providing care. safety and quality.

outcomes and resources. required and actual staffing. prioritization and time management. 11

The primary care provider plans chemotherapy for an elderly client with end-stage cancer. Which activity does the primary care provider undertake when honoring the principle ofnonmaleficence?

Incorrect. Keeping the client's chart from open view protects his/herconfidentiality.Incorrect. The client who is informed about the possible complications associated with procedures or drug therapies is accepting some of the risk for related sequalae. This is an example of autonomy, rather than nonmaleficence.Correct. Chemotherapy is used for curative of palliative care. Nonmaleficence is duty to do no harm. The principle of nonmaleficence is evident when the nurse gives the right dosage via the right route to the right client at the righttime for the right reason.Incorrect. Realizing that he/she does not possess the knowledge tomanage drug therapy in this situation, the primary care provider is honoring beneficence (doinggood), or perhaps simply adhering to the principle of fidelity, or holding to the promises of the medical oath. Keeping the client's chart from open view Informing the client of the possible side effects Administering the correct chemotherapy dosage Consulting another physician to manage the client's chemotherapy12

Responsibilities of the change agent during the refreezing phase include:

Correct. The refreezing phase of change involves integration and stabilization of the change. The key elements in this final phase are positive feedback and encouragement to reinforce the group members and their adaptive efforts.Incorrect. The identification of strategies to facilitate change occurs in the second phase of change, referred to as the movementphase.Incorrect. Examining alternatives for change occurs during the movement phase ofchange.Incorrect. During the unfreezing phase, the change agent must assess the extent ofinterest and nature of motivation of those involved in the proposed change. reinforcing adaptive efforts. identifying strategies for change. examining alternatives for change. assessing interest in and motivation to change. 13

A major advantage of centralized scheduling is:

Correct. A major benefit of centralized scheduling is fairness to employees through objective, unbiased, and consistent application of policies and better use of resources. This typeof scheduling relieves managers from this time-consuming task so that they are free to performother duties.Incorrect. There is no mention of reduced absenteeism as an advantage of centralized scheduling. In fact, absenteeism may indeed increase if scheduling is notindividualized or personalized to the employee.Incorrect. The lack of individualized scheduling

of staff members is a primary complaint with the centralized scheduling system.Incorrect. Selfscheduling is the only type of scheduling practice that has been associated with increased jobsatisfaction, improved team spirit, and decreased absenteeism. fairness.

reduced absenteeism. individualized staffing. increased job satisfaction. 14

One of the common pitfalls in progressive discipline is:

Incorrect. Inadequate documentation is more likely the problem. Conversations anddiscussions are sometimes not documented at all, and it becomes difficult to recall exact timesand dates later.Incorrect. Although there are times when employees may disagree with an evaluation, the greater tendency in progressive discipline is for the employee to not fullyunderstand what is expected.Correct. A behavior should not be allowed to go on until the manager overreacts; however, this is a common error because managers are busy, and there is also a tendency to wait and see if the behavior continues.Incorrect. A common pitfall in progressive discipline is for the manager to write an overly positive terminal evaluation. This would discredit any previous evaluative statements. too much documentation. the employee disagrees with the evaluation. intervention may not have occurred early enough. the unit manager writes an overly critical terminal evaluation. 15

In a hospital with a vertical line of authority, a nurse on the critical care unit with a suggested

change to the unit's standard policy and procedure should direct recommendations to the:

Incorrect. The chief nursing administrator represents upper management. In the traditional vertical structure, it is considered improper to bypass any level in the chain ofcommand.Incorrect. The critical care pharmacist would certainly be involved if the nurse's concern(s) involved drug administration. However, the nurse's concerns should be initially takento the immediate supervisor in an organization with a vertical structure.Incorrect. In a tall orvertical structure, the director of client care services is at the top of the organization. Although those at the top of a centralized organization are primary decision makers, communicationfollows a formal pathway and should not skip or "loop" any levels in the chain ofcommand.Correct. A vertical structure encompasses a chain of command or line authority; therefore, nurses on any given unit report to their immediate supervisor or nurse manager. chief nursing administrator. critical care pharmacist. director of client services. nurse manager. 16

Administrative staff is evaluating the institution's budget at the end of the first quarter. Which report tells the most about the status of the capital budget?

Incorrect. A nursing audit is one method used to monitor nursing care. It involves examining hospital records of a large number of cases and comparing the course of care acrosscases.Incorrect. The payroll is a list of salaried employees. Although the payroll provides

information about salary-related costs, it does not give any data related to the actual orproposed budget.Correct. The budget variance is a report on the difference between the planned and actual income and/or expenses.Incorrect. A productivity report would include information regarding supplies used, hours of labor, dollars spent on equipment, number and

type of personnel, and case mix. However, it would not provide comparative data on the status of the actual versus proposed budget. Nursing audit Payroll record Budget variance Productivity report17

The unit manager interviews the employee regarding poor work performance. Which statementis an example of the best way to deliver performance feedback?

Incorrect. It is important to avoid vague statements, either positive or negative. Ambiguity is more likely to inhibit communication rather than enhance it, and the employee is left unsureabout the exact nature of the evaluation.Incorrect. Generalities should be avoided during theevaluation process. Explicit examples of work performance should be given.Correct. Direct, straightforward examples help to communicate the exact nature of the problem. This avoids anymisunderstandings and facilitates goal setting.Incorrect. This is another generality that does not specify the exact nature of the problem. This comment does not adequately convey positive aspects of the employee's performance, and the exact problem with staff meetings is unclear. "Your assessment skills could improve." "The way you prioritize care is ineffective." "This pay period you reported late to work six times." "You are doing fine except for participation in staff meetings." 18

A member of a preferred provider organization (PPO) is billed for $500 of the total cost of the MRI taken at the first of the year. This represents the person's:

Incorrect. A copayment is a fixed amount that the enrollee pays for at each point of service (i.e., office visit, prescription).Correct. A deductible is the fixed amount determined by the healthcare policy that the member must pay per year before the insurer reimburses theprovider.Incorrect. Capitation payments are flat fees paid to the provider for every person enrolled in the health plan whether or not services are used.Incorrect. A premium is the rate thatis paid by an individual or employer for insurance coverage. copayment. deductible. capitation. premium. 19

Based on an understanding of Good Samaritan acts, a nurse who comes upon an accidentscene should:

Correct. To encourage people to offer assistance in emergency situations, Good Samaritan acts have been written to release care providers from legal liability for injuries causedunder such circumstances. However, nurses and other health professionals are still held to a level of care that would have been provided by any other nurse. Therefore, principles of

emergency care should be followed. This includes providing first aid, remaining with the client, and waiting for help to arrive.Incorrect. Good Samaritan acts encourage people to render firstaid by releasing them from legal liability for injuries caused as a result of emergency aid. Therefore, a health care professional such as a nurse (who is prepared to render emergencycare) is ethically obliged to do more than just stop and wait for someone else to come.Incorrect.

Regardless of the Good Samaritan act and its legal protection, no one should transport an injured individual other than emergency personnel.Incorrect. Not providing first aid could be viewed as a departure from the normal standard of care in terms of what any other nurse would do in the same situation. provide first aid and wait for help. stop and wait for emergency personnel. transport the injured to the nearest hospital. provide emotional support and send someone else to call 911. 20

A 19-year-old woman who is supporting herself working a part-time, minimum-wage job seeks prenatal care. Which payment source will cover the majority of her health care expenses?

Correct. Medicaid is a federally subsidized program to help provide health care benefits for adults and children with low income. The Medicaid program covers health care expenses for women for up to 60 days postpartum if they meet the income criteria. It is the primary funding source for most long-term care in the United States.Incorrect. Medicare is a federally subsidized program for all individuals over 65 years of age and those with qualifying disabilities. Medicare Part A covers inpatient hospital services, limited and extended care costs, assisted living expenses, and home health care costs.Incorrect. Medicare is a federally subsidized program for all individuals over 65 years of age and those with qualifying disabilities. Medicare Part B provides partial coverage of physician services and some outpatient services, and medical

supplies not covered by Medicare Part A.Incorrect. If the client has Medicaid, she should not be required to pay for health services out of pocket. Medicaid Medicare Part A Medicare Part B Out of pocket

21

Which activity represents the induction process of indoctrinating a new employee?

Incorrect. CPR certification or verification is an activity performed in the orientation phase of indoctrination. Other activities include review of unit policies, scheduling, staffing, and benefit plan.Incorrect. Induction provides information about the organization, while orientation provides more specific information about the position.Incorrect. Using role models, preceptors, and mentors can accomplish clarification of role expectations. This is part of the socialization process of indoctrination.Correct. Induction is the first phase of indoctrination and includes all activities that educate the employee about the organization. CPR verification Reviewing treatment policies Assigning a preceptor or mentor

Reviewing the employee handbook 22

Which client can legally give informed consent?

Incorrect. An individual must be over 18 years old to give informed consent. In most states, a parent or legal guardian must give consent before treatment can be given to a minor.Incorrect. To give informed consent, the individual must be of legal age (18 years). Had the person been married, she would be legally permitted to provide consent. The client's

language should not be an interfering factor. If given enough information (i.e., someone interpreting or reading the permit), the person is considered a competent adult and can makehealth decisions.Correct. A competent adult is defined as an individual who is over the age of 18years, conscious, and oriented. Although the client's hand tremors may interfere with signing a permit, they do not affect the individual's mental ability to give permission for treatment.Incorrect. A competent adult is defined as one who is conscious and oriented. A client who is confused or sedated is not considered functionally competent. A 13-year-old child at summer camp. A 17-year-old single female who does not speak English. A 58-year-old female who is oriented but has hand tremors. A 72-year-old male who is sedated following morphine administration. 23

Primary nursing is based on the assumption that:

Incorrect. A main focus of team nursing is the care conference that is designed to develop and revise the plan of care using the total group for input.Incorrect. The assumption underlying team nursing is that goals can be better achieved through group action.Correct. Primary nursing is based on the philosophy that clients should be the focus of professionalnurses. The primary nurse is responsible for the client's care from admission through discharge, thereby improving continuity of care.Incorrect. Functional nursing emphasizes efficiency (costreduction) and division of labor. care conferences facilitate problem solving. care improves when given by a team.

continuity of care is enhanced. cost of care decreases. 24

Gross negligence is clearly evident in the situation whereby the nurse:

Correct. Gross negligence involves extreme lack of knowledge, skill, or decision makingthat the person clearly should have known would be harmful. In addition, this constitutesnegligence in that the action was below standard practice expected of an ordinary, reasonable, and prudent nurse.Incorrect. This represents invasion of privacy because the nurse inappropriately released confidential client information. Written client permission is required to release such information.Incorrect. This is an example of assault, which is threatening to touch a client without their permission or without justification.Incorrect. This is an example of battery because the nurse touched the client without the client's permission. leaves a diabetic client with a heating pad without instruction. releases the client's chart/medical record to the client's clergyman. says he/she will be forced to give an injection if the client refuses an oral drug. administers a sedative by intramuscular injection without client consent. 25

The nursing staff of a general medical unit is opposed to changing to a total client care model, stating that the quality of client and family care will be diminished. Which response by the nursemanager is an important early strategy in reducing resistance to change in the delivery of care?

Incorrect. Emphasizing former failures may only escalate feelings of resistance sincepast experiences with change had a negative effect.Incorrect. Although it is true that nursing responsibilities may change under a new care system, the response by the unit manageraccents the possibility of nurses losing valued time doing certain activities; furthermore, the

response suggests that the group should accept the change based on the unit manager'sperception of this nursing care approach.Incorrect. In the early stages of the change process, itis best to attempt to reduce the forces that are bringing about resistance. By emphasizing thechange in staff mix, nurses may become more resistant, fearing their roles will be altered.Correct. A key intervention in dealing with resistance to change is to first determine why the change is being resisted. This also conveys trust and enhances the change process when the group can see the leader is sensitive to their needs. "Our former approach did not work, but I'm sure total client care can work if weare committed." "I realize nursing responsibilities will increase, but clients will benefit in the longrun." "The ratio of nurses to nursing assistants will change, but the clients will benefit." "I am interested in knowing some of the reasons you feel this way."

26

Which of the following is an example of a decentralized decision-making process?

Correct. Decentralized decision making occurs among many people, including thoseclosest to the situation.Incorrect. This represents centralized decision making, in which a few individuals in central positions hold most of the decision-making authority.Incorrect. Those at thetop have a great deal of control over primary decision-making in the centralizedorganization.Incorrect. In a centralized organization, decisions about equipment priorities, staffing patterns, and policies are made at top levels of authority. Unit managers staff their own units. Staff development provides new employee orientation. Administration makes primary decisions regarding nursing policy.

The vice president of client services determines hospital equipment needs. 27

A unit manager has received two client complaints regarding an employee and the care he/she received. The first thing the manager does is:

Incorrect. Employee suspension is initiated only after both verbal and writtenadmonishment.Correct. The ramifications of disciplinary action are very serious; therefore, managers must carefully collect the facts and check allegations. A general rule for effective discipline is to get the facts before acting.Incorrect. Although an employee's work record may bea component of the assessment, investigating the alleged incident(s) should be done first toavoid any bias in data gathering.Incorrect. Before any type of disciplinary plan can bedeveloped, the problem must be validated. suspend the employee. investigate the incident(s). look up the employee's past record. develop a plan for disciplinary action. 28

Which strategy in conflict management is being used when one individual gives in to the wishesof another to preserve harmony in the relationship?

Incorrect. Avoidance occurs when the person chooses not to address the problem. For example, a person may feel that the conflict will take care of itself and choose to ignoreit.Incorrect. Compromise or negotiation involves a balancing of the gains and loss on both sides

of the issue. We have all used compromise even in childhood when we negotiated to play with aparticular toy, perhaps offering the playmate a different toy.Incorrect. Competition in a conflict involves working toward a specific goal or desire to the exclusion of other ideas or goals. Forexample, a family cannot decide how to spend the evening and each has a different wish. The teenager approaches each family member and promotes the idea of going to the movies byconvincing other family members of his preference.Correct. Accommodation is a strategy whereby one person gives in to the desires of the other person in order to prevent discomfort orperhaps, the loss of friendship. Avoidance Compromise Competition Accommodation 29

Which element must be present to prove that nursing malpractice has occurred? The nurse:

Correct. One of the four elements that must be present to prove malpractice is that thenurse has acted outside a standard of care that would be expected of any other nurse in thesame situation.Incorrect. The plaintiff must prove that the nurse's actions, whether intended or not, actually caused injury or harm. Malpractice is actually considered an unintentional tort. Ifthe act were executed on purpose, it would be considered an intentional tort such as assault orbattery.Incorrect. Although a history of negligence would be detrimental to the nurse, it is not a defining element in malpractice.Incorrect. Malpractice is not determined by whether the nursecarries liability insurance. breached standards of care. harmed the client on purpose.

has a former history of negligence. did not carry professional liability insurance. 30

During a staff meeting, the nurse manager informs the staff that their decision regarding the uniform policy must be submitted the following day. This approach is an example of which leadership style?

Incorrect. In an autocratic or authoritarian style of leadership, the leader believes thegroup is incapable of making decisions and, therefore, assumes decision making for the group. In this case, the manager would inform the group of the new uniform policy after it wasrevised.Correct. The nurse manager assumed the group to be motivated, self-regulated, and therefore, capable of making the decision without any further direction. This is called a laissezfaire or nondirective, permissive style of leadership.Incorrect. A bureaucratic leader does not rely on, or have faith in self or others to make decisions, but instead depends on rules andpolicies of the organization to direct the decision. The manager in this example would develop the unit policy based on the hospital's uniform policy.Incorrect. A manager that uses a transactional style of leadership focuses on achieving the goal and promoting loyalty throughincentives. For example, the manager would offer a gift certificate to a uniform shop for the group who submits the best uniform proposal. Autocratic Laissez-faire Bureaucratic

Transactional

31

A client is receiving chemotherapy for breast cancer. An example of fixed costs in this situation is the:

Correct. Fixed costs do not vary based on volume. Mortgage payments or rental fees are examples of fixed costs.Incorrect. Chemotherapy drugs represent a variable cost that changes based on volume or usage.Incorrect. A mammogram represents a direct cost, a cost that is clearly identifiable as a product or service.Incorrect. Such items as an emesis basin and washcloths represent indirect costs. These are hidden costs. Other examples include housekeeping services and office supplies. office rental fee. chemotherapy. mammogram. emesis basin and washcloths. 32

The validity of an interview can be increased by using:

Correct. Validity means that the tool, in this case an interview, assesses or measures what it is supposed to: productivity as an employee. Because interviews are influenced by subjective judgments, using a team approach can increase validity. Having more than one individual interview the applicant decreases bias that is normally present.Incorrect. Studies show that structured interviews have a higher validity than unstructured interviews.Incorrect. Because each job has different requirements, interviews must be individualized to fit the position.Incorrect. Using the same interview for all individuals applying for the same job can increase the validity and reliability of the interview. a team approach. unstructured interviews. an identical interview for every position. a different interview for applicants applying for the same job. 33

Calculate the number of nursing care hours per client using the following information: Today's census is 16. The number of nursing care hours needed for each acuity level and the number of clients at each level is as follows: Category I = 2.3 hours (six clients) Category II = 2.9 hours (three clients) Category III = 3.4 hours (four clients) Category IV = 4.6 hours (three clients)

Incorrect. Double check math calculations: 2.3 hours times six clients = 13.8 hours 2.9 hours times three clients = 8.7 hours 3.4 hours times four clients = 13.6 hours 4.6 hours times three clients = 13.8 hours. The number of care hours equals 49.9 hours. Divide 49.9 by 16 clients = 3.1 hours per client.Incorrect. Double check math calculations: 2.3 hours times six clients = 13.8 hours 2.9 hours times three clients = 8.7 hours 3.4 hours times four clients = 13.6 hours 4.6 hours times three clients = 13.8 hours. The number of care hours equals 49.9 hours. Divide 49.9 by 16 clients = 3.1 hours per client.Correct. The total number of nursing care hours equals 49.9 hours. This was determined by multiplying each category (acuity level) by the number of clients falling into that category, and adding all of the hours from the four categories together. Then, the total number of nursing care hours (49.9) is divided by the number of clients (16), which equals 3.1 hours.Incorrect. Double check math calculations: 2.3 hours times six

clients = 13.8 hours 2.9 hours times three clients = 8.7 hours 3.4 hours times four 4.2 hclients =

13.6 hours 4.6 hours times three clients = 13.8 hours. The number of care hours equals 49.9 hours. Divide 49.9 by 16 clients = 3.1 hours per client. 1.8 hours 2.0 hours 3.1 hours 4.2 hours 34 A major difference between quality assurance and quality improvement programs is that the latter:

Incorrect. Quality improvement programs focus on just the opposite: importance isplaced on evaluating and improving care through the system versus faulting a person orgroup.Incorrect. Quality improvement programs endeavor to enhance quality by design(prevention), rather than emphasizing quality by inspection (correction).Incorrect. Qualityassurance programs of the past typically focused on problem detection. Quality improvementprograms of the present focus on problem prevention.Correct. Quality improvement includes allactivities of quality assurance, but moves beyond to include programs aimed at improvingquality of health care (not just correcting the error). In this way, the approach is proactive versusreactive. focuses on improving individual performance versus work processes. improves quality by inspection and correction. concentrates on problem detection. emphasizes a proactive approach. 35

A manager who utilizes Theory Y philosophy will:

Correct. McGregor believes that broadening individual responsibility motivates people tohigher levels and to be more productive.Incorrect. A manager who manages using reward for desirable behavior is supporting Herzberg's two-factor theory. The two factors that Herzberg identified as the highest motivators were recognition and achievement. Using this philosophy, the manager rewards extra efforts to reinforce positive behavior.Incorrect. The manager who leads by helping staff set goals supports the philosophy of McClelland, who believes a major motivator is achievement. Achievement-motivated individuals set moderately difficult but attainable goals. They are most interested in the accomplishment itself.Incorrect. The manager who shows genuine interest in staff development supports the philosophy of Maslow and McClelland, who focus on the importance of affiliation or acceptance as a primary motivator. Motivation is a function of the need to belong and the feeling that someone cares. delegate responsibilities. reward desirable behavior. facilitate staff members in setting attainable goals. show genuine interest in the developmental needs of staff members.

36

Nurse managers from the Emergency Department and Intensive Care units directly report to thenursing supervisor of the critical care area. The critical care supervisor represents which level ofmanagement?

Incorrect. A first-level manager works with non-managerial staff (i.e., nurses) and supervises the activities of a specific group. A unit manager is one example.Correct. Middlelevel managers supervise the work of several first-level managers (i.e., the managers of theEmergency Department and Intensive Care units).Incorrect. Although one could argue that the term intermediate is similar to middle, this is not cited in the literature in relation to levels ofmanagement.Incorrect. Upper-level managers (i.e., directors of nursing) are executives of theorganization who are responsible for administrative tasks, such as strategic planning. First Middle Intermediate Upper37

A nursing unit is changing from team nursing to total client care. One of the driving forces for change might be:

Incorrect. Staff is resistant because of a perceived increase in workload when providing total client care.Incorrect. Fear of losing control over client care is a restraining force. In otherwords, staff may be resistant to changing because of this fear.Correct. An anticipated benefit (driving force) of the change is a gain in time for client teaching.Incorrect. Comfort with thepresent system is also a force that pushes against the anticipated change. perceived potential increase in the workload. fear of losing control over client care. more time for client teaching. comfort with the present system.

38

An alert, oriented 22-year-old client is scheduled for an open reduction of the right femur. He voluntarily signed the operative permit. Based on this data, to determine if informed consentwas obtained, the nurse should validate the client's:

Correct. Informed consent is obtained when the client fully understands what is beingconsented to. Therefore, the nurse should question the client as to his knowledge of the procedure, alternative options, side effects, and potential complications.Incorrect. The scenario stated the client was alert. There is no reason to further assess or question the competency of this individual; furthermore, the law presumes that every adult is mentally competent untiljudged otherwise.Incorrect. Informed consent must be given voluntarily, without coercion. The scenario stated that the client voluntarily signed the operative permit.Incorrect. The client in this scenario is of legal age (age 18) to give informed consent. knowledge of the procedure. mental competency. free will. age. 39

When a client's plan of care is to be initiated within 24 hours of admission, which type ofstandard is represented?

Incorrect. Program standards guide the development and implementation of programs tomeet client needs (i.e., cancer support program).Correct. This is an example of a process standard that addresses methods of providing services. Protocols and procedures are examplesof process standards.Incorrect. An outcome standard is used to evaluate client

outcomes.Incorrect. A structural standard supports the provision of health care and includes number of personnel, their educational backgrounds, and assigned responsibilities. Program Process Outcome Structural 40

Which is an appropriate question to ask during the planning phase of the budgetary processprior to allocation of funds?

Correct. The preliminary or planning phase of budget preparation focuses on information gathering such as environmental assessment, setting operational objectives, and statingmission and goals.Incorrect. Actual budget preparation includes needs prioritization and asking the question "is the item necessary to support quality care and client safety?"Incorrect. Controlprocedures are generally in place in every organization to monitor or evaluate the budget. The focus is on determining if there are any variances between budgeted expenses and actualexpenditures.Incorrect. During actual budget preparation, meetings often include a review of thebudget(s) of the prior year and the relationship of one budget year to the next. "What are the strengths and weaknesses of the organization?" "Are resources allocated according to priorities?" "What were the actual personnel expenses?" "What do past budgets suggest?"

41

A new employee selects a primary care provider and realizes pre-authorization for any specialty care is required. This individual has which type of health plan?

Correct. To reduce costs, HMOs pay for specialty care only if the primary care physician has made a referral to the specialist.Incorrect. Retrospective payment plans are the typical feefor-service or private insurance health plans that offer a wide range of services without provider restrictions.Incorrect. Private insurance is also referred to as third-party reimbursement, in whichthe insurance company pays a fee for each service provided. There are no restrictions on the doctor or specialty services the client may select.Incorrect. Indemnity insurance is another termfor traditional private insurance, in which the client can select from a wide variety of careproviders. Health maintenance organization (HMO) Retrospective payment plan Private health insurance Indemnity insurance42

With regard to delegation, the ultimate authority by which nursing performance is judged is the:

Correct. The Nurse Practice Act is the ultimate authority by which a nurse's performance is judged.Incorrect. It is the state's Nurse Practice Act that determines the appropriateness of

nursing performance. Although the state board of nursing is responsible for discipline, its members do not have authority to judge nursing performance.Incorrect. The American Nurses Association is a professional organization that is involved in establishing a Code of Ethics andstandards of practice for nursing; however, they do not have authority to judge the performance

of individual nurses.Incorrect. Employee policies cannot override the laws and rules of nursing set by the Nurse Practice Act. Nurse Practice Act. State Board of Nursing. American Nurses Association. hospital's policies and procedures. 43

Shared governance is built on a foundation of:

Correct. Shared governance is built on a foundation of primary nursing, peer review, clinical advancement, and shared decision-making between staff and management.Incorrect. A bureaucracy represents the opposite of shared governance. Characteristics of bureaucracies include a well-defined hierarchy of authority with systematic control of superiors over subordinates.Incorrect. Shared governance is based on the concept of democratic forums and group work versus an authoritarian style of leadership.Incorrect. Although participatory management is the basis for shared governance, they are not the same. Shared governance gives nurses increased authority and control over their own nursing practice; whereas participative management means that someone (i.e., management) has ultimate control over the decision-making process. peer review. bureaucracy.

autocratic leadership. participative decision-making. 44

A key indicator of efficient nursing care is the client got better:

Incorrect. The client improving in 24 hours does not mean care was delivered in an efficient, cost-effective manner.Correct. Effectiveness of nursing care relates to its quality, safety, and appropriateness; efficiency of care relates to methods used to produce a service resulting in the maximum outcome (using the right resources for the job). Efficiency considers both client outcome and cost.Incorrect. Although client recovery without complications means less cost, it does not mean that the actual nursing care was delivered in an efficient, costeffective manner. While recovery without complications infers effectiveness of care, it does not ensure efficiency of care.Incorrect. The effectiveness of care is reflected in such measures as quality improvement data and evaluation of nursing care. Effective care means care was appropriate and of a quality nature. It does not mean that care was delivered in a cost-effective manner, however. within 24 hours. with minimal cost. without complications. with an appropriate plan of care. 45

The autocratic or authoritarian style of leadership is most effective in which situation?

Correct. An autocratic style of leadership is essential in situations such as emergencies, whereby external factors motivate the group, and the leader assumes the decision-making role.Incorrect. A staff meeting might require a variety of leadership styles; however, the most effective in this situation is the democratic style, which assumes participants are capable of making decisions and encourages group discussion. Transactional leadership may also be

effective in discussing work schedules because the leader may want to provide incentives to promote loyalty.Incorrect. The connective style of leadership is most appropriate in this situation since the leader's goal is to promote collaboration and teamwork between members of theplanning committee and the community.Incorrect. The laissez-faire, or shared leadership style, is most appropriate in groups where the leader assumes there are many leaders, and the groupis internally motivated and autonomous. Triage in the emergency room Staff meeting to discuss work schedules Planning committee for a community seminar Board meeting of a state's nurses association

46

When faced with a moral dilemma, nurses can refer to the nursing code of ethics for:

Incorrect. Codes of ethics are not legally enforceable.Correct. Nursing codes of ethicsserve as guidelines for ethical decision-making.Incorrect. Neither moral frameworks nor codesof ethics provide a guaranteed right answer to any given moral dilemma.Incorrect. A code of ethics is general in nature and does not provide specific standards on how to respond to anygiven situation. legal backing. ethical guidelines. the right ethical decision. standards regarding specific situations.

47

Rightsizing the workforce in health care emphasizes which goal?

Incorrect. Although downsizing may mean reducing the number of professional nursing staff, this is not the primary goal.Incorrect. While the use of part-time employees may reduce cost because of fewer benefits, increasing part-time employee numbers is not the goal ofdownsizing.Correct. Rightsizing or downsizing focuses on how many staff members are required to do the job, emphasizing the minimum number to reduce cost.Incorrect. Rightsizingthe workforce does include the bottom line, which is cost containment; however, the chief goal of downsizing is to achieve quality care with the right mix of people. Quality care should not besacrificed for cost. Reduction in professional nursing positions Increased use of part-time employees Least number of staff to do the job Minimum cost expenditure48

A staff nurse voluntarily quits his job on the telemetry unit because he does not agree with the manager's leadership style. He is asked to complete an exit interview. Studies show that the nurse will most likely give what type of reason for his decision?

Correct. Studies have shown that employees are reluctant to be totally truthful in atermination or exit interview. They tend to soften their reasons, or make safe responses for fear of the negative impact honesty may have on future references from that employer.Incorrect. This is a possible way for the employee to avoid a negative evaluation during an exitinterview.Incorrect. It is unlikely, according to the literature, that the employee would be this direct.Incorrect. Although this would be a very informative and helpful comment during a

termination interview, it is unlikely that an employee would be this forthright for fear of a negative future reference. "I found a better opportunity." "My manager and I did not see eye to eye." "I need more autonomy and my unit manager is too autocratic." "I prefer participating in decision-making, but my manager dictates policy." 49

The charge nurse is assessing staffing needs for the day shift on a surgical unit with 36 clients. A critical factor for determining staffing is:

Incorrect. Although number of days after surgery affects the level of acuity, it is not a consistent predictor. For example, a client who is four days post colon resection may still require intense care, even though most clients in the same situation have a lower acuity.Correct. Client classification systems help to determine the number of client care hours using different client acuity levels or levels of nursing care intensity.Incorrect. Type of surgery may be a consideration in a client classification system, but is not a reliable predictor of acuity level. The actual acuity of each client is more essential for determining staffing needs than the type of surgery each client received.Incorrect. Using numbers of clients alone has proven to be an inaccurate method for determining nursing care assignments. time spent in post-operative care.

acuity of each client. type of surgery. client census. 50

The nurse manager uses management by objectives as an appraisal tool. This means that the manager:

Incorrect. Used as an appraisal tool, management by objective means the manager helps employees establish personal objectives and standards of performance, rather than a set plan for daily activities. It is an outcome-based management model.Incorrect. The underlying philosophy of management by objective is that individuals perform best in an environment that encourages participation.Correct. Management by objectives emphasizes individual achievement and measures performance in terms of expected results, rather than personality characteristics.Incorrect. Management by objectives focuses on expected results versus the activities themselves. For example, making a worksheet (activity) improves time management (expected result). establishes daily objectives. sets goals for each staff member. measures performance in terms of results. identifies specific activities to be accomplished each day.

51

A strategic plan is most appropriate for which situation?

Incorrect. This represents an operational plan that focuses on the day-to-day operation of the organization. There can be many operational plans within a long-term strategic plan.Incorrect. Again, this situation calls for an operational plan aimed at getting the job done; in this case, it involves client care.Incorrect. A specific problem such as a natural disaster is addressed with a contingency plan that focuses on being proactive.Correct. Strategic planning

focuses on long-range issues with the goal of improving effectiveness. Planning for a gradual change in the method of care delivery in a long-term care facility involves this type of planning. Implementing a different method of dispensing medication Developing an immediate plan for obtaining new thermometers Ensuring that the emergency room can handle a natural disaster Improving nursing care at a long-term care facility over the next five years52

Collective bargaining is a process whereby:

Correct. Collective bargaining is a legal process used by organized employees tonegotiate with an employer regarding wages and other work-related issues. These negotiations result in an employment contract.Incorrect. The director of nurses is a member of management and, therefore, does not represent members of the bargaining unit. The bargaining unit is represented by the union organization, which is external to the health care institution.Incorrect. Collective bargaining is a process designed to protect workers against unfair working conditions. Management does not unionize.Incorrect. Labor relations and collective bargaining involve athird party or bargaining agent to represent them in negotiations for work-related conditions. Nurse collaboration with the health care team is a professional intervention and not a collectivebargaining activity. employees negotiate an employment contract. the director of nursing represents nurses in work-related issues. employers form a union to protect clients and ensure quality care. nurses collaborate with the health care team on the client's behalf. 53

The Nurse Practice Act in each state is a set of laws that originate from:

Incorrect. The United States Constitution gives authority to all the other sources of law (e.g., legislative, administrative, court); and is a system of laws that governs a nation.Incorrect. Court decisions or tort laws are made by the courts to interpret legal issues that are indispute.Incorrect. In civil actions (i.e., issues that deal with interpersonal relationships) thecourts interpret statutes to determine the verdict. These court decisions are also called tort law.Correct. The nurse practice acts define the practice of nursing and are made by the legislative bodies of each state. These laws are called statutory or legislative laws. the U.S. constitution. court decisions. tort law. statutory law. 54

A written record of all conferences and conversations with an employee during disciplinary actions must:

Incorrect. Not only must a copy of the records be given to the employee, but anopportunity must also be given for the employee to respond.Incorrect. Although the employee's signature is preferred, it is not essential. However, if an employee does not want to sign,

another person in management should be asked to sign as witness to the fact that thedocument was discussed with the employee, and that a copy was given.Correct. This is an essential standard for managers to maintain. It is imperative that documentation clearly showsthat all employees are being treated equitably. For example, the manager may overlook documenting an employee's tardiness since the employee has excellent performance

skills.Incorrect. An employee's performance in another job would not be relevant to a current complaint. be kept from the employee. include the employee's signature. show that the employee has not been singled out. consist of a history of the employee's prior employment. 55

The nurse manager on an orthopedic unit is skilled in setting up traction. A nurse from another unit seeks help from the manager to implement Bucks traction. Which type of leadership power is evident in this situation?

Incorrect. The ability to influence by offering incentives is referred to as reward power.Correct. It is evident in this situation that the nurse manager is respected by other nurses for his/her knowledge and/or skills. This is referred to as expert power.Incorrect. As the name suggests, coercive power refers to the ability to influence another by invoking fear of punishment.Incorrect. Referent power is somewhat similar to expert power in that it refers to the ability of a person to influence by his or her own abilities. Expert power is granted by a leader's expertise, whereas referent power is granted by a leader's ability to engage or persuade. Since the scenario stated the manager had specific knowledge that was sought by others, the power exerted by the manager represents expert power. Reward

Expert Coercive Referent

56

Which example suggests the most efficient care?

Correct. This is the best example of efficiency because the outcome (pain elimination) was achieved with the most cost-effective resource (non-pharmacologic intervention given by a nursing assistant).Incorrect. Reducing operative pain, the desired outcome of altered comfort, demonstrates care effectiveness; however, we are not certain morphine was the most costeffective intervention, nor do we know if the nurse was the most cost-effective use of personnel.Incorrect. The promotion of wound healing means that care was effective, but was surgical debridement the most cost-effective way to achieve wound healing? Could the same goal have been achieved using chemical debridement methods?Incorrect. Although this example reflects effective care through improved airway clearance, we do not know if tracheal suctioning was the most cost-effective means for achieving the goal. Perhaps the client may have been able to cough adequately to clear the airway; use of nursing staff to perform airway management may be more efficient. Eliminating back pain with massage given by a nursing assistant

Reducing operative pain with morphine sulfate administration by an RN Promoting wound healing from stage IV to III by surgical debridement Improving airway clearance with tracheal suctioning by a respiratory therapist 57

A nurse applying for a nursing position reviews the hospital's organizational chart showing the formal authoritative structure and hierarchy of positions. Which question, if asked by the applicant, tells about the informal structure of the hospital?

Incorrect. The chain of command (who reports to whom) identifies the path of authority and accountability at each level within an organization. This relationship is evident in the formal organizational chart.Incorrect. Job descriptions (an explanation of who does what) are a part ofthe formal organizational chart.Correct. The informal structure of an organization refers to the communication channels where employees know unwritten rules and expectations. The informalstructure provides a means for employees to develop a sense of belonging.Incorrect. The organizational chart depicts the formal channels of communication with the organization. The nursing staff is expected to take concerns to the next level (i.e., unit manager). "Who reports to whom?" "Who does what activities?" "Who are the unofficial leaders?" "To whom should nurses take their concerns?" 58

A nurse manager of a stroke rehabilitation unit wants to evaluate productivity. Which is a direct measure of the unit's output?

Incorrect. Type of stroke can be used to analyze output since a specific diagnostic group and related costs reflect client days for a specific case mix or cluster of clients (product). However, a more direct measure of output, or product in this case, is the quantity of services, such as the number of physical therapy referrals.Incorrect. This is another example of input. Thelabor required to produce the service includes consideration of the qualifications of the staff (e.g., educational level and/or experience).Incorrect. Inputs include the labor, materials, and equipment used in the product or service provided; therefore, hours of labor and dollars spenton supplies reflect input.Correct. Output represents the product, and is generally defined interms of

quantity of services (although we are also interested in the quality of the output, whichis more difficult to measure). In this case, one product of a stroke rehabilitation unit is the use ofphysical therapy. Type of stroke Qualifications of staff Expenses for hydrotherapy Number of physical therapy referrals59

The capitation payment system is evident in which situation?

Incorrect. This is an example of the fee-for-service where the provider, in this case, the physician, charges for services given.Incorrect. This is an example of per diem rates that are based on each day of service.Incorrect. This is an example of DRG payment system where a payment covers the expected costs for specific diseases, procedures, surgeries, etc.Correct. Inthe capitation system of payment, the plan contracts with a provider that is paid a flat amount forevery person enrolled, regardless of the services used. For example, the physician groupreceives $100 a month per member for every chest x-ray. The physician charges $50 for an office visit. The insurer is billed for three days of hospitalization. The hospital receives $1500 to cover costs for surgical hip pinning. The physician receives a flat fee of $100 a month per member for every chest xray. 60

A client's wife asks the nurse not to tell her husband that he has terminal cancer. Which moral principle would the nurse be using if the decision to inform the client were made?

Incorrect. Beneficence is a principle that means doing good. One could argue that tellingthe truth to the client is an act of beneficence; and although true, veracity is the principle directlyexemplified in this question.Incorrect. The principle of autonomy refers to the right to make one'sown decisions and choices; in other words, self-determination. This principle would be evident, for example, if the client chose not to hear the results of the biopsy.Correct. The nurse would be honoring the principle of veracity that refers to telling the truth. Although decisions to tell the truth are not always clear, deceiving a sick or dying client is rarely justified.Incorrect. Justice refers to the simple rule of fairness. In this situation, for example, justice is evident if the nurseconsiders the client rights of self-determination. Beneficence Autonomy Veracity Justice

61

Which agency represents a for-profit or proprietary health care organization?

Correct. For-profit health care organizations have shareholders who expect to make aprofit on their investment. Large corporations that own a chain of hospitals typically run forprofitorganizations.Incorrect. Charitable institutions, like American Red Cross, often fund not-

for-profithealth care organizations. Profits are funneled back into the organization for expansion orimprovement.Incorrect. Local public agencies that may receive federal and state funding includecounty long-term care facilities that have local governing boards composed of locally electedofficials. These types of organizations are not-for-profit.Incorrect. Government subsidies traditionally help fund not-for-profit health care agencies, such as those providing services tomilitary personnel. Corporate hospital American Red Cross County long-term care facility Veterans Administration Hospital 62

Variable staffing is a method that uses which approach?

Correct. Variable staffing is a method that uses client needs to determine the number of staff members. Some sort of client classification system is used.Incorrect. Moving staff membersto cover peak workload periods simply represents a variation in some of the traditionalscheduling plans discussed.Incorrect. Flextime is the term used to describe a type of schedulethat involves variations in the hours worked (e.g., 10- or 12-hour shifts versus 8hourshifts).Incorrect. Variable staffing does not involve rotating work shifts. It involves determining client needs or care hours required, and assigning a staff according to these needs. Therefore, scheduling will vary from day to day and shift to shift. Determining number of staff members based on client needs Moving staff members to cover peak workload periods Variation in number of hours worked

Rotating work shifts63

One of the primary aims of an integrated health care delivery system is to:

Correct. In an integrated delivery system, health care is provided across the continuumof care at the lowest cost; therefore, a chief goal is to promote wellness and keep clientshealthy.Incorrect. In an integrated health care system such as managed care, the focus is on primary care in order to control specialty care that is more expensive.Incorrect. Providers in theintegrated delivery system are under tighter control, both in terms of treatment options andreimbursement.Incorrect. Specific amounts of money are paid to provide care under theintegrated health care delivery system. If, for example, the provider prescribes tests orprocedures beyond the specifications for payment, the provider will be responsible for theadditional cost. This creates a greater financial risk for the provider. keep clients healthy. use more specialty care. give greater autonomy to the provider. create less financial risk for providers. 64

A 98-year-old client refuses to have hip surgery. The family believes the surgery should be doneto increase her comfort and mobility. Which statement by the nurse demonstrates a principlesbased perspective?

Incorrect. This is an illustration of consequence-based reasoning because the treatment would cause the least harm (the most good) to the most people (client and family).Correct. This is an example of a principles-based or deontological theory that emphasizes individual rights forself-determination.Incorrect. This is an example of a utilitarian or consequence-based

perspective, which views a good act as one that produces the best outcome.Incorrect. This is anexample of a relationship-based or caring theory that emphasizes the need to nurture andmaintain relationships, and promotes the common good or welfare of the group. "Conservative treatment would be less painful for everyone." "Everyone has the right to decide what happens to his/her body." "Consider the risks related to surgery, such as pain, medical complications, and death before reaching your decision about surgery." "I will protect the needs of your family and support you during this time." 65

A nurse delegates blood sugar checks to a nursing assistant she has worked with for 12 months. The nurse instructs the assistant to avoid finger sticks on a client with low platelets. Later, on rounds, the nurse finds the client holding cotton swabs on both index fingers. The appropriate conclusion is that the:

Incorrect. The nurse was aware of the abilities of the nursing assistant (NA) since they had a working relationship. Because the nurse knew the competencies of the assistant, and hadprevious opportunity to assess job performance, the nurse delegated appropriately.Incorrect. Will the state board discipline the nurse? Not at this point in time. The nurse assessed the situation, made a reasonable assignment, provided instruction, and provided follow-up. So far, he/she meets the expectations of a delegator. The action the nurse takes next will determine if his/her license is in jeopardy. In other words, the nurse must assess the client, notify the physician if necessary, and meet with the NA to make sure this does not happen again.Correct.

The assistant is accountable for both accepting the delegation and for his/her actions incompleting the task.Incorrect. The nurse manager would not be directly involved or accountable in this situation unless the nurse manager was not aware of the nurse's competencies as adelegator and/or the nurse was inappropriately assigned.

nurse delegated inappropriately. State Board may discipline the nurse. nursing assistant is responsible for any harm to the client. nurse manager is held accountable for the actions of the nurse and assistant.

66

During the process of performance evaluation, the halo effect is evident when the manager:

Incorrect. If an evaluator is hypercritical, this is called the horns effect. Managers who tend to be perfectionists may rate employees lower than they should. A recent mistake or a personality trait may cause the manager's rating to be lower than the person's work merits.Correct. The halo effect is the result of allowing a positive characteristic to influence the overall evaluation. For example, a manager might evaluate a nurse high on interaction with patients and compassion. Although the nurse needs considerable work on decision making, the manager does not give sufficient constructive feedback, allowing the positive aspects of the nurse's behavior to unduly influence an area that needs improvement.Incorrect. A tendency to overrate employee performance is called leniency error. This type of appraisal error is often the result of the manager wanting to be liked.Incorrect. A tendency to complete a performance rating based on the employee's recent contribution rather than over the entire evaluation period is known as recency error.

is hypercritical of the employee. rates everyone on the nursing staff above average. allows positive behaviors to influence negative behaviors. rates the employee's performance based on the most recent events. 67

Nurse practice acts all have the common purpose of:

Incorrect. Credentialing is the process of determining and maintaining competence in nursing practice.Correct. Although individual differences exist, all nurse practice acts legally define the scope of nursing practice; thereby, protecting nurses and the public.Incorrect. The American Nurses Association has developed standards of practice that are used to evaluate the quality of nursing care.Incorrect. A code of ethics is a formal statement that provides general guidelines for professional behavior and conduct. ensuring competency. protecting the consumer. evaluating quality of care. guiding professional behavior. 68

A nurse who is concerned about abusive conduct of a fellow employee should report such

behavior to the:

Correct. Each state's board of nursing is the authority that assumes the obligations of the licensing process, as well as disciplinary action, in nursing.Incorrect. The National League of Nursing (NLN) is an organization whose objective is to foster development and improvement of nursing services and education. The organization provides workshops, consultation, and educational aid. It also serves as an accreditation agency for schools of nursing.Incorrect. The American Nurses Association is the national organization for professional nursing in the United States. Its purpose is to further high standards of practice and advance educational and professional goals of nurses toward better nursing care.Incorrect. The International Council of

Nurses (ICN) is an organization through which members can collaborate to promote standards of nursing worldwide. State Board of Nursing. National League for Nursing. American Nurses Association. International Council of Nurses. 69

A nurse working in a cardiac care unit administers the wrong dose of an antihypertensive agent. Which documentation illustrates proper documentation for an incident with potential harm to client?

Correct. Incident reports should not be placed in the client's chart since they are considered confidential and cannot be used as evidence in lawsuits (as long as they remain confidential). Documentation of an incident should simply indicate what occurred (without blaming or naming an error), client assessment, actions taken, and the results.Incorrect. This statement openly suggests wrongdoing and client records should never state that an incident report was completed.Incorrect. Although it is important to note the client's blood pressure and condition, the documentation entry does not address the incident or action(s) taken.Incorrect. This entry openly suggests wrongdoing and does not provide enough information about the incident, the client, or what was done. Name and dosage of medication; date and time given; BP 110/68; physician

notified Inadvertently given wrong drug; incident report completed Blood pressure 110/68; client resting without complaints Incorrect medication administered; supervisor notified 70

McGregor's Theory Y supports the assumption that people:

Incorrect. McGregor's Theory X assumes that people generally avoid work because they dislike it.Incorrect. McClelland's motivation theory assumes that people are motivated by affiliation (need to belong), power (the need to control), and/or achievement (the need to succeed).Correct. McGregor's Theory Y assumes that workers are basically satisfied when conditions are favorable, and that they embrace opportunities to be self-directed and creative.Incorrect. McGregor's Theory X assumes that people find work distasteful and are motivated primarily by monetary rewards. avoid work because they dislike it. are motivated by the need to control. welcome opportunities to be creative. seek rewards in money and fringe benefits.

71

A senior client receiving Medicare benefits is admitted to the hospital for pneumonia and stays for five days. Which factor primarily determines the hospital bill?

Incorrect. Under the DRG mechanism for Medicare, the cost of professional care is not a factor in determining how much money will be reimbursed to that facility. Regardless of how costly care is in any given situation, costs incurred beyond the specific diagnostic-related payment are not reimbursed.Incorrect. The DRG mechanism covers the cost of surgery or episode of illness; if the client stays more than the predetermined number of days, the hospital

is expected to absorb the cost. The client's diagnosis is the key factor in determining how much the hospital will receive for care provided.Correct. In 1983, Diagnostic Related Groups (DRGs) created a prospective reimbursement mechanism for Medicare and Medicaid. In other words, aspecific payment is made for a specific diagnosis based on statistical averages.Incorrect. Although the client's age is a factor in eligibility for Medicare, age does not increase or decreasethe hospital bill. Cost of professional care Length of stay Diagnosis Age72

Which situation illustrates the concept of collaboration as it relates to conflict management?

Incorrect. The staff member has accommodated in a non-assertive manner, by giving in to the manager's request to work overtime; thereby, hoping to preserve harmony and building upa case for requesting an additional vacation day.Incorrect. Each nurse is giving up his or her usual schedule in order to gain a preferred weekend off. This approach involves negotiation andcompromise.Incorrect. In this example, the nurse is competing or forcing the issue of IV catheterselection by emphasizing the merits of a specific catheter that he/she favors.Correct. Collaboration requires that all participants come to a decision together, being supportive and considerate of one another's point of view. In this example, the nursing staff is working together to determine the best approach to floating. The staff member agrees to work on a day off, hoping to gain an additional vacation day. A nurse trades work schedules with another nurse so both can have a specific

weekend off. During a staff meeting, a nurse presents research supporting the use of a new IV catheter. The unit manager calls the staff together to discuss criteria for floating to other units. 73

The nurse manager evaluates the twelve nurses on a pediatric unit as average on allperformance criteria. This evaluation error is known as:

Incorrect. The halo effect occurs when the evaluator assigns ratings based on an overall positive impression or on one trait.Incorrect. The horns effect is essentially the opposite of the halo error. The evaluator rates the employee based on an overall negative impression or one negative trait or event.Incorrect. A leniency error occurs when the appraiser overrates the employee's performance. There may be a variety of reasons for doing this (e.g., wanting to be liked). Regardless, it can cause problems when, for instance, a nurse whose performance isaverage is given a lenient or better rating. Then it is difficult to take corrective action when necessary.Correct. The appraiser who hesitates to risk true assessment(s) falls into the trap of central tendency, and therefore, rates all employees as average. halo effect. horns effect. leniency error. error of central tendency. 74

Which question during an employment interview is unacceptable?

Correct. Any question about an applicant's marital status is legally unacceptable. Questions about marital status, age, race, sexual preference, financial status, national origin, orreligion can be grounds for discrimination.Incorrect. The name and address of a person to be notified in case of emergency is acceptable. It does not have to be a relative.Incorrect. It isacceptable to ask if the applicant is a citizen of the United States. It is unacceptable to inquireabout the person's birthplace, ancestry, or native language.Incorrect. Place of residence and theamount of time the person has resided in that city or state is acceptable. However, it is unacceptable to inquire about former addresses, who lives with the applicant, or if the individualowns or rents. "Are you married?" "Who can we notify in case of emergency?" "Are you a United States citizen?" "How long have you lived in your current home?" 75

The major driving force behind redesigning the health care system is an increase in the:

Correct. The increasing cost of health care is the major force behind most of the changes in health care delivery (i.e., managed care).Incorrect. Although the number of uninsured is growing in the United States, it is only one of the problems that ultimately have ledto an increase in the cost of health care delivery. It is not the major force behind reengineering

the health care system.Incorrect. The homeless population is increasing, which is another factorthat contributes to the rise in the cost of health care. However, it is not the major force behind re-engineering the health care system.Incorrect. An increase in the incidence of acute disease would significantly increase the cost of health care; however, this factor has not been cited as one of the major driving forces behind re-engineering of the health care system. cost of health care. number of uninsured. homeless population. incidence of acute disease.

76

The hospital is planning for the acquisition of a new computed tomography scanner. These funds are allocated from which type of budget?

Incorrect. A revenue-and-expense budget is another term used for the operating budget. The operating budget shows an analysis of expected revenues and expenses that dealsprimarily with salaries, supplies, and contracted services.Incorrect. The operating budget involves daily expenses that fluctuate. Examples include cost of repairs, utilities, andsupplies.Correct. The capital budget includes long-term plans for purchase of buildings or major equipment that is not used in daily operations.Incorrect. A labor, or personnel budget, estimates the cost of direct labor and includes salaries, recruitment, holidays, turnover, and orientation costs.

Revenue and expense Operating Capital Labor 77

A planning committee is meeting to discuss ideas for the next continuing education seminar. The leader would most likely respond with which statement when using a transformational styleof leadership?

Incorrect. This statement reflects a laissez-faire leadership style, in which the group is permitted to make the decision without any additional input.Incorrect. This type of response reflects a bureaucratic style where the leader does not feel the group is able to make anappropriate decision, and therefore, relies on rules or policies to guide the decision.Incorrect. This statement reflects the democratic style of leadership. The group members are considered capable of making an appropriate decision. The group leader simply encourages group discussion to collaborate decision-making.Correct. Transformational leadership fosters creativity, supports the sharing of values, and empowers the group to contribute to the hospital's vision/mission. "I'll give you a little more time to select the best topic for the next seminar." "According to unit policy, we have not presented enough hours on technology update." "Let's have more discussion on how you perceive the educational needs of thenurses." "Think about the word 'compassion' and brainstorm ways to connect our vision tothe topic." 78

The nurse manager of a respiratory care unit is planning a smoking cessation seminar. As a change agent, the manager plans to present the latest research on cancer and smoking. This strategy is referred to as:

Incorrect. The normative-reeducative strategy for effecting change uses group norms to socialize and influence individuals to change. In this situation, family, friends, and/or former smokers could educate the person about the effects of smoking in a social context (peer pressure).Correct. The rational-empirical approach to change assumes people are rational and will change when given the most current facts.Incorrect. A power-coercive strategy attempts to influence people to change through authority measures. For example, in this situation, thechange agent may restrict seminar participants from smoking in or near the building.Incorrect. The rational-empirical strategy may be used during the unfreezing phase of change, however, unfreezing is not a strategy. Unfreezing is a phase in the change process whereby the change agent intentionally raises the group's discomfort level, making them aware of the need forchange. normative-reeducative. rational-empirical. power-coercive. unfreezing. 79

A nurse administers the incorrect dose of medication to a client. The drug given was similar in name and container as the prescribed drug. A quality improvement approach to the problem is to:

Incorrect. The goal of a quality improvement program is to improve care through analysis of the system instead of faulting people.Correct. Quality improvement programs focus onexamining the system to identify factors that may be interfering with quality care rather

thanplacing blame on individuals.Incorrect. Providing classes for the nursing staff places blame on nursing, rather than looking at the entire system of drug administration.Incorrect. Focusing

evaluation on the pharmacy places blame on specific individuals, rather than the system as a whole. discipline the nurse. examine the drug dispensing system. plan remediation classes for the nursing staff. counsel the pharmacist who dispensed the medication. 80

The nurse who provides population-based care within the managed care environment is:

Incorrect. Population-based care does not revolve around geographic area. Rather, care focuses on a population of people with similar health needs.Correct. Population-based care within the managed care environment means that the focus is on improving care for a specific population (e.g., the elderly), that has common health care needs.Incorrect. Ensuring that clients receive the right care at the right time from the right provider defines the function of a triage coordinator.Incorrect. Finding ways to reduce the organization's financial risks, whether associated with a specific group of clients or not, defines the process of risk management. providing care to people in the same geographic area. enhancing care for a specific group with common health needs. ensuring that clients receive the right care at the right time from the right provider. finding ways to reduce financial risks associated with specific groups of clients.

ATI. NUTRITION

Which is a benefit of breastfeeding?

Incorrect: Breastfeeding may facilitate weight loss after delivery. The average caloric expenditure for breast milk production is 500 kcal/day.Correct: Breastfeeding protects the infant against many infections and diseases, especially in the respiratory system and gastrointestinaltract. Breastfeeding promotes the growth of Lactobacillus bifidus, which suppresses thecolonization with staphylococci, shiyella, and protozoa (cell disease causingorganisms).Incorrect: Children who have been breastfed develop fewer food allergies. The gastrointestinal tracts allows for the passage of whole proteins into the bloodstream. The absorption of these proteins can stimulate an allergic response in susceptible infants.Incorrect: An increase in infant oral motor development has been shown. Delays return to pre-pregnancy weight Provides immunologic protection to the infant Increases risk of food allergies in the infant Delayed infant oral motor development2

Solid foods should be introduced slowly at approximately 6 months of age. Which would be themost appropriate selection?

Correct: Rice cereal is the preferred first solid food introduced because of itshypoallergenic quality. It is the easiest of the first solid foods for the six-month-old baby to swallow and digest. Additionally, rice cereal is enriched with iron.Incorrect: Honey may cause infantile botulism and should never be given under one year of age.Incorrect: After solid foods

are introduced into the infant's diet, the customary practice is to delay strained fruits because ofthe strong human preference to sweets.Incorrect: Grapes may be a choking hazard. The infant at six months of age has immature mastication and swallowing function. Rice cereal Honey Strained fruits Grapes3

Which type of oil is an example of a polyunsaturated fatty acid?

Incorrect: Olive oil is a monounsaturated fatty acid.Correct: Corn oil, safflower oil andsunflower oil are all examples of PUFAs.Incorrect: Coconut oil is a saturated fat often called tropical oil.Incorrect: Peanut oil is a monounsaturated fatty acid. Olive Corn Coconut Peanut 4

Assessment findings that indicate that a client is having swallowing problems may include:

Incorrect: Spitting food out of the mouth occurs when the client uses a tongue thrustingmovement to avoid swallowing.Incorrect: Coughing that occurs after swallowing may indicatedysphagia. A weak or hoarse cough, decreased gag reflex or pocketing of food in the mouth may indicate dysfunction of the tongue, hypoglossal, or hypopharyngeal muscles. Otherassessment findings may include drooling, poor articulation, or facial drooping.Incorrect: Weight loss may result from inadequate intake of food and fluids leading to malnutrition anddehydration.Correct: These assessment findings indicate the client is experiencing difficultly swallowing. Dysphagia occurs as a result of mechanical or pathological disorders involving thetongue, pharynx, hypopharynx, or esophagus. Specific causes include: neuromotor dysfunction(stroke or multiple sclerosis), cardiovascular abnormalities (aortic aneurysm or enlarged heart), or mechanical obstruction (esophageal carcinoma, hiatal hernia or congenital defects). spitting food out of the mouth. coughing after swallowing. weight loss. all of the above. 5

The nurse interprets the laboratory values for a general health assessment for the client withcoronary artery disease. Which finding may indicate an abnormality?

Correct: A triglyceride is a molecule composed of three fatty acids joined to a glycerol molecule. Excess triglycerides are stored in adipose cells. High triglycerides in clients withobesity, diabetes mellitus, familial hyperlipidemia, and premature heart disease are at increasedrisk for coronary artery disease.Incorrect: High-density lipoproteins (HDL) are the good

proteins that acts as a scavenger to take cholesterol out of the serum and transport it to theliver. The higher the HDL levels, the lower the risk of heart disease.Incorrect: Low-density lipoproteins (LDL) are the bad proteins that causes atherosclerotic changes. High levels ofLDL are associated with obesity and myocardial infarction.Incorrect: Total cholesterol is comprised of high-density (HDL), low-density (LDL) lipoproteins and very lowdensitylipoproteins. The HDL's are the good proteins that remove cholesterol from the serum. The

LDL's are the bad proteins that contain more cholesterol and triglycerides than protein. Elevated LDL level is associated with an increased LDL and appear to be atherogenic. The VLDL's are precursors of LDL and appear to be athrogenic. The ratio of LDL to HDL proteins is an important consideration in the interpretation of the risk for coronary artery disease. High triglyceride level Low (HDL) cholesterol High (LDL) cholesterol Low total cholesterol 6

The nitrogen balance that occurs in normal, healthy adults is referred to as:

Incorrect: Positive balance occurs when more nitrogen is retained than excreted, suchas in rapid growth in infancy, children, and pregnancy.Correct: Zero balance is normal when growth is not needed, just maintenance.Incorrect: Negative balance occurs when more nitrogenis excreted from the body than is retained, secondary to malnutrition or severe illnesses, suchas liver and renal failure.Incorrect: The deamination process occurs when enzymes strip off amino acid groups during protein digestion. positive balance. zero balance or equilibrium. negative balance. deamination. 7

Clients who receive renal dialysis are required to observe a tightly restricted diet to minimizewide changes in blood chemistries. Phosphorus level is best controlled by which dietarymeasure?

Incorrect: Fluid restriction may be necessary for fluid volume regulation.Incorrect: There is some debate on how to restrict protein intake in the diet of the client receiving dialysis. Somestudies show the benefit of maintaining a daily intake of protein with high biologic value. Generally, the dietary recommendation for protien intake ranges from 1 gm/kg/day protein intaketo no restriction other than avoiding high-protein fad diets.Correct: Calcium carbonate may begiven with meals to bind phosphate to foods.Incorrect: Carbohydrates and fats are the primarysource of energy foods and calories in dialysis clients. It is important to provide nonproteincalories to prevent or reduce catabolism. One common recommendation is 40-50 kcal/kg/day ofcarbohydrates and fats. Fluid restriction High protein intake Calcium carbonate supplement Limit carbohydrates and fats8

The purpose of the assessment phase in nutrition therapy for the diabetic client is to:

Correct: The purpose of the assessment phase of nutrition therapy is to identify goals for diabetic care and to establish a rapport with the client.Incorrect: The effectiveness of interventions for nutrition therapy are determined in the evaluation phase.Incorrect: This is

accomplished in the planning phase of the nursing process.Incorrect: Meal planning is aneducational process that is part of the intervention phase. identify goals for diabetic care. determine the effect of medical interventions.

develop a plan of action. discuss individualized meal planning. 9

The type of dietary protein of an animal source that contains all the essential amino acidsnecessary to meet human needs is a(n):

Incorrect: A polypeptide is a chain of two or more amino acids that are linked by peptide bonds. A polypeptide can be found in animal or plant dietary sources.Correct: Complete proteinscontain the eight essential amino acids necessary for human health. They are usually found in an animal source product such as milk, eggs, and fish.Incorrect: Incomplete proteins lack one ormore of the eight essential amino acids that are needed by humans. These are usually found in plant sources such as grains and vegetables.Incorrect: A high quality protein source can be either plant or animal. Complete proteins contain the eight essential amino acids and areprimarily found in animal sources. Two examples of highest quality protein foods are eggs and human milk. polypeptide. complete protein. incomplete protein. high quality protein. 10

What is the most important advantage of a gastrostomy tube feeding?

Incorrect: There is an increased risk of aspiration pneumonia in clients with gastroesophageal reflux.Incorrect: Enteral feedings may be delayed while healing occurs afterthe gastrostomy placement.Correct: Gastrostomy feedings are well tolerated because thestomach chamber holds and releases feedings in a physiologic manner that promotes moreeffective digestion. As a result, the dumping syndrome is usually avoided.Incorrect: Gastrostomy feedings are frequently administered for clients with impaired swallowing ability. Decreased risk for aspiration pneumonia Enteral feedings may be started immediately post-op Reduction in "dumping syndrome" Use in clients with normal swallowing

11

The nurse providing nutritional counseling to the client with lactose intolerance understands thatlimited intake of lactose-containing foods may increase the risk for which nutritional deficiency?

Incorrect: Good sources of calcium include: milk, dairy products, green leafy vegetables, whole grains, nuts, legumes, and calcium enriched orange juice.Incorrect: Good sources ofriboflavin include milk and dairy products.Incorrect: Good sources of vitamin D include: fortifiedmilk, margarine, and breakfast cereals.Correct: The client who is not consuming dairy products or is on a lactose-restricted diet may be at risk for calcium, riboflavin, or vitamin D deficiency. Calcium

Riboflavin Vitamin D All of the above 12

A healthy snack for a school-aged child may include:

Incorrect: Ice cream is high in fats and sugar.Correct: Yogurt is an appropriate snack that is low in fat and contains calcium.Incorrect: French-fries are high in fats and salt.Incorrect: Muffins, cakes and pastries are high in fats and sugar. ice cream. fruit flavored yogurt. french-fries. blueberry muffins. 13

The nurse providing gastrostomy feedings to the client with gastroesohageal reflux implementswhich action to reduce the risk of aspiration pneumonia?

Incorrect: Tube placement should be checked before feedings are started. Confirmation of proper tube placement is easiest with a gastrostomy tube.Correct: The HOB should be elevated 30-45 degrees to reduce the risk of aspiration pneumonia.Incorrect: The length of time that feeding solutions are hung is not related to the risk of aspiration but instead tocontamination of the feeding.Incorrect: Coloring formulas may assist the nurse in determiningthe presence of enteral solution in the pulmonary mucous indicating aspiration. This is a measure to aid in the detection but not prevention of aspiration. Check tube placement immediately after each feeding. Elevate HOB 30-45 degrees. Do not hang feedings for longer than 4-6 hours. Add vegetable food coloring to formula. 14

The woman in her early 30s has a continuing need for adequate amounts of which substance?

Incorrect: The intake of vitamins A and C are generally not altered in this age group.Incorrect: Potassium and phosphorus are usually not deficient for persons in the agegroup who are not taking loop diuretics.Incorrect: Vitamins B1 and D are generally not altered by the customary eating habits in this age group.Correct: Calcium and iron are in essential forwomen of all age groups, especially during menstruation or lactation. Vitamins A and C Potassium and phosphorus Vitamins B1 and D Calcium and iron 15

Which dietary source is an example of the disaccharide, sucrose?

Incorrect: Honey is the monosaccharide, fructose, which is easily digested in thebody.Incorrect: Milk is the disaccharide, lactase.Correct: Sugar cane, sugar beets, and table sugar are examples of sucrose.Incorrect: Syrup is the monosaccharide, fructose, which israpidly digested by the body. Honey Milk Sugar cane

Maple syrup

16

Pregnant women are normally advised to increase caloric intake by 300 kcal/day during thesecond and third trimesters. What is the rationale for this adjustment to the diet duringpregnancy?

Incorrect: Personal preference may guide particular food choices for the extracalories.Incorrect: Although eating for two, the pregnant woman should not double her calories.Incorrect: Appetite may decrease during the first trimester.Correct: The basal metabolic rate (BMR) increases during the third trimester of pregnancy in response to the energy needs ofthe developing fetus. To meet the increased metabolic workload and spare protein for tissue building, the pregnant women will normally require an additional 300 kcal/day. Personal preference Need to eat for two Appetite increases Accommodates the rise in maternal BMR 17

The nurse providing nutritional support offers a full liquid diet to the client with:

Incorrect: A mechanical soft diet is indicated for the client with mouth pain. Poorly fitting dentures may lead to the client having difficulty chewing.Incorrect: A soft diet or mechanical soft diet is indicated for the client with restriction in chewing or difficulty swallowing.Correct: A full liquid diet is indicated for a client with fractures of the mandible or facial bones due to therestriction of the jaw for chewing.Incorrect: A progressive diet that is advanced slowly and

includes mechanically soft foods is more easily tolerated by the client and results in improveddigestion. poorly fitting dentures. limited chewing or swallowing ability. fractures of mandible or facial bones. parenteral nutrition advancing to solid foods. 18

A clear liquid diet is a good source of fluids, but may be lacking which of the following?

Incorrect: The food choices for a clear liquid diet are limited and clients might easily consume increased amounts of caffeine in coffee, tea, and salt, leading to sleeplessness and upset stomach.Incorrect: This diet can provide adequate amounts of vitamin C with non-pulp fruit juices, but is inadequate for almost all other nutrients.Incorrect: Water and fluid intake can be monitored carefully by maintaining an accurate record of intake and output.Correct: Clearliquid diets contain little protein or fat. The full liquid diets include foods such as: milk products, custard, thinned hot cereal, and soups. Caffeine Adequate amounts of vitamin C An adequate amount of fluid intake for 24 hours Adequate amounts of protein and fat19

A factor that favors calcium absorption is:

Correct: Vitamin D and sunlight play a major role for calcium and phosphorus absorption in the process of bone and mineral metabolism.Incorrect: Dietary fat can form insolublechemical bond with calcium, making them harder to digest.Incorrect: Laxatives cause food topass through the GI tract too quickly for calcium to be absorbed.Incorrect: Lack of physicalactivity leads to less bone density and osteoporosis.

vitamin D. dietary fat. laxatives. sedentary lifestyle. 20

Fluoride is necessary for whch bodily function?

Correct: Fluoride has been shown to strengthen the structure of the teeth making thembetter able to resist the bacterial acids that cause dental caries. Fluoridated water and dental hygiene products have reduced the incidence of caries in children by as much as 40%.Incorrect: Cobalt is necessary for red blood cell formation.Incorrect: Zinc has an important role in woundhealing, taste, and the immune response.Incorrect: Phosphorous is a component of the storageforms of energy (ADP and ATP). Reduction of dental caries Red blood cell formation Wound healing Energy storage

21

A primary intervention in nutritional therapy for the client with cystic fibrosis (CF) is:

Incorrect: Fat malabsorption occurs in clients with cystic fibrosis (CF) due to theabnormal secretion of the exocrine glands and impaired pancreatic enzyme secretion. This leads to fat steatorrhea and malnutrition. The dietary management for clients with CF includes increasing the fat content in the diet, although fat tolerances vary among individuals. Pancreaticenzymes are necessary for fat digestion and absorption.Correct: Cystic fibrosis is a hereditary, chronic disease characterized by abnormal secretion of exocrine glands. The most significant nutritional problems are malnutrition and poor growth because of impaired pancreatic enzymerelease. Nutrition management includes a high protein, high calorie, high salt, and low fatdiet.Incorrect: A high salt diet is necessary because of the client's excessive sodium and chloride losses in the sweat.Incorrect: Supplementation with fat-soluble vitamins is necessarydue to the pancreatic exocrine dysfunction, which causes impaired fat absorption. to limit the intake of fats. to exceed the RDA's for kilocalories of protein and salt. to restrict the intake of salt. to restrict the intake of fat-soluble vitamins. 22

A nurse conducting a nutritional screening for an elderly person identifies the client is at risk based on which most important factor?

Incorrect: Two major indicators that are highly predictive of morbidity and mortality are low body weight and rapid, unintentional weight loss, which is defined as a loss greater than 5% baseline weight, or greater than 10% in six months. Involuntary weight loss is an importantwarning sign that must not be ignored. Encouraging weight loss in the elderly should

beapproached with caution.Correct: Studies of predictive factors for mortality among persons aged70 and older indicate that body mass index (BMI) in the lowest quartile is the greatest commonfactor. The inverse relationship between weight and mortality may reflect on the effect of illness. Elderly persons who are underweight should be encouraged to gain weight to within normalparameters.Incorrect: The elasticity of the skin in the elderly person is diminished as a normal

part of the aging process. However, it is important for the nurse to assess the hydration status, as the elderly are more prone to dehydration. The decreased sensation of thirst due to changes in the thirst center of the hypothalamus increases the risk for fluid volume deficit.Incorrect: Difficulty chewing related to loss of teeth and periodontal disease may limit the intake to soft, easy-to-chew foods and liquids. The intake of many types of meat that are particularly difficult toingest, may therefore, be reduced in the diet. Care must be taken to provide a well-balanced, nutrient-dense diet based on the Daily Food Guide for the Elderly. However, the two predictive factors for morbidity and mortality in the elderly are low body weight and rapid, unintentionallyweight loss. Intentional weight reduction Low body weight Poor skin turgor Difficulty chewing23

Which strategy may the nurse recommend to help the client cope with a strict fluid restriction?

Incorrect: Fluids should be given between meals.Incorrect: Offer medicines with soft foods at meal times to conserve fluids.Correct: Lemon wedges stimulate saliva and moisten adry mouth.Incorrect: Keeping the mouth clean will increase comfort. Drink most of the fluid volume allowed with meals. Take medications with fluids between meals. Use sliced lemon wedges in beverages. Avoid brushing teeth unless necessary. 24

The elderly may have decreased taste and smell. Which action would be appropriate to makefoods more appealing?

Incorrect: Sipping water before and during the meal will keep the mouth moistened forenhanced flavor.Incorrect: To amplify flavors in food use natural seasonings and fruit juices and avoid extra salt.Correct: A variety of food textures and flavors are more appealing.Incorrect: Foods should be chewed thoroughly to fully release the flavor. Avoid drinking water during the meal. Use extra salt to bring out natural food flavors. Vary food textures and flavors. Use foods that are pureed and do not need to be chewed. 25

What nutritional strategy may the nurse use to increase the appetite of the client who isreceiving chemotherapay?

Correct: It is important to increase the protein and calorie intake of the client receivingcancer treatment; provide small, frequent meals, seasoned according to individual taste, andnutrientdense liquids such as instant breakfast, milk shakes, or commercial supplements. They are easily consumed and digested as they leave the stomach easily and breakdown readily. Unless aggressive nutritional support is integrated into the early course of cancer treatment, profound effects, including cachexia and death, can result.Incorrect: Chemotherapy may cause

the client to have a metallic taste in the mouth. Plastic utensils, instead of metal, may help toreduce this unpleasant side effect. Tart foods like citrus juices, pickles, or vinaigrettes may help to overcome the metallic taste.Incorrect: To decrease nausea, the client may better tolerate serving foods cold or at room temperature. Hot foods may contribute to nausea.Incorrect: The

client receiving cancer treatment may experience sporadic anorexia. When the client has a good day with lessened nausea and anorexia, the nurse should encourage the client to eat whenever hungry. Eat frequent, small meals. Avoid using plastic utencils. Serve foods hot. Discourage overeating on "good" days.

26

The Food Pyramid was developed in 1992 as a nutritional guide for daily dietary intake. Which category contains the recommended serving of 6-11 per day?

Incorrect: An intake of 2-3 servings is recommended for the protein group.Incorrect: An intake of 2-3 servings a day or more is recommended for dairy products.Incorrect: Fruit recommendations are 2-4 servings a day.Correct: The bread, cereal, rice, and pasta group contains the most recommended servings of 6-11 per day. Meat, poultry, fish, dry beans, eggs, and nut group Milk, yogurt, and cheese group Fruit group Bread, cereal, rice, and pasta group

27

The nurse caring for the client with ascites related to cirrhosis of the liver may anticipate that which therapy may be used?

Incorrect: All alcohol should be eliminated in cirrhosis to halt liver damage.Correct: Diuretic therapy is a treatment for ascites. Monitor potassium levels with loop diuretic therapy.Incorrect: Dietary intervention to control the development of ascites is salt restriction. Salt leads to water retentions, which increases ascites.Incorrect: An indwelling urinary catheter is not needed to measure urinary output. Reduction of alcohol intake Diuretic therapy Fluid restriction Foley catheter for strict output monitoring 28

The dietary goals for the client with prolonged immobilization are to promote healing and avoid complications related to altered body metabolism. How may the nurse accomplish this objective?

Incorrect: Zinc is a part of numerous enzyme systems that function in tissue growth, maintenance, and healing. Vitamin C is necessary for collagen formation. Collagen is a protein in fibrous tissue, cartilage, bone, tendon, skin, and dentin. Collagen acts as intracellular cement for maintenance of growth and repair of tissue.Incorrect: A high protein is beneficial in preventing skin breakdown and muscle wasting. Amino acids are the building block for muscle and also play a role in the prevention of infection by bolstering the immune response. The recommended intake of protein is 1.2 gms/kg/day to decrease nitrogen excretion and prevent negative nitrogen balance.Incorrect: During periods of prolonged immobility clients may experience anorexia and may benefit from small, frequent meals.Correct: Altered calcium metabolism is not nutritionally related, but is due to immobility with decreased activity, resulting in increased calcium resorption and bone loss. Normal calcium metabolism is restored after

remobilization. Bone replacement can only occur if sufficient calcium is available afterremobilization. Limit vitamin C and zinc Decrease protein intake Offer large, balanced meals Provide normal calcium intake 29

Vitamin A deficiency may cause which of the following symptoms?

Correct: Xerophthalmia is the drying and thickening of the epithelial tissues of the eye, which can cause blindness. All epithelial tissues are damaged by vitamin A deficiency, including sinus, throat, and buccal membranes. Vitamin A is also necessary for proper bone growth. Therefore, a deficiency in this fat-soluble vitamin can lead to poor bone growth and other relatedcomplications such as brain and spinal cord injury.Incorrect: Tetany occurs when there is inadequate ionized serum calcium resulting in muscular contractions and pain.Incorrect: Adult vitamin D deficiency may lead to, or contribute to, the development of osteomalacia.Incorrect: Cataracts are an opacification of the lens of the eye. Xerophthalmia Tetany Osteomalacia Cataracts 30

Studies show that the dietary preferences of Asian cultures include rice and vegetables and contain about 15% fat. Americans who eat Chinese food typically do not lower their heart disease risk. Why?

Incorrect: Metabolic rates are individual and not necessarily related to ethnicity.Correct: Americans increase the fat in cooking and adding other ingredients.Incorrect: Chinese mealsinclude rice and vegetables and little meat.Incorrect: There is no evidence to support this statement. The metabolic rate of the Chinese person is higher. Americans prepare Chinese food by adding more fats and cheeses. Authentic Chinese meals omit meat completely. Chinese people naturally have a lower cholesterol level.

31

Riboflavin deficiency is characterized by inflammation of the mucous membranes of the mouthand lips. This clinical sign is called:

Incorrect: Pellagra is a deficiency disease due to the lack of niacin, (vitaminB2).Incorrect: Pernicious anemia is the inadequate formation of red blood cells due to the lackof cobalamin, (vitamin B 12).Correct: A deficiency of riboflavin (vitamin B 2) can lead to

cheilosis, glossitis, alopecia, scrotal or vulvar seborrheic dermatitis, normocytic anemia orpersonality changes. Riboflavin needs increase as protein needs increase, such as thosepersons with extensive burns or skin and tissue injury. A person who avoids all dairy products could be at risk.Incorrect: Scurvy will occur in two to three months with a diet deficient in vitamin

C. The lack of fresh fruits and vegetables in the diet can lead to this nutritional deficiency. The condition is characterized by hemorrhagic manifestations and abnormal formations of bonesand teeth.

pellagra. pernicious anemia. cheilosis. scurvy. 32

Which causative microorganism of foodborne illness results in symptoms that occur within 1236 hours after ingestion and include dizziness, thirst, paralysis and death?

Incorrect: Camploybacteriosis normally occurs 2-5 days after eating contaminated food and causes bloody diarrhea and pain.Correct: The botulism toxin is extremely dangerous and is common in home canned foods.Incorrect: Escherishia coli may cause bloody diarrhea, dehydration, kidney dysfunction, or death.Incorrect: Salmonellosis occurs 12-24 hours after eating contaminated food, causing diarrhea, chills, and fever. Campylobacteriosis Botulism Escherichia coli Salmonellosis 33

Which method is safe for the administration of medication through an enteral tube feeding?

Incorrect: Flush with at least 20 cc water before and after the medicine.Correct: Gastric irritation can be avoided by the dilution of certain medicines with water before administration.Incorrect: Consult with a pharmacist or physician before using a time-released capsule for medication with any tube-feeding route.Incorrect: Sublingual medicines should not be administered via tube. Flush tubing with 5 mL of water before and after the medication is instilled. Dilute certain medicines with water before administering. Administer time-released capsules through the feeding tube. Crush sublingual medicines thoroughly. 34

What chemical acts to facilitate the suspension of one liquid in another?

Incorrect: An essential fatty acid (EFA) is a fat that cannot be produced by the body and must be consumed in the diet.Incorrect: Triglycerides are the largest class of lipids found in food and body fat.Correct: Emulsifiers like bile have an extensive role: acting as a suspension, they facilitate fat absorption by creating smaller fat droplets with increased surface area.Incorrect: Adipose tissue is the stored form of fat.

Essential fatty acid Triglyceride Emulsifier Adipose tissue 35

Intermittent enteral feedings that are administered via a nasogastric tube placed into the stomach has which advantage over a tube enterostomy?

Incorrect: Nasogastric tubes have a higher risk for aspiration than other delivery routes. Regurgitation of the stomach contents can occur as a result of the relaxation of the sphincter. This relaxation occurs secondary to the presence of the tube. Slowed gastric emptying time, inhibited cough reflex, or high fat content of the formula are also contributing factors. Improper

feeding position may aggravate reflux and increase the risk for aspiration pneumonia.Correct: Nasogastric feeding tubes use the stomach as a reservoir. Digestion occurs in a physiologic manner with the stomach emptying and digestive enzyme release from the biliary andpancreatic structures. The dumping syndrome is less likely with formula delivery into the stomach. Transnasal tubes are indicated for short-term use (less than 6 weeks) with a functionalGI tract.Incorrect: Diarrhea may occur in tube-fed clients for a number of reasons, mainlyattributed to the formula rather than the route of delivery. Potential causes may include: formula too cold, lactose intolerance, bacterially contaminated formula, excess volume of formula, hypertonic formula, low serum albumin, and side effects to antibiotics or other medication.Incorrect: Nasointestinal tubes (NI) are used because of enhanced absorption offormula even when digestion is impaired. Little risk of aspiration exists with feedings placeddirectly into the jejunum. Reduced risk of aspiration Normal GI function Less diarrhea Enhanced absorption36

Which category of vegetarianism consists of a food plan that includes only plant foods and dairyproducts?

Correct: The lacto-vegetarian food plan allows for cheese, milk, yogurt, and butter.Incorrect: A vegan food plan consists of only plant food and may lead to development of vitamin B12 deficiency.Incorrect: An ovo-lacto vegetarian food plan consists of plant foods, plus dairy and eggs.Incorrect: A pollo-vegetarian food plan allows limited quantities of chicken. Lacto-vegetarian

Vegan Ovo-lacto vegetarian Pollo-vegetarian37

What is the food labeling term for processed food that means one-third fewer calories or onehalf of the original fat?

Incorrect: Foods that are labeled fat-free contain less than 0.5 grams of fat per servingand tiny or insignificant amounts of cholesterol, sodium, sugar, and/or calories.Incorrect: Low fatmeans no more that 3 grams of fat are contained per serving.Correct: Descriptor terms like lightmust now meet the legal definitions developed by the federal government. Foods that arelabeled light contain one-third fewer calories or one-half the fat of the original product.Incorrect: The definition of extra lean means the food contains <5 grams fat, <2 grams of saturated fat, and <95 mg of cholesterol per serving. Fat free Low fat Light/lite Extra lean 38

The nurse who is providing discharge instructions for the client with diverticular diseaseprovides information about dietary modification and lifestyle changes for long-term prevention ofthe disease. What type of diet should the nurse recommend?

Incorrect: A low fiber diet may be the cause of diverticular disease. Over 35 gm of fiber are recommended daily.Correct: Fiber reduces symptoms for clients with diverticular disease and by producing soft, bulky stools that are easily passed. The benefit is decreased pressure in the colon, easing the feeling of bloating and distention. However, once a client has diverticular disease, there are foods that can become trapped in the diverticulum and cause inflammation. Such foods include: nuts, seeds, popcorn, tomatoes, and corn.Incorrect: A low residue diet is used for the acute phase of diverticular disease or with complications of intestinal bleeding, perforation, or abscess. Clients are treated with a low residue diet initially, but are switched to high fiber diet upon discharge.Incorrect: Iron intake may need to be increased or supplementedas clients with diverticular disease are at increased risk of anemia related to intestinal bleedingor perforation. Low fiber diet High fiber diet Low residue diet Low iron diet 39

Iron absorption can be enhanced by:

Incorrect: Ingestion of caffiene contained in coffee and tea can hinder iron absorption.Incorrect: Antacid use hinders iron absorption by competing for iron absorption sites.Correct: Vitamin C increases the absorption of iron. It acts with hydrochloric acid to keep iron in the most absorbable form (ferrous) and facilitates its uptake in the intenstine.Incorrect:

Foods rich in calcium impair the absorption of iron and should be avoided with iron rich foodintake. coffee and tea ingestion. chronic antacid use. drinking orange juice with iron-rich food. drinking milk with iron-rich food. 40

A strategy to decrease body fat in the middle age years is to:

Correct: Regular exercise may decrease body fat.Incorrect: An increase in fat is not a given fact.Incorrect: Vitamin supplementation has no effect on fat storage.Incorrect: Protein needs remain constant during middle years. increase regular exercise for calorie expenditure. accept that there is no way to prevent this inevitable fat increase. supplement with vitamins. increase the amount of protein.

41

Maternal smoking may lead to which complication?

Incorrect: Fetal alcohol syndrome is a condition characterized by mental and physicalabnormalities in the infant caused by ingestion of alcohol during pregnancy. Typical manifestations include a characteristic facies noted by a flattened nasal bridge, thin vermillion ofthe lip and absent philthrum.Incorrect: Smoking does not appear to cause pregnancy inducedhypertension (PIH).Correct: Low birth weight, prematurity, abruptio placentae and increased perinatal mortality are all associated with maternal smoking. The mechanism for low birth weightis the vascoconstrictive effects of nicotine, which leads to uteroplacental insufficiency. A

compromised blood supply to the growing fetus results in impaired growth, affecting the weight parameter first. The fetus exposed to maternal nicotine use may suffer asymmetric intrauterine growth retardation.Incorrect: Gestational DM is associated with family history, maternal age, andobesity. Fetal-alcohol syndrome Pregnancy-induced hypertension Low birth weight Gestational diabetes 42

The nurse and client collaboratively plan the dietary modifications necessary after acute renalfailure. Which food containing high potassium does the nurse counsel the client to avoid?

Incorrect: The potassium content of butter is not high.Incorrect: Strawberries are a low potassium food.Correct: Dried beans, peas, and soy are high in potassium.Incorrect: Apple juiceis not high in potassium. Butter Strawberries Dried beans Apple juice43

All childbearing women must have adequate dietary intake or supplementation of which Bcomplex vitamin to reduce incidence of neural tube defects in early pregnancy?

Incorrect: Pyridoxine or vitamin B6 is needed for energy production.Incorrect: Ascorbic acid (or vitamin C) is not a B complex vitamin.Incorrect: Tocopherol is fat-soluble vitamin E and a powerful antioxidant.Correct: Folic acid deficiency results in impaired cell division and proteinsynthesis. Increased folic acid needs occur during periods of rapid growth, including the fetalperiod, infancy, and adolescence. Pyridoxine Ascorbic Acid Tocopherol Folate 44

Which of the following statements best describes vitamins?

Incorrect: Vitamins are organic molecules.Correct: Vitamins are essential micronutrients and must be provided through dietary intake.Incorrect: Each vitamin performs a specificmetabolic function.Incorrect: Vitamins are not synthesized by the body but are supplied in very small amounts in the diet. Vitamins are inorganic molecules. Vitamins are essential micronutrients. Each vitamin performs a wide variety of functions. Our bodies synthesize vitamins. 45

Glycosylated hemoglobin is sampled to determine the average blood glucose in the previous100120 days. This laboratory result is most likely to be altered by: Incorrect: Because the glycosylated hemoglobin represents the glucose level over athree-month period of time (the life cycle of a red-blood cell), it is used to determine the average

glucose control of the diabetic client. The level is not fluctuant with periods of fasting. Additionally, fasting causes hypoglycemia and should be avoided.Incorrect: Recent strenuous exercise would not change the glycosylated hemoglobin levels. The value represents the glucose level over a three-month period of time (the life cycle of a red-blood cell), and is used todetermine the average glucose control of the diabetic client. The level is not fluctuant with episodic acitivity.Correct: Use of an individualized meal plan is the dietary measure used to normalize daily glucose levels over time. Consistent glucose control is reflected in thislaboratory value.Incorrect: Infection increases the metabolic need thereby affecting gluconeogenesis, although an upper respiratory infection is typically short in duration and wouldnot affect the level of glycosylated hemoglobin. fasting. recent strenuous exercise. meal planning. upper respiratory infection.

46

A symptom of anorexia nervosa is refusal to maintain normal body weight through self-imposed starvation. Which is a symptom of anorexia nervosa?

Incorrect: Bingeing and purging is a feature of bulimia.Correct: The starvation causes hormone imbalances and reproductive disorders. Other symptoms of anorexia nervosa includeloss of 25% of body weight, decreased resting energy expenditure, pre-occupation with food, body image disturbances, intense fear of obesity, and abnormal sleep patterns.Incorrect:

Overeating and snacking can lead to obesity.Incorrect: Bulimia causes dental enamel erosion secondary to vomiting. Repetitive food binges accompanied by purging Absence of at least three consecutive menstrual cycles Eating inordinately large amounts of food Dental enamel erosion 47

The functional definition of dehydration is:

Incorrect: Fluid volume excess leads to fluid retention, edema, and thirdspacing.Incorrect: Hypervolemia describes fluid volume excess caused by water and sodiumretention in the same proportion that normally exists in the extracellular fluid. The isotonic retention of body fluid leads to edema, distended neck veins, adventitious breath, tachycardia, increased weight, polyuria, and shortness of breath.Correct: Dehydration refers to fluid volumedeficit whereby depletion of water occurs on a vascular, interstitial, and cellular level. Severe fluid volume deficit occurs when body fluid levels fall below 10% of body weight.Incorrect: Homeostasis is a state of internal physiologic balance of the internal environment achieved byautomatic monitoring and regulating mechanisms. fluid volume excess. hypervolemia. fluid volume deficit. homeostasis.

48

The liver is an important accessory organ for digestion. Which is a function of the liver?

Incorrect: Production of bicarbonate is a function of the pancreas.Incorrect: Nutrient andwater absorption is a function of the small intestine.Incorrect: Bile storage is a function of thegallbladder.Correct: The liver is a digestive organ that aids in the metabolism of all the energy nutrients and bile production, as well as, many other important functions. Production of bicarbonate to neutralize stomach acid Absorption of nutrients and water molecules Storage of bile for use in the digestion and absorption of fats Metabolism of many biological molecules for energy production49

The process of converting glycogen back to glucose is called:

Correct: Glycogen stored in the liver and muscle tissue is converted to glucose throughthe process of glycogenolysis.Incorrect: Glycogen is the carbohydrate energy form that is storedin the liver and muscles.Incorrect: Glycogenesis is the process of converting glucose toglycogen.Incorrect: Gluconeogenesis is the process of producing glucose from fat and protein. glycogenolysis. glycogen. glycogenesis. gluconeogenesis. 50

The majority of the digestion of fat occurs in which organ?

Incorrect: Only mechanical digestion of fats takes place in the mouth throughmastication.Incorrect: Peristalsis and some chemical digestion occur in the stomach.Correct: The majority of fat digestion occurs in the small intestine with the release of specificenzymes.Incorrect: Some fats are partially digested in the large intestine, and the rest passesthrough unchanged. Mouth Stomach Small intestine Large intestine51

The nurse evaluating the client's understanding of the signs and symptoms of tube feedingintolerance determines that further education is necessary if the client states:

Incorrect: Abdominal cramping, distention and diarrhea indicate feeding intolerance. The care provider should be notified to investigate the cause. Possible sources of the problem maybe: formula too cold, bacterially contaminated formula, lactose intolerance, feeding rate toorapid, volume of formula too great, hypertonic formula, nasogastric tube misplaced in duodenumcausing Dumping Syndrome, low serum alsumin, secondary to antibiotics, and highfatformula.Correct: Constipation is a common problem for the person receiving tube feedings. The low-residue content of the formula and decreased activity related to the feeding tube apparatusmay be contributing factors.Incorrect: A dry mouth is a common discomfort for the person receiving a tube feeding. NPO can cause dryness and irritation to the mucous membranes. Lipbalms may help. The nurse may encourage ice chips, gum, and hard candy, if possible, to stimulate salivation. Oral hygiene, including mouth rinses and dental care, can reduce halitosisand dryness of the mouth. The nurse can also encourage nose breathing as much as

possible.Incorrect: The upright position or elevated head during and after feedings reduces the rise of aspiration. The gastroesophageal sphincter relaxes due to the pressure of a feeding

tube, leading to regurgitation of the content of the stomach. Additionally, the cough reflex may be inhibited, which can increase the risk of aspiration pneumonia. A high-fat content of formula delays gastric emptying time. "I feel crampy, distended and have diarrhea. I will call my health care provider." "I have been constipated for two days. I will call my health care provider." "I often have a dry mouth. Hard candies and gum help to relieve it." "I will remain in an upright position for feedings and for half an hour or so afterwards." 52

The nurse educates the pregnant client to avoid increased supplementation of which vitamin or mineral?

Correct: Excessive vitamin A consumption during pregnancy can cause birth defects because the liver stores large amounts of vitamin A and toxicity results.Incorrect: The need for vitamin C is increased during pregnancy because it is necessary for the development of tissue proteins and the absorption of iron.Incorrect: Folate is important in the prevention of neural tube defects and should be supplemented in all pregnant women, particularly during the first trimester.Incorrect: Calcium needs during pregnancy increase because of bone development in the developing fetus. Vitamin A Vitamin C

Folate Calcium 53

The nurse educates an adolescent mother client regarding formula preparation. Which instruction is most accurate?

Incorrect: Tap water may be used and allows for varied formula temperature; however, caution is advised in the checking of the water temperature before feeding it to the baby. Bottled water is preferred if water is obtained through lead pipes.Correct: Microwaves tend to heat food unevenly and leads to hot spots in a liquid. Additionally, microwave cooking may destroy the structure of proteins and vitamins.Incorrect: Unused formula in bottles should be discarded after 2 hours because of the risk of contamination by saliva and bacteria. Prepared formula must be stored in the refrigerator until ready for use.Incorrect: Diluting formula can result in inadequate caloric and nutritional intake and eventually failure to thrive and malnutrition. The WIC supplementation program provides free formula under certain guidelines to prevent this practice. Tap water should never be used to prepare concentrate or powdered formula. Caution should be taken when formula is heated in a microwave. Unused formula is stable for up to twelve hours. Formula may be diluted with extra water to prolong its freshness. 54

Lifestyle habits that assist in the control of non-insulin dependent diabetes mellitus may include:

Correct: Nutrition therapy is the cornerstone of treatment for all diabetics. The spacing of meals throughout the day results in better stabilization of blood glucose levels. Traditionally, assumptions that sugar causes the blood sugar to rise too high and too quickly in the diabetic are no longer true. It is now known that the consistency of total carbohydrates consumed is the more important factor in glucose control. Eating meals 4-5 hours apart allows for postprandial glucose levels to return to the baseline.Incorrect: Exercise is another important aspect of the

care for the diabetic client, unless contraindicated for other medical reasons. Physical activity lowers serum glucose levels by increasing the number of insulin receptors for glucose control. To improve glucose sensitivity, it is recommended that diabetics exercise at least three to four times a week. Clients should be encouraged to exercise within two hours after eating and stop when symptoms of hypoglycemia develop.Incorrect: Blood glucose levels need more frequent monitoring, especially if an oral hypoglycemic is used.Incorrect: The diabetic client needs to plan nutritionally adequate meals, but total fat must be reduced, particularly saturated fats. The current recommendations for the distribution of calories are: 55-60% carbohydrates, 30% fats, and 12-20% proteins. spacing meals throughout the day. exercising once a week. monitoring blood glucose levels monthly. planning nutritionally adequate meals without restriction of fats. 55

What is the primary function of dietary proteins in the body?

Incorrect: Carbohydrates (CHO) are the primary energy source released from the cells. The metabolic process of the Kreb cycle results in the production of ATP. Proteins are not used as efficiently as CHO. The nitrogenous wastes resulting from protein metabolism burdens the kidney and requires energy for excretion. Additionally, sources of dietary protein are often high in fat.Incorrect: Fats enhance the palatability and flavor of food and influence its texture. Dietary

fat intake delays gastric emptying time and provides a feeling of satiety.Correct: The function of dietary protein is to provide adequate amino acids for the synthesis of protein in the body (anabolism). Proteins are used to repair, maintain, and synthesize new tissue. Proteins are also an important component of body secretions, hormones, plasma, protein enzymes, histamines, and antibodies. Protein is necessary for normal blood clotting. Fluid balance is regulated with albumin to maintain the oncotic pressure. Proteins also buffer acids and bases to regulate the acid-base balance. Proteins function as a transport vehicle through the blood. Dietary protein plays a minor role in energy production, although the process is inefficient and a burden on the kidneys for excretion.Incorrect: Temperature regulation through the process of evaporation is a function of water. Primary energy source Palatability of food Growth and maintenance of tissues Temperature regulation

56

What is a major advantage to the exchange lists for meal planning in diabetics?

Incorrect: Exchanging food selection from one group to another is not a recommended practice.Correct: The exchange list uses regular foods. The nurse must emphasize to the

client that they are not eating a diabetic diet, but rather a balanced meal plan.Incorrect: The meal plan needs to be re-evaluated at least every 3-6 months.Incorrect: All plans are individualized to each client's health care needs. Foods can be traded from one list to another. No special or diet foods are needed. The meal plan never needs to be modified or changed. All plans are based on 1500 kilocalories/day. 57

The nurse obtains a health history from a client who seeks health care for a routine wellnessassessment. What lifestyle conditions may the nurse identify as risk factors for coronary arterydisease?

Incorrect: Family history of coronary artery disease increases a person's risk two to fivefold. Genetic susceptibility emerges as a major factor in heart disease.Incorrect: Exercise raisesthe serum levels of high-density lipoproteins HLD's, which are the scavenger proteins that carrycholesterol to the liver for the manufacture of bile and later excretion in the feces. Additionally, exercise helps to control weight, perhaps the single determinant of cholesterol level.Incorrect: To lower serum cholesterol, the nurse should counsel the client to reduce total dietary fat and replace saturated fat intake with polyunsaturated fatty acids (PUFA).Correct: A high serum cholesterol level is associated with the development of atherosclerosis, which is a diseaseprocess involving cholesterol and lipid deposit in the walls of the vessels. The liver makes most of the cholesterol in the bloodstream from saturated fats. Therefore, a diet high in saturated fats has an impact on serum cholesterol levels. To reduce the risk of coronary artery disease expertsrecommend limiting the intake of saturated fat to less than 10% of the total calories. Negative family history Vigorous exercise High PUFA: saturated fat ratio Diet high in saturated fat58

Lifestyle modifications used in the treatment of essential hypertension include:

Incorrect: Persons with a genetic susceptibility to sodium-induced hypertension benefitfrom sodium restriction, not increase. Considering the average American diet contains 5 grams of sodium per day, a reduction to 2 grams per day should reduce a hypertensive client's blood pressure by 16/9 points.Correct: Smoking increases blood pressure and coronary artery disease(CAD) risks. The nicotine in cigarettes is a vasoconstrictor leading to increased blood pressure. Additionally, nicotine lowers the cigarette smoker's high-density lipoproteins (HDL), which is the type of cholesterol that acts as a scavenger to take cholesterol out of the blood.Incorrect: Potassium levels that are too high or too low can cause muscle weakness and lethal cardiacdysrythmias. Potassium is the major intracellular cation and is essential for contractions of theskeletal or cardiac muscles.Incorrect: Low to moderate levels of alcohol dilate peripheralvessels to lower blood pressure. High blood alcohol levels, more than 2 ounces per day, will have the opposite effect and raise blood pressure. increasing sodium intake. cessation of smoking. decreasing intake of potassium. limiting alcohol intake to 2 glasses per day. 59

The developmental task during the toddler period is gaining independence and autonomy. Which of the following dietary practices are most appropriate for this age group?

Correct: Children respond well to predictable routines but allow choices within foodgroups.Incorrect: Self-feeding is imperative to develop motor skills.Incorrect: Hot dogs, popcorn,

and many other foods may create choking hazards in this age group.Incorrect: Feeding from thebottle should be discontinued early in this age group to prevent dental caries and orthodonticmisalignment. Weaning from the bottle also contributes to the growing sense of independence.

Young children who wean from the bottle subsequently take in less of their total caloric intake from formula or breast milk, and substitute it with a solid nutritive source. Consistency of mealtimes is important. Self-feeding should be discouraged. Hot dogs and popcorn are introduced. Allow bottle-feeding to facilitate sleep. 60

Which is a complication of total parenteral nutrition (TPN) therapy?

Incorrect: Pneumothorax can occur with insertion of the central line into the superior vena cava. It is not actually a complication of the TPN fluid.Correct: Hyperglycemia is a metabolic complication that can occur as a result of high glucose load in the solution. It is the most frequent complication when beginning TPN therapy until the pancreas increases the production of insulin to meet the glucose load. When hypertonic solution is abruptly stopped, the high insulin levels can lead to a rebound hypoglycemia.Incorrect: Glucose in excess of body needs can lead to retention of carbon dioxide (hypercarbia) and acidosis.Incorrect: Renal function is not significantly altered. Instead, liver and gallbladder complications can occur. Total parenteral nutrition without oral intake for greater than one month causes gallbladder stasis and sludging of the bile. The effect of altered gallbladder contractility is cholelithiasis (gallbladder stone formation). Pneumothorax

Hyperglycemia Hypocarbia Renal dysfunction 61

Which nutritional source is considered a "Healthy Heart" selection?

Incorrect: Skim or 1%, and soymilk are lowfat choices.Incorrect: Tuna and canned fish should be packed in water.Correct: Soy, dried beans, and legumes may be used as protein substitutes.Incorrect: Self-basting turkeys are high in saturated fats. 2% milk Tuna Soy protein Self-basting turkey 62

Which intervention for monitoring stable clients receiving continuous parenteral nutrition is commonly used?

Incorrect: Intake and output is normally calculated over a 24-hour period.Incorrect: Monitor the TPN infusion rate closely to avoid systemic complications, such as cardiopulmonary overload, or local effects to the site including extravasation injury. Solutions that are infused too rapidly can cause hyperosmolar diuresis, leading to seizures, coma, or death. Solutions that are administered too slowly prevent optimal nutritional intake.Correct: The nurse inspects the solution for an oily appearance or a layer of fat on top of the solution, which is known as cracking. This can occur if the calcium and phosphorous content is high or if low-salt albumin is added. A cracked TPN solution cannot be infused. Notify the pharmacy or the clinician, who may need to adjust the contents of the solution.Incorrect: Baseline information for cholesterol and triglyceride levels are commonly obtained at the initiation of TPN therapy and continued every 1-2 weeks during the infusion as clinically indicated.

Calculate intake and output every 4-8 hours. Monitor the flow rate every shift. Inspect the solution for "cracking". Check the cholesterol and triglyceride levels daily. 63

The eating patterns of a preschool aged child are characterized by:

Incorrect: Toddlers attempt and learn to self-feed.Incorrect: The developmental phase of the toddler-aged child is characterized by a need to establish routines for daily living, including mealtime.Incorrect: Increased body fat is found in the pre-pubertal child.Correct: The preschoolaged child begins demonstrating independent eating styles modeled by the food preferences and dietary habits of adults . beginning attempts to self-feed. increased need for consistency of meal times. food selection leading to an increase of body fat. independent eating styles modeled by adults. 64

The nurse educates the client with gastroesophageal reflux disease (GERD) about ways to reduce heartburn. What strategy is likely to be effective?

Incorrect: Smoking relaxes the lower esophageal sphincter and may increase symptoms of heartburn.Incorrect: Drinking liquids and eating large meals may increase pressure to esophageal sphincter, which causes reflux symptoms.Correct: The client with GERD should avoid eating meals or snacks for at least 2 hours before lying down. This allows for stomach emptying and digestion to occur. Recurrent reflux is related to reduced lower esophageal sphincter pressure. The gastric acids and digestive contents backflow into the esophagus causing inflammation and discomfort.Incorrect: Foods that relax the lower esophageal sphincter and increase heartburn should be avoided. Some factors related to heartburn include: cigarette smoking, ingestion of alcohol, caffeine, chocolate, fat, peppermint, and spearmint oils. Smoke a cigarette after meals. Drink liquids with large meals. Wait two hours before lying down after a meal. Eat foods that relax the lower esophageal sphincter. 65

Which physical finding may indicate deficiency of potassium?

Incorrect: Osteoporosis refers to a loss of bone mass. The decreased bond density can result in fractures disproportionate to the degree of trauma. Osteoporosis is thought to be more

closely associated with a deficiency of calcium rather than with a deficiency of Vitamin D.Correct: Decreased potassium may cause muscle weakness, confusion, and cardiac arrhythmias.Incorrect: Thyroid enlargement or goiter can be associated with iodine deficiency.Incorrect: Dehydration and edema may occur from sodium and fluid imbalances. Reduced bone density Muscle weakness Thyroid enlargement Dehydration

66

The nurse understands that a reduction in protein and phosphorus may slow the progression ofchronic renal insufficiency. Which food is low in phosphorus?

Incorrect: Baked potatoes have high protein and phosphorus content.Correct: Blueberries, strawberries, and raspberries have very low phosphorus content.Incorrect: All nuts, including almonds, have high phosphorus content.Incorrect: All dairy products have high phosphorus content, including ice cream. Baked potato Blueberries Almonds Ice cream 67

Recurrent renal calculi may be reduced by which dietary intervention?

Incorrect: Clients must be instructed to restrict foods high in calcium, oxalates, andpurines. A low purine diet is used to prevent uricacid stone formation.Incorrect: Increased fiber in the diet binds with oxalate in the GI tract and thereby decreases urinary calciumexcretion.Incorrect: An increased fluid intake helps to dilute the concentration of the minerals in the urine. The high urinary output also acts to mechanically flush out the precipitates for stone formation. Fluid intake of 3-4 liters/day is recommended.Correct: A low oxalate, low calcium diet is used to prevent calcium or oxalate recurrence, leading to stone formation in the kidney.

Increased purine in the diet Decreased fiber intake Fluid restriction Low oxalate diet 68

What is the hormone secreted by the stomach which stimulates the secretion of hydrochloricacid to aid in digestion?

Correct: Gastrin is one of the most potent hormones released by the stomach. This hormone stimulates the secretion of HCl by the stomach to aid in digestion.Incorrect: Cholescytokinin is a hormone that is secreted causing the release of bile into the duodenum andthe secretion of pancreatic juice.Incorrect: Bile is a substance that emulsifies fats to aid in thedigestion of lipids.Incorrect: Secretin is a powerful stimulant for bicarbonate secretion by the pancreas. Gastrin Cholecystokinin Bile Secretin 69

The nurse and client collaboratively plan dietary modifications to achieve the goal of thereduction of body mass. The nurse understands that regarding food selection the most commonfactor for the general population cited was:

Incorrect: Price was an important factor to 75% of those surveyed although taste wasidentified as the most influential factor to 90% of those surveyed.Incorrect: Product safety wasan important factor to 75% of those surveyed although taste was identified as the mostinfluential factor to 90% of the sample.Correct: Taste was identified as the most influential factor determining food selection in 90% of those surveyed.Incorrect: Ease of preparation was

important to less than 50% of those surveyed. Taste was identified as the most influential factor determining food selection in 90% of those surveyed. price. product safety. taste. ease of preparation. 70

The nurse instructs clients taking MAO inhibitors to avoid foods and drugs that contain tyramine. Which food is high in tyramine?

Correct: Pickled and smoked foods are high in tyramine. When combined with an MAO inhibitor, they may cause a hypertensive crisis.Incorrect: Green vegetables are not high in tyramine.Incorrect: Citrus fruits do not contain tyramine. Avoid raspberries, avocados, and bananas which are moderately high in tyramine.Incorrect: Whole milk does not contain tyramine, but most aged cheeses, yogurt, and sour cream produced from milk are high in tyramine and should be avoided while taking MOA inhibitors. Smoked fish Green, leafy vegetables Citrus fruits Whole milk

71

Which fat-soluble vitamin is important in normal blood clotting?

Incorrect: Vitamin A maintains epithelial tissues, vision, bone growth, and reproduction.Incorrect: Vitamin D stimulates DNA to bind with calcium and phosphorus, which are necessary for bone mineralization.Incorrect: Vitamin E functions as an antioxidant to bind oxygen free radicals that can cause tissue damage.Correct: Vitamin K is necessary for the liver to make factors II (prothrombin), VII, IX, and X. These factors are key to the production of blood clots. Vitamin A Vitamin D Vitamin E Vitamin K 72

The nurse may anticipate providing which type of diet to the client with chronic diarrhea?

Incorrect: For the client with acute diarrhea a clear liquid diet is encouraged to allow the gastrointestinal tract rest and recovery. This dietary intervention is recommended for short-term treatment, generally 24-48 hours.Incorrect: Clients that have difficulty chewing or swallowing use the full liquid diet as a transition between clear liquid and soft diet.Correct: Chronic diarrhea can result in fluid and electrolyte imbalance, as well as metabolic acidosis, and progresses to weight loss and nutritional deficiency. A low-residue diet slows GI transit time and reduces stool bulk. Food high in pectin help to firm the stools (firm bananas, white rice, applesauce, dry toast). Increase protein, calorie, and potassium intake to replenish losses.Incorrect: A high fiber diet is used to alleviate constipation. The insoluble dietary fiber component is effective in promoting the passage of stool by increasing stool bulk and stimulating peristalsis. Clear liquid Full liquid Low-residue

High fiber73

Which is a common assessment finding of dehydration in an older adult?

Incorrect: Decreased skin turgor is a normal change that occurs with aging. The nurse would not anticipate increased skin turgor in the elderly client.Correct: Decreased urinary outputis an early sign of dehydration. It is one of the most sensitive and reliable indicators of fluidstatus in persons of all ages.Incorrect: Pulse is usually rapid in dehydration. The heart rate will increase to compensate for decreased blood volume.Incorrect: Skin is normally hot and dry withdehydration. The body compensates by conserving water and will not normally release the needed moisture with perpiration. Increased skin turgor Less frequent urination Slow, bounding pulse Cool, moist skin74

The nurse encourages which lifestyle modifications for relief of symptoms of peptic ulcerdisease (PUD)?

Incorrect: Do not drink caffeinated beverages on an empty stomach or before bedtime. Cigarette smoking should be avoided entirely.Correct: Elimination of spicy and acidic foods or

beverages is needed if it causes discomfort. Smoking is associated with PUD and delayshealing of the ulcer.Incorrect: Bedtime eating stimulates acid secretion during sleep; antacids have limited use in the treatment of PUD. The nurse should educate the client with PUD to avoid eating h.s.Incorrect: These measures are recommended to prevent Helicobacter pylori infection. Consume caffeinated beverages and cigarette smoking in moderation only. Eliminate spicy foods, alcohol, and smoking. Take antacids when eating prior to sleep. Be careful about consuming contaminated food and water. 75

The nurse is likely to educate the client with gallbladder disease to avoid which food?

Incorrect: Fat soluble vitamin deficiencies can occur as a result of impaired bilesecretion, making water soluble forms of vitamin A, D, E, and K necessary.Incorrect: Yogurt is low in fat. High fat foods lead to gallbladder stimulation and bile duct inflammation causing thepain associated with a gallbladder episode.Incorrect: Green leafy vegetables are healthy, low fat, and not contraindicated for the client with gallbladder disease.Correct: The role of dietary treatment of gallbladder disease is to reduce stimulation of the gallbladder by minimizing fatintake. Fatty, greasy foods cause symptoms of gallbladder attacks and should be strictly avoided. A low fat diet (20 to 60 gms/day of fat) is recommended to manage symptoms. Food with water-soluble vitamins Yogurt Green leafy vegetables Cheesecake

76

The majority of persons with diabetes have which type?

Incorrect: Diabetes Mellitus, type I, insulin-dependent diabetes, is the form that occurs inapproximately 5-10% of all diabetics.Incorrect: Gestational diabetes is associated with 2-5% ofall pregnancies.Correct: Diabetes Mellitus, type II, is the non-insulin dependent diabetes thatoccurs in approximately 90% of all diabetics.Incorrect: Secondary diabetes results from otherdiseases, genetics, trauma, medications and hormonal changes in approximately 2% of alldiabetics. Diabetes Mellitus, type I Gestational diabetes Diabetes Mellitus, type II Secondary diabetes77

Which assessment finding is typical of bulimia nervosa?

Incorrect: Anorexia nervosa is a mental disorder characterized by 25% or greater loss of usual body weight, an intense fear of becoming fat, and self-induced starvation.Incorrect: The intense fear of weight gain is a symptom of anorexia nervosa and results in refusal to maintainnormal body weight.Correct: Bulimia is characterized by excessive food intake followed byextreme methods to rid the body of foods eaten, such as the use of laxatives, diuretics, andpurging.Incorrect: Bulimia is characterized by excessive food intake followed by extrememethods to rid the body of foods eaten such as the use of laxatives, diuretics, and purging. The bingeing and purging activity is intermittent rather than the continued high calorie snacking andgrazing throughout the day. Weight loss leading to body weight <85% of that expected Intense fear of gaining weight and becoming fat Self-induced vomiting and misuse of laxatives and diuretics Snacking and grazing throughout the day on high fat foods78

A deficiency of thiamin characterized by muscle weakness, wasting or edema, anorexia, mental disorientation, and cardiac enlargement is known as:

Incorrect: Cheilosis is the inflammation of the mucous membranes of the mouth and lips, whereby the tissue becomes reddened and develops fissures at the angles. It is frequently seenin riboflavin (vitamin B2) deficiency.Correct: Beriberi is caused by the deficiency of thiamin (vitamin B1). It is characterized by neurologic, cerebral, and cardiovascular abnormalities. Without this vitamin, the myelin sheath covering the peripheral nerves eventually degenerates, resulting in paralysis and muscle atrophy. Thiamin is found in pork and whole grains. The enrichment of foods has almost eliminated the disease in this country, except in alcoholic persons.Incorrect: Glossitis is a general term describing inflammation of the tongue. Stomatitisand glossitis are seen with niacin (vitamin B3) deficiency. Some forms are herpetic in origin. Theacute form is characterized by thickened saliva, ulcer formation, fever, and malaise.Incorrect: Pellagra is a disease that occurs as a result of a niacin (vitamin B3) deficiency. It is characterized by cutaneous (dry, scaly skin); gastrointestinal (nausea, vomiting, and diarrhea); neurologic (organic psychosis, memory impairment, and confusion), and mucosal (ulcers in themouth, esophagus, and vagina) abnormalities. The condition may occur secondary to alcoholism or gastrointestinal disease. cheilosis. beriberi. glossitis. pellagra.

79

Malnutrition is a common problem of HIV infection and may be caused by:

Incorrect: Anorexia and nausea are physical symptoms that lead to malnutrition.Incorrect: Depression and dementia are psychosocial symptoms that contribute toanorexia leading to malnutrition.Incorrect: The side effects, quantity, and timing of medications often impair appetite and can cause malnutrition. Some medications cause a metallic taste; other types suppress the appetite. Nausea, vomiting, and diarrhea are frequent manifestationsof medication therapies.Correct: All factors contribute to malnutrition. As many as 80% of all clients with HIV or AIDs have malnutrition. Poor nourishment and ingesting less than bodily needs may accelerate the progression from HIV to AIDS. anorexia and nausea. depression and dementia. medications all of the above. 80

The passive movement of water across a semi permeable membrane from a less concentratedsolution to a more concentrated solution is called:

Incorrect: Diffusion describes the process whereby particles move from an area of high

concentration to that of lower concentration.Correct: Osmosis describes the movement of water across a semi permeable membrane from an area of fewer particles to one of moreparticles.Incorrect: Active transport occurs when a particle or substance binds with another, leading to movement within or external to the cell.Incorrect: Hydrostatic pressure is created bythe pumping action of the heart on the fluid in the blood vessels. diffusion. osmosis. active transport. hydrostatic movement.

ATI. MED-SURG. RENAL-URINARY

A nurse is educating a client about the management of mixed (urge and stress) incontinence. Which of the following statements made by the client indicates an understanding of theteaching?

Clients with mixed incontinence should be instructed to consume fluids throughout theday and avoid drinking a large amount at one time.Oxybutynin (Ditropan) decreases leaking ofurine, relieving symptoms of stress incontinence.Clients with incontinence should avoid caffeine because it stimulates the bladder and has a diuretic effect.Surgery does not cure mixed incontinence. "I should drink 16 oz of water with each meal." "My Ditropan will help my bladder to stop leaking urine." "I must limit my caffeinated coffee intake to 1 cup in the morning."

"My incontinence can be cured with surgery." 2

A client is admitted with nephrotic syndrome. The nurse should anticipate which of the following client findings?

Proteinuria is the primary finding in a client with nephrotic syndrome.Clients withnephrotic syndrome experience hypertension due to fluid retention.Exophthalmos is a conditionassociated with hyperthyroidism and is not associated with nephrotic syndrome.Clients withnephrotic syndrome experience hypoalbuminuria. Five g of protein in urine/24 hr Hypotension Exophthalmos Serum albumin 3.5 g/dL3

A nurse is providing instruction regarding reduced dietary intake of potassium to a client with renal failure. Which of the following client food selections contains the lowest potassiumcontent?

Cantaloupe is high in potassium: 494 mg per serving.Raw spinach is high in potassium: 470 mg per serving.Of the listed foods, an apple is the lowest in potassium: 158 mg perserving.Skim milk is high in potassium: 406 mg per serving. Cantaloupe Raw spinach Apple Skim milk

A nurse is providing education regarding cyclosporine (Sandimmune) to a client who had a renaltransplant 2 days ago. Which of the following statements by the nurse is appropriate?

One of the most common adverse effects of cyclosporine is hirsutism.Clients must take cyclosporine daily for the life of the transplanted organ.Renal damage may be intensified byusing nonsteroidal anti-inflammatory medications.Cyclosporine increases the risk of infection. Discontinuing cyclosporine places the client at risk for organ rejection. You may experience hair loss due to the medication therapy you'll be taking. "You will need to continue taking this medication to protect your new kidneys." Use an over-the-counter anti-inflammatory medication for aches and pains. You will be at an increased risk for infection if you stop taking this medication. 5

A nurse is planning care for a client who is scheduled to begin hemodialysis. Which of the following places the client at risk for seizures?

Hypokalemia does not place the client at risk for seizures.An increase in catecholaminesdoes not place the client at risk for seizures.A rapid decrease in fluid can result in cerebral edema and increased intracranial pressure, placing the client at risk for seizures.Hypocalcemiadoes not place the client at risk for seizures. Hypokalemia

A rapid increase of catecholamines A rapid decrease in fluid Hypocalcemia6

A client with renal failure has just completed a round of peritoneal dialysis (PD). Which of the following assessments should the nurse report to the primary care provider?

A 15 mm Hg drop in systolic blood pressure is an expected finding after dialysis due to fluid removal.A 1.5 kg (3.3 lb) weight loss is an expected finding due to fluid removal.Cloudy or opaque effluent is the earliest sign of peritonitis; therefore, the primary care provider should benotified immediately, because infection can be a life-threatening complication.The pain associated with PD is usually at the beginning of treatment, or the inflow of the dialysate, whenthe client first starts PD therapy. This pain usually resolves within 1 to 2 weeks of PD. 15 mm Hg drop in systolic blood pressure 1.5 kg (3.3 lb) weight loss Cloudy dialysate effluent Report of pain during inflow7

A nurse should understand that hemodialysis is contraindicated for a client who

Anticoagulants are required for clients receiving hemodialysis to prevent clot formation. Therefore, hemodialysis is contraindicated for a client who cannot receiveanticoagulants.Hemodialysis is not contraindicated for a client who is immobile.Hemodialysis isnot contraindicated for a client who is immunocompromised.Hemodialysis is not contraindicatedfor a client who is allergic to iodine. cannot receive anticoagulants. cannot ambulate. is immunocompromised.

is allergic to iodine. 8

A nurse is providing care to a hospitalized client who received hemodialysis 1 hr ago. When evaluating the client's status after dialysis, which of the following should the nurse assess first?

Potassium level should be assessed following hemodialysis. However, it is not the first priority.Body weight should be assessed following hemodialysis. However, it is not the first priority.It is important to assess the client's creatinine level. However, it is not the first priority.The greatest risk to this client is hypotension. Therefore, vital signs should be assessed first. Potassium level Body weight Creatinine level Vital signs 9

A nurse is caring for a client who is postoperative following a transurethral resection of the prostate with continuous bladder irrigation. Upon detecting an output obstruction, which of thefollowing should the nurse do first?

Irrigating the catheter with normal saline is appropriate, but it is not the first action thenurse should take.Notifying the primary care provider is appropriate, but it is not the first actionthe nurse should take.The greatest risk to the client is injury to the bladder. The first action the nurse should take is to turn off the continuous bladder irrigation to prevent additional fluid from entering the bladder.Providing PRN pain medication is appropriate, but it is not the first action the nurse should take. Irrigate the catheter with normal saline using a large syringe.

Notify the primary care provider. Turn off the irrigation system. Provide PRN pain medication. 10

A nurse is caring for a client who has been admitted to the medical surgical floor after a transurethral resection of the prostate. The client reports a strong urge to urinate. Which of the following actions should the nurse anticipate taking?

To maintain proper positioning of the catheter, the balloon must remain fully inflated.Decreasing the rate of bladder irrigation does not eliminate or decrease the urge to void. The rate of bladder irrigation needs to be maintained to prevent the formation of clots.Irrigating the catheter does not eliminate or decrease the urge to void. The catheter is irrigated to maintain patency.The urge to void is caused by bladder spasms, which can be treated with belladonna and opium suppositories. Remove 5 mL of fluid from the urinary catheter's balloon. Decrease the bladder irrigation rate. Irrigate the catheter. Administer a belladonna and opium suppository.

11

A nurse should plan to monitor which of the following clients for signs of nephrotoxicity?

Aminoglycoside antibiotics can injure cells of the proximal renal tubules. Aminoglycoside-induced nephrotoxicity usually manifests as acute tubular necrosis.Digoxin does not cause nephrotoxicity.Methylprednisolone does not cause nephrotoxicity.Propranolol does not cause nephrotoxicity. A client receiving gentamicin (Garamycin) for treatment of a wound infection A client receiving digoxin (Lanoxin) for treatment of heart failure A client receiving methylprednisolone (Solu-Medrol) for treatment of severe asthma A client receiving propranolol (Inderal) for treatment of hypertension 12

A nurse is planning dietary teaching for a client who has gout. The nurse should plan to advise the client to limit intake of which of the following to decrease the development of renal stones? (Select all that apply.)

Poultry is correct. Clients with gout need to limit foods high in purines, such as poultry

products.

Fish is correct. Clients with gout need to limit foods high in purines, such as fish.

Chocolate is incorrect. There is no restriction on chocolate for clients with gout.

Milk is incorrect. Milk is high in calcium and phosphate, which are not restricted for clients with gout.

Red wine is correct. Red wine is high in purines, so intake will need to be limited.Poultry is correct. Clients with gout need to limit foods high in purines, such as poultry products.

Fish is correct. Clients with gout need to limit foods high in purines, such as fish.

Chocolate is incorrect. There is no restriction on chocolate for clients with gout.

Milk is incorrect. Milk is high in calcium and phosphate, which are not restricted for clients withgout.

Red wine is correct. Red wine is high in purines, so intake will need to be limited. Poultry Fish Chocolate Milk Red wine 13

A client has just returned from a renal transplant. The nurse providing care should recognize which of the following client findings as a possible indication of a delay in functioning of thetransplanted kidney?

This blood pressure reading is within normal limits following a renaltransplant.Tenderness at the incision site is expected during the early postoperative period.Pink and bloody urine is an expected finding immediately after surgery.A minimum urine output of 30 mL/hr is desired following a renal transplant. Blood pressure 110/58 mm Hg

Incisional tenderness Pink and bloody urine Urine output of 50 mL/2 hr14

A nurse is obtaining a urine culture and sensitivity for a client with symptoms of a urinary tract infection. Which of the following actions should the nurse take?

The specimen cup must be sterile for a urine culture and sensitivity.The client should be instructed to pass a sterile container into the urine stream after initiating the flow of urine.A urinespecimen for culture and sensitivity does not need to be the first morning voiding.It is notnecessary to add a preservative to a specimen for culture and sensitivity. Collect the client' s urine in a clean specimen container. Instruct the client to initiate the flow of urine before collecting the specimen. Obtain the client' s first morning voiding on the following day. Place the client's urine specimen in a container with a preservative. 15

A nurse is providing dietary teaching to a client with chronic renal failure. Which of the following statements indicates that the client understands the teaching?

A client following a 1,200-calorie per day diet will not have adequate calories to prevent protein catabolism.A client with chronic renal failure cannot take milk of magnesia for constipation because it is too high in sodium.Weight gain may indicate fluid retention caused by poorly functioning kidneys. Weight loss may be the result of anorexia from uremic syndrome.Aclient with chronic renal failure cannot use a salt substitute because it contains potassium. I will follow a 1,200-calorie diet. I will take milk of magnesia if I'm constipated.

I will weigh myself each morning. I will use a salt substitute in my diet.

16

A client with chronic renal failure has laboratory results as follows: BUN 196 mg/dL, sodium 152 mEq/L, and potassium 7.3 mEq/L. Which of the following interventions should the nurseanticipate taking?

Fluids with normal saline should be restricted due to elevated sodium level.Spironolactone is a potassium-sparing diuretic. With chronic renal failure, potassium isretained at high levels.Regular insulin given IV with 10% dextrose in water may be administeredto move potassium out of the intravascular fluid back into intracellular fluid.Polystyrene sulfonateis an exchange resin that will absorb excess potassium. Initiate an IV of 0.9% normal saline. Give oral spironolactone (Aldactone). Provide NPH insulin by subcutaneous injection. Administer a sodium polystyrene sulfonate (Kayexalate) enema. 17

A client with diabetes mellitus presents to the emergency department with reports of costovertebral angle tenderness, nausea, and vomiting. For which of the following laboratoryvalues should the nurse notify the primary care provider?

The WBC count is greater than the normal value and indicates presence of an infection. This laboratory value should be reported to the primary care provider.A BUN of 15 mg/dL is within the normal range.A urine specific gravity of 1.020 is within the normal range.A urine pH of

5.5 is within the normal range. WBC 15,000/mm3 BUN 15 mg/dL Urine specific gravity 1.020 Urine pH 5.518 A client is admitted with a diagnosis of acute glomerulonephritis. Based on this diagnosis, the nurse should expect which of the following assessment findings?

A client with acute glomerulonephritis usually has sodium and fluid retention that leads to an elevated blood pressure.A client with acute glomerulonephritis usually has a decreased urineoutput.A client with acute glomerulonephritis usually has urine that is a dark, reddishbrown color.A client with acute glomerulonephritis usually gains weight due to fluid retention. Low blood pressure Polyuria Dark-colored urine Weight loss 19

A nurse is caring for a client with acute renal failure (ARF). Which of the following findings should the nurse expect? (Select all that apply.) BUN 30 mg/dL is correct. This BUN level is elevated. In ARF, the BUN may increase to 80 to 100 mg/dL within 1 week.

Urine output of 70 mL in past 3 hr is correct. In ARF, the client is usually oliguric (less than 400 mL/24 hr).

Potassium 3.6 mEq/L is incorrect. This potassium level is within the normal range. In ARF, the potassium level is usually elevated.

Serum calcium 9.8 mg/dL is incorrect. This serum calcium level is within the normal range. In ARF, the calcium level is usually decreased.

Hematocrit 30% is correct. This Hct is decreased, which is an expected finding for clients with ARF.BUN 30 mg/dL is correct. This BUN level is elevated. In ARF, the BUN may increase to 80 to 100 mg/dL within 1 week.

Urine output of 70 mL in past 3 hr is correct. In ARF, the client is usually oliguric (less than 400 mL/24 hr).

Potassium 3.6 mEq/L is incorrect. This potassium level is within the normal range. In ARF, the potassium level is usually elevated.

Serum calcium 9.8 mg/dL is incorrect. This serum calcium level is within the normal range. In ARF, the calcium level is usually decreased.

Hematocrit 30% is correct. This Hct is decreased, which is an expected finding for clients with ARF.

BUN 30 mg/dL Urine output of 70 mL in past 3 hr Potassium 3.6 mEq/L Serum calcium 9.8 mg/dL Hematocrit 30% 20

An older adult client presents to the clinic stating that she has urine leakage when she sneezes, coughs, or laughs. She says she is embarrassed by this. Which of the following should thenurse recommend to the client?

Citrus fruit intake will acidify urine to help prevent urinary tract infections. However, it will not prevent stress incontinence.Kegel exercises for stress incontinence strengthen the musclesof the pelvic floor, decreasing incontinence.Limiting fluid intake after dinner may help prevent nocturia but will not prevent stress incontinence.Nitrofurantoin is a broad-spectrum antimicrobialdrug used for urinary tract infections. This medication will not improve stress incontinence, which is caused by weakened muscles. Increase citrus fruit intake. Perform Kegel exercises. Limit fluid intake after dinner. Obtain a prescription for nitrofurantoin (Macrobid).

21

A client with nephrotic syndrome is taking prednisone. For which of the following assessment findings should the nurse monitor and report to the primary care provider?

Glucocorticoids depress the natural immune system and increase the client's risk for infection. A sore throat may indicate an infection.Frequent stools are not an adverse reaction related to prednisone therapy.Headache is not an adverse reaction related to prednisone therapy.Tremors are not an adverse reaction related to prednisone therapy. Sore throat Frequent stools Headache Tremors 22

A nurse is providing teaching to a client with urge urinary incontinence. The nurse should include which of the following instructions?

The sound of running water is a sensory stimulus that promotes normal micturition butdoes not help reduce urinary incontinence.Bladder training will teach the client to hold urinedeliberately until the scheduled times. By increasing the bladder's ability to hold and suppressurine, continence can be maintained.Scheduled toileting is successful in reducing incontinencein clients with urge incontinence. This method of bladder control includes establishing a toileting schedule based on the client's voiding pattern.Self-catheterization is used for functional urinaryincontinence, not urge incontinence. Sit on the toilet with water running every 4 hr. Respond immediately to the urge to void. Set an interval for toileting based on previous voiding pattern. Self-catheterize daily following a regular voiding. 23

A nurse is caring for a client who presents to a clinic with urinary urgency and dysuria. Which of the following additional findings supports a diagnosis of a urinary tract infection (UTI)?

Curd-like vaginal discharge is seen in a vaginal infection, not a UTI.WBCs present in theurine indicate infection.Glucosuria indicates hyperglycemia and is not a usual finding with aUTI.Elevated serum creatinine is a finding usually indicating renal impairment. It is not a usualfinding with a UTI. Curd-like vaginal discharge Pyuria Glucosuria Elevated serum creatinine 24

A nurse is caring for a client the night before a scheduled intravenous urography. Which of the following is the highest priority intervention?

The client must be NPO after midnight. However, this is not the highest priority.Obtaining the client's vital signs is important. However, this is not the highest priority.Administering a bowelprep is important. However, this is not the highest priority.Radiopaque contrast media may cause an allergic reaction in iodine-sensitive clients and is the greatest risk to a client's safety. Therefore, this is the priority action. Inform the client he must be NPO after midnight.

Obtain the client's vital signs. Administer a bowel prep to the client. Determine if the client has an allergy to iodine or shellfish. 25

A nurse is providing care for a client with acute glomerulonephritis. The nurse should plan to provide which of the following interventions?

Fluid retention should be monitored by weighing the client daily.The client's fluid allotment is based on her previous 24-hr urine output plus 500 to 600 mL.Clients with acuteglomerulonephritis need to conserve energy and should rest as much as possible.Phosphorouslevels are increased with acute glomerulonephritis. Therefore, the dietary intake should be decreased. Weigh the client on a daily basis. Encourage the client to drink 2 to 3 L of fluid/day. Instruct the client to ambulate frequently. Increase the amount of phosphorous-rich foods in the client's diet.

26

Following lithotripsy for urolithiasis for calcium phosphate renal calculi, a nurse should plan toinstruct the client to

The urine pH should be monitored to detect the acidity of the urine and the potential forcalcium precipitation.Ingestion of animal proteins may lead to acidic urine and the potential forcalcium precipitation.The client should drink at least 3 L per day to promote urine flow, decreaserisk of stone precipitation, and prevent dehydration.The client should strain urine to monitor forstone fragments as they leave the body. monitor urine for ketones.

increase intake of animal proteins. limit fluid intake to 1.5 L per day. strain all urine for possible stone fragments. 27

A client in acute renal failure exhibits a significant drop in blood pressure and urine output. The nurse should anticipate which of the following medication orders?

Low dose (1 to 3 mcg/kg) dopamine is given to clients with oliguric renal failure toenhance blood flow to the kidneys and to increase blood pressure.Captopril is used to reduceblood pressure; therefore, it is not an appropriate medication for this client.For clients with renalimpairment, sodium bicarbonate can cause systemic alkalosis. Absorption of sodium can increase blood pressure.Erythropoietin is given only to clients with chronic renal failure to treatassociated anemia. Dopamine (Intropin) Captopril (Capoten) Sodium bicarbonate Erythropoietin (Epogen) 28

A nurse is providing care to a client who is scheduled to undergo lithotripsy for urolithiasis. Which of the following actions should the nurse anticipate taking?

The client should be placed in a supine position on a flat x-ray table with the lithotripteraimed at the stone.The client is not intubated for lithotripsy. Conscious sedation is used.An indwelling urinary catheter is not necessary, but the urine should be strained to monitor the passage of stone fragments.Cardiac rhythm is monitored during lithotripsy by ECG. Shockwaves are delivered in synchrony with the R-wave on the ECG monitor.

Placing the client in a semi-Fowler's position Assisting with the client's intubation Inserting an indwelling urinary catheter Applying electrodes for cardiac monitoring29

A nurse is preparing a teaching plan for a male client who had a continent internal ileal reservoircreated to treat bladder cancer. Which of the following should be included in the teaching plan?

Creation of a continent internal ileal reservoir results in impotence in men. These clients are referred for additional treatment to address problems with sexual functioning.There is nosensation of bladder fullness in a continent internal ileal reservoir. The client must learn to void on a scheduled basis.There is no need to avoid certain foods. The continent internal ileal reservoir is not attached to the gastrointestinal tract.Intermittent catheterization is required todrain urine from the continent internal ileal reservoir. You can resume sexual intercourse when you feel ready. You should empty your new bladder when it feels full." You will need to avoid foods that produce intestinal gas. "You must insert a catheter through your stoma to drain the urine." 30

A nurse is caring for a client who is hospitalized with acute pyelonephritis. The nurse should anticipate including which of the following interventions when planning care for this client?

The client should drink 2 to 3 L/day to promote renal blood flow and urineproduction.Acute pyelonephritis is an acute bacterial infection that is treated with IVantibiotics.There is no indication to monitor the client's blood glucose level.There is no indicationto limit the client's dietary protein. The client should follow a balanced diet for adequate healing. Providing up to 1,500 mL of fluid/day Administering IV antibiotics Monitoring the client's blood glucose level Limiting the client's dietary protein intake

ATI. MED-SURG. CARDIOVASCULAR

A client presents to the emergency department with a blood pressure of 254/139 mm Hg. The nurse recognizes that the client is in a hypertensive crisis. Which of the following actions shouldthe nurse take first?

Obtaining laboratory values is an important action to assess the client's condition, but itis not the first action the nurse should take.Assessing the client's vision is an important actionfor a client in hypertensive crisis, but it is not the first action the nurse should take.Monitoringblood pressure is an important intervention for this client, but it is not the first action the nurseshould take.A client in hypertensive crisis requires IV antihypertensive medications to lower the blood pressure as quickly and safely as possible. Therefore, initiating an IV is the first action thenurse should take.

Obtain laboratory values. Assess vision.

Monitor blood pressure. Initiate an IV. 2

A nurse is reviewing a client's ECG monitor and notes that the client's rhythm has changed from a normal sinus rhythm to supraventricular tachycardia. The client is conscious with a heart rate of 200 to 210/min and has a faint radial pulse. The nurse should anticipate assisting with which of the following interventions?

A precordial thump is not indicated for supraventricular tachycardia.Vagal stimulation may help the client's heart return to a normal sinus rhythm temporarily.Atropine is not indicated for supraventricular tachycardia.Defibrillation is not indicated for supraventricular tachycardia. Delivery of a precordial thump Vagal stimulation Administration of atropine IV Defibrillation 3

A nurse is caring for a client who is 2 days postoperative following a heart transplant. Which of the following should the nurse include in the client teaching?

Heart transplant clients usually are no longer able to feel chest pain due to the denervation of the heart.The client's activity tolerance should gradually improve as the healing process progresses.The client will need to continue a restricted sodium and fat diet.The client will remain on immunosuppressants for the remainder of his life to help prevent rejection of the heart. You may no longer be able to feel chest pain. Your level of activity intolerance will not change. After 6 months, you will no longer need to restrict your sodium intake. You will be able to stop taking immunosuppressants after 12 months. 4

A nurse is preparing a client for coronary angiography. The nurse should report which of the following findings to the client's primary care provider prior to the procedure?

A hemoglobin of 14.4 g/dL is within the normal range and does not warrant reporting.This procedure involves access through large arteries into the heart and is not affected by peripheral arterial disease.The contrast medium used is iodine-based, and clients with a history of allergic reaction to shellfish often react to iodine.An output of 200 mL in 4 hr is within the normal range. Hemoglobin 14.4 g/dL

History of peripheral arterial disease Previous allergic reaction to shellfish Urine output 200 mL/4 hr 5

A nurse is planning a presentation about hypertension for a community women's group. Which of the following lifestyle modifications should the nurse include? (Select all that apply.) Moderate alcohol intake is correct. Alcohol intake should be limited for people with hypertension.

Regular exercise program is correct. A regular exercise program will help reduce blood pressure.

Moderate sodium intake is correct. Sodium should be limited for people with hypertension.

Reduced potassium intake is incorrect. There is no indication to reduce potassium intake.

Smoking cessation is correct. Smoking exacerbates hypertension.

Hormone replacement therapy post menopause is incorrect. Adding hormone replacement therapy may potentially increase blood pressure.Moderate alcohol intake is correct. Alcohol intake should be limited for people with hypertension.

Regular exercise program is correct. A regular exercise program will help reduce blood pressure.

Moderate sodium intake is correct. Sodium should be limited for people with hypertension.

Reduced potassium intake is incorrect. There is no indication to reduce potassium intake.

Smoking cessation is correct. Smoking exacerbates hypertension.

Hormone replacement therapy post menopause is incorrect. Adding hormone replacement therapy may potentially increase blood pressure. Moderate alcohol intake

Regular exercise program Moderate sodium intake Reduced potassium intake Smoking cessation Hormone replacement therapy post menopause

When caring for a client following coronary artery bypass graft surgery, the nurse should recognize which of the following as a sign of cardiac tamponade?

Sternal instability is an expected finding because of the splitting of the sternum duringsurgery.An increased white cell count may indicate an infection, but it is not related to cardiac tamponade.Pulsus paradoxus is an indicator of cardiac tamponade when the blood pressure is10 mm Hg or higher on expiration than on inspiration.Premature atrial contractions with anormal heart rate may be caused by caffeine intake and lack of sleep; however, this is not an indication of cardiac tamponade. Sternal instability Increased WBC count Blood pressure of 140/82 mm Hg on inspiration and 154/90 mm Hg on expiration Sinus rhythm with occasional premature atrial contractions and heart rate of 88/ min 7

A nurse is admitting a client who has a leg ulcer and a history of diabetes mellitus. Which of the following focused assessments will help the nurse differentiate between an arterial ulcer and a venous stasis ulcer?

Family history is important, but does not help to differentiate between arterial and venous ulcers.Both arterial and venous ulcers cause varying degrees of pain.Knowing aboutclaudication will help differentiate vascular ulcers. Clients with arterial ulcers experience claudication, while those with venous ulcers do not.Both arterial and venous ulcers have the potential for infections. Explore the client's family history of peripheral vascular disease. Note the presence or absence of pain at the ulcer site. Inquire about the presence or absence of claudication. Ask if the client has had a recent infection. 8

A nurse is caring for a client in the first hour following an aortic aneurysm repair. Which of the following may indicate shock and should be reported to the primary care provider?

Drainage during the first postoperative hours is expected to be serosanguineous and isnot a sign of shock.Coughing is painful during postoperative hours; however, it is not a sign of shock.Urine output less than 30 mL/hr may indicate shock because it reflects decreased bloodflow to the kidneys.A change in temperature is not associated with shock. Serosanguineous drainage on dressing

Severe pain with coughing Urine output of 20 mL/hr Increase in temperature from 36.8 C (98.2 F) to 37.5 C (99.5 F) 9

A nurse is caring for a client who has pulmonary edema related to heart failure. Which of the following findings indicates effective treatment of the client's condition?

Adventitious breath sounds occur when there is fluid in the lungs, therefore the absenceof adventitious breath sounds indicates that the pulmonary edema is resolving.A moist, productive cough usually accompanies pulmonary edema, however the presence of anonproductive cough does not indicate that the problem is resolving.The respiratory rate usuallydecreases while at rest, and therefore is not an indicator of effective treatment.This value is below the normal range, and is therefore not an indicator of effective treatment. Absence of adventitious breath sounds Presence of a nonproductive cough Decrease in respiratory rate at rest SaO2 86% on room air 10

A nurse is assessing a client who has left-sided heart failure. Which of the following manifestations should the nurse expect to find?

Increased abdominal girth is a finding related to systemic congestion resulting from rightsided heart failure.Weak peripheral pulses are related to decreased cardiac output resulting from left-sided heart failure.Jugular venous neck distention is a finding related to systemiccongestion resulting from right-sided heart failure.Dependent edema is a finding related tosystemic congestion resulting from right-sided heart failure. Increased abdominal girth Weak peripheral pulses

Jugular venous neck distention Dependent edema

11

When caring for a client in the first 8 hr following a coronary artery bypass graft (CABG) surgery, the nurse should report which of the following client findings?

Mediastinal drainage of up to 150 mL/hr is expected during this time.Elevated bloodpressure following a CABG procedure should be reported because increased vascular pressuremay cause bleeding at incision sites.A normal body temperature is expected following a CABG procedure.A potassium level of 3.8 mEq/L is within the normal range. Mediastinal drainage of 100 mL/hr Blood pressure of 166/94 mm Hg Temperature of 37.1 C (98.8 F) Potassium level of 3.8 mEq/L12

A nurse is caring for a client who is scheduled for a coronary artery bypass graft in 2 hr. Which of the following client statements indicates a need for further clarification by the nurse?

Clients scheduled for a coronary artery bypass graft should not take aspirin for at least aweek prior to the surgery to prevent excessive bleeding.Medication to treat high blood pressuremay be given prior to surgery if indicated by the primary care provider.Clients scheduled for a coronary artery bypass graft should not take anticoagulants such as warfarin (Coumadin) for atleast a week prior to the surgery to prevent excessive bleeding.An increased pulse rate isexpected when regularly scheduled digoxin (Lanoxin) is not given. Digoxin is usually withheld 12hr before surgery. My arthritis is really bothering me because I haven't taken my aspirin in a week. My blood pressure shouldn't be high because I took my blood pressuremedication this morning. I took my Coumadin last night according to my usual schedule. My pulse is faster because my doctor told me to stop taking my Lanoxin 2 daysago. 13

A client is receiving heparin therapy for treatment of deep vein thrombosis. Which of the following interventions should the nurse anticipate taking if the client's aPTT is 96 seconds?

Decreasing the heparin infusion flow rate is not the appropriate action to take.Continuingto monitor the heparin infusion as ordered is not the appropriate action to take.It is notappropriate to wait 2 hr before taking any corrective action.The aPTT level is above the therapeutic range. The heparin infusion should be stopped immediately to prevent harm to the client. Decrease the heparin infusion flow rate by 2 mL/hr. Continue to monitor the heparin infusion as ordered. Request another aPTT level in 2 hr. Stop the heparin infusion immediately.

14

A nurse is caring for a client who had an onset of chest pain 24 hr ago. The nurse should recognize that an increase in which of the following is diagnostic of a myocardial infarction (MI)?

While myoglobin is found after an MI, it is not specific to the cardiac muscle and may riseif other muscles are injured.C-reactive protein rises soon after the beginning of an inflammatoryprocess, such as rheumatoid arthritis.Creatine kinase-MB is the enzyme specific to themyocardium and is elevated when that muscle is injured.Homocysteine levels will rise as aprecursor to heart disease and may be used as a risk factor marker. Myoglobin C-reactive protein Creatine kinase-MB Homocysteine15

A client recently diagnosed with hypertension is prescribed an ACE inhibitor. The nurse instructsthe client about side effects of the medication. The client demonstrates an understanding of the teaching by stating that he will notify his primary care provider if he experiences which of thefollowing?

Bruising is not a side effect of ACE inhibitors.A persistent cough is an adverse effect of ACE inhibitors, and the medication should be discontinued if it occurs.Frequent urination is anexpected outcome of this medication.Constipation is a common side effect of calcium-channel blockers. Bruising Persistent cough Frequent urination Constipation

16

A nurse is observing a student nurse apply a transdermal nitroglycerin patch to a client who has a history of angina. Which of the following actions by the student nurse demonstrates a need forintervention?

Rotation of patch sites will prevent local irritation.Patches may be applied to any hairlessarea of skin.All traces of nitroglycerin should be removed and remain off for 10 to 12 hr per day.Dating and initialing the patch allows others to see when the patch was placed and when it needs to be taken off, as well as who applied the patch if verification is needed. Applies the new patch to the same site as the previous patch Places the patch below the client's right clavicle Removes the old patch and wipes the area Dates and initials the transdermal nitroglycerin patch17

A client with a history of deep vein thrombosis is receiving warfarin (Coumadin). Which of the following client findings provides the nurse with the best evidence regarding the effectiveness ofthe warfarin therapy?

A hemoglobin of 14 g/dL is a normal finding, but this is not the best evidence of effective warfarin therapy.Minimal bruising or no bruising is desired, but this is not the best evidence of effective warfarin therapy.Decreased circumference of the affected extremity is a desired effect, but this is not the best evidence of effective warfarin therapy.An INR of 2.5 is within the desired

therapeutic range and is the best indication of effective warfarin therapy. A hemoglobin of 14 g/dL Minimal bruising of extremities

Reduced circumference of affected extremity An INR of 2.5 18

A nurse is caring for a client following an abdominal aortic aneurysm resection. Which of the following is the priority assessment for this client?

Monitoring for neck vein distention is important, but it is not the priorityassessment.Monitoring bowel sounds is important, but it is not the priorityassessment.Monitoring for peripheral edema is important, but it is not the priorityassessment.The greatest risk to this client is graft occlusion or dissection. Monitoring urineoutput is the priority assessment. Neck vein distention Bowel sounds Peripheral edema Urine output19

A nurse is caring for a client admitted for treatment of left-sided heart failure with intravenous loop diuretics and digitalis therapy. The client is experiencing weakness and an irregular heart rate. Which of the following actions should the nurse take first?

Obtaining a current weight is important, but it is not the first action a nurse shouldtake.Weakness and irregular heart rate may indicate electrolyte imbalance. The first action the

nurse should take is to review the client's electrolyte values.Determining the time of the lastdigoxin dose is important, but it is not the first action a nurse should take.Checking the client'surine output is important, but it is not the first action a nurse should take. Obtain current weight. Review serum electrolyte values. Determine the time of the last digoxin dose. Check the client's urine output. 20

A nurse is caring for a client diagnosed with dilated cardiomyopathy. The client reports increasing difficulty completing her daily 1-mile walks. The nurse should recognize that this is a finding of which of the following?

Activity intolerance is a finding of left ventricular failure that is associated with dilatedcardiomyopathy.Peripheral vasodilation is not a finding associated with dilated cardiomyopathy.Pericardial effusion is not a finding associated with dilated cardiomyopathy.Decreased vascular volume is not a finding associated with dilated cardiomyopathy. Left ventricular failure Peripheral vasodilation Pericardial effusion Decreased vascular volume

21

A nurse in the emergency department is caring for a client who has experienced an anterior myocardial infarction. The client's history reveals he is 1 week postoperative. The nurse should recognize that which of the following interventions is contraindicated?

Administering IV morphine is an appropriate intervention related to painmanagement.Supplemental oxygen may increase myocardial tissue perfusion and is anappropriate intervention to assist the client.Using a bedside commode is less stressful thanusing a bedpan, and most clients are allowed to use a commode following a myocardialinfarction.The nurse should recognize that surgery within the previous 10 days is acontraindication for thrombolytic therapy. Administration of IV morphine sulfate Application of oxygen at 2 L/min per nasal cannula Helping the client to the bedside commode Assisting with thrombolytic therapy22

A client is admitted with a diagnosis of second-degree heart block. Which of the following assessment findings requires immediate intervention?

A bradydysrhythmia may lead to inadequate circulation causing dizziness and lightheadedness. This is a serious consequence that requires immediate intervention.Weight gain may result from decreased cardiac output related to second-degree heart block. However, this does not require immediate intervention.Diminished peripheral pulses may result fromdecreased cardiac output related to second-degree heart block. However, this does not require immediate intervention.While constipation may lead to increased straining and vagal stimulationwith defecation, it does not require immediate intervention. The client reports being dizzy and light-headed. The client experiences a weight gain of .02 kg (0.5 lb) in 1 day. The client's peripheral pulses are diminished. The client's last bowel movement was several days ago. 23

A nurse is reviewing the laboratory results of several clients diagnosed with peripheral arterial disease. The nurse should plan to provide dietary teaching for the client with which of the following laboratory values?

These laboratory values do not indicate the need for dietary teaching.These laboratoryvalues do not indicate the need for dietary teaching.Normal cholesterol is less than 200 mg/dL, normal HDL is above 40 mg/dL, and normal LDL is less than 130 mg/dL.These laboratory values do not indicate the need for dietary teaching. Cholesterol 130 mg/dL, HDL 70 mg/dL, LDL 90 mg/dL Cholesterol 145 mg/dL, HDL 50 mg/dL, LDL 120 mg/dL Cholesterol 170 mg/dL, HDL 35 mg/dL, LDL 165 mg/dL Cholesterol 190 mg/dL, HDL 55 mg/dL, LDL 125 mg/dL 24

A nurse is providing discharge teaching to a client with heart failure. Which of the following signsand symptoms should the client report immediately to the primary care provider?

A weight gain of 1 to 2 lb in a day is an indication of fluid retention resulting from worsening heart failure. This should be reported immediately.While an increase in systolic blood pressure of 10 mm Hg should be noted, this finding does not need to be reported immediately.If

dyspnea occurs on exertion, but not at rest, it does not need to be reportedimmediately.Medications taken by clients with heart failure may result in dizziness when rising too quickly, but this does not need to be reported immediately. Weight gain of 1 kg (2.2 lb) in 24 hr

Increase in systolic blood pressure of 10 mm Hg Dyspnea with exertion Dizziness when rising quickly25

A client with a history of angina presents to the primary care provider's office for a stress test at 1100. Which of the following statements by the client requires the nurse to contact the primary care provider for possible rescheduling?

It is not necessary for the client to be NPO prior to this procedure.It is appropriate for theclient to be instructed not to take cardiac medication prior to this procedure.Smoking prior to thetest may change the outcome and places the client at additional risk, so the test should berescheduled.Episodes of chest pain are not a contraindication to this test. I'm still hungry after the bowl of cereal I ate at 7 a.m. I didn't take my heart pills this morning because the doctor told me not to. I smoked a cigarette this morning to calm my nerves about having thisprocedure. I have had chest pain a couple of times since I saw my doctor in the office lastweek.

26

Following insertion of a permanent pacemaker, which of the following client statements indicatesa potential complication of the insertion procedure?

Hiccups may indicate that the pacemaker is stimulating the chest wall or diaphragm.

This could be caused by lead wire perforation and may indicate complications.Dizziness is not acomplication of the insertion procedure and may be normal initially as the client adjusts to thepacemaker.Pain or stinging at the incision site is an expected finding after insertion of a permanent pacemaker.Headache is not a complication of the insertion procedure; however, it may signal other problems related to the disease process. I can't get rid of these hiccups. I feel dizzy when I stand. My incision site stings. I have a headache. 27

Which of the following should a nurse recognize as a potential complication for a clientdiagnosed with endocarditis?

Ventricular depolarization occurs during a normal cardiac cycle and is not a potential complication of endocarditis.Guillain-Barr syndrome is not a potential complication ofendocarditis.Myelodysplastic syndrome is not a potential complication of endocarditis.Valvular disease or damage often occurs as a result of inflammation or infection of the endocardium. Ventricular depolarization Guillain-Barr syndrome Myelodysplastic syndrome Valvular disease 28

A nurse is providing health teaching to a group of clients. Which of the following clients is at risk for development of peripheral arterial disease?

Hypothyroidism is not associated with peripheral arterial disease.Diabetes mellitusplaces the client at risk for peripheral neuropathy and progressive peripheral arterialdisease.Twenty-five percent is within the recommended range for daily fat intake and therefore, this diet does not place the client at risk for development of peripheral arterial disease.Two bottles of beer a day is considered moderate alcohol intake and does not place the client at riskfor development of peripheral arterial disease. A client with hypothyroidism A client who has diabetes mellitus A client whose daily caloric intake consists of 25% fat A client who consumes two bottles of beer a day 29

A client being treated for heart failure is taking digoxin (Lanoxin) and furosemide (Lasix). The nurse should plan to monitor for which of the following as an adverse reaction to thesemedications?

Digoxin and furosemide are used to manage shortness of breath. This is not an adverse reaction to these medications.Furosemide can cause a substantial drop in blood pressureresulting in light-headedness.A dry cough is not an adverse reaction to digoxin and furosemide.A metallic taste is not an adverse reaction to digoxin and furosemide. Shortness of breath Light-headedness Dry cough Metallic taste 30

A client admitted with a diagnosis of heart failure experiences atrial fibrillation. The nurse shouldplan to monitor for and report which of the following findings to the primary care provider?

Slurred speech may indicate inadequate circulation to the brain because of an embolus. This should be reported to the primary care provider.An irregular pulse is an expected finding fora client with atrial fibrillation and does not need to be reported.Dependent edema is an expectedfinding for a client with heart failure and does not need to be reported.Fatigue is an expectedfinding for a client with heart failure and does not need to be reported. Slurred speech Irregular pulse Dependent edema Extreme fatigue

ATI. MED-SURG. RESPIRATORY

A client is diagnosed with acute respiratory failure. The nurse should expect which of the following laboratory findings?

Respiratory failure results in respiratory acidosis, causing a decreased pH level.Thecarbon dioxide level will rise with acute respiratory failure.Oxygen saturation will decrease withacute respiratory failure.The PaO2 is decreased with acute respiratory failure.

Arterial pH 7.50 PaCO2 25 mm Hg SaO2 92% PaO2 58 mm Hg2

A nurse should plan to administer which of the following medications to a client during an acute asthma attack?

Cromolyn sodium is an anti-inflammatory agent used for maintenance therapy ofasthma.Albuterol is a short-acting beta2 agonist, which acts quickly to produce bronchodilationduring an acute asthma attack.This inhaler combines a glucocorticoid (fluticasone) and a longacting beta2 agonist (salmeterol) for maintenance therapy of asthma.Short-term use ofprednisone may be indicated after a severe exacerbation for anti-inflammatory effects. Cromolyn sodium (Intal) Albuterol (Proventil) Fluticasone and salmeterol (Advair Discus) Prednisone (Deltasone) 3

When planning the care for a client with COPD, the nurse should include which of the followinginterventions?

Respiratory treatments should be scheduled before meals.Short activity periods morefrequently are better for the client.Clients with COPD have difficulty obtaining enough caloriesand protein due to fatigue and early satiety. Therefore, the food that is consumed should be highin calories and protein.Pacing activities is important to prevent fatigue, but short rests betweenactivities are better than long activities and long rests. Schedule respiratory treatments following meals. Have the client sit in a chair for 2-hr periods three times a day. Provide a diet that is high in calories and protein. Combine activities to allow for longer rest periods between activities. 4

A nurse is caring for a client following the insertion of a chest tube. The nurse should plan to have which of the following items in the client's room?

It is not necessary to have an extra drainage system in the client's room.Suture removal is not indicated following the chest tube insertion.The nurse should plan to have a pair ofpadded clamps in the event that the tubing becomes disconnected.Nonadherent pads are notnecessary following the insertion of a chest tube. Extra drainage system Suture removal set Pair of padded clamps Nonadherent pads5

A nurse is assessing a client who has a chest tube in place following thoracic surgery. Which of

the following findings indicates a need for further action? Fluctuation of drainage in the tubing is an expected occurrence withinspiration.Continuous bubbling in the water seal chamber suggests an air leak, indicating a

need for further action.Drainage greater than 100 mL in 1 hr would require further action.Small, dark-red clots are a common finding after chest surgery. Fluctuation of drainage in the tubing with inspiration Continuous bubbling in the water seal chamber Drainage of 75 mL in the first hr after surgery Several small, dark-red blood clots in the tubing

A nurse is caring for a client who has COPD. Which of the following findings should the nurse report to the primary care provider?

Clients with COPD have low arterial oxygen levels due to the difficulty of oxygen movingfrom diseased lungs into the blood.A productive cough with green sputum indicates an infection. This should be reported to the primary care provider.Clubbing of fingers is a sign of chronic arterial oxygen levels, which is an expected finding for clients with COPD.Pursed-lip breathingshould be used by clients with COPD to improve oxygenation when exercising. An oxygen saturation of 89% Productive cough with green sputum Clubbing of fingers Use of pursed-lip breathing with exertion7

A nurse is caring for a client who was just diagnosed with a pulmonary embolism. Which of the

following interventions is the highest priority?

A client with a pulmonary embolism should be provided with a quiet environment to promote rest and conserve oxygen. However, this is not the highest priority.A client with a pulmonary embolism should use an incentive spirometer to improve oxygenation andventilation. However, this is not the highest priority.A client with a pulmonary embolism is at risk for cardiac dysrhythmias. However, applying electrodes for continuous cardiac monitoring is not the highest priority.A client with a pulmonary embolism is at greatest risk for respiratory arrest related to extension of the clot. Administration of heparin will prevent further clot formation; therefore, this is the highest priority. Provide for a quiet environment. Encourage use of incentive spirometry every 1 to 2 hr. Apply electrodes for continuous cardiac monitoring. Administer IV heparin at 1,300 units per hr. 8

A nurse is caring for a client receiving mechanical ventilation. The low pressure alarm sounds. Which of the following should the nurse recognize as a cause of the alarm?

An excess of secretions in the airway causes the high pressure alarm to sound.Kinks inthe tubing can cause an obstruction, which causes the high pressure alarm to sound.An artificialairway cuff leak interferes with oxygenation and causes the low pressure alarm to sound.Biting on the endotracheal tube causes the high pressure alarm to sound.

Excess secretions Kinks in the tubing Artificial airway cuff leak Biting on the endotracheal tube9

A nurse is caring for a client who has just had a thoracentesis. Which of the following is the priority assessment finding?

Hemoptysis is an important finding; however, it is not the priority assessment finding.Insertion site pain is an important finding; however, it is not the priority assessment finding.The greatest risk following a thoracentesis is development of a pneumothorax. Decreased breath sounds may indicate a pneumothorax and is the priority assessmentfinding.A temperature of 37.3 C (100 F) is an important finding; however, it is not the priority assessment finding. Hemoptysis Insertion site pain Decreased breath sounds Temperature of 37.3 C (100 F) 10

A nurse is providing discharge instructions to a client following a tracheostomy. Which of the following statements by the client indicates a need for further instruction?

This is an appropriate action to take following a tracheostomy.This is an appropriate cleaning technique.Accidental decannulation may be prevented by waiting to remove the oldties until the new ties are in place.Suction should only be applied on withdrawal of the catheterto prevent tracheal tissue trauma. I need to inspect the stoma for signs of infection or skin irritation.

I will clean the cannula with half-strength peroxide and rinse with saline. I can remove the old twill tape once the new tape is in place. I should apply suction while inserting the catheter into my tracheostomy.

11

A nurse is caring for a client with adult respiratory distress syndrome (ARDS). Which of the following assessment findings indicates that the client's work of breathing has worsened?

An increase in respiratory rate indicates increased work of breathing and the need forimprovement in oxygen delivery.An increase in oxygen saturation indicates that gas exchange isimproving.A decrease in carbon dioxide retention indicates that gas exchange is improving.Adecrease in adventitious breath sounds indicates a decrease in secretions. Increase in respiratory rate Increase in oxygen saturation Decrease in carbon dioxide retention Decrease in adventitious breath sounds 12

A nurse should recognize that which of the following factors places a client at risk for a pulmonary embolus?

The surgical procedure and prolonged immobilization will place the client at risk fordevelopment of a pulmonary embolism.The clot formation that might occur with an IV catheterwill not usually produce an embolus.An indwelling urinary catheter does not place the client atrisk for development of an embolus. In addition, the client is able to move about and is unlikelyto develop an embolus.An embolus is unlikely to form after a laparoscopic cholecystectomy. Total hip arthroplasty Peripheral IV catheter

Indwelling urinary catheter Laparoscopic cholecystectomy13

When caring for a client with lung cancer, the nurse should expect which of the following assessment findings?

Sputum may be blood-tinged if the tumor bleeds.Fremitus is usually increased due toairspaces being replaced with tumor tissue or fluid.Masses present in the lungs will sound dullor flat on percussion.Peripheral edema is usually not associated with lung cancer. Blood-tinged sputum Decreased tactile fremitus Resonance with percussion Peripheral edema14

A client who is postoperative is hypoventilating secondary to general anesthesia effects and incisional pain. Which of the following ABG values support the nurse's suspicion of respiratoryacidosis?

These ABG values do not indicate respiratory acidosis.These ABG values do not indicate respiratory acidosis.These ABG values do not indicate respiratory acidosis.These ABG values indicate respiratory acidosis. The pH is below 7.35, and the PaCO2 is greater than 45 mm Hg, which indicates respiratory acidosis.

pH 7.50, PO2 95 mm Hg, PaCO2 25 mm Hg, HCO3- 22 mEq/L pH 7.50, PO2 87 mm Hg, PaCO2 35 mm Hg, HCO3- 30 mEq/L pH 7.30, PO2 90 mm Hg, PaCO2 35 mm Hg, HCO3- 20 mEq/L pH 7.30, PO2 80 mm Hg, PaCO2 55 mm Hg, HCO3- 22 mEq/L15

A nurse is caring for a client diagnosed with bacterial pneumonia. In this situation, the nurse should expect which of the following assessment findings?

A cough related to pneumonia is usually productive in nature.This is a normal oxygen saturation, and with pneumonia, a level below 95% is expected.A temperature elevation is expected with bacterial pneumonia.In an attempt to increase oxygen levels, respirations willincrease, not decrease. Nonproductive cough SaO2 96% Temperature of 38.8 C (101.8 F) Bradypnea

16

A nurse is assessing a client with emphysema. The nurse should report which of the following client assessment findings?

Clients with emphysema have chronic fatigue due to poor oxygenation.Cyanosis of thelips indicates that the client is not being well oxygenated. This finding should be reported to the primary care provider.Chronic lung overinflation and diaphragm flattening lead to the appearance of a barrel-shaped chest, which is an expected finding in emphysema.Crackles inthe posterior chest are an expected finding with emphysema. Fatigue

Cyanotic lips Barrel-shaped chest Crackles in posterior chest17

A nurse receives an order from the primary care provider to perform nasopharyngeal suctioning for each of the following clients. The nurse should question the provider's order for which client?

Increased intracranial pressure often follows a closed-head injury. Nasopharyngeal suctioning can further increase intracranial pressure and should be avoided.Nasopharyngealsuctioning is not contraindicated for this client.Nasopharyngeal suctioning is not contraindicatedfor this client.Nasopharyngeal suctioning is not contraindicated for this client. A client admitted with a closed-head injury who is lethargic A client admitted with a fractured femur who is in severe pain A client admitted with a ruptured appendix who has a temperature of 39 C (102.2 F) A client admitted with emphysema who has a respiratory rate of 36/min 18

A nurse is caring for a client in the emergency department following chest trauma. Which of the following findings should the nurse recognize as indicating a tension pneumothorax?

Neck veins are not collapsed on the affected side.Neck veins are not collapsed on the unaffected side.The trachea does not deviate to the affected side.A pneumothorax will cause the trachea to deviate to the unaffected side. Collapsed neck veins on the affected side Collapsed neck veins on the unaffected side Tracheal deviation to the affected side Tracheal deviation to the unaffected side 19

A nurse is positioning a client with emphysema to promote effective breathing. The nurse shouldplace the client in which of the following positions?

A side-lying position makes breathing more difficult.The client should be encouraged to sit upright leaning slightly forward with both arms supported on the overbed table to ease thework of breathing.A semi-Fowler's position does not allow the client to lean forward to betterexpand the lungs.A supine position will place pressure on the chest and not allow for the best expansion. Lateral position with a pillow over the chest to support the arm High-Fowler's position with arms supported on the overbed table Semi-Fowler's position with pillows supporting both arms Supine position with the head of the bed elevated 15 20

A nurse is preparing a client for discharge following a bronchoscopy. Which of the following is the priority assessment?

The greatest risk to the client is aspiration due to a depressed gag reflex. The priority assessment is to determine the return of the gag reflex.Percussion of the lung fields is importantfollowing this procedure, but it is not the priority assessment.Auscultating heart sounds isimportant following this procedure, but it is not the priority assessment.Palpating peripheralpulses is important following this procedure, but it is not the priority assessment.

Assessing gag reflex Percussing lung fields Auscultating heart sounds Palpating peripheral pulses

21

A client with pulmonary tuberculosis is being discharged with a prescription for rifampin (Rifadin). The nurse should plan to include which of the following in the client's dischargeteaching plan?

Tinnitus is not a side effect of rifampin.The client will always have a positive purified protein derivative skin test, even after the disease is no longer active.Rifampin will turn the urineand other secretions orange.The medication should be taken on an empty stomach once a day. Ringing in the ears is expected. Purified protein derivative skin test results will improve in 4 months. Urine and other secretions will be orange in color. Medication should be taken with meals. 22

A nurse should plan to monitor a client receiving albuterol (Proventil) for which of the following side effects?

Hypokalemia is not a side effect of albuterol.Albuterol is used to treat dyspnea.Tachycardia is a common side effect, especially if albuterol is used excessively.Candidiasis is a side effect of inhaled glucocorticoids, such as beclomethasone (QVAR). Hypokalemia Dyspnea Tachycardia Candidiasis 23

A client who is postoperative develops an acute onset of severe chest pain that is worse with inspiration. The client is anxious and tachypneic. The nurse should anticipate taking which of the following actions first?

The greatest risk to this client is respiratory compromise. Therefore, the first action the nurse should take is to apply supplemental oxygen.Auscultating the lungs is an important action, but it is not the first action the nurse should take.Administering pain medication is important, butit is not the first action the nurse should take.Initiating heparin therapy is important, but it is notthe first action the nurse should take. Apply supplemental oxygen. Auscultate lung sounds. Administer pain medication.

Initiate heparin therapy. 24

A nurse is caring for a client in respiratory distress. Which of the following devices should the nurse use to provide the highest level of oxygen via a low-flow system? The oxygen flow rate via nasal cannula is 2 to 6 L/min and provides oxygen at aconcentration of 28 to 44%.A nonrebreather mask is made up of a reservoir bag from which the

client obtains the oxygen, a one-way valve to prevent exhaled air from entering the reservoirbag, and exhalation ports with flaps that prevent room air from entering the mask. This delivers greater than 90% FIO2 (fraction of inspired oxygen), which provides the highest level ofoxygen.A simple face mask delivers oxygen concentrations between 40 and 60% and has open exhalation ports that allow room air in and exhaled air out. It does not provide the highest levelof oxygen.The partial rebreather mask only delivers oxygen concentrations of 60 to 75%. The exhalation ports are open, which will allow room air in and exhaled air out. It does not providethe highest level of oxygen. Nasal cannula Nonrebreather mask Simple face mask Partial rebreather mask 25

A nurse is obtaining a blood sample for ABG determination from a client's radial artery. Which ofthe following interventions are correct when performing this procedure? (Select all that apply.) Holding pressure at the puncture site for 5 min is correct. It is necessary to hold pressureat the puncture site for 5 min to ensure adequate clotting and prevent bleeding.

Aspirating the specimen into a heparinized syringe is correct. Heparinization is required for anaccurate sampling result.

Inserting an air bubble into the syringe before capping is incorrect. Air in the sample will change the results and will not provide accurate results.

Transporting the specimen to the laboratory within 30 min is incorrect. The sample must be transported immediately.

Performing the Allen test prior to obtaining the specimen is correct. This prevents use of an artery that has insufficient blood flow, which can damage the hand.Holding pressure at the puncture site for 5 min is correct. It is necessary to hold pressure at the puncture site for 5 minto ensure adequate clotting and prevent bleeding.

Aspirating the specimen into a heparinized syringe is correct. Heparinization is required for anaccurate sampling result.

Inserting an air bubble into the syringe before capping is incorrect. Air in the sample will change the results and will not provide accurate results.

Transporting the specimen to the laboratory within 30 min is incorrect. The sample must be transported immediately.

Performing the Allen test prior to obtaining the specimen is correct. This prevents use of an artery that has insufficient blood flow, which can damage the hand. Holding pressure at the puncture site for 5 min Aspirating the specimen into a heparinized syringe Inserting an air bubble into the syringe before capping Transporting the specimen to the laboratory within 30 min Performing the Allen test prior to obtaining the specimen

26

When caring for a client immediately following a total laryngectomy for laryngeal cancer, the nurse should give priority to which of the following assessments?

The greatest risk to the client is airway obstruction. The priority assessment is to determine airway patency.Oxygen saturation is an important assessment, but it is not the highest priority.Breath sounds are an important assessment, but it is not the highest priority.Agag reflex is an important assessment, but it is not the highest priority. Airway patency Oxygen saturation Breath sounds Gag reflex27

A nurse is providing instruction to a client on how to use montelukast (Singulair) to treat chronic asthma. The nurse should recognize that the client understands the teaching when he states,

This medication should not be taken with each meal.This medication is not used to terminate an acute attack.This is not the appropriate schedule for taking thismedication.Montelukast is used for prophylaxis of asthma exacerbation and should be taken ona daily basis in the evening. I will take this medication with each meal."

I will take this medication during my asthma attacks." I will take this medication up to three times a day when I begin to wheeze." I will take this medication every evening, even when I do not have symptoms." 28

A nurse should take which of the following actions when providing endotracheal suctioning for a client who is in respiratory distress?

Suctioning should be performed using sterile technique.Rotating the suction catheterwhile withdrawing facilitates the removal of secretions that may stick to the sides of theendotracheal tube and therefore, this reduces the risk of tissue trauma.Suctioning of thenonsterile oropharyngeal cavity should be performed after suctioning of the endotracheal tube toprevent cross contamination.Sputum is not produced on a continual basis, and scheduledsuctioning is not necessary for most clients. Additionally, hypoxia can occur with frequent suctioning. Suction the client's endotracheal tube using clean technique. Remove the suction catheter using a rotating motion. Suction the oropharyngeal cavity prior to suctioning the endotracheal tube. Suction the client' s endotracheal tube every 2 hr. 29

A client diagnosed with active tuberculosis is hospitalized. Which of the following isolation precautions should the nurse plan to implement?

Tuberculosis is a respiratory infection that is spread through the air and requires airborne isolation.This type of isolation protects clients from outside infections, such as clients who arereceiving bone marrow transplants.This type of isolation is used for infections transmitted bydirect contact, such as scabies.This type of isolation is used for infections transmitted bydroplets, such as rubella.

Airborne Neutropenic Contact Droplet30

A client in acute respiratory failure is receiving mechanical ventilation. Which of the following is the priority assessment the nurse should use to evaluate effectiveness of the mechanical ventilation?

Blood pressure will provide important information regarding the client's circulatory status, but it is not the priority assessment to evaluate effectiveness of mechanical ventilation.Breath sounds will provide important information regarding tube placement and adequacy of airexchange, but it is not the priority assessment to evaluate effectiveness of mechanical ventilation.Arterial blood gases will provide important information regarding the acidbasebalance of the client's blood. This information will evaluate if the mechanical ventilator is providing adequate oxygenation to maintain lung function.Heart rate will provide importantinformation regarding the client's circulatory status, but it is not the priority assessment toevaluate effectiveness of mechanical ventilation. Blood pressure Breath sounds Arterial blood gases Heart rate

ATI. MED-SURG. GASTROINTESTINAL

A nurse is providing teaching to an older adult client who has mild diverticulitis. Which of the following client statements indicates understanding of the teaching?

Intestinal bleeding is not common with mild diverticulitis.The client should refrain fromany activity that increases intra-abdominal pressure, which can result in the perforation of thediverticula.A client with active diverticulitis should eat a low-fiber diet.Laxatives should be avoided because they increase intestinal motility. I should expect occasional bloody stools. I will remain active by working in my garden every day. I should eat foods that are low in fiber. I will use a mild laxative every day. 2

A client is admitted for an acute onset of bacterial gastroenteritis. The nurse should question a prescription for which of the following?

In an effort to allow the bacteria causing the gastroenteritis to be eliminated, medications such as diphenoxylate should be avoided because they decrease gastrointestinal motility.The

client with acute gastroenteritis is at risk for dehydration; fluid replacement isanticipated.Ciprofloxacin is an appropriate antibiotic that may be prescribed for bacterialgastroenteritis.The client with acute gastroenteritis is at risk for dehydration; fluid replacement isanticipated.

Diphenoxylate (Lomotil) 0.45% sodium chloride IV Ciprofloxacin (Cipro) 5% dextrose in water IV 3

Which of the following laboratory results should a nurse report to the primary care providerregarding a client who has had alcoholic cirrhosis for several years?

This albumin level is within the normal reference range and does not need to bereported.This INR is within the normal reference range and does not need to be reported.Thisbilirubin level is within the normal reference range and does not need to be reported.Increasedserum ammonia levels commonly occur in clients with cirrhosis. This may indicate portalsystemic encephalopathy and should be reported to the primary care provider. Albumin 4.0 g/dL INR 1.5 Bilirubin 0.2 mg/dL Ammonia 180 mg/dL4

A nurse is providing discharge teaching to a client with peptic ulcer disease. Which of the following should the nurse include in the teaching?

Sucralfate should be given on an empty stomach 1 hr before meals.It is not necessary toavoid lying down for 1 hr after each meal, and doing so may interfere with the client's ability toget adequate rest.Eating frequent, small meals and bedtime snacks stimulates gastric acidsecretion and should be avoided.NSAIDs deplete endogenous prostaglandins, resulting inmucosal injury by breaking down the mucosal barrier. Take sucralfate (Carafate) with meals. Avoid lying down for 1 hr after each meal. Eat six small meals daily, including bedtime snacks. Avoid taking nonsteroidal anti-inflammatory medications. 5

A nurse is teaching a client with cholecystitis about her diet after discharge from the hospital. The nurse should recognize that the teaching was effective when the client selects which of the following foods?

Eggs are high in cholesterol and should be avoided by a client with cholecystitis.Broccolitends to cause flatus and should be avoided by a client with cholecystitis.Milk is high in fat andcholesterol and should be avoided by a client with cholecystitis.Bananas are an acceptable foodchoice for a client with cholecystitis. Eggs Broccoli Whole milk Bananas 6

A client with alcoholism presents to the emergency department reporting a sudden onset of severe back pain with vomiting for the past 18 hr. Which of the following laboratory results indicates that pancreatitis is the cause of the client's symptoms?

A serum blood glucose of 110 mg/dL is within the normal range. Serum blood glucose levels are usually increased with pancreatitis.Amylase levels are increased with pancreatitis.AWBC of 9,000/mm3 is within the normal range. WBC levels are usually increased withpancreatitis.Bilirubin levels are increased with pancreatitis. Serum blood glucose of 110 mg/dL Increased amylase WBC of 9,000/mm3 Decreased bilirubin 7

A nurse is assessing a client who has acute hepatitis B. Which of the following is an expected assessment finding related to this condition?

Joint pain is an expected finding for a client with acute hepatitis B.Obstipation, or failureto pass stools, is an expected finding for a client with a complete bowel obstruction.This is anexpected finding for a client with a small bowel obstruction.This is an expected finding for aclient with intra-abdominal bleeding. Arthralgia Obstipation Epigastric abdominal distention Periumbilical grey-blue discoloration8

A client is receiving instruction from a nurse on how to manage chronic hepatitis C at home.

Which of the following statements made by the client indicates an understanding of the nursinginstructions?

The client with hepatitis C, regardless of the infectious state, is always at risk for furtherliver damage and should avoid all alcohol consumption.Acetaminophen (Tylenol) can cause additional liver damage and should be avoided.Special diets are not required for clients withhepatitis C.Since many medications are metabolized in the liver, the medication dosage may need to be reduced for a client with hepatitis C. I will avoid alcohol until I'm no longer contagious. I will avoid medications containing acetaminophen. I will try to eat more dark green vegetables. I will need increased doses of most medications. 9

A client who has portal-systemic encephalopathy (PSE) is admitted with an elevated ammonia level. The client asks the nurse if she can have a larger portion of beef for dinner. Which of the following is an appropriate response by the nurse?

Chicken is not an appropriate substitute for this client's diet. Vegetable proteins are preferred.This response is dismissive of the client's concern.Clients with increased ammonialevels due to liver disease should adhere to a low-protein diet.The client with PSE has increasednutritional requirements and calories should not be limited. Beef is too high in fat, but you can have chicken. I will discuss this with your primary care provider.

You need to limit your protein intake, but you can have more fruits or vegetables. You need to limit calories. Would you like some sugar-free gelatin? 10

A nurse is caring for a client admitted with cirrhosis. Which of the following is the highest priority assessment finding?

This is an expected finding for a client with cirrhosis.This is an expected finding for aclient with cirrhosis.The greatest risk to this client is hemorrhage due to impaired coagulation. Bloody stools are an indication of bleeding in the gastrointestinal tract; therefore, this is thehighest priority assessment finding.This is an expected finding for a client with cirrhosis. Spider angiomas Fetor hepaticus Bloody stools Peripheral neuropathy11

A nurse is caring for a client with ascites who has just undergone a paracentesis. Which of the following is the best indication that the treatment has been effective?

Increased abdominal fluid can limit the expansion of the diaphragm and prevent theclient from taking a deep breath. Once ascitic fluid is removed, the diaphragm will expand morefreely. This is the best indication that the treatment has been effective.Removal of 1,000 mL of fluid is an expected outcome of a paracentesis, but it is not the best indicator that the treatmenthas been effective.A blood pressure reading that is within the normal range indicates that the client is hemodynamically stable, but it is not the best indicator that the treatment has beeneffective.Increased activity tolerance is an expected outcome of a paracentesis, but it is not the

best indicator that the treatment has been effective. Decreased shortness of breath The removal of 1,000 mL of fluid Blood pressure of 110/80 mm Hg Increased activity tolerance12

A nurse is caring for a client who has a duodenal ulcer. Which of the following client reports should the nurse anticipate?

Pain associated with a duodenal ulcer is not described as spasms in the right lowerquadrant.Pain associated with a duodenal ulcer is not described as pressure in theepigastrium.Pain associated with a duodenal ulcer does not occur as soon as food has enteredthe stomach.Pain associated with a duodenal ulcer occurs when the stomach is empty, which is typically 1 to 3 hr after meals and during the night. Pain is described as spasms in the right lower quadrant. Pain is described as pressure in the epigastrium. Pain occurs as soon as food has entered the stomach. Pain occurs 1 to 3 hr after meals and during the night. 13

A nurse is assessing a client diagnosed with Crohn's disease. Which of the followingassessment findings should the nurse expect?

Steatorrhea (fatty stool) is common in clients with Crohn's disease.The client is more likely to have hypokalemia due to diarrhea.The client will most likely lose weight due to chronicinflammation and malabsorption.The client usually has constant pain located in the lower rightquadrant. Fatty, diarrheal stools

Hyperkalemia Weight gain Sharp epigastric pain14

A client has returned to the nursing unit following a gastrectomy. The nurse's priority is to assessfor

The greatest risk to the client is retention of gastric secretions that can lead to gastricdilation. This can be prevented by nasogastric tube patency. Therefore, this is the highest priority assessment.It is important for the nurse to assess for incisional drainage, but this is notthe highest priority assessment.It is important for the nurse to assess for postoperative paincontrol, but this is not the highest priority assessment.It is important for the nurse to assess forthe presence of bowel sounds, but this is not the highest priority assessment. nasogastric tube patency. incisional drainage. postoperative pain control. presence of bowel sounds. 15

A nurse is teaching a client how to prepare for a colonoscopy scheduled for the following afternoon. Which of the following should the nurse include in the teaching?

This is the correct procedure to prepare a client for a colonoscopy.This is not the

appropriate preparation prior to a colonoscopy.This is not the appropriate preparation prior to a colonoscopy.This is not the appropriate preparation prior to a colonoscopy. Drink clear liquids for 24 hr prior to the procedure, and then take nothing bymouth for 6 to 8 hr before the procedure. Drink full liquids for breakfast the day of the procedure, and then take nothing bymouth for 2 to 4 hr prior to the procedure. Drink an oral liquid preparation for bowel cleansing on the morning of theprocedure. Use an enema for bowel cleansing on the morning of the procedure.

16

A client is receiving information regarding dietary management to prevent dumping syndrome following a gastrectomy. The nurse should encourage the client to include which of the following foods in his diet?

Foods high in carbohydrates lead to rapid gastric emptying due to high osmolarity andshould therefore be avoided.Eggs are encouraged for a client with dumping syndrome, as highprotein diets are recommended.Beverages high in carbohydrates lead to rapid gastric emptyingdue to high osmolarity and should therefore be avoided.Salty foods, such as broths, should beavoided as they lead to rapid gastric emptying due to high osmolarity.

Milkshakes Eggs Cranberry juice Vegetable bouillon

17

A nurse is monitoring a client who has peptic ulcer disease. Which of the following assessment findings indicates peritonitis secondary to ulcer perforation?

This sign is associated with peptic ulcer disease, not acute peritonitis.A board-like, distended abdomen is a sign of peritonitis.This sign is indicative of subcutaneous intraperitonealhemorrhage.Diminished bowel sounds are characteristic of peritonitis. Bloody diarrhea Board-like abdomen Periumbilical cyanosis Increased bowel sounds 18

When assessing a client with colorectal cancer, the nurse should expect which of the following findings?

Hemoglobin is expected to be decreased due to occult intestinal bleeding.CEA levels are elevated in 70% of people with colorectal cancer.A hematocrit of 43% is within the normal reference range.A fecal occult blood test is often positive due to intermittent bleeding associatedwith the tumor. Hemoglobin 9.1 g/dL Decreased serum carcinoembryonic antigen (CEA) level Hematocrit 43% Negative fecal occult blood test19

A nurse is providing discharge teaching to a client who has undergone an ileostomy. The nurse should instruct the client to contact the primary care provider if which of the following occurs?

This is an expected finding that does not need to be reported to the provider.This is an expected finding that does not need to be reported to the provider.This is a sign of bowel ischemia that should be reported to the provider.The client can take action for this finding without contacting the provider. The client has a 1 kg (2.2 lb) weight gain. The ileostomy output is liquid. The stoma is a reddish-purple color. The ileostomy drainage bag is leaking. 20

A client who is being discharged with a new colostomy is concerned about flatus and odor. To avoid these problems, the nurse should instruct the client to consume which of the followingfoods?

Eggs can cause flatus and increase foul stool odor.Fish can increase foul stool odor.Yogurt can help prevent flatus.Broccoli can cause flatus and increase foul stool odor. Eggs

Fish Yogurt Broccoli

21

A client is admitted to the hospital with a diagnosis of acute pancreatitis. Which of the following is the priority intervention that the nurse should anticipate implementing?

Clients with acute pancreatitis are at risk for paralytic ileus. Insertion of a nasogastrictube will help prevent this complication, but this is not the highest priority.Clients with acute pancreatitis are at risk for infection but administration of an antibiotic is not the priorityintervention.Clients with acute pancreatitis are at greatest risk for pain and the priorityintervention is to provide patient-controlled analgesia with opioids.Clients with acute pancreatitisare at risk for hyperglycemia, but monitoring for this condition is not the highest priority. Insert a nasogastric tube. Administer ceftazidime (Ceptaz). Provide patient-controlled analgesia. Monitor for hyperglycemia. 22

A client with peptic ulcer disease is prescribed a medication to decrease the production of hydrochloric acid. A nurse should anticipate administering which of the following medications?

This medication destroys the cell wall of Helicobacter pylori and does not affect gastric acid secretion.This medication is an H2 antagonist that is used to decrease gastricacidsecretion.This medication is an antacid that neutralizes gastric acid.This medication is used toform a protective coating over the gastric ulcer. Bismuth subsalicylate (Pepto-Bismol) Ranitidine (Zantac) Aluminum hydroxide (Amphojel) Sucralfate (Carafate) 23

A client with GERD is being discharged from the hospital. The nurse recognizes that the client understands disease management principles when the client states,

Foods that promote gastric acid should be removed from the diet.Chewable antacidsshould be followed with a glass of water.Eating three meals a day without snacking is not appropriate for clients with GERD.The client is advised to remain upright for 1 to 2 hr aftermeals. I will decrease the amount of carbonated beverages I drink. I will avoid drinking liquids for 30 minutes after taking a chewable antacid tablet. I will eat three nutritious meals a day and avoid snacks between meals. I will lie down on my bed for at least 30 minutes after eating each meal. 24

A client has been diagnosed with GERD after an esophagogastroduodenoscopy. The nurse should recognize the need for further teaching when the client states,

Sleeping with the head of the bed elevated and on the left side prevents reflux.Water aerobics is an appropriate form of exercise for a client with GERD since it does not involveheavy lifting, straining, or bending over.Some medications, such as alprazolam (Xanax), can cause lower esophageal sphincter pressure and reflux.Large meals increase the volume andpressure in the stomach and delay gastric emptying. "I will sleep on my left side with the head of the bed elevated 6 inches." "I will start a water aerobics class."

"I can continue to take Xanax for anxiety when needed." "I will eat four to six small meals a day instead of three larger meals." 25

A nurse is caring for a client who has GERD and has a prescription for metoclopramide (Reglan). The nurse should plan to monitor for which of the following adverse effects?

Blood dyscrasias are not side effects of metoclopramide.Hearing loss is not a side effect of metoclopramide.Tachycardia is not a side effect of metoclopramide.Like other dopamine agonists, metoclopramide may cause extrapyramidal reactions, such as involuntary movements. Blood dyscrasias Hearing loss Tachycardia Involuntary facial movements

26

A nurse is providing care to a client who had a traditional cholecystectomy yesterday. The nurse notes that the client's T-tube has drained 200 mL of greenish-brown fluid in the past 12 hr. Which of the following actions should the nurse anticipate taking?

The T-tube drainage bag should be below the level of the incision to promote drainage from the bile duct.Clamping the T-tube an hour before and after meals allows the nurse to determine if the client is able to tolerate food.The client should remain in a semi-Fowler's position to maintain drainage from the bile duct.This client's T-tube output does not indicate a need to evaluate for hypovolemic shock.

Raise the T-tube drainage bag above the level of the incision. Clamp the T-tube an hour before and after meals. Maintain the client in a supine position. Assess the client's vital signs for indications of hypovolemic shock. 27

A client is being admitted to the medical-surgical unit with appendicitis. Which of the following should the nurse expect to find when assessing this client? (Select all that apply.)

Oral temperature of 38.4 C (101.2 F) is correct. A client with acute appendicitis may have an elevated temperature.

White blood cell count of 6,000/mm3 is incorrect. WBC count is 10,000 to 18,000/mm3 in thepresence of acute appendicitis.

Loose, blood-streaked bowel movements is incorrect. Loose, blood-streaked bowel movementsare not an expected finding with appendicitis.

Recent history of nausea and vomiting is correct. Nausea and vomiting are frequently present inacute appendicitis.

Right lower quadrant pain is correct. Right lower quadrant pain is an expected finding ofappendicitis.Oral temperature of 38.4 C (101.2 F) is correct. A client with acute appendicitis may have an elevated temperature.

White blood cell count of 6,000/mm3 is incorrect. WBC count is 10,000 to 18,000/mm3 in thepresence of acute appendicitis.

Loose, blood-streaked bowel movements is incorrect. Loose, blood-streaked bowel movementsare not an expected finding with appendicitis.

Recent history of nausea and vomiting is correct. Nausea and vomiting are frequently present inacute appendicitis.

Right lower quadrant pain is correct. Right lower quadrant pain is an expected finding ofappendicitis. Oral temperature of 38.4 C (101.2 F) White blood cell count of 6,000/mm3 Loose, blood-streaked bowel movements Recent history of nausea and vomiting Right lower quadrant pain28

A nurse is caring for a client admitted with reports of abdominal pain and frequent, bloody stools. The primary care provider suspects that the client has ulcerative colitis. The nurse shouldrecognize that which of the following is a risk factor for ulcerative colitis?

Clients with a long-term history of steroid use are more likely to have pepticulcers.Clients with pulmonary tuberculosis may have manifestations of intestinal tuberculosis, not ulcerative colitis.This is not the most frequently affected population.Ulcerative colitis generally occurs in clients from 15 to 25 and 55 to 65 years of age. Women are more often affected than men. The client has a long-term history of steroid use. The client has a history of pulmonary tuberculosis. The client is an older adult man. The client is an adolescent girl. 29

A client with cirrhosis of the liver has a daily prescription for lactulose (Cephulac). The nurse should plan to include which of the following when teaching the client how to administer thismedication at home?

Bloating is an expected side effect of lactulose.The purpose of administering lactulose is to promote the excretion of ammonia in the stool. The client will have more frequent stools to achieve this.The client with high ammonia levels should adhere to a low-proteindiet.Dehydration may result from increased stool frequency, so oral fluid intake should not be restricted. Notify the primary care provider if bloating occurs. Expect to have between two and five soft stools per day. Increase the intake of red meat and eggs. Limit oral fluid intake to 1,500 mL/day of clear liquids. 30

A nurse should plan to take which of the following actions when caring for a client following surgical placement of a portal-systemic shunt for treatment of cirrhosis?

The amount of ascites should decrease post placement of a portal-systemic shunt. Measuring abdominal girth daily will determine if this is occurring.The client with cirrhosis is atrisk for bleeding due to coagulation abnormalities caused by liver disease. IM injections

shouldbe avoided as much as possible to avoid the risk of bleeding.The client is at risk for fluidoverload because ascitic fluid is diverted to the venous system.Breath sounds should be

monitored more frequently than once every shift following this procedure due to the risk of fluid overload from reabsorption of ascitic fluid. Measure abdominal girth daily. Administer IM opioids for pain control. Monitor for signs of shock. Assess breath sounds once every shift.

ATI. MED-SURG. FLUID, ELECTROLYTE & ACID-BASE

A nurse is performing a neurological assessment on a client who has dehydration. The nurse notes that the client's mental status has changed, and she now appears confused and lethargic. Which of the following is the priority nursing action?

The nurse will need to document the change in mental status, but this is not the prioritynursing action.The nurse should continue to reassess the client's neurological status, but actionneeds to be taken sooner than in 2 hr.The IV flow rate will likely need to be increased, but this isnot the priority nursing action.The greatest risk to the client is worsening dehydration. The priority nursing action is to contact the primary care provider to obtain further orders. Document the change in status in the nursing notes. Reassess neurological status in 2 hr.

Increase the IV flow rate. Report this finding to the client' s primary care provider. 2

A nurse is caring for a client who has hypomagnesemia. Which of the following is an expected finding?

Hyperactive deep tendon reflexes are an expected finding for a client withhypomagnesemia.Decreased bowel sounds are an expected finding for a client withhypomagnesemia.Shallow respirations are an expected finding for a client withhypomagnesemia.Increased blood pressure is an expected finding for a client withhypomagnesemia. Hyperactive deep tendon reflexes Increased bowel sounds Deep respirations Decreased blood pressure3

A nurse notes that a client's serum sodium is 155 mEq/L. The primary care provider prescribes IV fluids. The nurse should question which of the following orders?

This isotonic solution should decrease serum sodium.This hypotonic solution shoulddecrease serum sodium.This hypertonic solution will increase serum sodium.This hypotonicsolution should decrease serum sodium. 1,000 mL 0.9% saline 1,000 mL 5% dextrose with water 1,000 mL 3% saline 1,000 mL 5% dextrose and 0.45% saline

Which of the following assessment findings should a nurse expect when caring for a client whohas a serum potassium level of 3.0 mEq/L?

Hyporeflexia is likely to occur with hypokalemia.Orthostatic hypotension is an expectedfinding with hypokalemia.Respirations become shallow with hypokalemia.Pulses are usuallyweak and thready with hypokalemia. Hyperactive deep tendon reflexes Orthostatic hypotension Rapid, deep respirations Strong, bounding pulse5

A nurse is planning dietary teaching for a client who has hypermagnesemia. Which of the following food choices should the nurse plan to instruct the client to avoid?

Eggs do not contain magnesium.Cheese does not contain magnesium.Rhubarb doesnot contain magnesium.Spinach contains 88 mg of magnesium per serving. Eggs Cheese Rhubarb Spinach

A nurse is providing teaching to a client who has a low serum calcium level and an allergy to milk. Which of the following statements made by the client indicates to the nurse that sheunderstood the teaching?

Cheese is a milk product and will not be tolerated.Vitamin D is required for calcium absorption and is not related to a milk allergy.The client should take the supplements with food to decrease gastrointestinal upset, not discontinue taking them.Green beans and broccoli aregood sources of calcium as alternatives to milk products. I will eat extra cheese because I can't drink milk. I need to avoid foods with vitamin D because I am allergic to milk. I will stop taking my calcium supplements if they irritate my stomach. I will add broccoli and green beans to my diet. 7

A nurse is caring for a client who has experienced excessive fluid loss. Which of the following should the nurse include in the plan of care? (Select all that apply.) Administer prescribed IV fluids is correct. A client with excessive fluid loss will most likely receive replacement fluids by IV.

Encourage the client to drink fluids is correct. A client with excessive fluid loss should drink fluidsif there is no nausea or vomiting.

Recommend that the client use mouthwash is incorrect. Mouthwash may dry the mucousmembranes out; therefore, causing more fluid loss.

Apply humidified oxygen is correct. Applying humidified oxygen will help prevent hypoxia.

Weigh the client each morning is correct. Weighing the client each morning will provide information regarding fluid balance.Administer prescribed IV fluids is correct. A client with excessive fluid loss will most likely receive replacement fluids by IV.

Encourage the client to drink fluids is correct. A client with excessive fluid loss should drink fluids if there is no nausea or vomiting.

Recommend that the client use mouthwash is incorrect. Mouthwash may dry the mucous membranes out; therefore, causing more fluid loss.

Apply humidified oxygen is correct. Applying humidified oxygen will help prevent hypoxia.

Weigh the client each morning is correct. Weighing the client each morning will provide information regarding fluid balance. Administer prescribed IV fluids. Encourage the client to drink fluids. Recommend that the client use mouthwash. Apply humidified oxygen. Weigh the client each morning. 8

A nurse is providing teaching to a client who has venous insufficiency of the lower extremities. The nurse evaluates that the teaching has been effective when the client makes which of the following statements?

The provider does not need to be called, as edema is expected with this diagnosis.Elastic stockings should be put on upon awakening and before getting out of bed in the morning.Venous insufficiency makes it difficult for blood to return to the heart, and elevating the feet will increase the return.The seams should be on the outside of the stocking, not next to the skin, to prevent skin breakdown. I'll call my doctor when I first notice any swelling. I'll put on my elastic stockings at the first sign of swelling. When I sit down to watch television, I'll be sure to put my feet up. When putting on my elastic stockings, I'll have the seam on the inside. 9

A client with heart failure is admitted following 3 days of vomiting. He is currently taking 40 mg furosemide (Lasix) and 0.125 mg digoxin (Lanoxin) daily. The nurse suspects hypokalemia. Which of the following should the nurse anticipate administering?

This medication is administered to treat hyperkalemia.This IV solution will provide adequate fluid and potassium replacement to offset the losses related to vomiting.There is no indication to administer an additional dose of digoxin.Lactated Ringer's solution will not provide adequate potassium replacement needed to offset the losses related to vomiting. Sodium polystyrene (Kayexalate) 30 g/day Normal saline with 40 mEq/L K+ at 100 mL/hr Digoxin 0.125 mg/day Lactated Ringer's solution at 100 mL/hr 10

A client is admitted with status asthmaticus. An ABG shows pH 7.32, PaO2 74 mm Hg, PaCO2 56 mm Hg, and HCO3- 26 mEq/L. The nurse correctly interprets these laboratory values as

Status asthmaticus causes inadequate gas exchange resulting in a low pH and PaO2, an elevated PaCO2, and a normal HCO3-. These laboratory values support respiratory acidosis.These laboratory values do not support respiratory alkalosis.These laboratory valuesdo not support metabolic acidosis.These laboratory values do not support metabolic alkalosis. respiratory acidosis. respiratory alkalosis. metabolic acidosis. metabolic alkalosis.

11

A nurse is caring for a client on continuous cardiac monitoring and identifies a prolonged PR interval and a widened QRS complex. Which of the following laboratory values supports thisfinding?

Hypernatremia does not cause a prolonged PR interval and widened QRS complex.Thischloride level is within the normal reference range and should not result in a prolonged PRinterval and widened QRS complex.This magnesium level is within the normal reference rangeand should not result in a prolonged PR interval and widened QRS complex.Hyperkalemia maycause a prolonged PR interval and widened QRS complex. Sodium 152 mEq/L

Chloride 102 mEq/L Magnesium 1.8 mEq/L Potassium 6.1 mEq/L12

A nurse is caring for a client who is receiving nasogastric suctioning. Which of the following sets of laboratory results indicates that the client is experiencing metabolic alkalosis?

An elevated pH and HCO3- with normal PaCO2 indicate metabolic alkalosis.Theselaboratory values do not indicate metabolic alkalosis.These laboratory values do not indicatemetabolic alkalosis.These laboratory values do not indicate metabolic alkalosis. pH 7.51, PaO2 94 mm Hg, PaCO2 36 mm Hg, HCO3- 31 mEq/L pH 7.48, PaO2 89 mm Hg, PaCO2 30 mm Hg, HCO3- 26 mEq/L pH 7.31, PaO2 77 mm Hg, PaCO2 52 mm Hg, HCO3- 23 mEq/L pH 7.26, PaO2 84 mm Hg, PaCO2 38 mm Hg, HCO3- 20 mEq/L13

A nurse is caring for a client who has heart failure and is receiving furosemide (Lasix). The clientis experiencing general muscle weakness and decreased bowel sounds. Which of the followingactions should the nurse anticipate taking?

Eating whole grain breads will not increase the potassium level.The findings indicatehypokalemia likely caused by furosemide. The nurse should anticipate a prescription for a

potassium-sparing diuretic.Potassium is not given by IV push, but in an IV solution to deliver 10mEq/L.There is no indication that fluid should be restricted. Offer whole grain breads with meals. Administer a potassium-sparing diuretic. Give potassium 20 mEq/L by IV push. Restrict oral fluids.

14

A nurse is caring for a client with a serum potassium of 6.2 mEq/L. The client is on potassium chloride (K-Dur), furosemide (Lasix), and has IV fluid infusing. Which of the following actionsshould the nurse anticipate implementing?

A potassium supplement should not be given to a client with an elevated potassium level.There is no indication to withhold the furosemide, as it can facilitate removal of excess potassium from the body.An IV solution with a high dextrose concentration and Regular insulin will cause the potassium to move from the blood into the cells and decrease the serumpotassium level.This client has an elevated potassium level and should not be receivingadditional potassium. Give the prescribed potassium chloride with meals. Withhold the prescribed furosemide each morning. Administer D50W and Regular insulin IV. Add potassium to the IV fluid. 15

A nurse is caring for a client who has insulin-dependent diabetes mellitus and is experiencing DKA. The client is receiving IV insulin therapy, and laboratory values are being monitored. Which of the following findings should the nurse report to the primary care provider?

Urine output of 30 mL/hr is a normal amount.A blood glucose of 180 mg/dL is elevated, but that is to be expected with DKA.This serum potassium level is below the normal referencerange and needs to be addressed by the primary care provider.A BUN of 18 mg/dL is within the normal reference range. Urine output of 30 mL/hr Blood glucose of 180 mg/dL Serum potassium of 3.0 mEq/L BUN of 18 mg/dL

16

A client diagnosed with acute heart failure is prescribed furosemide (Lasix) daily. During the client's morning assessment, he tells the nurse that he is feeling weak in the legs. Which of thefollowing actions should the nurse take first?

The furosemide may need to be withheld, but this is not the first action the nurse shouldtake.The nurse needs to collect more data before taking any action; therefore, the first actionthe nurse should take is to review the daily laboratory results.The primary care provider willneed to be notified, but this is not the first action the nurse should take.The client should havean adequate breakfast, but this is not the first action the nurse should take. Withhold the furosemide. Review the daily laboratory results. Notify the primary care provider of the client's symptoms. Provide an adequate breakfast for the client. 17

A nurse enters the room of a client receiving PCA following a thoracotomy. The client is short of breath, appears restless, and has respirations of 28/min. Arterial blood gases are obtained and reveal a pH of 7.52, PaO2 89 mm Hg, PaCO2 28 mm Hg, and HCO3- 24 mEq/L. Which of thefollowing interventions should the nurse anticipate taking?

Coughing will not increase the carbon dioxide blood level.Ambulation is not appropriatefor this client and may exacerbate the client's respiratory distress.This client is likelyhyperventilating and needs to be calmed down to allow for carbon dioxide to build up.It isimportant to provide adequate pain control for this client, and a PCA is an appropriate method. Uncontrolled pain may increase the risk of hyperventilation. Instruct the client to cough forcefully. Assist the client with ambulation. Provide calming interventions. Discontinue the PCA. 18

Which of the following assessment findings should a nurse expect to find in a client who hashyperkalemia?

Muscle weakness may result from hyperkalemia.Increased gastric motility may occurfrom hyperkalemia.Bradycardia may result from hyperkalemia.Hypotension may result fromhyperkalemia. Decreased muscle strength Decreased gastric motility Increased pulse rate Increased blood pressure19

A nurse is providing teaching to a client who is at risk for developing respiratory acidosis following surgery. Which of the following statements made by the client indicates understanding

of the teaching?

Clients at risk for respiratory acidosis should be encouraged to move about as much aspossible.The longer a client waits for pain medication, the longer it takes to resolve the pain. The client should be encouraged to use the PCA when the pain begins to worsen.Respiratory acidosis may be related to dehydration. The client should be encouraged to drink at least 1,500 mL of fluid daily.Use of an incentive spirometer postoperatively will promote adequate gas exchange and decrease the risk of respiratory acidosis. I should conserve energy by limiting my physical activity. I will wait until my pain is at least a 6 out of 10 before I use the PCA. I will limit my daily fluid intake to 2 to 3 glasses. I will use the incentive spirometer every hour. 20

Which of the following is the priority assessment a nurse should make when caring for a clientwho has a calcium level of 8.1 mg/dL?

Hyperactive deep tendon reflexes may occur, but this is not the priority assessment.Hypocalcemia may cause increased gastrointestinal motility, but bowel sounds are not the priority assessment.Peripheral pulses may be diminished, but this is not the priorityassessment.The greatest risk to this client is cardiac dysrhythmias. The priority assessment is the client's cardiac rhythm. Deep tendon reflexes

Bowel sounds Peripheral pulses Cardiac rhythm

21

A client is admitted with isotonic dehydration following 2 days of vomiting and diarrhea. Which ofthe following laboratory results should indicate to the nurse that the client has responded to fluidtherapy?

A BUN reading of 26 mg/dL is above the normal range; therefore, this is not an indication that the client has responded to fluid therapy.Isotonic dehydration causes loss of water only. Aserum sodium of 138 mEq/L indicates that the fluids are being replaced and that the client has responded to fluid therapy.A Hct of 56% is above the normal range; therefore, this is not an indication that the client has responded to fluid therapy.A urine specific gravity of 1.035 is above the normal range; therefore, this is not an indication that the client has responded to fluidtherapy. BUN 26 mg/dL Serum sodium 138 mEq/L Hct 56% Urine specific gravity 1.03522

A client is suspected of having metabolic acidosis. Which of the following laboratory values should the nurse expect to find?

These laboratory values do not indicate metabolic acidosis.These laboratory values donot indicate metabolic acidosis.These laboratory values do not indicate metabolic acidosis.Adecreased pH and HCO3- with a normal PaCO2 indicate metabolic acidosis.

pH 7.51, PaO2 94 mm Hg, PaCO2 38 mm Hg, HCO3- 29 mEq/L pH 7.48, PaO2 89 mm Hg, PaCO2 30 mm Hg, HCO3- 24 mEq/L pH 7.30, PaO2 77 mm Hg, PaCO2 52 mm Hg, HCO3- 26 mEq/L pH 7.26, PaO2 84 mm Hg, PaCO2 38 mm Hg, HCO3- 20 mEq/L23

A nurse is caring for a client who is receiving hydrochlorothiazide (Hydrodiuril) 25 mg/day. Assessment findings reveal that the client is confused and lethargic. Which of the followinglaboratory values should the nurse report to the primary care provider?

This is a low sodium level, which commonly causes changes in mental status.This iswithin the normal reference range.This is within the normal reference range.This is within thenormal reference range. Sodium 128 mEq/L Potassium 4.8 mEq/L Calcium 9.1 mg/dL Magnesium 2.0 mg/dL24

A nurse is caring for a client who reports difficulty breathing and tingling in both hands. His respiratory rate is 36/min and he appears very restless. Which of the following values should thenurse expect to be abnormal if the client is experiencing respiratory alkalosis?

With respiratory alkalosis, the PaO2 is normal.With respiratory alkalosis, the PaCO2 isdecreased.With respiratory alkalosis, the sodium level should be normal.With respiratoryalkalosis, the chloride level should be normal. PaO2 PaCO2

Sodium Chloride 25

A client is experiencing respiratory distress as a result of acute pulmonary edema. Which of the following actions should the nurse take first?

Assisting with intubation may be necessary, but this is not the first action the nurse should take.The least invasive action is to place the client in high-Fowler's position to allow forgreater lung expansion and ease of breathing. This is the first action the nurse should take.It is important to assess the client's lung sounds, but this is not the first action the nurse shouldtake.Notifying the primary care provider may be necessary, but it is not the first action the nurse should take. Assist with intubation. Place the client in high-Fowler's position. Assess the client's lung sounds. Notify the primary care provider. 26

During a physical assessment, a nurse notes that a client is experiencing 2+ pitting ankleedema. Which of the following serum laboratory results supports the client's physical findings?

A 2+ pitting edema indicates fluid overload, which may result in a decreased hematocrit.This sodium level is within the normal reference range.This BUN level is within thenormal reference range.This creatinine level is within the normal reference range. Hematocrit 34% Sodium 138 mEq/L BUN 12 mg/dL Creatinine 0.7 mg/dL27

Which of the following actions should a nurse plan to take for a client with a serum calcium levelof 8.4 mg/dL?

The client is at risk for seizures due to the lowered excitation threshold. Seizure precautions should be implemented to prevent injury to the client.Calcium may be given orally orby IV, but not intramuscularly.Restricting fluids will not increase calcium levels.Ambulation will not decrease bone resorption and may be difficult because of neuromuscular changes. Implement seizure precautions. Administer calcium by IM injection. Place the client on a fluid restriction. Encourage ambulation to prevent bone resorption. 28

A nurse is preparing to administer oral potassium. The client's potassium level is 5.5 mEq/L. Which of the following actions should the nurse take?

This is not the correct action.This is not the correct action.This client's potassium level ishigh; therefore, the nurse should withhold the medication and notify the primary careprovider.This is not the correct action. Administer the medication as prescribed because the laboratory result is low. Discontinue the medication because the laboratory result is within normal limits. Withhold the medication because the laboratory result is high.

Give the medication because the laboratory result is approaching high. 29

A nurse is caring for a client who has Guillain-Barr syndrome. Which of the following arterial pHvalues indicates that the client is experiencing acidosis?

An arterial pH of 7.30 indicates acidosis.An arterial pH of 7.35 does not indicateacidosis.An arterial pH of 7.40 does not indicate acidosis.An arterial pH of 7.45 does notindicate acidosis. 7.30 7.35 7.40 7.45 30

A client diagnosed with dehydration is receiving IV fluids. When assessing for complications, thenurse should recognize which of the following as a sign of fluid overload?

Increased specific gravity indicates greater concentration of urine and is associated withdehydration, not fluid overload.A decrease in bowel sounds is associated with dehydration, not fluid overload.Fluid overload results in increased vascular volume and places a greaterworkload on the heart. Thus, an expected finding is bounding peripheral pulses.Fluid overload places a greater workload on the respiratory system, and an increased respiratory rate is anexpected finding.

Increased urine specific gravity Hypoactive bowel sounds Bounding peripheral pulses Decreased respiratory rate

ATI. MED-SURG. NEUROSENSORY & MUSCULOSKELETAL

When caring for a client following cataract surgery, the nurse should recognize which of the following client statements as indicative of a need for further teaching?

Changes of vision need to be reported immediately, because there should be an improvement in vision after the surgery.Aspirin should be avoided because it prevents clotting and can lead to bleeding in the eye.Bending over should be avoided because it may increaseintraocular pressure.It is not necessary to sleep upright for the first 24 hr to maintain normalintraocular pressure. I should call my doctor if I experience a decrease in my vision. I should not take aspirin following the surgery. I will need to avoid activities that require bending over. I will need to sleep upright for the first 24 hours. 2

A nurse is caring for a client in balanced suspension skeletal traction who reports intermittent muscle spasms. Which of the following actions should the nurse take first?

The client may need to be repositioned, but this is not the first action the nurse shouldtake.The primary care provider may need to be notified, but this is not the first action the nurseshould take.A muscle relaxant may need to be administered, but this is not the first action the nurse should take.The first action the nurse should take is to gather more data as to thepossible cause of the client's discomfort. Reposition the client. Notify the primary care provider. Administer a muscle relaxant. Check position of weights and ropes. 3

A client presents to the emergency department with sudden, severe eye pain with blurred vision. A diagnosis of angle-closure glaucoma is made. The nurse should anticipate administration of which of the following medications?

The use of IV osmotic diuretics is appropriate emergency treatment for angle-closureglaucoma due to the ability of these medications to rapidly reduce intraocular pressure.Mydriaticophthalmic drops are contraindicated for angle-closure glaucoma because they cause pupillarydilation. Pupillary constriction is the desired effect to facilitate flow and drainage of aqueous humor.Corticosteroid ophthalmic drops are not indicated for angle-closure glaucoma.Epinephrine-containing medications are contraindicated for angle-closure glaucomabecause they cause pupillary dilation. Pupillary constriction is the desired effect to facilitate flow and drainage of aqueous humor. IV osmotic diuretics

Mydriatic ophthalmic drops Corticosteroid ophthalmic drops IV epinephrine4

When caring for a client who has multiple sclerosis, the nurse should recognize that thisdisorder is characterized by

This finding is a manifestation of Guillain-Barr syndrome and amyotrophic lateralsclerosis.This finding is a manifestation of Guillain-Barr syndrome.The course of multiplesclerosis is unpredictable and is manifested by remissions and exacerbations that may occur forvarying lengths of time.This finding is a manifestation of Alzheimer's disease and amyotrophic lateral sclerosis. respiratory failure. ascending paralysis. remissions and exacerbations. gradual loss of cognitive function. 5

A nurse is prioritizing care for a client who is hospitalized for a brain tumor. Which of the following actions is the highest priority?

Although adequate nutrition is important, it is not the highest priority.The greatest risk to

the client is brain injury due to increased intracranial pressure resulting from growth of thetumor. The priority intervention is to monitor for increased intracranial pressure.Although maintaining skin integrity is important, it is not the highest priority.Headaches are a common feature of brain tumors and should be treated with analgesics; however, this is not the highest priority.

Assess the client for adequate nutritional intake. Monitor the client for signs of increased intracranial pressure. Position the client to maintain skin integrity. Administer analgesics for headache. 6

A nurse is caring for a client who is a quadriplegic secondary to a cervical fracture at vertebral level C5. The client reports a throbbing headache and nausea, and the nurse notes facial flushing and a blood pressure of 220/110 mm Hg. Based on the client's report and findings, which of the following actions should the nurse perform first?

While this is an appropriate action, it is not the first action the nurse should take.These findings are usually due to a noxious stimulus in the lower part of the body. While identifying and eliminating the source of the stimulus are appropriate actions by the nurse, they are not the first action the nurse should take.These findings are usually due to a noxious stimulus in the lower part of the body. While identifying and eliminating the source of the stimulus are appropriate actions by the nurse, they are not the first action the nurse should take.This is the first action the nurse should take to lower the blood pressure because it takes advantage of the postural hypotension typically experienced when a client who is quadriplegic is moved from a supine to an upright position. Notify the primary care provider.

Check the bladder for distention. Determine if the client has a fecal impaction. Elevate the head of the bed. 7

A client who is recovering from a cerebrovascular accident has right-sided homonymous hemianopsia. To help the client adapt, the nurse should take which of the following actions?

This intervention will not assist the client with homonymous hemianopsia.This intervention will not assist the client with homonymous hemianopsia.This intervention will not assist the client with homonymous hemianopsia.Since the client with right-sided homonymous hemianopsia has lost vision in the right visual field of both eyes, he may not be able to see half of his meal tray. This intervention is appropriate and can help the client compensate for his visual loss. Check the client' s cheek on his affected side after eating to be sure no food remains there. Encourage the client to sit upright with his head tilted slightly forward during meals. Provide the client with eating utensils that have large handles. Remind the client to consciously look at both sides of his meal tray. 8

A client with a possible head injury is admitted to the hospital following a motor-vehicle crash. The nurse should recognize that which of the following findings indicates increasing intracranial pressure?

Behavioral changes, such as restlessness or irritability, are early signs of increased intracranial pressure.While dizziness may or may not be present after head trauma, it is not a sign of increased intracranial pressure.Hypotension is not a sign of increased intracranial pressure. The Cushing reflex, manifested by hypertension and a widening pulse pressure, is a late sign of increased intracranial pressure.While a client with head trauma may develop fever, it

is either in response to infection or due to hypothalamic damage, not due to increasedintracranial pressure. Restlessness Dizziness Hypotension Fever 9

A nurse is caring for a client who is at risk for experiencing a tonic-clonic seizure. Which of the following actions should the nurse plan to take?

A client experiencing a tonic-clonic seizure may require IV antiepileptic medications, so IV access should be maintained.It is unnecessary to maintain a client at risk for seizures on bedrest.Forcing objects, such as padded tongue blades or an airway into the client's mouth during aseizure, is more likely to cause injury to the client. If supplemental oxygen is needed, it can beadministered by nasal cannula or mask.A change in blood pressure does not correlate with an increased incidence of seizure activity. Establish IV access. Maintain the client on bed rest. Keep an airway at the bedside. Assess regularly for a spike in blood pressure. 10

Which of the following assessment findings should a nurse expect when caring for a client withrheumatoid arthritis?

Rheumatoid arthritis usually occurs bilaterally and symmetrically. Osteoarthritis usually occurs unilaterally.The inflammation that occurs in the small joints can make them susceptible todeformity from daily use. Ulnar deviation, or lateral deviation of the fingers, can occur fromopening jars and other similar motions.Compression fractures of the spine are more common inclients who have osteoporosis.A client with rheumatoid arthritis will exhibit an increased sedimentation rate related to the body's response to this inflammatory connective tissuedisorder. Unilateral joint involvement Ulnar deviation Deformity of the spine Decreased sedimentation rate 11

A nurse is providing instruction regarding carbidopa/levodopa (Sinemet) to a client with Parkinson's disease. Which of the following client statements indicates a need for furtherinstruction?

One of the side effects of carbidopa/levodopa is orthostatic hypotension.High-protein foods may delay the absorption of carbidopa/levodopa or reduce the amount absorbed into thebloodstream.Secretions such as saliva, urine, and sweat may darken in color when takingcarbidopa/levodopa.It is important for clients to understand that relief from symptoms may notbe experienced for weeks to months after starting this medication.

I may feel lightheaded while taking this medication. I should take my medication with a high-protein food. I should expect my urine to be a darker color. I will expect it to take weeks to months for this medication to work.

12

When caring for a client who has a retinal detachment, the nurse should expect the client toreport which of the following findings in the affected eye?

This is not an expected finding.This is not an expected finding.Flashes of bright light orfloating dark spots in the eye are symptoms of a detached retina.A detached retina is usually painless, because there are no pain fibers located in the retina. Photophobia Complete blindness Flashes of bright light Pain 13

A client newly diagnosed with epilepsy is prescribed phenytoin (Dilantin). Which of the following instructions should the nurse include in the client's medication teaching plan?

Antiepileptic medications should be taken on a regular schedule to maintain therapeuticblood levels and achieve maximum effect.Development of a rash should be reported to the provider as the medication will need to be discontinued.The client needs to continue takingantiepileptic medications even in the absence of seizures. Stopping the medication may lead tothe return of seizures or the complication of status epilepticus.The client should not take anextra dose of the antiepileptic medication if a dose is missed.

Take medications at a consistent time each day to maintain therapeutic blood levels. Use an over-the-counter antihistamine if a rash develops. Slowly taper the medication after 6 consecutive months without seizure activity. Take an extra dose the following day if a dose is missed. 14

A client is admitted to the emergency department after suddenly losing consciousness and falling in her home. The client is diagnosed with an embolic cerebrovascular accident. Which of the following medications should the nurse anticipate administering?

Tissue plasminogen activator is a thrombolytic and is administered to dissolve the blood clot that resulted in the cerebrovascular accident.This medication is administered for hemophilia.Nitroglycerin is a coronary vasodilator that is administered for angina.Lidocaine is anantidysrhythmic agent used for ventricular dysrhythmias. In addition, adverse CNS effects may occur, such as drowsiness and confusion. Tissue plasminogen activator (Alteplase) Recombinant factor VIII Nitroglycerin (Nitro-Bid IV) Lidocaine 15

A nurse is providing teaching to a client and his family about the diagnosis and treatment of Alzheimer's disease. Which of the following statements by the family indicates an understandingof the teaching?

There is no specific test for Alzheimer's disease. Alzheimer's disease is diagnosed by ruling out other diseases.None of the medications currently available reverses the course of thedisease.Early signs include short-term memory loss, forgetfulness, and shortened attention

span.Medications used to treat Alzheimer's disease increase the level of acetylcholine allowing for greater response from cholinergic neurons in the brain. "There is a test for Alzheimer's disease that can establish a reliable diagnosis." "The goal of medication therapy is to reverse the degenerative changes that can occur in brain tissue." "Early signs of Alzheimer's disease include mild tremors and muscular rigidity." "The drugs used to treat Alzheimer's disease may help delay cognitive changes."

16

A nurse is caring for a client who is admitted with ascending Guillain-Barr syndrome. The nurse should give priority to which of the following assessment findings?

Altered sensory perception is an expected finding in clients with Guillain-Barr syndrome, but this is not the highest priority.The greatest risk to this client is respiratory failure as indicated by the presence of adventitious breath sounds.Loss of bowel and bladder control is an expected finding in clients with Guillain-Barr syndrome, but this is not the highest priority.Weak or incomplete eye closure is an expected finding in clients with Guillain-Barr syndrome, but this is not the highest priority. Altered sensory perception

Presence of adventitious breath sounds Loss of bowel and bladder control Weak or incomplete eye closure 17

A nurse is planning care for a client following a lumbar puncture. Which of the following actions should the nurse plan to include?

It is not necessary to apply a pressure dressing to the site.The client's fluid intake should be increased to 3,000 mL unless contraindicated for the next 24 to 48 hr to facilitate cerebrospinal fluid production.The client should lie flat for 4 to 8 hr to prevent cerebrospinal fluid leakage from the puncture site.The client does not usually require pain medication following a lumbar puncture. Apply a pressure dressing to the site for 12 hr. Restrict the client' s fluid intake for 24 hr. Ensure the client lies flat for 4 to 8 hr. Administer pain medication every 3 to 4 hr. 18

When developing a teaching plan for a client diagnosed with Mnire's disease, the nurse should include which of the following instructions to decrease symptoms?

Slow head movements decrease the stimulation of vertigo.This intervention does not relieve the symptoms of Mnire's disease.This intervention does not relieve the symptoms of Mnire's disease and may actually precipitate an exacerbation.This intervention will not relieve the symptoms of Mnire's disease and may actually worsen symptoms secondary to increased fluid retention. Move head slowly to decrease vertigo. Apply warm packs to the affected ear during acute attacks. Increase fluid and sodium intake to maintain hydration. Administer corticosteroids during acute attacks. 19

A client has been diagnosed with a basilar skull fracture following a fall from a ladder. Which of the following assessments should be reported to the primary care provider?

A score between 13 and 15 characterizes a mild traumatic brain injury. Subsequently, a score of 14 does not need to be reported to the primary care provider, but a decrease in the score should be reported.An intracranial pressure reading of 15 mm Hg is within the normalrange of 10 to 15.This is likely due to a contusion because of a fall and does not need to bereported.This finding indicates cerebral spinal fluid is leaking from the skull fracture and shouldbe reported. Glasgow Coma Scale score of 14 Intracranial pressure reading of 15 mm Hg Ecchymosis at base of skull Clear drainage from nose20

A nurse is planning care for a client with a closed head injury who is receiving mechanical ventilation. Which of the following is the highest priority intervention?

The greatest risk to the client is increased intracranial pressure. PaCO2 should bemaintained approximately at 35 mm Hg to prevent hypercarbia. The vasodilation effects of carbon dioxide can lead to intracranial pressure increases.Opiate pain medications should beadministered by the nurse in small increments, as they tend to mask changes in the level ofconsciousness. However, this is not the highest priority.Clients with head injuries commonly experience a fever secondary to the body's response to the trauma or hypothalamic damage. However, this is not the highest priority.The client is at risk for seizures. However, this is not the

highest priority. Maintain a PaCO2 of approximately 35 mm Hg. Provide small IV doses of opioid analgesia. Monitor body temperature every 1 to 2 hr. Administer an antiepileptic agent.

21

A nurse is performing a pain assessment on a postoperative client. Which of the following should the nurse use to determine the severity of the client's pain?

A change in vital signs may identify that pain is present, but this does not identify the severity of the pain.A report of the type of pain identifies the character of the pain, but this does not indicate the severity of the pain.Use of a standardized pain scale is the best method todetermine the severity of the client's pain.Facial grimacing may identify that pain is present, butthis does not identify the severity of the pain. Vital sign changes Client's report of the type of pain Client's report of pain on a standardized pain scale Nonverbal communication 22

A client with myasthenia gravis presents to the emergency department. Based on presenting

symptoms of increasing muscle weakness, the nurse suspects the client is experiencing amyasthenic crisis. The nurse should give priority to which of the following actions?

The client with myasthenia gravis may have dry eyes due to inability to close the eyescompletely; however, this is not the priority action.Further assessment is necessary to determinethe cause of the client's symptoms. This test will determine if the client is experiencing a myasthenic crisis or a cholinergic crisis.Immunosuppressants can be administered to reduce thesymptoms of myasthenia gravis; however, this is not the priority action.Plasmapheresis can be administered to reduce the symptoms of myasthenic crisis; however, this is not the priority action. Instill artificial tears. Assist with Tensilon test. Administer immunosuppressants. Prepare for plasmapheresis. 23

A nurse is caring for a client with advancing amyotrophic lateral sclerosis. Which of the followingshould be the priority nursing intervention?

Implementing a bowel training program is important; however, it is not the priority intervention.Maintaining adequate pain control is important; however, it is not the priority intervention.The greatest risk to this client is respiratory compromise due to progressiveparalysis of respiratory muscles. The priority intervention is to monitor oxygen saturation to identify respiratory compromise as soon as possible.Promoting adequate intake is important; however, it is not the priority intervention. Implement a bowel training program. Medicate for pain around the clock.

Monitor pulse oximetry findings. Promote adequate oral intake. 24

A nurse is providing teaching to a client who is prescribed alendronate (Fosamax) for osteoporosis. Which of the following should the nurse include in the teaching?

Alendronate does not decrease the risk for development of degenerative joint disease.To prevent esophagitis or esophageal ulcers that may result from alendronate therapy, the client should be instructed to sit up for 30 min after taking this medication.Bisphosphonates arestructural analogs of normal bone constituents; therefore, they are incorporated into the bonetissue and then prevent bone resorption.Alendronate absorption is not increased by vitamin Dintake. This medication decreases the risk for degenerative joint disease. This medication should be taken before breakfast and in an upright positionmaintained for 30 min. This medication increases the serum calcium level. This medication's rate of absorption can be enhanced by increasing intake ofvitamin D. 25

A nurse is caring for a client with osteoarthritis. Which of the following should the nurse expect? (Select all that apply.) Crepitus with joint movement is correct. Crepitus occurs due to irregular cartilage in thejoints.

Decreased range of motion of the affected joint is correct. Decreased range of motion occurs due to the pain experienced by the client from movement.

Joint stiffness that is worse in the morning is incorrect. This is an expected finding with rheumatoid arthritis.

Warm, red joints is incorrect. This is an expected finding with rheumatoid arthritis.

Joint pain that resolves with rest is correct. The pain associated with osteoarthritis increases with activity and diminishes with rest.Crepitus with joint movement is correct. Crepitus occursdue to irregular cartilage in the joints.

Decreased range of motion of the affected joint is correct. Decreased range of motion occurs due to the pain experienced by the client from movement.

Joint stiffness that is worse in the morning is incorrect. This is an expected finding with rheumatoid arthritis.

Warm, red joints is incorrect. This is an expected finding with rheumatoid arthritis.

Joint pain that resolves with rest is correct. The pain associated with osteoarthritis increases with activity and diminishes with rest. Crepitus with joint movement Decreased range of motion of the affected joint Joint stiffness that is worse in the morning

Warm, red joints Joint pain that resolves with rest

26

A client who is 2 days postoperative following a left above-the-knee amputation reports pain in his left foot at a pain level of 9 on a scale from 0 to 10. Which of the following interventionsshould the nurse take first?

This is an appropriate intervention for phantom limb pain, but it is not the first action thenurse should take.This is an appropriate intervention to minimize pain, but it is not the firstaction the nurse should take.The greatest need for this client is pain control. Phantom limb painshould be treated as real pain. Gabapentin is an effective medication for phantom limb pain.Placing the client in a prone position several times a day will help prevent hip flexioncontractures, but it is not the first action the nurse should take. Use therapeutic communication to facilitate grieving for the lost limb. Change the client's dressing using minimal movements. Administer an oral dose of gabapentin (Neurontin) to the client. Place the client in a prone position several times a day. 27

One hour after application of a cast for an ulnar fracture, a client reports a pain level of 8 on ascale from 0 to 10 that is unrelieved by analgesics and the application of a cold pack. Based onthe client's report, which of the following actions should the nurse take first?

The client may require additional pain medication, but this is not the first action the nurseshould take.The greatest risk to the client is neuromuscular injury resulting from compartmentsyndrome. These findings require immediate intervention to relieve pressure in the extremity and should be reported to the primary care provider.The nurse should document the findings, but this is not the first action the nurse should take.The client's arm and cold pack may need tobe repositioned, but this is not the first action the nurse should take. Administer additional pain medication. Report the findings to the primary care provider. Document the findings. Reposition the client's arm and cold pack. 28

A nurse is providing teaching to a client with a spinal cord injury who has a spastic bladder. Which of the following actions should the nurse recommend to the client to help stimulatemicturition?

A client who has a spastic bladder can stimulate micturition by stroking the inner thigh, pulling on the pubic hair, or tapping the abdomen over the bladder to stimulate the detrusor muscle.Intermittent catheterization is used to drain the bladder manually, but it cannot be used to stimulate micturition.The Cred's maneuver, or direct pressure applied over the bladder, can be used to express urine from a flaccid bladder. It is not effective with a spastic bladder due to the spasticity of the external sphincter.The Valsalva maneuver, or tightening of the stomach muscles, can be used to express urine from a flaccid bladder. It is not effective with a spastic bladder due to the spasticity of the external sphincter. Stroking the inner thigh

Intermittent catheterization Cred's maneuver Valsalva maneuver 29

A nurse is instructing an assistive personnel on proper care of a client who has had a total hip arthroplasty. Which of the following instructions should the nurse include?

Thromboembolic hose should be worn postoperatively to prevent the development ofemboli in the lower extremities. Care should be taken, however, when manipulating the affected extremity to avoid flexure greater than 90.A client who has had a hip arthroplasty should not flex her affected hip more than 90.A client who has had a total hip arthroplasty can be placed ineither an upright wheelchair or one with a back that reclines.A client who has had a total hip arthroplasty should not adduct her hips due to risk of dislocation of the surgical hip. The use of an abductor pillow will maintain the client's hips in abduction. Avoid applying thromboembolic hose to the affected leg. Have the client lean forward prior to standing to maintain her center of gravity. Discourage the client from sitting in a wheelchair with the back reclined. Use an abductor pillow between the client's legs if the client becomes restless. 30

A nurse is assessing a client who has experienced a right hemispheric stroke. Which of the following neurological deficits should the nurse expect?

A client who has experienced a right hemispheric stroke should not experience aphasia. This occurs when the left side of the brain has been affected.A client who has had a right hemispheric stroke would demonstrate neurological deficits on the left side of the body and not

the right side. The combination of hemianesthesia and hemiplegia can result in one-sided neglect because the client can neither feel nor move that side of the body.A client who has had a right hemispheric stroke will demonstrate impulsive behavior, poor judgment, and lack of awareness of neurological deficits.A client who has had a right hemispheric stroke should not experience difficulties in reading due to inability to discriminate different letters and words. This occurs when the left side of the brain has been affected. Aphasia Right-sided neglect Impulsive behavior Inability to read

ATI. MED-SURG. PERIOPERATIVE

A nurse's postoperative assessment of a client who had a vaginal hysterectomy 2 days ago reveals the right calf to be red, edematous, and warm to the touch. Which of the followingactions should the nurse anticipate taking?

A client with these findings should not have an ice pack applied to her right calf.An extremity with deep vein thrombosis should be elevated to promote venous return.A client with these findings should not be encouraged to ambulate.A client with these findings should not have her right calf massaged.

Apply an ice pack to the right calf. Elevate the right extremity. Encourage ambulation. Gently massage the right calf. 2

A nurse is monitoring a client receiving succinylcholine (Anectine) during a surgical procedure. Which of the following actions should the nurse anticipate taking if the client develops malignanthyperthermia?

Dantrolene is administered IV at 2 to 3 mg/kg to reverse malignant hyperthermia.A client with malignant hyperthermia is not at risk for seizures.A client with malignant hyperthermia is notat risk for hypoglycemia.It is not indicated to give atropine to a client who has malignanthyperthermia. Administer dantrolene (Dantrium). Institute seizure precautions. Obtain a stat blood glucose. Give IV atropine. 3

To help prevent neuromuscular complications during surgical procedures on a client, the nurse should anticipate taking which of the following actions?

Administering IV fluids will help prevent dehydration and maintain adequate bloodvolume, but it will not prevent neuromuscular complications.Massaging lower extremities is notan appropriate action to prevent neuromuscular complications.Padding on bony prominences

will help prevent pressure on nerves and blood vessels and reduce the potential for neuromuscular complications.Joints should be placed and maintained in a slightly flexed position to prevent neuromuscular complications. Administer an IV bolus of normal saline. Massage lower extremities during procedure. Support bony prominences with foam padding. Extend joints and maintain position with padded straps. 4

A nurse is caring for a postoperative client with a Jackson-Pratt drain in place. Which of the following interventions should the nurse use to ensure proper functioning of the drain?

Compressing the reservoir produces the suction necessary for the drain to function properly.The drain is not clamped because continuous drainage is required.Secure the drain to the client's gown or clothing to prevent pulling when the client moves.The drain should be kept lower than the incision to facilitate drainage. Empty and compress the drain reservoir as needed. Clamp the drain when the client is ambulating. Secure the drain to the client's bed sheet. Keep the drain higher than the surgical incision. 5

A nurse is caring for a client who is postoperative following abdominal surgery. Which of the following nursing interventions should the nurse perform to prevent respiratory complications?

The client should inhale deeply from the incentive spirometer and hold his breath to expand the lungs.The client will breathe deeper if there is less pain, so pain medications should be given as needed.Splinting the incision supports the surgical site and allows for deeper coughing and clearing of the lungs.Position should be changed at least every 2 hr until the client is ambulating frequently. Instruct the client to exhale into the incentive spirometer every 1 to 2 hr. Minimize the amount of pain medication the client receives to prevent sedation. Advise the client to splint the surgical incision when coughing and deep breathing. Reposition the client twice every 8 hr for the first 48 hr.

Which of the following observations by a scrub nurse will require an intervention to maintain surgical asepsis in the operating room?

Finger and wrist jewelry, which can harbor bacteria, should be removed to maintain surgical asepsis.The outside wrapping of instrument packages is unsterile and therefore should be opened before applying sterile gloves to maintain surgical asepsis.Anything below the waist is considered unsterile, and therefore hands should be held above the waist to maintain surgical asepsis.A surgical scrub requires 3 to 5 min of washing followed by a rinse to maintain surgical asepsis. A team member is wearing a watch under his scrubs. A team member opens instrument packages before applying sterile gloves. A team member has her hands folded 2 inches above the waist. A team member scrubs her hands for 3 to 5 min, and follows with a rinse. 7

A nurse is providing teaching to a client who is scheduled to undergo conscious sedation for a bronchoscopy. The nurse verifies the client's understanding of the procedure when the clientstates which of the following?

A bowel prep is not necessary prior to a bronchoscopy.The client should be NPO for 6 to 8 hr prior to a bronchoscopy.The medications used for conscious sedation are administered intravenously, which results in a quick onset and recovery.Following conscious sedation, the client should expect to feel drowsy for several hours and should avoid all activities requiringconcentration. "I will need to complete a bowel prep the day before the procedure. "I will drink plenty of fluids the morning of the procedure. "I will not need an IV for this procedure. "I can expect to feel sleepy for several hours after the procedure. 8

A nurse is caring for a client in the PACU. Based on a score of 10 on the PACU recovery scale, which of the following assessment findings indicates that the client is ready to be dischargedfrom the PACU?

A 25% or greater drop in blood pressure requires investigation before discharge from the PACU.The client needs to maintain an oxygen saturation level of at least 92% on room air to be discharged from the PACU.The client should be able to move all four extremities on command before discharge from the PACU.If the client requires tactile stimulation to awaken, the client is

not ready for discharge from the PACU. The client's preoperative blood pressure was 140/90 mm Hg and thepostoperative blood pressure is 100/65 mm Hg. The client' s SaO2 is 90% on room air. The client is able to move all four extremities on command. The client requires tactile stimulation to awaken. 9

A nurse who is working in the operating room (OR) should recognize that the room is maintained at a cool temperature with low humidity because it reduces

A cool room temperature with low humidity does not decrease the risk for infection.A cool room temperature with low humidity does not decrease the amount of anesthetic agentsneeded.A cool room temperature with low humidity will reduce the risk of fires in the OR secondary to volatile anesthetic agents used.A cool room temperature with low humidity does not decrease the amount of oxygen the client needs. the risk for infection. the amount of anesthetic agents needed. the risk of fire. the amount of oxygen needed by the client. 10

A nurse is taking a preoperative medication history on a client scheduled for surgery. Which of the following medications should the nurse recognize as placing the client at risk forcomplications due to interaction with anesthetic agents?

Antihypertensive agents, such as captopril, may cause a hypotensive crisis when givenwith anesthetic agents.Antilipemics, such as atorvastatin, have no interaction with anesthetic

agents.H2 receptor blockers, such as ranitidine, have no interaction with anesthetic agents.Antibiotics, such as ciprofloxacin, have no interaction with anesthetic agents. Captopril (Capoten) Atorvastatin (Lipitor) Ranitidine (Zantac) Ciprofloxacin (Cipro)

11

A client had an open transverse colectomy 5 days ago. The nurse enters the client's room and recognizes that the wound has eviscerated. After covering the wound with a sterile, salinesoaked dressing, which of the following actions should the nurse take?

A client who has experienced evisceration should not be left alone. The nurse should press the call light to seek assistance.The nurse should make no attempt to reinsert the eviscerated contents.The client should be placed in a supine position with hips and knees bent.The nurse should take vital signs to assess the client's current status. Go to the nurses' station to seek assistance. Reinsert the organs into the abdominal cavity. Place the client in reverse Trendelenburg. Obtain vital signs to assess for shock.

12

A nurse is planning to perform a preoperative assessment for a client who is a Jehovah's Witness. Which of the following should the nurse recognize as a situation that could pose special care needs for this client?

There are no restrictions on providing blood samples by this religious group.Clients in this religious group are willing to give a sexual history.Clients in this religious group are not allowed to accept blood or blood products from other persons.Clients in this religious group usually do not refuse care by health care providers of the opposite gender. Having preoperative blood drawn Giving information about sexual history Providing informed consent to receive blood products Receiving care from a nurse of the opposite gender 13

Which of the following is an unexpected finding for a nurse when caring for a client in the postoperative period?

Urine output should be at least 30 mL/hr. Decreased output may indicate dehydration and poor perfusion of the kidneys.A slightly lower body temperature may occur related to the temperature of the operating room or to the surgery itself and is not an unexpected finding in the postoperative period.A small amount of bloody drainage is expected following surgery.Drainage from a Jackson-Pratt drain of 30 mL/hr is not unexpected. Urine output of 20 mL/hr Temperature of 36.5 C (97.7 F) A 3-cm area of bloody drainage on the dressing Jackson-Pratt drainage of 30 mL/hr 14

A nurse is providing preoperative teaching to a client scheduled for a gastrectomy to be performed in 1 week. The client is anxious about the upcoming surgery. The nurse should take which of the following actions for this client?

Sympathizing with the client's feelings is nontherapeutic and will not help to decreaseanxiety.The client will be anxious until measures are taken to reduce the anxiety.Watching a video will not allow the client time to ask questions, and viewing a video regarding the proceduremay increase anxiety.Providing concise, factual information allows for open communication and gives the nurse the opportunity to dispel fears the client is having. Sympathize with the client's feelings. Reschedule the teaching for another time. Give the client a video to watch. Provide concise, factual information. 15

A nurse is providing care to a postoperative client. To prevent formation of thrombi in the postoperative period, the nurse should do which of the following?

The client's position should be changed at least every 2 hr.This exercise will help prevent venous stasis in the lower legs and decrease the likelihood of thrombophlebitis.Placing pressureon the popliteal area with a pillow may increase the risk of thrombus formation.Assessing thepedal and posterior tibial pulses provides an indication of arterial flow to the feet. It does notprevent or assess thrombus formation in the veins of the lower extremities.

Change the client's position every 4 hr. Instruct the client to perform dorsal/plantar flexion of the feet every hr. Place the client in bed with a pillow under the knees. Assess pedal and posterior tibial pulses every 2 hr.

16

A nurse has been assigned to care for a client who has a surgical wound with a Penrose drain inplace. Which of the following interventions should the nurse plan to perform?

A dressing with drainage should be changed, not reinforced, to prevent skin breakdown and infection.A dressing with a Penrose drain should be changed using sterile technique.Signs of infection should be assessed at least once every shift.A Penrose drain is an open system anddrains by gravity. Reinforce the dressing if drainage is noted. Use sterile technique when performing dressing changes. Observe for signs of infection daily. Apply negative pressure when emptying the drain. 17

A client has a nasogastric (NG) tube set to continuous low suction following a gastrectomy. Which of the following findings should the nurse report to the primary care provider?

Gastric distention is a sign that the NG tube may not be patent. This should be reported to the primary care provider to prevent complications at the anastomosis.Absent bowel soundsare a normal finding after abdominal surgery.Incisional pain is a normal finding after abdominal surgery and should be treated with prescribed analgesics.A small amount of bloody drainage is a normal finding after a gastrectomy. Gastric distention Absent bowel sounds

Incisional pain of 9 on a scale from 0 to 10 Small amount of bloody drainage in the NG tube18

A nurse is assessing a client's recovery from spinal anesthesia. Which of the followingsensations should the nurse expect to return first?

The first sensation to return is touch.The sensation of cold is the fourth to return.The sensation of pain is the second to return.Warmth is the third sensation to return. Touch Cold Pain Warmth 19

A client is transferred from the surgical suite to the PACU following oral surgery. While monitoring the client's vital signs, the nurse finds that the tongue has become swollen and isobstructing the airway. The nurse should take which of the following actions first?

The anesthesiologist should be notified, but it is not the first action the nurse shouldtake.Intubation may be necessary, but it is not the first action the nurse should take.The flow of oxygen may need to be increased, but it is not the first action the nurse should take.Thegreatest risk to the client is airway obstruction. A nasal airway should be inserted for a client

following oral surgery. Contact the anesthesiologist. Assist with endotracheal intubation. Increase the flow of oxygen. Insert a nasal airway. 20

A nurse is admitting a client from the operating room to the PACU following surgery. The client had spinal anesthesia for a total knee arthroplasty. During the initial assessment, which of the following indicates the need to notify the primary care provider?

A recovery score of 8 indicates that the client is recovering from the anesthesia.Spinal anesthesia will probably still be providing its intended anesthetic effect at this time. Documentation of motor and sensory function should be done until the client's function has returned.These are signs of acute pain indicating that the client needs to be medicated for pain. The primary care provider does not need to be notified for expected postoperativepain.Following a total knee arthroplasty, a client is at risk for pulmonary embolism. Reports of chest pain and shortness of breath may indicate pulmonary embolism and should be reported tothe primary care provider immediately. A recovery score of 8 Spinal anesthesia at the T10 level Elevated heart rate and blood pressure Reports of chest pain and shortness of breath

21

A nurse is providing teaching to a client scheduled to have a mastectomy. Which of the followingstatements made by the client indicates a need for further teaching?

This activity can be resumed in about 3 weeks.The client should start doing exercises(e.g., squeezing a ball) the first day after surgery. This can be initiated by the nurse.Blood pressures and blood draws should not be taken from the arm on the side of themastectomy.Showering is permitted after drain removal. I should be able to resume water aerobics in a few weeks. I'll start arm exercises after I see the physical therapist. I shouldn' t have blood drawn from the arm on the side of my mastectomy. I'll be able to shower after the drain is removed. 22

A nurse is providing preoperative teaching to a client. Which of the following prescribed medications should the nurse instruct the client to discontinue 48 hr prior to the surgery?

Furosemide is not contraindicated before surgery.Digoxin is not contraindicated before surgery.Prednisone is not contraindicated before surgery.Warfarin will increase the likelihood of bleeding during and following surgery and should be discontinued several days before surgery. Furosemide (Lasix) Digoxin (Lanoxin) Prednisone (Deltasone) Warfarin (Coumadin) 23

A nurse is providing teaching to a client in the immediate postoperative period who has a PCApump. Which of the following should the nurse include in the teaching?

Because the tolerance level for opioids varies among clients, it may still be possible for aclient on a PCA pump to have an adverse reaction.It is important that no one else besides the client push the PCA button to prevent excessive dosing. The client should be awake to receive adose of the medication.The PCA button should be pinned to the client's gown for easy access and to prevent it from falling out of reach.Pain relief is more effective and lower doses of opioidsare required if the PCA pump is used before the pain gets severe. "You may push the PCA button as often as you want without overdosing." "Do not allow your family to push the PCA button if you are sleeping." "You may leave the PCA button on your bedside table so you can reach it." "Do not push the PCA button until your pain reaches a severe level." 24

A nurse is providing discharge instructions to a client after abdominal surgery. Which of the following client statements indicates the need for further teaching?

An increase in temperature or drainage may indicate an infection and should bereported.Protein and vitamin C are essential for healing tissue.Clients are usually allowed toreturn to work in about 6 weeks following abdominal surgery.Remaining on bed rest places the client at risk for development of venous thrombosis. Ambulation and activity should be encouraged. I will call my doctor if I have an increase in temperature or wound drainage.

I will eat foods high in protein and vitamin C during my recovery. I will plan to return to work in about 6 weeks. I will remain on bed rest until my follow-up appointment with my doctor. 25

A client is scheduled for surgery to be performed under general anesthesia. The nurse should report which of the following findings to the provider?

This potassium level is low and places the client at risk for cardiac dysrhythmias.This iswithin the normal reference range for sodium.This is within the normal reference range forINR.This is within the normal reference range for BUN. Potassium level 2.8 mEq/L Sodium level 140 mEq/L INR 1.5 BUN 12 mg/dL

26

A nurse is preparing a client for surgery. The client appears apprehensive and asks multiple questions about the risks of the procedure. Which of the following actions should the nurse takebefore witnessing the client's signing of the informed consent form?

Explanation of surgical risks and benefits is the responsibility of the surgeon.Thesurgeon is responsible for explaining the surgery and its risks and benefits.Reassurance will notprovide the client with the answers needed.It is not appropriate to document the client's lack of preoperative teaching. Explain the risks and benefits of the surgery to the client. Ask the surgeon to speak to the client for clarification. Reassure the client that the procedure is necessary for recovery.

Document the client's lack of preoperative teaching. 27

A nurse has received afternoon report on four clients who have returned from the PACU this morning. The nurse should first assess which of the following postoperative clients?

This client should be assessed first because this is an unexpected finding and should bereported to the provider immediately.This finding is expected for a client who had intestinal surgery. Bowel sounds are usually absent for 24 to 36 hr postoperatively.Coffee-ground emesis is an expected finding for a client following a tonsillectomy. This is a result of swallowing blood during the surgery.Postoperative pain of 6 on a scale from 0 to 10 is an expected finding for a client with a total knee arthroplasty in the first 24 hr. A client post thoracotomy who has a chest tube with 150 mL of bright red blood in the collection chamber in the past hour. A client post small bowel resection with a temporary colostomy who has absent bowel sounds in all four quadrants. A client post tonsillectomy who is vomiting emesis with a coffee-ground appearance. A client post total knee arthroplasty with a PCA pump who is reporting a pain level of 6 on a scale from 0 to 10 at the operative site. 28

A nurse is caring for a client who had a small bowel resection 4 days ago. Assessment findings reveal a hard, distended abdomen with no bowel elimination in the last 2 days and absent bowelsounds. After notifying the primary care provider, which of the following actions should the nurseanticipate taking next?

Measuring abdominal girth is an important action, but it is not the next action the nurseshould take.The greatest risk to the client is fluid and electrolyte imbalance as a result of

accumulated fluid and gas in the gastrointestinal tract. The next action the nurse should take is to put in a nasogastric tube to decompress the bowel.Inserting an indwelling urinary catheter is an important action, but it is not the next action the nurse should take.Placing the client on strict intake and output is an important action, but it is not the next action the nurse should take. Measure the client's abdominal girth. Put in a nasogastric tube. Insert an indwelling urinary catheter. Place the client on strict intake and output. 29

A nurse is caring for a client receiving conscious sedation with midazolam (Versed) and fentanyl (Sublimaze). The client's respiratory rate decreases from 16/min to 6/min and the oxygen saturation decreases from 92% to 85%. Which of the following actions should the nurse anticipate taking first?

Assisting with intubation may be necessary, but it is not the priority action.Obtaining equipment necessary for CPR may be indicated, but it is not the priority action.The greatest risk to the client is respiratory arrest. Increasing the client's respiratory rate by administering a reversal agent is the priority action.Obtaining stat ABGs may be indicated, but it is not the priority action. Assist with intubation. Obtain equipment necessary for CPR.

Administer reversal agents. Obtain stat ABGs. 30

A nurse is caring for a client status post total hip arthroplasty. Which of the following assessment data indicates the client is at an increased risk for infection?

Most herbal remedies do not decrease a client's resistance to infection.Use of corticosteroids will inhibit leukocyte response, thus decreasing resistance to infection.Excessive exposure to sunlight can precipitate changes in the skin, but it does not increase risk for infection.A diet high in cholesterol can precipitate cardiovascular changes, but it does not increase risk for infection. Use of herbal remedies Long-term use of corticosteroids Excessive exposure to sunlight Diet high in cholesterol

ATI. MED-SURG. IMMUNE, AIDS & CANCER

A nurse is caring for a client who has a history of COPD and is admitted with a diagnosis of viral pneumonia. Which of the following findings should indicate to the nurse a need to contact the primary care provider?

This is an expected finding with pneumonia.A left shift in the WBC differential indicates that the pneumonia is of bacterial origin, rather than viral, and should be reported to the primary care provider.This oxygen saturation level is acceptable for a client with COPD.This is an expected finding for a client with pneumonia.

Consolidation in lower lobes by chest x-ray Left shift seen in WBC differential Oxygen saturation of 91% Orthostatic hypotension2

A nurse should recognize that which of the following clients is at the highest risk for pneumonia?

This client does not have the highest risk factors.This client does not have the highestrisk factors.This client does not have the highest risk factors.Two of the highest risk factors for pneumonia are being an older adult and living in a long-term care facility. A school-age child with a history of allergies and asthma A young adult client living in a college-based dormitory A middle adult client using an incentive spirometer following surgery An older adult client in a long-term care facility with dysphagia3

A nurse is planning to educate a group of high school teachers who will be taking students on a hike. What information should the nurse include in her plan regarding Lyme disease?

Testing should be done in 4 to 6 weeks. Earlier testing is not reliable.Ticks should be removed gently with tweezers or the fingers.Light colors should be worn so ticks on the bodycan be easily seen.Stage I Lyme disease is manifested by flu-like symptoms, a "bull's-eye" rash, muscle and joint pain, and stiffness.

"If bitten by a tick, you should get tested immediately." "If you have a tick embedded in your skin, apply a lit match to remove it." "You should wear dark-colored clothing to deter ticks from biting." "If you develop pain and stiffness in your joints, you should see your doctor." 4

A client is admitted with enlarged lymph nodes and a fever. To confirm the diagnosis of bacterial pharyngitis, the nurse should anticipate which of the following diagnostic tests?

Sedimentation rate is used to determine presence of inflammation in the body and isuseful in the diagnosing and monitoring of conditions such as rheumatoid arthritis.A chest x-ray is used to identify conditions such as pneumonia and pleural effusions.A throat culture is used toconfirm a diagnosis of bacterial pharyngitis by identification of specific micro-organisms presentin the pharynx.Peak flow rate is used to evaluate the effectiveness of asthma management. Sedimentation rate Chest x-ray Throat culture Peak flow rate 5

A nurse is planning an education program about testicular cancer for a group of male adolescents. Which of the following information should the nurse include?

Testicular cancer is common in men 15 to 40 years of age.The 5-year survival rate for this cancer, when diagnosed and treated early, is 95%.A testicular self-examination should be done on a monthly basis by examining the testicles after a bath or shower to allow for easierpalpation.Ultrasounds are not used to screen for testicular cancer. Testicular cancer is common in men older than 50. With early treatment, the survival rate is 50%.

Examine your testicles immediately after showering. Schedule a yearly ultrasound to screen for testicular cancer.

A nurse is performing a breast examination on a client. Which of the following findings by the nurse supports a suspicion of breast cancer?

Breast asymmetry may be a normal finding.An orange-peel appearance of the skinindicates advanced breast cancer due to blockage of lymph channels.The presence ofMontgomery's tubercles on the areola is a normal finding.A mass that is associated with breast

cancer is usually nonmoveable. Asymmetrical size of breasts Breast tissue with an orange-peel appearance Presence of Montgomery's tubercles on the areola Moveable mass in the left-lower breast quadrant7

A nurse is educating a client who is being admitted for a kidney transplant. Which of the following information should the nurse include in the teaching regarding hyperacute rejection?

This is an example of acute rejection.This finding is consistent with chronic rejection.Thisis the only treatment for hyperacute rejection.Immunosuppressive therapy is not used forhyperacute rejection, but it may be used to prevent chronic rejection. Hyperacute rejection may occur during the first few weeks after the transplant. If hyperacute rejection occurs, the organ may develop scar-like tissue. The organ will have to be removed if hyperacute rejection occurs. Immunosuppressive therapy decreases the incidence of hyperacute rejection. 8

A nurse is providing teaching to a client diagnosed with Hodgkin's lymphoma who is undergoingexternal radiation treatment. Which of the following should the nurse include in the care plan forthe irradiated area?

Antibacterial soaps are too harsh and should be avoided.Lotion can damage skinexposed to radiation and should be avoided.Skin should be patted instead of rubbed dry toavoid damage to the skin.Sun exposure can be damaging to skin exposed to radiation andshould be avoided. Use an antibacterial soap to cleanse the skin. Use lotion to moisturize irradiated skin. Avoid patting the skin when drying. Avoid direct sun exposure to the skin. 9

A client is diagnosed with non-Hodgkin's lymphoma and is receiving chemotherapy. Which of the following is a priority assessment finding?

Loss of hair is an adverse effect of chemotherapy, but it is not the priority finding.Anorexia is an adverse effect of chemotherapy, but it is not the priority finding.Oral mucositis is an adverse effect of chemotherapy, but it is not the priority finding.The greatest risk to the client is infection at the central IV site. Erythema at the site may indicate infection; therefore, this is the priority assessment finding. Loss of body hair

Report of anorexia Mucositis of the oral cavity Erythema at the IV insertion site10

A nurse is caring for a client who has an elevated prostate-specific antigen level. The nurse should anticipate that the client will undergo which of the following diagnostic tests?

This procedure is not a reliable diagnostic test for prostate cancer.Human chorionic gonadotropin is used to diagnose testicular cancer.This test determines the size and consistency of the prostate, thus assisting with differentiating between benign prostatic hypertrophy and prostate cancer.A pelvic ultrasound is not used to diagnose prostate cancer. Palpation of testes and lymph nodes Human chorionic gonadotropin level Digital rectal examination Pelvic ultrasound

11

A nurse is providing care to a client who has leukemia. The nurse reviews the client's laboratorydata and finds the platelet count to be 48,000/mm3. The nurse should anticipate implementing which of the following nursing interventions?

A diet low in vitamin K may further decrease coagulation.There is no indication for contact precautions.Epoetin alfa is not effective in increasing platelet production.A client who is thrombocytopenic is at risk for occult bleeding. Provide a diet low in vitamin K. Place the client on contact precautions. Administer subcutaneous epoetin alfa (Epogen). Test urine and stool for occult blood. 12

A nurse is providing teaching to a client with rheumatoid arthritis with a prescription for methotrexate (Trexall) therapy. Which of the following should the nurse include in the teaching?

Methotrexate is usually administered once a week.It takes 4 to 6 weeks for thesymptoms of rheumatoid arthritis to respond to methotrexate.Clients are advised to increaseintake of folic acid, not vitamin C, to help decrease the side effects of methotrexate.The dose of methotrexate for rheumatoid arthritis is low; therefore, hair loss is not usually a side effect. Daily injections will be required. Expect symptoms to subside in 4 to 6 weeks. Increase intake of vitamin C. Hair loss is expected. 13

A nurse should recognize that which of the following places a client at risk for hepatitis B?

This client is at risk because hepatitis B is transmitted by sexual contact.This client is notat risk because hepatitis B is not transmitted by casual contact.This client is not at risk becausehepatitis B is not transmitted by chemical waste.This client is not at risk because hepatitis B isnot transmitted by casual contact. Engaging in unprotected sexual intercourse Residing in an institutional setting

Working with hazardous chemical waste materials Traveling to a foreign country 14

A nurse is providing teaching to a client with systemic lupus erythematosus (SLE). Which of the following statements made by the client indicates an understanding of the teaching?

Clients with SLE should use sunscreens with an SPF of at least 30.SLE is a chronic autoimmune disease and cannot be cured with long-term immunosuppressivetherapy.Raynaud's syndrome commonly accompanies SLE and can cause painful vasoconstriction in the fingers when they are exposed to cold temperatures.SLE can affect all systems in the body and can subsequently cause pleural effusions and pneumonia. I should use a sunscreen with an SPF of at least 15. Long-term immunosuppressive therapy could cure this disease. I should wear gloves when it is cold outside. SLE should not affect my lungs or breathing. 15

A nurse is reviewing the daily laboratory work for a female client with acute leukemia. Which of the following is an expected finding?

A client with acute leukemia usually has an increased aPTT.A client with acute leukemia usually has a decreased hemoglobin level.A client with acute leukemia usually has a decreased Hct.A client with acute leukemia usually has a decreased platelet count. aPTT 90 seconds Hemoglobin 14 g/dL Hct 40% 170,000/mm3 platelet count

16

A nurse is planning education for a client who had a mastectomy and is being discharged. Which of the following information should the nurse plan to include in the teaching?

Numbness is normal along the arm on the side of the mastectomy.After 4 to 5 days, pain medication may only be needed at bedtime or as needed.The client should not shower for 7 to10 days, or until the drains and staples are removed.The drains should be emptied twice a dayand the output recorded so that it can be reported to the primary care provider. "Call your doctor if you experience numbness along the arm on your surgicalside." "You should take pain medication regularly every 4 to 6 hours for the first 2 weeks." "You can shower when you get home." "Empty your drains twice a day and write down the amount of fluid collected." 17

A nurse is caring for a client who has rheumatoid arthritis. The client states, I am in pain all the time. I don't know what to do. Which of the following recommendations by the nurse is thehighest priority?

Wearing comfortable, nonrestrictive clothing may help keep the client comfortable, but this is not the highest priority.The most effective way to manage chronic rheumatoid arthritis pain is to maintain a regular schedule of medications. Therefore, recommending taking

medications on a regular schedule is the highest priority.Using progressive relaxation techniques may help keep the client comfortable, but this is not the highest priority.Having family members assist with household chores may give the client an opportunity for rest, but this is not the highest priority. Wear comfortable, nonrestrictive clothing. Take your medications on a regular schedule. Use progressive relaxation techniques. Ask family members to help with household chores. 18

A nurse is caring for a client with a prescription for clindamycin (Cleocin) for acute pelvic inflammatory disease. Which of the following adverse reactions is the highest priority finding and should be immediately reported to the primary care provider?

The greatest risk to this client is pseudomembranous colitis, which is manifested by watery diarrhea. This finding should be immediately reported to the primary care provider, as the medication should be discontinued.Vaginitis is an adverse reaction of clindamycin, but it is not the highest priority finding.Urinary frequency is an adverse reaction of clindamycin, but it is not the highest priority finding.Nausea and vomiting are adverse reactions of clindamycin, but they are not the highest priority findings. Watery diarrhea

Vaginitis Urinary frequency Nausea and vomiting 19

A client notes a skin change on her arm and asks the nurse about it. Which of the following findings by the nurse supports a suspicion of skin cancer?

The appearance of this skin change supports the suspicion of malignant melanoma.A linear crack in the epidermis is not a sign of skin cancer.A raised vesicle that is filled with clear fluid is not a sign of skin cancer.A macule that is flat and oval shaped is not a sign of skin cancer. An irregularly shaped papule that is pigmented A linear crack in the epidermis A raised vesicle that is filled with clear fluid A macule that is flat and oval shaped 20

A nurse is assessing a client who is positive for HIV. Which of the following findings should cause the nurse to suspect that the client's diagnosis has progressed to AIDS?

The presence of small, purplish-brown raised skin lesions, indicating Kaposi's sarcoma, signifies that the client has developed AIDS.Fever and diarrhea indicate category B disease.Persistent, generalized lymphadenopathy indicates category A disease.A diagnosis of AIDS requires the CD4 count to be below 200 cells/mm3. Small, purplish-brown raised skin lesions Fever and diarrhea lasting longer than 1 month Persistent, generalized lymphadenopathy CD4+ cells decreased to 750 cells/mm3 21

A nurse is providing instructions to a client with a prescription for amoxicillin (Amoxil) for a respiratory infection. Which of the following statements by the client indicates to the nurse aneed for further teaching?

Antibiotics accelerate the elimination of birth control pills, making them lesseffective.Amoxicillin is one of the penicillins that should be taken with food.Opening the capsule or crushing the tablet is not going to alter the effectiveness of the medication.Antibiotics need to be taken until finished, even if the condition improves. My birth control pills are less effective while I am on this medication. I must take the medication on an empty stomach. The capsules can be opened and mixed with liquids if it is hard for me toswallow them. I will keep taking the medication until I have finished the entire bottle. 22

A nurse is caring for a client in an outpatient clinic who is receiving chemotherapy and has laboratory data revealing bone marrow suppression. The nurse should plan to instruct the client to do which of the following?

The client with bone marrow suppression is at an increased risk for bleeding and shouldnot be given acetylsalicylic acid or other platelet inhibitors.Rinsing a toothbrush with warm waterwill not adequately kill micro-organisms.Fresh fruits and vegetables can contain bacteria; therefore, the client with bone marrow suppression who has an increased risk of infection needsto avoid exposure to micro-organisms.Sun exposure does not pose a risk to a client receivingchemotherapy.

Take aspirin for minor aches and pains. Rinse toothbrush after each use with warm water. Avoid eating fresh fruit and vegetables. Wear clothing that will minimize sun exposure. 23

A nurse is providing teaching to a client who is to undergo a Pap smear. The nurse should instruct the client that she is being tested for which of the following?

A Pap smear is not used to diagnose uterine cancer.A Pap smear is used to diagnose cervical cancer.A Pap smear is not used to diagnose HPV.A Pap smear is not used to diagnose MRSA. Uterine cancer Cervical cancer Human papillomavirus (HPV) Methicillin-resistant Staphylococcus aureus (MRSA) 24

A nurse is planning to provide teaching to a client who is allergic to peanuts. Which of the following is the highest priority instruction that the nurse should include in the teaching plan?

It is important that other health care providers are aware of the client's allergy, but this is not the highest priority.It is important that the client wear a medical alert identifier, but this is not the highest priority.The greatest risk to the client is an anaphylactic reaction; therefore, the highest priority is for the client to be prepared for emergency treatment.It is important that theclient read food labels to identify hidden sources of potential exposure to peanuts, but this is notthe highest priority. Inform other health care providers of the allergy.

Wear a medical alert identifier. Carry an emergency anaphylaxis kit. Read food labels. 25

For a client diagnosed with HIV, which of the following laboratory findings suggests that medication therapy is effective in controlling disease progression?

This WBC count is low and does not indicate effectiveness of therapy.Clients with HIV often have lymphocyte values below 1,500/mm3. This finding does not indicate effectiveness of therapy.This finding indicates that the viral protein amount is decreasing in the blood, and there is a positive response to treatment.A low or decreasing ratio of CD4+/CD8+ indicates disease progression. WBC 3,500 cells/mm3 Lymphocyte count 1,500 cells/mm3 Decreased viral load Low CD4+/CD8+ ratio

26

A nurse is performing an assessment on a client who has systemic lupus erythematosus (SLE).

Which of the following are signs of complications associated with SLE? (Select all that apply.) Subcutaneous nodules is incorrect. Subcutaneous nodules are found with rheumatoid arthritis.

Decreased urine output is correct. Decreased urine output due to kidney damage is a commoncomplication of SLE.

Renal calculi is incorrect. Lupus nephritis, not renal calculi, is a complication of SLE.

Dyspnea is correct. Pleural effusion is a common respiratory complication of SLE.

Report of chest pain is correct. Pericarditis, including symptoms of chest pain, is the mostcommon cardiovascular manifestation of SLE.Subcutaneous nodules is incorrect. Subcutaneous nodules are found with rheumatoid arthritis.

Decreased urine output is correct. Decreased urine output due to kidney damage is a commoncomplication of SLE.

Renal calculi is incorrect. Lupus nephritis, not renal calculi, is a complication of SLE.

Dyspnea is correct. Pleural effusion is a common respiratory complication of SLE.

Report of chest pain is correct. Pericarditis, including symptoms of chest pain, is the mostcommon cardiovascular manifestation of SLE. Subcutaneous nodules Decreased urine output Renal calculi

Dyspnea Report of chest pain

27

A nurse is providing discharge teaching to a client who is HIV positive. Which of the following should the nurse include in the teaching?

This action destroys micro-organisms that may be present on the toothbrush.Usingseparate eating utensils is not necessary.The client does not need to wear a mask while outsideof the home.Clients who are HIV positive should not spend extended periods of time outside inplaces where contact with disease-causing micro-organisms is likely. Instruct the client to clean his toothbrush daily in the dishwasher. Recommend that the client use separate eating utensils from his family. Tell the client to wear a mask while outside of the home. Encourage the client to work in his garden for exercise. 28

A nurse is caring for a client who has neutropenia. Which of the following indicates a need for intervention?

A school-age child attending school places the client at risk for infection due to his immunocompromised status.These foods are allowed for a client who is immunocompromisedbecause they are part of a low-bacteria diet.An artificial flower arrangement poses no risk to theclient.Supplies should be placed in the room and only used by the client who isimmunosuppressed.

The client's granddaughter is visiting and telling him about her first day ofkindergarten. The client has a grilled ham and cheese sandwich, a banana, and yogurt on hislunch tray. The client's family brings in a silk flower arrangement. The client's assistive personnel places paper cups and plastic utensils in his room. 29

A nurse should recognize that which of the following places the client at risk for developing skin cancer?

Occupational exposure to chemical carcinogens places the client at risk for skincancer.A lowfiber diet places the client at risk for colon cancer.A high alcohol intake places the client at risk for liver cancer.Exposure to HPV places the client at risk for cervical cancer. Occupational chemical exposure Low-fiber diet High alcohol intake Exposure to human papillomavirus (HPV) 30

A nurse is caring for a client with Hodgkin's lymphoma. Which of the following findings should the nurse expect?

An overgrowth of B-lymphocyte plasma cells is found in clients with multiplemyeloma.ReedSternberg cells are cancer cells specific to Hodgkin's lymphoma usually locatedin nodal tissue.The Epstein-Barr virus is associated with Burkitt's lymphoma.HIV is associatedwith Kaposi's sarcoma. Overgrowth of B-lymphocyte plasma cells Reed-Sternberg cells

Epstein-Barr virus HIV virus

ATI. MED-SURG. 2.0

A nurse is preparing a client for a blood transfusion. Which of the following is a correct step for the nurse in preparation for the procedure?

Normal saline is the solution to administer with blood products.Baseline information isalways required to determine if the client can tolerate the transfusion and to help identifytransfusion reactions.Hemolytic reactions most often occur with the first 50 mL of the infusion. The nurse should reassess the temperature and vital signs 15 min after the start of thetransfusion and remain with the client during the first 15 to 20 min of the transfusion.Using alarge-bore (18or 19-gauge) needle allows the blood cells to flow more easily without occludingthe lumen of the catheter. Obtain lactated Ringer's solution to administer with the blood product. Obtain vital signs and temperature immediately before starting the transfusion. Monitor the client every 30 min after beginning the transfusion. Start an intravenous infusion with a 22-gauge needle. 2

A client experiences a decreased level of consciousness, expressive aphasia, ataxia, and

hemiparesis. The client is admitted to the hospital with the diagnosis of cerebrovascular accident (stroke). Which of the following is an appropriate nursing intervention for a client in theimmediate period following a stroke?

During the first 72 hr after a stroke, the client is at an increased risk for increasedintracranial pressure. Extreme hip flexion should be avoided because this can increase theintrathoracic pressure, which in turn can increase intracranial pressure.When performingnursing care or assisting a client to perform activities of daily living, the nurse should userepetition and cues. Breaking down tasks into steps helps the client overcome the possibledifficulties of receptive aphasia.Clustering nursing procedures should be avoided becauseperforming several activities within a short period of time can dramatically elevate theintracranial pressure.Because of sensory and perceptual changes and deficits, the nurse shouldapproach the client from the unaffected side. This helps the client perceive who is approaching. The client's unaffected side should also be positioned facing toward the door of the room. Elevate the head of the client's bed to a high-Fowler's position. Use frequent verbal and tactile cues, breaking down tasks into discrete steps. Cluster nursing procedures to minimize the client's fatigue. Approach the client from the side of the body that is affected by the stroke. 3

A client with a pack per day smoking history is 24 hr post abdominal surgery. Client findings include a temperature of 38 C (100.4 F), tachycardia, and pain rated at 3 on a scale of 1 to 10that increases to 7 with deep breathing. Which of the following nursing actions will most improvethe client's respiratory status?

A side-lying position may enhance the client's comfort, but it will not improve the client's respiratory status and may even compromise it further.Analgesia is an important nursing intervention, but if the client is too sedated by the medication, respiratory status may be furthercompromised.Use of incentive spirometry will encourage the client to breathe deeply. Coughing, deep breathing, and movement will maintain an open airway and encourage mobilization ofsecretions, which will improve respiratory status.This action may increase the client's comfort inthe short term, but it will not improve the client's respiratory status. Pillows should be removedfrom beneath the knees periodically to prevent blood clots from forming in the client's legs. Place the client in a side-lying position with a pillow against the client's back. Administer opioid analgesics to the client as often as prescribed. Encourage the client to use an incentive spirometer every hour while awake. Place one or two pillows beneath the client's knees when the client is in bed. 4

A client with a new colostomy asks the nurse about the best way to get a wafer (skin barrier) to stick to the skin. The nurse explains that the most effective means of ensuring adherence is to

Moisturizing soap and cream interfere with adhesion.The lotion will produce a moistbarrier between the skin and the stoma, thus preventing the wafer from securely adhering to theskin.The gauze creates a barrier and will not allow the wafer to adhere to the skin, increasingthe risk for stool to irritate the peristomal skin.The skin sealant protects the skin, provides betteradhesion, and has the added benefit of making the appliance easier to remove. gently cleanse the area with a moisturizing soap before applying the wafer. place a thin layer of lotion on the peristomal area before applying the wafer. place a gauze dressing on the peristomal skin and apply the wafer.

apply a skin sealant and allow it to dry before applying the wafer. 5

A nurse is assigned to care for a client who has been readmitted to the hospital three times for different complications related to congestive heart failure (CHF). Which of the following should be included in the discharge teaching by the nurse to prevent future hospital admissions due tocomplications of CHF?

The client should count and record his pulse before taking each dose of digoxin. Achange in pulse rate or rhythm or abnormal rate should be reported to the primary careprovider.Clients with CHF should be encouraged to stay as active as possible and to develop a regular exercise regimen.Diuretics like furosemide (Lasix) deplete potassium and often causehypokalemia. Potassium supplements may be prescribed.Rapid weight gain - 1.4 kg (3 lb) in aweek or 0.5 to 0.9 kg (1 to 2 lb) overnight - is a symptom of worsening heart failure. Count pulse after taking digoxin (Lanoxin). Encourage activity restrictions. Restrict dietary intake of potassium. Report any rapid weight gains.

The priority nursing intervention to protect client safety when providing tracheostomy care is

Preventing decannulation during tracheostomy care is critical, particularly whenchanging the trach ties.The pad is replaced when it becomes soiled, which may not occur witheach suctioning. This is not the highest priority intervention.Gathering the necessary equipment is important for any procedure, but it does not necessarily protect client safety while providing

tracheostomy care.Suctioning may not be necessary before providing care. Routine unnecessary suctioning causes mucosal damage, bleeding, and bronchospasms. securing the tracheostomy tube when changing the ties. replacing the pad beneath the tube after suctioning. gathering the necessary equipment before providing care. suctioning the tracheostomy tube before providing care. 7

A nurse is teaching pursed-lip breathing to a client newly diagnosed with chronic obstructive pulmonary disease. The nurse recognizes that the client understands the concept when the client says,

Diaphragmatic breathing is appropriate during low activity times, such as before going to bed.This is improper pursed-lip breathing technique. The client should breathe in slowly through the nose and exhale slowly through pursed lips to delay airway compression and reduce air trapping.The pursed-lip breathing technique is best performed during periods of activity and when feeling short of breath. The client should breathe in slowly through the nose and exhale slowly through pursed lips to delay airway compression and reduce air trapping.Pursed-lip breathing technique requires slow breathing to prevent air trapping and collapse of the alveoli. "I should perform pursed-lip breathing before going to bed." "When I'm fatigued, I should inhale slowly through pursed lips." "Pursed-lip breathing works best during activities such as walking up stairs."

"I will exhale several short, quick breaths while pursing my lips." 8

To help manage the pain associated with herpes zoster of a lower extremity, the nurse should

Astringent compresses such as Burow's solution promote healing but do not relieve pain.Irritation or rubbing the area may intensify the pain and/or exacerbate pruritus associated with the condition.Antibiotics may be administered to prevent secondary infections, but they do not relieve pain.Salicylic acid is a keratolytic agent used in the treatment of warts. apply warm compresses. use a bed cradle to keep sheets off affected areas. administer antibiotics. apply salicylic acid to affected dermatome. 9

A client is admitted with upper gastrointestinal distress. The nurse should suspect metabolic alkalosis if the client has

Vomiting causes excessive loss of acid, which can result in metabolic

alkalosis.Excessive blood glucose levels are associated with metabolic acidosis, not alkalosis.Salicylate intoxication is associated with metabolic acidosis, not alkalosis.The potassium level will be lower than normal in metabolic alkalosis, and this level is high. a recent experience of prolonged vomiting. diabetes mellitus and an elevated glucose level. routinely ingested large amounts of aspirin. a potassium level of 6.1 mEq/L. 10

A college student living in a campus dormitory presents to the emergency department with a recent history of acute sinusitis. Laboratory tests confirm bacterial meningitis. Upon monitoringthe client, the nurse should observe which of the following symptoms?

An extremely stiff and rigid neck (nuchal rigidity) is a sign of meningitis.A flexed trunk with a stooped posture is typical of clients with Parkinson's disease.A positive Kernig's sign(resistance to extension of the client's leg from a flexed position) is an indicator of meningitis. A positive Brudzinski's sign (flexion of extremities occurring with deliberate flexion of the client's neck) is another indicator of meningitis.A masklike facial expression is typical of clients with Parkinson's disease. Flaccid neck with generalized poor muscle tone Flexed trunk with stooped posture Resistance to extremity extension from flexed position Masklike facial expression

11

A client is suspected of having a myocardial infarction. The nurse correctly prepares the client for the electrocardiogram (ECG) by completing which of the following steps?

The client should be in a supine or semi-reclined position.Conduction gel is applied toeach electrode prior to placement. The gel should be moist and fresh.Cleansing the skin

reduces skin oils and improves electrode contact.The placement of the electrodes is crucial inobtaining an accurate ECG, and the electrodes should be placed on the anterior surface. Position the client in the left lateral Sims' position. Assure that each electrode is dry before application. Cleanse the client's skin prior to electrode placement. Place the electrodes around the client's chest and back. 12

A healthcare employee accidentally sprays disinfectant in her right eye. Which of the following isthe priority intervention?

The eye may be patched later for comfort, but this is not the priority intervention.Thepriority intervention is to immediately flush the chemical out of the eye to prevent furtherdamage.Although the incident should be reported to the supervisor, this is not the first action.The neutralizing agent may result in a chemical reaction generating heat. Place a patch over her right eye. Irrigate the affected eye with water. Report the incident to the supervisor. Instill a neutralizing agent into the eye. 13

A nurse receives a needle stick following a blood draw on a client who is HIV-positive. An enzyme-linked immunosorbent assay (ELISA) screening test for HIV is scheduled. Which of thefollowing statements by the nurse indicates an accurate understanding of the test?

The ELISA tests for the presence of antibodies that have been produced by the client's immune system in response to HIV.Not all clients need to be retested. Retesting is dependent on the timing of exposure and the client's risk of exposure. Retesting may be needed if the clientexperiences a recent exposure to the virus, and insufficient time has elapsed for the body toproduce detectable levels of the antibody, which takes from 3 to 12 weeks.A negative test indicates that antibodies have not been detected but does not reflect conclusively whether or

not the client has been infected. If the exposure was recent, the client should be counseled to have follow-up testing.The ELISA does not determine the amount of HIV in the blood. "This test detects antibodies in the blood produced by the HIV virus." "I will need to be retested in 2 weeks for the virus." "If this test is negative, I can be confident that I have not been infected with HIV." "This test determines the amount of HIV present in my blood." 14

A client is scheduled for an early morning surgical procedure. Which of the following preoperative nursing actions should the nurse take?

It is likely that most of the client's regularly scheduled medications will be held the day of a surgical procedure.The primary care provider is responsible for giving an explanation of the procedure. The nurse can clarify any information that is unclear.Part of preoperative teaching includes the use of an incentive spirometer, deep breathing, and coughing to clear the lungs of anesthesia to prevent respiratory complications.Specimen collection and diagnostic laboratory studies should have been performed by this point. The nurse is responsible for reviewing laboratory findings and notifying the primary care provider of any abnormal values. Administer regularly scheduled medications to the client. Explain the procedure to the client and how it is performed. Reinforce deep breathing and coughing techniques with the client. Collect specimens from the client for diagnostic laboratory studies.

15

A client who has been diagnosed with esophageal varices is receiving discharge instructions from the nurse. Which of the following should the nurse plan to incorporate into the client teaching with regard to reducing the risk for bleeding?

There is no clear evidence that poorly chewed food causes variceal bleeding.There is no clear evidence that reflux of gastric acid causes variceal bleeding.Elevating the head of the bed will not prevent variceal bleeding.Esophageal varices result from portal hypertension as blood from the liver backs up and enters the esophageal and gastric vessels. Bleeding of the esophageal varices is a life-threatening situation. Any activity that increases abdominal pressure, such as vigorous physical exercise or weight lifting, can precipitate variceal bleeding. "Carefully chew your food before swallowing." "Take medication to prevent gastric reflux." "Sleep with the head of your bed elevated." "Avoid heavy lifting and vigorous physical exercise."

16

A client is being discharged from the hospital following a gastrectomy for ulcers. The nurse

recognizes that the discharge teaching has been effective when the client states that dumping syndrome can be prevented by

Liquids at meal times are discouraged because they facilitate the rapid movement of gastric contents into the small intestine.Starchy vegetables, rice, pasta, and potatoes are encouraged and do not cause dumping syndrome.A diet that is high-protein, high-fat, and lowto moderate-carbohydrate is the recommended dietary measure for the management of dumping syndrome.Concentrated sweets, such as sweetened fruit or juice, are excluded from the diet because they can lead to dumping syndrome. avoiding liquids at meal times.

excluding starchy vegetables. avoiding high-protein meals. eating sweetened fruits. 17

A nurse is caring for a male client with benign prostatic hypertrophy (BPH). The nurse recognizes that this condition places the client at increased risk for

Stasis of urine in the bladder from incomplete emptying related to BPH promotes the growth of bacteria and places the client at increased risk for urinary tract infections.BPH is not a cause of renal calculi, which are related to dietary and genetic factors.Neurogenic bladder results from neurological injuries or lesions and is not associated with BPH.BPH is not a risk factor for renal cancer. urinary tract infections. renal calculi. neurogenic bladder. renal cancer. 18

A client diagnosed with cellulitis of the right lower leg is to have a wound debridement when the dressing is changed. The nurse is to administer two tablets of hydrocodone/acetaminophen

(Vicodin) in preparation for the procedure, which will begin at 1100. For the most effective pain management, the nurse should administer the medication at

The effectiveness of the medication has peaked and is starting to decrease by 1100, not providing the best pain control.The procedure should be started close to the time when the medication is most effective, so that pain relief continues throughout and after the procedure.The medication's onset starts at 10 to 30 min. The peak action occurs between 30 to 60 min. After administration, the medication's duration of effect is from 4 to 6 hr.Pain will not be controlled immediately following medication administration. 0900. 0930. 1030. 1100. 19

Which of the following is the priority nursing action for a client who displays a sudden change in mental status following a femur fracture?

Altered mental status related to fat embolism is caused by low arterial oxygen level. Immediate pulse oximetry monitoring will determine oxygenation level. The client may require

oxygen therapy.A client with a femur fracture is most at risk for a fat embolism. Increasing blood flow to the brain will not assist with altered mental status caused by fat embolism.The client may be experiencing a fat embolism or pulmonary embolism. Both are medical emergencies requiring further data and assessment by the primary care provider prior to administering medications.A client with a femur fracture is most at risk for a fat embolism. A CT scan does not address this risk. Obtain a pulse oximetry reading. Place the client in Trendelenburg position. Administer a daily dose of warfarin (Coumadin). Request an order for a computerized tomography (CT) scan. 20

A client who has just undergone a bronchoscopy reports that her throat is sore, and she requests a glass of water. The nurse determines that the client's gag reflex has not returned. Which of the following actions should the nurse take in response to the client's request?

The client is at high risk for aspiration because her gag reflex has not returned. Ice chipsshould be withheld until the client's gag reflex has returned.The client is at high risk foraspiration because her gag reflex has not returned. Throat lozenges should be withheld until theclient's gag reflex has returned.The client is at high risk for aspiration because her gag reflexhas not returned. Water should be withheld until the client's gag reflex has returned.This intervention does not require the client to swallow, thus reducing the risk of aspiration while still promoting client comfort. Provide the client with ice chips, which she can suck on for moisture. Encourage the client to use a throat lozenge to numb her throat. Allow the client to have small sips of water in the nurse's presence. Give the client a mouth swab to moisten the inside of her mouth.

21

A nurse knows that a client with hypertension understands lifestyle modifications appropriate for this disease when the client states,

Shellfish are high in cholesterol and should be avoided in clients with hypertension.Overthecounter medications may contain sodium and may interact with prescriptionmedications.Secondary smoke substantially reduces blood flow velocity in the

coronaryarteries.Antihypertensive medication therapy should continue as prescribed despite an absenceof symptoms until discontinuation is discussed with the provider. "I need to eat more seafood. I like shrimp and lobster, so I'll increase them in my diet." "I need to consult my physician before taking any over-the-counter medications." "I will try to quit smoking, but my family doesn't have to change their smokinghabits." "I can stop taking my medication on a routine basis once I start feeling better." 22

A client returns to the nursing unit after abdominal surgery. The nurse notes that the client appears anxious and is pale with moist skin. Vital signs are: blood pressure 88/53 mm Hg, pulse116 beats/min, respirations 22/min, and temperature 37.3 C (99.2 F). Which of the following actions should the nurse perform first?

The client's symptoms are indicative of shock. Because the client has just returned fromsurgery, the most likely cause of this complication is bleeding.While estimated blood loss and fluid intake are important indicators of fluid balance, accessing this information is not the criticalnursing action at this time.Notification of the primary care provider or surgeon is appropriate, butthe nurse needs to gather additional information first.While replacing lost blood volume isimportant, the nurse first needs to determine the cause of the problem. Increasing the rate ofinfusion would not be an independent nursing decision. Check the client's surgical site for signs of bleeding. Review the surgical record for fluid intake and estimated blood loss. Notify the client's surgeon of these findings. Increase the flow rate of the intravenous normal saline infusion.

23

A nurse is admitting a client with cholecystitis. Which of the following client findings is a manifestation of this disease process that the nurse should expect?

Urine may be dark if there is an obstructive process and jaundice. Otherwise, urine coloris normal with chronic cholecystitis.Episodic or vague upper-abdominal pain or discomfort thatcan radiate to the right shoulder is an expected client finding.Steatorrhea may occur fromdecreased fat absorption secondary to lack of bile. However, alternating constipation and diarrhea are not manifestations of cholecystitis.Cholecystitis primarily affects the gastrointestinaltract and does not alter the skin or fingernails. Obstruction of the bile duct, however, can cause jaundice. Bright yellow-colored urine output Pain radiating to the right shoulder Alternating constipation and diarrhea Dry skin and brittle fingernails24

A client is receiving total parenteral nutrition (TPN). Which of the following nursing actions is a priority to best prevent complications?

Weights, input, and output should be obtained daily. The client is at an increased risk for fluid imbalance while on TPN due to the hyperosmolarity of the TPN solutions.Diarrhea can develop with the administration of parenteral nutrition. Stool softeners are contraindicated whileon TPN.This is an important intervention to keep a client who is taking nothing by mouth comfortable and to prevent the mouth from becoming dry or developing sores. This is not the

priority intervention to prevent complications.Because of the amino acid dextrose component ofTPN solutions, clients are at high risk for hyperglycemia. Finger stick blood glucose isperformed by the nurse to monitor for hyperglycemia as often as every 4 hr while the client is onTPN, especially if the client is on insulin. Obtain weekly weights. Administer stool softeners. Offer premoistened oral swabs. Check finger stick blood glucose. 25

A primary care provider has ordered Mucinex 1,200 mg twice a day for a client. The Mucinex is available in 600 mg capsules. How many capsules should the client plan to take each day? capsules 1,200 mg x 2 times per day = 2,400 mg daily

2,400 mg daily = 4 capsules per day600 mg per capsule1,200 mg x 2 times per day = 2,400 mg daily

2,400 mg daily = 4 capsules per day600 mg per capsule

26

A client taking digoxin (Lanoxin) starts to experience confusion, anorexia, nausea and vomiting, and visual changes. The client is also taking potassium chloride (K-Dur), famotidine (Pepcid), furosemide (Lasix), and levothyroxine (Synthroid). Which of the following medications mostlikely has contributed to the development of digitalis toxicity?

Adequate potassium levels may prevent digitalis toxicity.Famotidine has no documented interaction with digoxin.Levothyroxine can decrease digoxin serum levels and minimize thetherapeutic effect.Furosemide increases potassium loss, which leads to hypokalemia and increases the risk of digitalis toxicity and ventricular dysrhythmias. Potassium chloride Famotidine Levothyroxine Furosemide 27

A woman is admitted to the hospital with a suspected left ovarian cyst without rupture. The nurse recognizes that which of the following findings is indicative of this medical diagnosis?

Ovarian cysts cause tenderness over the affected ovary, but they do not produce abdominal rigidness unless ruptured.Fever is not a sign of ovarian cysts unless they haveruptured.Urinary frequency is not a sign of ovarian cysts.If the ovary is enlarged, the womanmay feel pelvic fullness and tenderness over the affected ovary. Abdominal rigidness Fever of 38.9 C (102 F) orally Urinary frequency Pelvic tenderness 28

When caring for a client with an arteriovenous fistula, the nurse correctly concludes that thearea of anastomosis is patent when a

The presence of a thrill is one finding indicative of a patent AV fistula, but visualization is not the correct method of checking for a thrill.A palpable thrill (felt by the fingertips) indicates a patent AV fistula.A bruit is one finding indicative of a patent AV fistula, but visualization is not thecorrect method of checking for a bruit.A bruit is indicative of a patent AV fistula, but it cannot be palpated. thrill can be easily visualized. thrill can be readily palpated. bruit can be easily visualized. bruit can be readily palpated. 29

A client with a new C-7 spinal cord injury is participating in an acute rehabilitation program. The nurse should alert the primary care provider if the client

The client or caregiver is encouraged to do this to prevent skin breakdown.Wearing shoes is beneficial because it prevents foot drop.Clients are often placed on stool softeners tofacilitate easy passage of stool and prevent constipation from lack of mobility.To prevent a urinary tract infection and urinary stasis, the post void residual should be no more than 60 to100 mL. This finding needs to be reported. performs lifts in the wheelchair every 2 hr.

insists on wearing shoes even though not ambulatory. has soft, unformed stools on a regular basis. has a post void residual volume of 130 mL. 30

The temperature of a client who has just been anesthetized is rapidly increasing. The anesthesiologist alerts the surgeon that the client may be experiencing malignant hyperthermia. The circulating nurse should immediately prepare to administer

The anesthesiologist is responsible for maintaining oxygenation of the client during thisevent and will administer oxygen through the client's endotracheal tube.Intravenous dantrolenesodium is the drug of choice for reversing malignant hyperthermia. It is the nurse's responsibilityto prepare the medication and either administer it or assist with its administration.The client withmalignant hyperthermia experiences hyperkalemia as potassium leaves the cells. The administration of IV potassium is contraindicated. Insulin is administered to move potassiumback into the cell and reverse hyperkalemia.Calcium-channel blockers are contraindicated astreatment for malignant hyperthermia because the client is experiencing hypercalcemia. supplemental oxygen. dantrolene sodium. intravenous potassium. calcium-channel blockers.

31

A nurse is checking the understanding of a client newly diagnosed with type 1 diabetes mellitus who will be performing blood glucose monitoring at home. The client demonstrates an accurate understanding of the procedure by stating, "I will check my blood glucose level

If insulin is based on after-meal levels, which are high, the client may receive too muchinsulin, resulting in hypoglycemia.Type 1 diabetes mellitus is difficult to control, and frequent glucose checks are necessary to regulate the amount and type of insulin needed. It is best tocalculate insulin dosage based on before-meal glucose levels to prevent hypoglycemia.Thismonitoring schedule is not often enough for a client with type 1 diabetes mellitus to ensure goodglucose control.This monitoring schedule is not often enough for a client with type 1 diabetesmellitus to ensure good glucose control. Glucose monitoring should be conducted routinely. following each meal." each morning and before meals." at 9 a.m. and 9 p.m. each day." when I feel that it is elevated." 32

A 55-year-old female client with reports of vaginal dryness and night sweats is being evaluated for possible hormone replacement therapy (HRT). The nurse is obtaining the client's health history. Which of the following conditions is a contraindication for HRT?

The age at which menopause symptoms began is not a contraindication for HRT.HRThas been associated with deep vein thrombosis. Therefore, a history of blood clots and treatment is a contraindication for HRT.Neither Tylenol use nor migraine headaches are contraindications for HRT. However, HRT can exacerbate migraine headaches for some women.Hypercholesterolemia is not a contraindication for HRT. However, cardiac disease may be a contraindication in some women. Menopause symptoms beginning at age 50 History of treatment for blood clots

Tylenol usage for migraine headaches Increased serum cholesterol levels 33

A nurse is providing discharge teaching regarding diet to a client with Crohn's disease. Which ofthe following dietary interventions is important for the nurse to reinforce?

Crohn's disease is an inflammatory bowel disease that involves the entiregastrointestinal tract, most commonly the small intestine and terminal ileum. The disease typically extends through all layers of the bowel. Long-term dietary treatment consists of a lowfiber, low-residue diet. The client may also need vitamin B12 injections because the small intestine cannot absorb it.This is the appropriate long-term diet for a client withdiverticulitis.Crohn's disease does have implications regarding vitamin absorption, but limitingfat-soluble vitamin intake is not indicated for Crohn's disease.Crohn's disease does have implications regarding vitamin absorption, but increasing fat-soluble vitamin intake is notindicated for Crohn's disease. Long-term dietary treatment for Crohn's disease includes a low-fiber, low-residue diet. Long-term dietary treatment for Crohn's disease includes a high-fiber, highresidue diet. Long-term dietary treatment for Crohn's disease includes limiting fat-solublevitamin intake. Long-term dietary treatment for Crohn's disease includes increasing fat-solublevitamin intake. 34

Which of the following nursing actions is most important during the first 24 hr following surgeryto prevent postoperative complications?

Pain medications should be titrated to provide effective pain control because pain can

prevent the client from coughing, deep breathing, turning, sitting up, or moving the legs, all ofwhich help prevent postoperative complications.Splinting the abdomen reduces pain associatedwith coughing and deep breathing. This allows the client to more effectively perform these tasks, which help to prevent postoperative complications.The client should be assisted with positionchanges at least every 2 hr following surgery to prevent atelectasis and skin pressure.A pillow orrolled towel under the knees restricts circulation and increases the risk for thrombophlebitis. Titrate pain medication to the lowest prescribed dose. Reinforce splinting the abdomen during coughing and deep breathing. Assist the client with position changes every 4 hr. Encourage placing a pillow or rolled towel beneath the client's knees. 35

A nurse caring for a client with acute glomerulonephritis should expect which of the following findings?

Scleral jaundice is present with liver disease due to accumulation of bilirubin, but it is notobserved in clients with acute glomerulonephritis.Exophthalmos is protrusion of the eye beyondits normal position within the bony orbit. It is characteristic of hyperthyroidism.Blepharitis isredness, swelling, and crusting along the lid margins of the eye. This is often due to bacterial invasion of the lid margins, but it is not observed in clients with acute glomerulonephritis.Theedema associated with acute glomerulonephritis initially appears in lowpressure areas such asaround the eyes (periorbital edema). Scleral jaundice Exophthalmos Blepharitis

Periorbital edema

36

An older adult client with Parkinson's disease is being discharged. Which of the followingsuggestions should the nurse make to the client's caregiver to help prevent the client fromfalling at home?

Falls can be prevented by having the client look forward rather than looking toward thefeet.All liquids should be thickened to prevent aspiration or choking.Small rugs increase thelikelihood of tripping and falling.Clients should be encouraged to participate as much as possiblein the performance of self-care. Adaptive devices can be used to promote independence while protecting safety. Remind the client to look forward when ambulating. Encourage the client to drink plenty of water. Use small carpeted rugs in the home for traction. Assume clothing and feeding responsibility for the client. 37

A nurse is caring for a client who had a sigmoid colon resection 3 days ago. Which of the following atypical findings should the nurse report?

Drainage from the stomach is usually dark green.Small amounts of serosanguineousdrainage is a normal postoperative finding.After 48 hr, a temperature over 38.4 C (101 F) may

indicate an infection.It may take several days following surgery for bowel sounds to return tonormal as peristalsis returns. Hypoactive bowel sounds are likely normal, but should continue tobe monitored. Dark green drainage from the nasogastric tube Scant serosanguineous drainage on the dressing Oral temperature of 38.4 C (101.2 F) Hypoactive bowel sounds in all quadrants38

Which of the following is the priority nursing intervention when caring for a client who is in a halofixation device with a vest following a cervical fusion?

Pin sites should be checked once every 8 hr, not once a day.Any changes in movement or sensation might indicate spinal cord compression from the unstable fracture. The primary care provider should be notified of any changes in neurovascular status.The halo vest should beremoved only in a medial emergency for cardiopulmonary resuscitation. A wrench is attached to the vest at all times in case of a medical emergency.The client should never be moved by pulling or holding onto the halo device. Check all pin sites once daily and document. Monitor the client for changes in movement and sensation. Remove the vest each shift to monitor pressure points. Reposition the client every 2 hr by holding the halo device. 39

A nurse is able to validate that a client correctly understands the lithotripsy procedure when the client states,

An ultrasound procedure uses reflected sound waves to visualize thestones.Extracorporeal shockwave lithotripsy is a noninvasive procedure that uses shock wavesto pulverize the stones so they can be eliminated in the urine.A cytoscopy is the insertion of a

cystoscope into the bladder to visualize the stones and possibly remove them.No incision ismade during a lithotripsy. "Sound waves will be sent into my bladder and then pictures will be taken toshow where the stones are located." "I will be lying down as sound shock waves are applied to my body to break upthe stones." "An instrument will be passed into my bladder so that the stones can be removedif they are visible." "I'll receive sedation while the physician destroys the stones through a smallincision." 40

A client admitted with deep vein thrombophlebitis reports heaviness in her lower extremities. Anticoagulation therapy is initiated as prescribed. The nurse best promotes safety with which of the following interventions?

The affected extremity should not be massaged due to the risk of the thrombus dislodging and becoming an embolus, which can then travel to the pulmonary circulation.Theclient should be instructed to avoid activities or positions that produce pressure on the legmuscles and might dislodge the thrombus.Aspirin should be avoided for a client receivinganticoagulant therapy.A positive Homans' sign is present in only 10% of clients with deep vein thrombosis, and false-positive findings are common. For this reason, monitoring for a positiveHomans' sign is not advised. Instead, the limb with pain should be assessed for induration, warmth, redness, and edema, and it should be compared to the contralateral side. Gently massaging the affected extremity Encouraging bedrest with diversional activities

Administering aspirin as prescribed for discomfort Monitoring for and reporting a positive Homans' sign

41

A nurse is reviewing the laboratory results for a client who is suspected of having liver cancer. Which of the following findings should be reported to the primary care provider because itsupports the presence of liver cancer?

Liver cancer damages the liver, causing an increase in the ALT.Acute liver disease, such as cancer, causes an elevation in total serum complement levels.Hepatic cell destruction from processes such as liver cancer cause an increase in serum LDH.Fecal urobilinogen isdecreased when obstructive liver disease occurs from processes such as cancer. Decreased serum alanine aminotransferase (ALT) Decreased total serum complement Increased serum lactate dehydrogenase (LDH) Increased fecal urobilinogen42

Bowel retraining has been recommended for an older adult client who is admitted to therehabilitation unit following a spinal cord injury. Bowel retraining for this client should include

A consistent time for bowel elimination should be initiated, taking into account the client's

normal time for bowel elimination. Any routine that may help as a triggering mechanism to facilitate and ensure an acceptable bowel evacuation is recommended in the re-establishmentof a defecation pattern.Digital stimulation with or without suppositories may help promote bowel

evacuation and assist in re-establishing a bowel routine.Stimulant laxatives promote intestinal peristalsis. They should be avoided when attempting to establish a bowel retraining program. The overuse of stimulant laxatives can decrease rectal tone and diminish the sensation of the urge to defecate.A high-fiber diet is recommended along with increased fluids (if not contraindicated) to assist in the re-establishment of a bowel pattern. Increasing dietary fiber is most effective if dietary fat is reduced. a consistent toileting schedule for bowel elimination. avoiding rectal stimulation before attempting to defecate. taking a stimulant laxative as needed to promote elimination. decreasing the dietary bulk to ease the passage of stool. 43

A client is receiving daily radiation therapy to the neck for laryngeal cancer. The client has developed redness and dry skin at the site. To help alleviate this problem, the nurse should instruct the client to

Soft material will prevent additional irritation to the site.The client should avoid applying lotions or creams to the site unless prescribed by the primary care provider. They may further irritate the skin.The radiation site should be gently cleansed with a mild soap and water to prevent further irritation and drying of the skin.Warm, moist heat is contraindicated because it may increase the risk of further skin breakdown. wear shirts with soft material.

use moisturizing lotions or creams. scrub the site with soap and water. apply warm, moist heat to the site three times daily. 44

An older adult client is moving to an assisted-living facility. The nurse is reviewing the client's immunization history to determine the risk for community-acquired infections. The client's record indicates that the client received the hepatitis B vaccine series 10 years ago, the tetanus vaccine 3 years ago, and the influenza and pneumococcal vaccines 1 year ago. Which of the following immunizations does the client need at this time?

A hepatitis B vaccine is not indicated for this client. A titer could be obtained if there is doubt concerning the client's immunity.Influenza vaccines should be administered annually, especially for older adult clients.A pneumococcal vaccine is not indicated for this client. The current recommendation is for clients to receive this vaccine every 5 years.The tetanus vaccine is currently recommended every 10 years, so it is not indicated for this client. Hepatitis B Influenza Pneumococcal Tetanus 45

A nurse is reinforcing teaching to a client with systemic lupus erythematosus (SLE). Which of the following should the nurse include?

Clients who have systemic lupus erythematosus should protect their skin by avoiding prolonged exposure to sunlight and ultraviolet light. Long sleeves and wide-brimmed hats should be worn when outside as well as sun-blocking agents.Cortisone, a topical corticosteroid preparation, is applied to the facial maculopapular rash to reduce inflammation and promote healing of skin lesions. Chronic steroid therapy is also used to treat the systemic disease

process.The skin should be well-moisturized. Lotions should be applied liberally to the skin. Cosmetics, if worn, should contain moisturizers.Pregnancy can create stress for women of childbearing age and can exacerbate the disease. Pregnancy is not advised for women with SLE, especially if there is cardiac, renal, or central nervous system involvement. Avoid sunlight and ultraviolet light. Avoid topical or systemic steroids. Avoid lotions and creams on the skin. Avoid the use of birth control pills.

46

A client has arrived at an outpatient clinic for an influenza vaccination. While performing the health screening, the nurse recognizes that the vaccine is contraindicated if the client

Cardiac disease and hypertension are not contraindications for the influenza vaccine. Persons with these conditions are encouraged to get this immunization.Pregnancy is not a contraindication for the influenza vaccine according to the Centers for Disease Control and Prevention.An allergy to eggs is a contraindication for the influenza vaccine because it contains egg protein.A history of cancer is not a contraindication for the influenza vaccine. has a history of cardiac disease and hypertension. is 7 months pregnant.

has a severe allergy to eggs. has a history of cancer. 47

A nurse is performing chest compressions on a client who has no pulse or respirations. Another nurse is providing slow breaths using a bag after the client has been intubated. In compliance with American Heart Association (AHA) guidelines, the first nurse should

This ratio is appropriate if there are one or two rescuers performing CPR.This ratio of compressions to breaths is no longer in compliance with AHA guidelines.The ratio of chest compression to breaths is important to maintain. Therefore, the nurses should alternate between chest compressions and breaths.This is not the appropriate ratio for two rescuers performing CPR on an intubated client based on the most recent AHA guidelines released in May 2006. perform 30 chest compressions for every two breaths. perform 15 chest compressions for every two breaths. perform as many chest compressions as possible until reaching exhaustion. perform five chest compressions for every breath. 48

A client is being evaluated for peptic ulcer disease. The nurse recognizes that the presence of

which of the following findings supports the diagnosis of this condition?

Peptic ulcer disease pain is typically located in the midepigastric area and is described as sharp, burning, or gnawing. It may radiate to the right or toward the back.Diarrhea is not associated with peptic ulcer disease unless there is a large amount of blood in the stool from gastric or duodenal bleeding.Appetite is generally maintained in clients with peptic ulcer disease unless pyloric obstruction is present.Hematocrit and hemoglobin are not affected by peptic ulcer disease unless bleeding is present. Bleeding causes a decrease in both. Sharp midepigastric pain Intermittent diarrhea

Loss of appetite Elevated hematocrit level 49

A client is admitted for a total hip replacement. Which of the following would be included in the postoperative nursing care?

Clients are usually allowed out of bed the day after surgery, and physical therapy is started. It is important to avoid immobility due to the high risk of thromboembolic complicationsfollowing a total hip replacement.One of the highest priorities is to prevent hip flexion greaterthan 90 postoperatively. This can lead to the common complication of subluxation (partial dislocation) or a total dislocation of the hip. If the client does get up to a chair several hours aday, care must be taken to keep the angle of hip flexion greater than 90. Prolonged sitting or standing should be avoided.The lower extremities should be kept abducted using a wedge orpillows. The client should be instructed to avoid allowing the extremities to cross beyond the midline of the body. This can lead to a dislocation of the hip.Massaging the extremity on the affected side following hip replacement is contraindicated because of the high risk of dislodging a thrombus, which would then become an embolus. Maintain the client on bedrest for 3 to 4 days postoperatively. Get the client up to a straight-back chair for several hours a day postoperatively. Prevent the client's lower extremities from crossing beyond the midline. Massage the extremity on the affected side to improve circulation. 50

A client underwent a laryngectomy 24 hr ago and is unable to communicate verbally. At this time, the nurse should

After a total laryngectomy, a client will have no natural voice. Therefore, a pen and paper will provide the client with an effective initial means for communication.Esophageal speech should be learned after the client has healed from surgery. It is not appropriate this early in the client's recovery.Preventing communication with the client is inappropriate and fails to address the client's need to communicate.An artificial laryngeal device may prove frustrating to the clientduring the early phase of recovery. The client should be instructed in the use of this device several days after surgery by a speech therapist. provide the client with a pen and paper for communication. refer the client to a speech therapist for instruction in esophageal speech. encourage the client to try to remain quiet. have the client use an artificial laryngeal device.

51

Which of the following are characteristics of a skin lesion that requires immediate attention by aprimary care provider?

This is commonly referred to as senile keratosis and is an age-related change.Cherryhemangiomas are commonly seen in older adult clients. Treatment is usually not

prescribed.This is a description of a melanoma and must be further evaluated by a primary careprovider immediately.These are termed pigmented nevi. They are common in light-skinned clients. Thick and scaly Round and reddish-purple Variegated color with irregular borders

Dark brown and located randomly over the body52

A client who underwent a thoracotomy 1 day ago has a chest tube in place. Which of the following nursing actions is appropriate for this client in regard to the chest tube?

Chest drainage systems need to be below chest level to facilitate gravity drainage andprevent possible back-up of fluid into the chest cavity.Emptying the collection chamber breaks the system, increasing the possibility of air being returned to the pleural cavity. The drainage system should not be emptied. The entire chest drainage system should be changed if the drainage chamber becomes full.Milking or stripping of the chest tubes is done only if there isevidence of clotting in the tubes. Milking increases pleural pressure and is not doneroutinely.Bubbling should occur in the suction control chamber, indicating that suction is being applied and functioning properly. Keep the tubing and drainage system below the level of the client's chest. Measure and empty the drainage compartment at the end of each shift. Strip or milk the chest tube at least once each shift. Retape the tubing connections if bubbling occurs in the suction control chamber. 53

A nurse is caring for a client newly diagnosed with Cushing's syndrome. Which of the following findings is an expected manifestation of this disorder?

Because cortisol degrades collagen, there is often the presence of reddish-purple striaeon the abdomen, thighs, and upper arms.Thinning of skin is a manifestation of Cushing'ssyndrome due to excess cortisol and an increase in the breakdown of tissue protein. Skin canhave a paper-like appearance, and this occurs primarily on the back of the hands.Tetany (hyperexcitability of nerves and muscles) is a manifestation of hypoparathyroidism caused byhypocalcemia, not Cushing's syndrome.Excess cortisol causes the redistribution of fat on theface, trunk, and upper back. Fat on the upper back is also referred to as a buffalo hump. Decreased pigmentation of abdominal skin Thickened skin on the back of the hands Hyperexcitability of nerves and muscles Fat pads on the upper back54

A client is 5 days postoperative following abdominal surgery. The client coughs and then reports to the nurse, "I feel as if I just split open." The nurse sees a protrusion of internal organs. After calling for help, which of the following should the nurse do first?

This should never be done by the nurse.This should be done, but it is not the priorityintervention at this time.The priority action is to cover the wound with a sterile nonadherentdressing moistened with sterile saline to keep the tissues from drying and prevent the dressingfrom sticking to the wound.This is an intervention for wound evisceration, but it is not the priorityat this time. Attempt to reinsert the protruding organ or viscera. Check the client's vital signs and temperature. Apply a sterile, saline-soaked nonadherent dressing over the wound. Place the client in the supine position with the hips and knees bent. 55

A client is being prepared for surgery. Which of the following rationales describes why it is important for the nurse to check the client's serum potassium level before taking the client to thesurgical suite?

There is no specific need for intravenous potassium during surgery.Hyperkalemia is associated with muscle weakness rather than muscle rigidity.Recovery from anesthesia depends on the agent used, the age and condition of the client, and the metabolism of themedication. Serum potassium levels do not influence recovery from anesthesia.Hyperkalemia orhypokalemia places the client at increased risk for cardiac irritability and dysrhythmias during surgery. Intravenous potassium is often administered during surgical procedures. Hyperkalemia could prevent adequate muscle relaxation during surgery. Hypokalemia slows the client's recovery from general anesthesia. Abnormal potassium levels may cause cardiac dysrhythmias.

56

A client is to receive 12 units NPH insulin and 3 units Regular insulin subcutaneously every morning. After obtaining correct insulin, the nurse should continue to prepare the medication by performing the following steps in what order? (List the numbers without any spaces orpunctuation.)

1. Draw 3 units of Regular insulin. 2. Clean the tops of the insulin bottles with alcohol. 3. Roll insulin bottles between the palms of both hands.

4. Draw 12 units of NPH insulin. 5. Inject air into NPH insulin. 6. Inject air into Regular insulin. Rolling the insulin bottles in the hands will warm and mix the solution. Cleaning thetops of the insulin bottles prevents contamination of the insulin. It follows mixing because thetops of the bottles could become contaminated during the mixing. Air should be injected first intothe NPH and then into the Regular insulin bottles to decrease pressure. Draw clear insulin(Regular) first. The fast-acting insulin needs to remain fast-acting by not contaminating it with longer lasting insulin. Long-acting (NPH) insulin is drawn up last. If a small amount of Regularinsulin is injected into the long-acting insulin, the implications are minimal.Rolling the insulinbottles in the hands will warm and mix the solution. Cleaning the tops of the insulin bottlesprevents contamination of the insulin. It follows mixing because the tops of the bottles couldbecome contaminated during the mixing. Air should be injected first into the NPH and then into the Regular insulin bottles to decrease pressure. Draw clear insulin (Regular) first. The fastacting insulin needs to remain fast-acting by not contaminating it with longer lasting insulin. Long-acting (NPH) insulin is drawn up last. If a small amount of Regular insulin is injected intothe long-acting insulin, the implications are minimal. 57

A 20-year-old client returns to the clinic for her 6-week postpartum check. All of her prior PAPsmears have been normal. The nurse knows that client teaching about future PAP smears is effective when the client states,

This is not the appropriate schedule for the client's next PAP smear.This is not the appropriate schedule for the client's next PAP smear.Women 20 years old and older should

receive annual PAP smears unless abnormal cells are detected.This is not the appropriate schedule for the client's next PAP smear. "I will make an appointment 2 years from today for my next PAP smear." "My next PAP smear will be when I turn 25 years old." "If this PAP exam is normal, I will return in 1 year for the next one." "I won't need to have a PAP exam until I become pregnant again." 58

A client with AIDS is admitted with Pneumocystis carinii. The client is receiving pentamidine isethionate (Pentam) 124 mg IV daily, passive nebulizer treatments with albuterol four times a day, and intravenous fluids at 125 mL/hr. Which of the following client findings indicates that the therapeutic regimen is effective?

This finding may indicate that the pneumonia is worsening rather than improving.Increased tolerance of activity and improved respiratory status indicate that the therapeutic regimen is effective.The client report of improved breathing after a nebulizer treatment does not indicate that the client's pneumonia is resolving.This finding does not indicate improvement in the client's condition. The head of the bed is elevated to maximize respiratory effort and minimize dyspnea. Diminished lung sounds in the lower lobes bilaterally Minimal shortness of breath while ambulating 25 feet

Client report of feeling better after nebulizer treatments Client sleeps intermittently with the head of the bed elevated 59

Which of the following signs and symptoms indicate angina? (Select all that apply.)

Substernal chest discomfort radiating to the left arm and precipitated by exertion is correct. Angina is often described as substernal and may spread across the chest and back and down the arms. Pain lasting less than 15 min is correct. Angina usually lasts less than 15 min. Pain that occurs without cause, primarily in the morning is incorrect. This characteristic usually describes myocardial infarction pain. Pain that worsens with inspiration is incorrect. This characteristic usually describes pleuropulmonary chest pain. Pain that is relieved with rest is correct. Angina occurs frequently in response to exertion. It is often relieved with rest, nitrate administration, or oxygen therapy. Pain that is commonly precipitated by exposure to cold weather is correct. Angina pain is usually in response to exertion, emotion, or extremes in temperature.Substernal chest discomfort radiating to the left arm and precipitated by exertion is correct. Angina is often described as substernal and may spread across the chest and back and down the arms. Pain lasting less than 15 min is correct. Angina usually lasts less than 15 min. Pain that occurs without cause, primarily in the morning is incorrect. This characteristic usually

describes myocardial infarction pain. Pain that worsens with inspiration is incorrect. This characteristic usually describes pleuropulmonary chest pain. Pain that is relieved with rest is correct. Angina occurs frequently in response to exertion. It is often relieved with rest, nitrate administration, or oxygen therapy. Pain that is commonly precipitated by exposure to cold weather is correct. Angina pain is usually in response to exertion, emotion, or extremes in temperature. Substernal chest discomfort radiating to the left arm and precipitated by exertion

Pain lasting less than 15 min Pain that occurs without cause, primarily in the morning Pain that worsens with inspiration Pain that is relieved with rest Pain that is commonly precipitated by exposure to cold weather60

A nurse is caring for a client who is experiencing intracranial hemorrhage. The nurse should monitor for which of the following as an early sign of complications?

Irritability, restlessness, and decreased level of consciousness are all first signs of increased intracranial pressure. Detection of subtle neurologic changes early can preventpotentially life-threatening complications.Changes in vital signs are late signs of increasedintracranial pressure, with bradycardia occurring first.A change in blood pressure with a widening pulse pressure between systolic and diastolic is a late sign of increased intracranialpressure.Although temperature is important to monitor in any hospitalized client, this is not themost important action for the nurse to take. Neurological changes Tachycardia Widening pulse pressure Elevated temperature

61

A nurse is caring for a client with recurrent urinary tract infections whose urine culture is positivefor Escherichia coli. To prevent reinfection, the nurse should instruct the client to take which of the following actions?

Douching involves the vaginal area and will not prevent Escherichia coli from enteringthe urinary bladder.Irritating substances, such as nylon underwear and bubble baths, should be avoided. Loose-fitting cotton underwear is recommended.Wiping from front to back prevents theintroduction of feces contaminated with Escherichia coli from entering the urethra.The bladdershould be emptied before and after intercourse. Urinating after intercourse flushes the urethraand prevents the upward migration of semen or bacteria. Douche immediately following intercourse. Wear nylon underwear. Wipe front to back after defecation. Do not urinate immediately after intercourse. 62

A client's central venous pressure (CVP) line has been inserted and is ready for use. Which of the following nursing actions is most important before initiation of an infusion through the line?

Tip location of the CVP must be confirmed in the correct location before initiating an infusion.It is important to observe for infection at the insertion site, and this should be done on acontinuous basis. This is not the priority prior to the initiation of an infusion through the newly inserted line.A CVP dressing should be applied using sterile technique, but this is not the most

important nursing action prior to starting an infusion.In most cases, the client can be assisted tothe restroom after the infusion is started. Review the chest x-ray to verify the catheter-tip location. Observe the insertion site for redness, warmth, and drainage. Place a 4x4 inch dressing over the site using sterile technique.

Have the client ambulate to the restroom to void. 63

A client is experiencing an acute exacerbation of rheumatoid arthritis and has joint redness, swelling, and warmth. Which of the following nursing interventions will best promote the client'scomfort?

Cold packs reduce inflammation by decreasing circulation and swelling of the joints, which increases the client's comfort.During an exacerbation, the joints are allowed to rest withan emphasis on maintaining the joints in the correct anatomical alignment. Support andimmobilization of the affected joint will help to promote healing and avoid further injury.Elevating the affected joint above the level of the heart can decrease edema by using gravity to facilitate fluid drainage.A firm mattress should be provided to offer support. The client does need to be repositioned on a regular schedule, taking care to position in proper body alignment. Comfortcan be increased by premedicating the client prior to repositioning. Apply cold packs to the inflamed joint to decrease circulation and reduceinflammation. Perform active range-of-motion exercises to keep the joint supple and flexible. Increase tissue perfusion by positioning the affected joint below the level of the heart. Avoid repositioning and provide a soft mattress to relieve pressure points. 64

A nurse is monitoring a client who is in diabetic ketoacidosis (DKA). Which of the following

respiratory patterns is an expected manifestation of this condition?

This breathing pattern is not associated with DKA.This breathing pattern is notassociated with DKA.Ventilations that are increased in number, deeper than normal (hyperventilation), and accompanied by dyspnea are associated with DKA in an attempt to blow off excess carbon dioxide. This breathing pattern is also known as Kussmaul's respirations. There will be a fruity odor to the breath from exhaled acetone.DKA does not decrease respiratory rate and depth. Ventilations that are decreased in number and depth (hypoventilation) result in low serum levels of carbon dioxide. This leads to a slowing of respirations in an attempt to retain carbon dioxide in the arterial blood. Normal breathing pattern with intermittent periods of apnea Varied breathing pattern with intermittent periods of apnea Increased rate and depth of respirations Decreased rate and depth of respirations65

A client with dyspnea and orthopnea is taking deep breaths through his mouth. Oxygen is being administered at 2 L/min per nasal cannula. The nurse notes that the client's oxygen saturation level has dropped from 95 to 90%. Which of the following actions should the nurse take?

Clients who breathe through their mouths receive little or no benefit from oxygendelivered by nasal cannula. Changing from a cannula to a mask should improve oxygen deliveryand increase the client's oxygen saturation. The condition causing the dyspnea and orthopnea needs to be addressed as well.This type of breathing may be impossible for a client who isexperiencing respiratory difficulty.Clients who breathe through their mouths receive little or no

benefit from oxygen delivered by nasal cannula. So increasing the rate of the oxygen will notimprove oxygen delivery. Increasing the oxygen administration rate is not an independent nursing action. It requires a prescription from the primary care provider.Using a nonrebreather

mask is not indicated until other measures have been attempted. Increasing the oxygen administration rate is not an independent nursing action. It requires a prescription from the primary care provider. Change to a simple face mask while delivering the same concentration of oxygen. Instruct the client to take slow, deep breaths in through his nose and out through his mouth. Increase the rate of the oxygen flow from 2 L/min to 6 L/min. Use a nonrebreather mask and increase the oxygen flow to 10 L/min.

66

Immediately following a coronary artery bypass graft, a nurse should initially give priority to monitoring the client for which of the following?

Checking for urinary output is not necessary for an initial postoperative assessment. However, it is an important assessment throughout the duration of postoperative care.Chest tube patency is needed for re-expansion of the lungs and to maintain breathing.Pain management is required during the immediate postoperative period, but not as a priority.Altered temperature is important to detect in the postoperative period for hypothermic or hyperthermic reactions to anesthesia and infections. However, it is not an immediate priority.

Bladder distention Chest tube drainage Increased pain postoperatively Elevated temperature 67 Prior to a procedure, a client is in a state of high anxiety and begins hyperventilating. The nurse should know that which of the following acid-base imbalances can result?DIGRAM HERE (Click on the correct image from the four choices below.)

Respiratory alkalosis Respiratory alkalosis occurs when the plasma pH rises above 7.45. This can result from hyperventilation in which the client blows off too much CO2 and the plasma pH becomes alkalotic. Respiratory alkalosis can also result from hypoxia, high altitudes, fever, pregnancy, or the initial stages of a pulmonary emboli.Metabolic alkalosis Metabolic alkalosis occurs when the plasma pH rises above 7.45. It can result from loss of gastric secretions, potassium-wasting diuretics, or overuse of antacids.Metabolic acidosis Metabolic acidosis occurs when the plasma pH falls below 7.35. It can result from a salicylate overdose, shock, sepsis, diarrhea, hyperkalemia, diabetic ketoacidosis, or renal failure.Respiratory acidosis Respiratory acidosis occurs when the plasma pH drops to below 7.35. It can result from hypoventilation, drug overdose, airway obstruction (such as in COPD), pulmonary edema, or chest trauma. It can also result secondary to a neuromuscular disease that compromises ventilation.

68

A client has an infected wound that has a large amount of purulent exudate. Which of the following types of dressing should the nurse apply to promote healing?

A dry gauze dressing is absorbant and can be used on wounds with purulent exudate.Transparent film is a nonabsorbent, self-adhesive form of a moisture-retentive dressing that is used on a wound with minimal drainage. Transparent films are limited to superficial lesions.Biologic dressings provide the most "natural" covering and can act as a catalyst for

healing. These dressings can be made from pigskin or cadaver skin. They are frequently used as a skin graft on burns and will not absorb purulent drainage.Wet-to-dry dressings are used for aggressive mechanical debridement of a necrotic wound to obtain a wound bed with pinkhealthy tissue. Dry gauze dressing Transparent film dressing Biologic dressing Wet-to-dry dressing 69

A client was hospitalized 3 days ago with a deep vein thrombosis. The client reports chest pain and hemoptysis. Vital signs are: blood pressure 144/88 mm Hg, pulse 122 beats/min, respirations 28/min, temperature 38 C (100.4 F), and oxygen saturation 86% on room air. The nurse should recognize that the client is most likely experiencing manifestations of which of thefollowing complications?

The client is at great risk for pneumonia, but this is not the most likely complication.Whilethe client's symptoms are similar to those of congestive heart failure, this is not the most likelycomplication.The client's symptoms and the diagnosis of DVT make it most likely that the client is experiencing a pulmonary embolus as a result of clot mobilization.The symptoms the client isexhibiting are not consistent with a myocardial infarction. Pneumonia Heart failure Pulmonary embolus

Myocardial infarction70

A client develops hypovolemic shock following a severe burn injury. The client is receiving dopamine hydrochloride (Intropin). Which of the following findings is an adverse effect of dopamine hydrochloride that should be reported to the primary care provider immediately?

Dopamine mimics stress reactions such as increased heart rate. Tachycardia is an adverse effect of dopamine hydrochloride that should be reported to the provider.Skin changes occur in shock because of decreased perfusion.Hypertension is a symptom of dopaminehydrochloride overdose.Dopamine hydrochloride increases renal perfusion and urinary output. Bradycardia Cool and dry skin Hypertension Decreased urinary output

71

A nursing student is assigned a client with Guillain-Barr syndrome (GBS). Which of the following statements by the student indicates a need for further study about GBS?

The most common clinical pattern of GBS is ascending weakness and paresthesias. It isan acute autoimmune disorder characterized by progressive motor weakness and paralysis thatstarts in the lower extremities and moves upward. This statement indicates understanding of GBS.In descending GBS, initial weakness and paralysis occurs in the face and head. Weaknessthen progresses downward to the sternocleidomastoid muscles and eventually to the lowerextremities. This statement indicates understanding of GBS.Autonomic manifestations of this GBS include a labile blood pressure, cardiac dysrhythmias, and tachycardia. This statement

indicates understanding of GBS.Autonomic manifestations of this GBS include a labile bloodpressure, cardiac dysrhythmias, and tachycardia, not bradycardia. "It can have a clinical pattern of ascending weakness and paresthesias starting inthe lower extremities and progressing upward." "It can have a clinical pattern of descending weakness and paresthesias startingin the face and head and progressing downward." "It is characterized by autonomic manifestations of cardiac dysrhythmias andtachycardia." "It is characterized by autonomic manifestations of cardiac dysrhythmias andbradycardia." 72

A client is newly diagnosed with carpal tunnel syndrome. The nurse is reinforcing client instructions about management of the condition in hope of preventing surgical interventions. Which of the following statements by the client indicates understanding of the instructions?

Nonsurgical management of carpal tunnel syndrome consists of the administration ofnonsteroidal anti-inflammatory drugs (NSAIDs) for pain relief and reduction of inflammation on aregularly scheduled basis, and immobilization of the wrist prevents further injury to the mediannerve.Flexion and extension exercises of the wrist would cause further inflammation of the structures within the carpal tunnel and exacerbate the condition.One of the major componentsof nonsurgical management of carpal tunnel syndrome is the administration of nonsteroidal antiinflammatory drugs (NSAIDS) for pain relief and reduction of inflammation.One of the majorcomponents of nonsurgical management of carpal tunnel syndrome is the administration ofnonsteroidal antiinflammatory drugs (NSAIDs) for pain relief and reduction of inflammation. Themedication should not be taken intermittently as needed for pain, but rather on a regularlyscheduled basis to relieve inflammation. "I will avoid repetitive hand movements."

"I will perform flexion and extension exercises of my wrist." "I will avoid taking aspirin or ibuprofen." "I will take aspirin or ibuprofen as needed for pain." 73

A nurse is caring for a client with excessive diaphoresis and diuresis. Which of the following clinical manifestations should cause the nurse to suspect that the client is experiencing lowserum sodium levels?

Urine output will increase with hyponatremia.Behavioral changes are the most obviousproblem with hyponatremia. These changes may consist of either depressed activity, increased activity, or both. It is important to compare current behavior to the client's baseline.There will be increased bowel motility with hyponatremia, so bowel sounds will be hyperactive.Rapid, shallowrespirations are characteristic of hyponatremia. Decreased urine output Behavioral changes Hypoactive bowel sounds Slow, deep respirations 74

A nurse is instructing a client with newly diagnosed multiple sclerosis (MS) on methods to reduce the possibility of exacerbations. Which of the following actions should the nurse include?

Upper respiratory infections are common triggers for exacerbations of MS.Extremetemperatures can exacerbate MS.It is important for the client to avoid overexhaustion andoverheating, because this can exacerbate symptoms of MS.Alcohol has not been identified ashaving any effect on the course of MS. Avoid exposure to anyone with a cold or influenza. Maintain a warmer temperature in the bedroom. Perform cardiovascular exercise 3 to 5 times per week. Drink no more than two alcoholic drinks per week. 75

A client who was admitted 3 days ago with gastrointestinal bleeding experiences drops in hematocrit and hemoglobin. Epoetin alfa (Procrit) is prescribed. Which of the following findingsindicates that the client is responding well to the epoetin alfa?

In anemia, the skin is usually pale or cyanotic because of decreased perfusion.Bowelsounds are usually active with normal activity in a client with anemia.Clients may experienceshortness of breath, dyspnea upon exertion, and weakness due to decreased perfusion of thelungs.In anemia, extremities may have reduced or absent pulses. Skin is warm, dry, and pink with good turgor. Bowel sounds are active in all four quadrants. The client reports no shortness of breath while at rest. Peripheral pulses are present but weak in all four extremities.

76

A 27-year-old client is having a seizure. In addition to staying with the client, which of the following actions should the nurse take when intervening for this client?

Clients are not restrained during seizures because doing so may cause greaterharm.Restrictive clothing should be loosened, but there is no need for oxygen unless the clientis critically ill or debilitated.Padded tongue blades should never be placed in a client's mouthduring a seizure because it may chip teeth and cause aspiration. The client's movements may be guided, but his arms should not be restrained.Turning the client's head to the side prevents aspiration and allows secretions to drain. The client should not be restrained, but movement may be guided if necessary. Restrain the client and monitor vital signs following the seizure. Loosen the client's clothing and administer oxygen by nasal cannula. Place a padded tongue blade in the client's mouth and hold his arms at his sides. Turn the client's head to the side and protect the client from injury. 77

A client in acute renal failure has a potassium level of 6.0 mEq/L. Which of the following foods should the client experiencing hyperkalemia avoid?

These foods are not high in potassium and are not restricted for clients withhyperkalemia.These foods are not high in potassium and are not restricted for clients withhyperkalemia.These foods are not high in potassium and are not restricted for clients withhyperkalemia.All of these foods are high in potassium and should be restricted for clients withhyperkalemia. Eggs, apples, and grapefruit

Peas, onions, and squash Cranberries, green beans, and celery

Cantaloupe, bananas, and potatoes78

A client receiving chemotherapy is placed on neutropenic precautions due to leukopenia. Which of the following actions should the nurse implement?

Uncooked fruits and vegetables are contraindicated for a client on neutropenicprecautions due to the risk of infection from bacteria on the fruit and vegetables.Children whomay not be symptomatic could still expose the client to bacteria or viruses. The illness increasesthe client's risk for serious infection.The client does not need a mask while visitors are present. Visitors with infections, such as a cold, should wear masks or avoid the client entirely.The client is at a greatly increased risk for mouth infections from Candida albicans (thrush). The client's mouth should be inspected much more often than weekly, at least every 8 hr. Promote the consumption of fresh fruit and vegetables. Limit visitors, particularly young children. Encourage the client to wear a mask when visitors are present. Check the client's mouth weekly for signs of infection. 79

A client with end-stage liver cirrhosis is having an abdominal paracentesis to control ascites. Following the procedure, which of the following complications requires immediate attention?

Drainage on the dressing is an expected outcome.These are signs and symptoms ofspontaneous bacterial peritonitis (SBP). Paracentesis does not cause SBP, but it is used as a

diagnostic tool to examine ascitic fluid for infection to determine the cause of fever and appetiteloss. Antibiotic therapy is prescribed for SBP based on the determination of the type of offendingorganism found on cytologic studies.A drop in blood pressure and tachycardia may indicate shock from fluid loss and should be reported immediately.Removing fluid from the abdomen relieves pressure on the lungs and relieves respiratory distress. A change in respiratory effort is expected following paracentesis. Serosanguineous drainage on the 4x4 inch dressing Low-grade fever and loss of appetite Hypotension and tachycardia Decreased respiratory rate and effort 80

Which of the following clients is at the highest risk for developing a cystocele or rectocele?

A cystocele is a protrusion of the bladder through the vaginal wall. A rectocele is a protrusion of the anterior rectal wall through the posterior vaginal wall. Both are caused by aweakening of the pelvic muscles and relaxation of structure support. Risk factors for thedevelopment of a cystocele or rectocele include advanced age with a decrease in estrogen, difficult or multiple childbirths, and obesity.This client is not at risk for developing a cystocele or rectocele.Follicular cysts usually occur in young women who are menstruating and grow as aresult of menstrual influences. The cyst may enlarge and become neoplastic. Ovarian cysts do not place the client at risk for developing a cystocele or rectocele.This client is not at risk fordeveloping a cystocele or rectocele. A postmenopausal woman with four children

An older adult man with urinary stress incontinence A young adult woman with a history of follicular ovarian cysts A young adult man with a history of benign prostatic hypertrophy

81

A client with tuberculosis (TB) is placed in respiratory isolation. Prior to entering the client's room, the nurse must put on

A gown is unnecessary to protect the nurse from contracting TB.Since mycobacterium tuberculosis is an airborne pathogen, a mask is the best choice. Regular masks are not effectiveagainst TB because of the size of the pathogen.Gloves do not need to be worn prior to entering the room. The nurse cannot get TB from skin contact with the client's sputum. However, the nurse should always wear gloves when coming in contact with any blood or body fluids, regardless of the disease process.Goggles are not appropriate for entering the client's roombecause TB cannot be transmitted through contact with the eyes. a fluid-proof disposable gown. a high-efficiency filtration mask. gloves. goggles. 82

A client is admitted to the emergency department with third-degree burns to his face, arms, and legs. The first thing the nurse should do is

Knowledge of circumstances surrounding the burn is extremely valuable in themanagement of a burn victim, but stabilizing the client first is most important.Although IV fluidsare a high priority, this is not the initial focus of care.The first thing to do in the emergent phase of a burn is to secure the airway and ensure adequate breathing. Support of circulation wouldbe the next focus of care.Although this will eventually need to be done, stabilizing the client firstis more important. ask about the circumstances surrounding the injury. obtain venous access for isotonic fluid replacement. check for airway patency. remove the client's clothing. 83

Which of the following should be increased in the diet of a client at risk for developingosteoporosis?

Calcitonin requires an adequate calcium intake and acts as an inhibitor of boneresorption aiding in fracture reduction. Phosphorus is a mineral of bones and teeth used inenergy transfer and in buffer systems that maintain acid-base balance.High protein intake aids in the excretion of calcium and may accelerate bone loss. Protein deprivation also stimulatescalcium losses. High iron intake may be beneficial in reversing osteoporosis.The major role offluoride is to promote mineralization of developing tooth enamel. Fluoride is also incorporated inthe bone matrix providing increased density and inducing bone formation. Because of thefluoridation of municipal water in the second half of the 20th century, it is not necessary to increase dietary sources of fluoride. Magnesium is also deposited in bone. There is also significant magnesium content in water that is classified as "hard" water, if the magnesium has not been removed by a water softener. Therefore, increasing dietary sources of magnesium is not necessary.Calcium intake of older adult clients is typically low, and calcium absorption

declines after the age of 65. The kidneys do not activate vitamin D as well as they did earlier in life, and vitamin D promotes calcium absorption. Calcitonin and phosphorus Protein and iron

Fluoride and magnesium Calcium and vitamin D 84

A nurse is completing discharge teaching for a client who has undergone bilateral cataract surgery. Which of the following statements by the client indicates a need for further reinforcement of teaching?

Coughing may increase intraocular pressure. This statement indicates understanding by the client.Mild itching, reddened eyes, and slight swelling of the eyelids are normal for a clientwho is 1-day postoperative following cataract surgery. Cool compresses applied to the site and acetaminophen can help with discomfort.Bending over or any kind of straining may cause anincrease in intraocular pressure. This statement indicates understanding by the client.The client's depth perception may be disturbed following surgery. This statement indicates understanding by the client. "I need to avoid coughing." "I need to call my physician if my eyes itch or are bloodshot." "I need to avoid bending over." "I need to be very careful walking for the next few days." 85

A client with AIDS has developed a severe fungal infection and is currently being treated with

amphotericin B 35 mg IV daily. Which of the following laboratory findings needs to be reported because it indicates that the client is experiencing a side effect of this medication?

This potassium level is within normal limits.This serum albumin level is low. However, serum albumin is not affected by this medication. The nurse should evaluate the client's nutritional status.These levels are indicative of compromised renal function. Because thismedication is nephrotoxic, the nurse should notify the provider.While these levels are somewhatlow, they do not indicate significant bone marrow suppression from medication therapy. Future values should be compared to detect trends that may signify bone marrow suppression. Potassium level of 4.8 mEq/L Serum albumin level of 3.1 g/dL BUN 34 mg/dL, creatinine 2.8 mg/dL Hemoglobin 11.4 g/dL, hematocrit 32.8%

86

A nurse is caring for a client who has Addison's disease. The nurse should suspect adrenal crisis and notify the primary care provider immediately if which of the following manifestationsare noted?

Excessive diaphoresis and low blood glucose levels are manifestations of adrenalcrisis.Weight loss and decreased serum sodium are manifestations of adrenal crisis.These are

not manifestations of adrenal crisis.Adrenal crisis is highlighted by low cortisol levels andprofound weakness. Dry skin and high blood glucose levels Weight gain and increased serum sodium Pitting edema and high cortisol levels Profound weakness and low cortisol levels 87

A client is being treated for a nosebleed. Which of the following instructions should the nurse give to family members who will be caring for the client at home?

A nosebleed lasting more than 20 min represents a serious situation, and the family should seek emergency care rather than trying to pack the nose with tissue.Cold causesconstriction of vessels, which stops bleeding and relieves pain and swelling. Keeping the coldpack in place for at least 20 min should help prevent recurrences of bleeding.Saline nasalsprays can help to lubricate the nostrils and prevent bleeding that occurs due to dry nasalpassages.Having the client tilt his head back increases the risk of aspiration and disguises theseverity of the nosebleed, which could create a delay in seeking medical attention for a seriousnosebleed. The client should sit upright and lean slightly forward. If a nosebleed continues for more than 20 min, pack the nostril with tissue. Apply an ice pack to the nose for at least 20 min if bleeding recurs. Discourage the client from using saline nasal sprays. If bleeding recurs, have the client tilt his head back. 88

A female client reports dyspnea, shortness of breath, and a productive cough. On auscultation, the nurse notes that the client has coarse crackles in the right lower lobe. The client has poor skin turgor and appears dehydrated. Laboratory results include a white blood cell count of15,000 mm3, hemoglobin 14.0 g/dL, hematocrit 40%, BUN 23 mg/dL, and sodium 148 mEq/L. Which of the following laboratory results support the presence of pneumonia and dehydration? (Select all that apply.)

White blood cell count is correct. Leukocytosis (elevated white blood cell count) is acommon finding in pneumonia. This white blood cell count is elevated. The normal range is 5,000 to 10,000 mm3. Hemoglobin is incorrect. Hemoglobin is not a laboratory indicator for pneumonia. The normal hemoglobin range is 14 to 18 g/dL for men and 12 to 16 g/dL for women. The client's hemoglobin value is within normal limits. Hematocrit is incorrect. Hematocrit is not a laboratory indicator for pneumonia. The normal hematocrit range is 42 to 52% for men and 37 to 47% for women. The client's hematocrit is within normal limits. BUN is correct. The client's BUN is elevated. The normal range for BUN is 10 to 20 mg/dL. An elevated BUN may occur with pneumonia as a result of dehydration secondary to fever anddecreased fluid intake. Sodium is correct. The client's sodium level is elevated. The normal range for sodium is 136 to 145 mEq/L. An elevated sodium level may occur with pneumonia as a result of dehydration secondary to fever and decreased fluid intake.White blood cell count is correct. Leukocytosis(elevated white blood cell count) is a common finding in pneumonia. This white blood cell count is elevated. The normal range is 5,000 to 10,000 mm3. Hemoglobin is incorrect. Hemoglobin is not a laboratory indicator for pneumonia. The normal hemoglobin range is 14 to 18 g/dL for men and 12 to 16 g/dL for women. The client's hemoglobin value is within normal limits. Hematocrit is incorrect. Hematocrit is not a laboratory indicator for pneumonia. The normal hematocrit range is 42 to 52% for men and 37 to 47% for women. The client's hematocrit is within normal limits. BUN is correct. The client's BUN is elevated. The normal range for BUN is 10 to 20 mg/dL. An elevated BUN may occur with pneumonia as a result of dehydration secondary to fever anddecreased fluid intake.

Sodium is correct. The client's sodium level is elevated. The normal range for sodium is 136 to 145 mEq/L. An elevated sodium level may occur with pneumonia as a result of dehydration secondary to fever and decreased fluid intake. White blood cell count Hemoglobin Hematocrit BUN Sodium 89

On daily rounds, a nurse encounters a client who appears to be unconscious. The nurse taps the client on the shoulder and calls out the client's name with no response. The nurse finds that the client has no pulse or respirations. Which of the following nursing actions takes priority atthis time?

There is no evidence of aspiration. Therefore, back blows would not be indicated at this time.Chest compressions should not be given at this time.Obtaining a blood pressure and pulseis not the priority. It would, however, be appropriate to confirm the absence or presence of a pulse.Opening the airway and administering two slow breaths when finding an unresponsiveclient follows the American Heart Association guidelines for CPR. Administer two back blows or abdominal thrusts. Begin chest compressions at a rate of 100/min. Obtain a blood pressure and pulse. Open the airway and administer two slow breaths.

90

A nurse is teaching a client with hepatitis A about preventing the spread of the virus to others. The client demonstrates understanding of transmission of hepatitis A by stating,

This response does not address transmission, and hepatitis vaccines are specific toeach type of the disease.Hepatitis A is not a bloodborne pathogen and cannot be transmitted from a pregnant women to the fetus.Hepatitis A is not a bloodborne pathogen and cannot be transmitted through donated blood.Hepatitis A is spread by the fecal-oral route. Therefore, transmission can be prevented by thoroughly washing hands prior to preparing food. "The hepatitis vaccine could protect my family from several types of hepatitis." "If I become pregnant, I could transmit this disease to my unborn baby." "I should not donate blood because someone could become infected." "Washing my hands before handling food can prevent the spread of the virus."

También podría gustarte